100% found this document useful (6 votes)
4K views151 pages

Rusczyk R. Introduction To Geometry 2ed 2007

Uploaded by

Patrick Luo
Copyright
© © All Rights Reserved
We take content rights seriously. If you suspect this is your content, claim it here.
Available Formats
Download as PDF, TXT or read online on Scribd
100% found this document useful (6 votes)
4K views151 pages

Rusczyk R. Introduction To Geometry 2ed 2007

Uploaded by

Patrick Luo
Copyright
© © All Rights Reserved
We take content rights seriously. If you suspect this is your content, claim it here.
Available Formats
Download as PDF, TXT or read online on Scribd
You are on page 1/ 151

I ______________________________________________________Table of Contents

Contents and Introduction


Table of Contents
Legal Information
How to Use This Book
Acknowledgements

1 What's in a Name?
1.1 Why Names and Symbols?
1.2 Points, Lines, and Planes
1.3 Round and Round
1.4 Construction: Copy a Segment
1.5 The Burden of Proof
1.6 Summary

2 Angles
2.1 What is an Angle?
2.2 Measuring Angles
2.3 Straight and Vertical Angles
2.4 Parallel Lines
2.5 Angles in a Triangle
2.6 Exterior Angles
2.7 Parallel Lines Revisited
2.8 Summary
Review Problems
Challenge Problems

3 Congruent Triangles
3.1 Introduction
3.2 SSS Congruence
3.3 SAS Congruence
3.4 ASA and AAS Congruence
3.5 SSA Not-Necessarily Congruence
3.6 Isosceles and Equilateral Triangles
3.7 Construction: Equilateral Triangle and Perpendicular Bisector
3.8 Summary
Review Problems
Challenge Problems

4 Perimeter and Area


4.1 Perimeter
4.2 Area
4.3 Same Base/Same Altitude
4.4 Summary
Review Problems
Challenge Problems

5 Similar Triangles
5.1 What is Similarity?
5.2 AA Similarity
5.3 SAS Similarity
5.4 SSS Similarity
5.5 Using Similarity in Problems
5.6 Construction: Angles and Parallels
5.7 Summary
Review Problems
Challenge Problems

6 Right Triangles
6.1 Pythagorean Theorem
6.2 Two Special Right Triangles
6.3 Pythagorean Triples
6.4 Congruence and Similarity Revisited
6.5★ Heron's Formula
6.6 Construction: Perpendicular Lines
6.7 Summary
Review Problems
Challenge Problems

7 Special Parts of a Triangle


7.1 Bisectors
7.2 Perpendicular Bisectors of a Triangle
7.3 Angle Bisectors of a Triangle
7.4 Medians
7.5 Altitudes
7.6★ Challenging Problems
7.7 Construction: Bisectors
7.8 Summary
Review Problems
Challenge Problems

8 Quadrilaterals
8.1 Quadrilateral Basics
8.2 Trapezoids
8.3 Parallelograms
8.4 Rhombi
8.5 Rectangles
8.6 Squares
8.7 If and Only If
8.8★ Quadrilateral Problems
8.9 Summary
Review Problems
Challenge Problems

9 Polygons
9.1 Introduction to Polygons
9.2 Angles in a Polygon
9.3 Polygon Area
9.4 Polygon Problems
9.5 Construction: Regular Polygons
9.6 Summary
Review Problems
Challenge Problems

10 Geometric Inequalities
10.1 Sides and Angles of a Triangle
10.2 Pythagoras - Not Just For Right Triangles?
10.3 The Triangle Inequality
10.4 Summary
Review Problems
Challenge Problems

11 Circles
11.1 Arc Measure, Arc Length, and Circumference
11.2 Area
11.3 Funky Areas
11.4 Summary
Review Problems
Challenge Problems

12 Circles and Angles


12.1 Inscribed Angles
12.2 Angles Inside and Outside Circles
12.3 Tangents
12.4 Problems
12.5 Construction: Tangents
12.6 Summary
Review Problems
Challenge Problems

13 Power of a Point
13.1 What is Power of a Point?
13.2 Power of a Point Problems
13.3 Summary
Review Problems
Challenge Problems

14 Three-Dimensional Geometry
14.1 Planes
14.2 Prisms
14.3 Pyramids
14.4 Regular Polyhedra
14.5 Summary
Review Problems
Challenge Problems

15 Curved Surfaces
15.1 Cylinders
15.2 Cones
15.3 Spheres
15.4 Problems
15.5 Summary
Review Problems
Challenge Problems

16 The More Things Change...


16.1 Translations
16.2 Rotations
16.3 Reflections
16.4 Dilation
16.5 Changing the Question
16.6 Construction: Transformations
16.7 Summary
Review Problems
Challenge Problems

17 Analytic Geometry
17.1 Lines
17.2 Circles
17.3 Basic Analytic Geometry Problems
17.4 Proofs with Analytic Geometry
17.5 Distance Between a Point and a Line
17.6 Advanced Analytic Geometry Problems
17.7 Summary
Review Problems
Challenge Problems

18 Introduction to Trigonometry
18.1 Trigonometry and Right Triangles
18.2 Not Just For Right Triangles
18.3 Law of Sines and Law of Cosines
18.4 Summary
Review Problems
Challenge Problems

19 Problem Solving Strategies in Geometry


19.1 The Extra Line
19.2 Assigning Variables
19.3 Proofs
19.4 Summary
Challenge Problems
Copyright © 2006, 2007,2009,2013, 2014,2015 AoPS Incorporated. All Rights Reserved.
Reproduction of any portion of this book without the written permission of AoPS Incorporated is strictly prohibited, except for ‫״‬fair use" or other noncommercial
uses as defined in Sections 107 and 108 of the U.S. Copyright Act.
This online book may be used solely under the terms of AoPS Incorporated's Online Book License Agreement, available HERE.
Published by:
AoPS Incorporated
10865 Rancho Bernardo Rd Ste 100
San Diego, CA 92127-2102
(858) 675-4555
[email protected]
Print Version ISBN: 978-1-934124-08-6 (text), 978-1-934124-09-3 (solutions)
Cover image designed by Vanessa Rusczyk using KaleidoTile software. Cover includes a satellite image of the Mississippi River Delta from NASA Earth
Observatory and a photo of protractor and compasses by Vanessa Rusczyk.
How to Use This Book

Learn by Solving P roblem s

This book is probably very different from most of the math books that you have read before. We believe that the best way to learn
mathematics is by solving problems. Lots and lots of problems. In fact, we believe that the best way to learn mathematics is to
try to solve problems that you don’t know how to do. When you discover something on your own, you'll understand it much better
than if someone just tells it to you.

Most of the sections of this book begin with several problems. The solutions to these problems will be covered in the text, but try
to solve the problems before reading the section. If you can’t solve some of the problems, that’s OK, because they will all be fully
solved as you read the section. Even if you solve all of the problems, it’s still important to read the section, both to make sure that
your solution is correct, and also because you may find that the book’s solution is simpler or easier to understand than your own.

If you find that the problems are too easy, this means that you should try harder problems. Nobody learns very much by solving
problems that are too easy for them.

N avig atin g T h is Book

From any page in the book, you can click on the image of the book’s cover in the top-left corner to view the table of contents. On
large-screen devices, you can also click anywhere on the left-side navigation bar to jump to the corresponding section of the
book. You can use the left and right arrows at the top of the page to move to the previous or next chapter or section of the book.

Most sections begin with all of the problems that appear in the section, as explained in the “Learn by Solving Problems”
subsection above. Clicking on the 4 Jump to Solution link will jump forward in the section to where that problem and its solution
appear in the text.

Hovering over a paragraph will create a small j icon in the left margin. Clicking on that icon will pop up a window with a
permanent link to that paragraph. You can cut-and-paste this link into an email message or community post.

In teractive Features

There are several interactive features built into the book. All online books are linked to the AoPS community, so that students
using the online book can discuss the book with other students. Click on the t Community icon next to a section or problem to
view all of the discussions about that section or problem. Click the V New Topic icon to start a new discussion topic about the
section or problem.

This book is also linked to Alcumus, Art of Problem Solving's innovative online learning system. Clicking on the 9 Alcumus link at
the top of a section takes you to Alcumus. This gives you the opportunity to work on additional practice problems that reinforce
the material in your current section of the book. (Not all sections are linked to Alcumus, so the icon may not appear at the top of
some sections.) To learn more about Alcumus, click here.

Explanation of Icons

Throughout the book, you will see various shaded boxes and icons.

Concept: This will be a general problem-solving technique or strategy. These are the “keys” to
becoming a better problem solver!
1
Important: This will be something important that you should learn. It might be a formula, a
solution technique, or a caution.
Z
WARNING!! Beware if you see this box! This will point out a common mistake or pitfall.

0
Game: Remember, math is fun! This box will contain a game to think about.

Sidenote: This box will contain material which, although interesting, is not part of the main
material of the text. It’s OK to skip over these boxes, but if you read them, you might
11 learn something interesting!

Bogus Solution: Just like the impossible cube shown to the left, there’s something wrong with any
“solution” that appears in this box.

Extra! This is an “Extra!" and might be a quote, some biographical or historical background, or
perhaps an interesting idea to think about.

Exercises, R eview P roblem s, and C hallen g e P roblem s

Most sections end with several Exercises. These will test your understanding of the material that was covered in the section that
you just finished. You should try to solve all of the exercises. Exercises marked with a ★ are more difficult.

Most chapters have a section containing Review Problems. These are problems which test your understanding of the material
covered in the chapter. Your goal should be to solve most or all of the Review Problems for every chapter - if you’re unable to do
this, it means that you haven’t yet mastered the material, and you should probably go back and read the chapter again.

All of the chapters end with a section containing Challenge Problems. These problems are generally more difficult than the other
problems in the book, and will really test your mastery of the material. Some of them are very, very hard - the hardest ones are
marked with a ★ . Don't necessarily expect to be able to solve all of the Challenge Problems on your first try - these are difficult
problems even for experienced problem solvers. If you are able to solve a large number of Challenge Problems, then
congratulations, you are on your way to becoming an expert problem solver!

You can type your solution or notes for any Exercise, Review Problem, or Challenge Problem directly into the book. Your work will
automatically be saved. You won’t be able to view the solution to a problem until you type something in the solution box.

Many problems come with one or more hints. You can view any available hint by clicking on the Hint link after the problem
statement. You can then hide the hint by again clicking on the Hint link. It is very important that you first try to solve the problem
without peeking at the hints. Only after you've seriously thought about a problem and are stuck should you look at a hint. Also, for
problems which have multiple hints, use the hints one at a time; don't go to the second hint until you’ve thought about the first
one.

The solutions to all of the Exercises, Review Problems, and Challenge Problems are built into the book. Clicking the Show
Solution button will display the solution, but you won't be able to view the solution until you’ve made an attempt to solve the
problem and typed something into the solution box. Once you’ve viewed the solution, you can add notes to the solution in a
separate box. You can also click the Reset button to clear your solution and notes and start fresh.

Here are some very important things to keep in mind about the solutions:

1. Make sure that you make a serious attempt at the problem before looking at the solution. You should think hard about a
problem before deciding to give up and look at the solution. Remember, once you view a solution, you can’t change what
you typed for your solution.

2. After you solve a problem, it’s usually a good idea to read the solution, even if you think you know how to solve the problem.
Our solution might show you a quicker or more concise way to solve the problem, or it might have a completely different
solution method that you might not have thought of.

3. If you have to look at the solution in order to solve a problem, make sure that you make a note of that problem. You can
then come back to the problem in a week or two to make sure that you are able to solve it on your own without resorting to
the solution.

R esources

Here are some other good resources for you to further pursue your study of mathematics:

■ Art of Problem Solving has a complete library of books (both print and online) specifically designed for avid math students:

■ The Introduction series: Prealgebra, Introduction to Algebra, Introduction to Counting & Probability, and Introduction to
Number Theory, designed for students in grades 6-10.

■ The Intermediate series: Intermediate Algebra, Intermediate Counting & Probability, Precalculus, and Calculus,
designed for students in grades 9-12.

■ The Problem Solving series: designed for students preparing for math competitions. In addition to our classics the
Art of Problem Solving, Volume 7: the Basics (for students in grades 7-10 preparing for MATHCOUNTS and the AMC
8/10/12 contests) and the Art of Problem Solving, Volume 2: and Beyond (for students in grades 9-12 preparing for
advanced contests such as the AIME), we also have Competition Math for Middle School.

■ Beast Academy: a full, rigorous, entertaining curriculum for aspiring math beasts in grades 2-5.

■ The Art of Problem Solving website contains many other resources for students.

■ The AoPS Community has tens of thousands of members (if you are reading this online book, you’re a member too!)
and millions of posts on a variety of math, problem solving, and other fun topics.

■ Alcumus, our free adaptive online learning system containing over 13,000 practice problems.

■ For the Win!, our free interactive online game inspired by the MATHCOUNTS Countdown Round.

■ Our vast video library contains hundreds of videos featuring AoPS founder Richard Rusczyk.

■ Learn LaTeX, the mathematical typesetting system used by most professional mathematicians and scientists, from
our widely-used LaTeX guide, and practice your LaTeX skills with the TeXeR.

■ AoPS Community members collaborate to build the AoPSWiki.

■ We have a collection of articles on a variety of problem-solving topics.

■ You can hone your problem solving skills (and perhaps win prizes!) by participating in various math contests. Please see
the Acknowledgements section of this book for more information.

A N ote to T each ers

We believe that students learn best when they are challenged with hard problems that at first they may not know how to do. This
is the motivating philosophy behind this book.

Rather than first introducing new material and then giving students exercises, we present problems at the start of each section
that students should try to solve before the new material is presented. The goal is to get students to discover the new material on
their own. Often, complicated problems are broken into smaller parts, so that students can discover new techniques one piece at
a time. Then the new material is formally presented in the text, and full solutions to each problem are explained, along with
problem-solving strategies.

We hope that teachers will find that many students will discover most of the material in this book on their own by working
through the problems. Other students may learn better from a more traditional approach of first seeing the new material, then
working the problems. Teachers have the flexibility to use either approach when teaching from this book.

The book is linear in coverage. Generally, students and teachers should progress straight through the book in order, without
skipping chapters. Sections denoted with a ★ contain supplementary material that may be safely skipped. In general, chapters
are not equal in length, so different chapters may take different amounts of classroom time.
Acknowledgements

C ontests

We would like to thank the following contests for allowing us to use a selection of their problems in this book:

■ The American Mathematics Competitions, a series of contests for U.S. middle and high school students. The AMC 8, AMC
10, and AMC 12 contests are multiple-choice tests that are taken by over 350,000 students every year. Top scorers on the
AMC 10 and AMC 12 are invited to take the American Invitational Mathematics Examination (AIME), which is a more
difficult, short-answer contest. Approximately 10,000 students every year participate in the AIME. Then, based on the
results of the AMC and AIME contests, about 500 students are invited to participate in the USA Junior Mathematical
Olympiad (USAJMO) and USA Mathematical Olympiad (USAMO), 2-day, 9-hour examinations in which each student must
show all of his or her work. Results from the USA(J)MO are used to invite students to the Math Olympiad Summer
Program, at which the U.S. team for the International Mathematical Olympiad (IMO) is trained. More information about the
AMC contests can be found on the AMC website at h ttp : / / www.maa.org/math-competitions.

■ MATHCOUNTS®, the premier contest for U.S. middle school students. MATHCOUNTS is a national enrichment, coaching,
and competition program that promotes middle school mathematics achievement through grassroots involvement in every
U.S. state and territory, with over 160,000 students participating in 2013-14. President Barack Obama, and former
Presidents Bush, Clinton, Bush and Reagan have all recognized MATHCOUNTS in White House ceremonies. The
MATHCOUNTS program has also received two White House citations as an outstanding private sector initiative. More
information is available at h t t p : //www. mathcounts. org.

■ The Mandelbrot Competition, which was founded in 1990 by Sandor Lehoczky, Richard Rusczyk, and Sam Vandervelde.
The aim of the Mandelbrot Competition is to provide a challenging, engaging mathematical experience which is both
competitive and educational. Students compete both as individuals and in teams. The Mandelbrot Competition is offered
at the national level for more advanced students and the regional level for less experienced problem solvers. The
Mandelbrot Competition is currently on hiatus until 2017. More information can be found at h t t p : / /
www.m andelbrot. org.

■ The Harvard-MIT Mathematics Tournament (HMMT), which is a twice-annual math tournament for high school students,
held at MIT and Harvard. It is run exclusively by MIT and Harvard students, most of whom themselves participated in math
contests in high school. More information is available at h t t p : //hmm t. m it. edu.

■ The USA Mathematical Talent Search (USAMTS), which was founded in 1989 by Professor George Berzsenyi. The
USAMTS is a free mathematics competition open to all United States middle and high school students. As opposed to
most mathematics competitions, the USAMTS allows students a full month to work out their solutions. Carefully written
justifications are required for each problem. More information is available at h ttp ://www. usamts.org.

■ The American Regions Math League (ARML), which was founded in 1976. The annual ARML competition brings together
nearly 2,000 of the nation's finest students. They meet, compete against, and socialize with one another, forming
friendships and sharpening their mathematical skills. The contest is written for high school students, although some
exceptional junior high students attend each year. The competition consists of several events, which include a team round,
a power question (in which a team solves proof-oriented questions), an individual round, and two relay rounds. More
information is available at h t t p : / / a r m l. com.

H ow W e W rote This Book

This book is a collaborative effort of the staff of the Art of Problem Solving. Richard Rusczyk was the lead author for this book,
and wrote most of the text. Some of the Sidebar and Extra sections were prepared by Ashley Reiter Ahlin, Vanessa Rusczyk, and
Naoki Sato. The solutions were written by Ruozhou Jia, Brian Rice, Richard Rusczyk, and Naoki Sato. Extensive proofreading of
the manuscript was done by Mathew Crawford, Lisa Davis, Amanda Jones, David Patrick, Tim Lambert, Naoki Sato, and Jake
Wildstrom. Vanessa Rusczyk designed the cover and also contributed greatly to the interior design of the book. David Patrick,
Naoki Sato, Ravi Boppana, Meena Boppana, Valentin Vornicu, Greg Brockman, Larry Evans, and Joseph Laurendi contributed
problems and proofreading to the second edition.

The author would also like to thank Josh Zucker, whose comments about how he learned mathematics inspired the questions-
before-the-lessons approach of the text.

The print version of this book was written using the LaTeX document processing system, and the diagrams were prepared using
Metapost and Asymptote. We thank the authors of the various LaTeX packages that we used while preparing this book, and also
the brilliant authors of The LaTeX Companion for writing a reference book that is not only thorough but also very readable.

The source files for the print book were initially converted to this online book using a script written in the Python programming
language. Palmer Mebane managed the conversion of this book from print to online, and Paul Salerno also wrote tools used in
the conversion. Jason Batterson and the Jacob Tyler Creative Group designed the look and feel of this online book. James Fung,
Shelley Garg, Kyle Guillet, Tasha Moyer, David Patrick, Shannon Rogers, Amy Szczepanski, Deven Ware, Phyllis Xu, and Laura
Zehender all helped to review and edit the online book content.

D edication

For Professor Harold Reiter, who brought me back to education, and for Vanessa Rusczyk, whose confidence in me and love of
the desert has kept me here ever since.
The Golden Ratio Spiral

Do not laugh at notations; invent them, they are powerful. In fact, mathematics is, to a large extent, invention o f better notations. - Richard Feynman

CHAPTER
I
I What's in a Name?

Each of these images helps tell a story. Throughout this book well share these stories with you, but before we tell these stories,
we have to name our characters.

1.1 Why Names and Symbols?


To convince you that names and symbols are useful, well start at the end of the book instead of the beginning. Here's the final
example problem in this book, written without any special symbols or names.

Draw three points and connect each to the other two with straight paths. Also, draw the circle that passes through all
three of these points. Then, draw a line through one of those three points such that the line goes inside the region you
just formed and is equally close to the two other straight paths you formed initially through this point. Draw the circle
that goes through the one of your three first points you just drew a line through, through the point where this line hits
the straight path that connects the other two of your first three points, and through the point that is half-way between
these two other points.

Consider the two paths from the point we drew the extra line through to the other two of our first three points. These
paths hit our second circle before they hit these other two points. Show that the distance from where the circle hits
these paths to the points where these paths end is the same for both paths.

If you can make much sense of this problem, you're a much more careful reader than I am! We need some special names and
symbols so we can communicate mathematical ideas more simply.
1.2 Points, Lines, and Planes
P•

Figure 1.1: A Point

A dot. A speck. In geometry, it’s a point. If you lived on a point, you'd be awfully bored. There would be no up and down, no right
and left. You couldn't move any amount in any direction. Since you can't move on your point in any direction, we say a point has 0
dimensions. In order to tell one point from another, we usually label them with capital letters, such as point P above.

A •----------------------------- • b

Figure 1.2: A Segment

Now, say you got so bored on one point that you just had to go to another point. If there were a straight path from one point to
another, that path would be called a line segment, or just a segment. The two points at the ends of a segment are cleverly called
the endpoints of the segment. We use these endpoints to label the segment. For example, A I3 is the segment from A to B. To
denote the length of the segment, we omit the bar. For example, A B equals 1.5 inches in Figure 1.2.

Figure 1.3: A Midpoint and Marking Segments of Equal Length

The endpoints aren't the only points on a segment. There are infinitely many points, since between any two points on the
segment, we can find another point. One special point on a segment is the segment's midpoint, which is the point halfway
between the endpoints. Because the midpoint is the same distance from both endpoints, we say it is equidistant from the
endpoints. In Figure 1.3, M is the midpoint of X Y . We show that X M = M Y in the diagram with the little tick marks along
X M and M Y . If we have multiple sets of equally long segments, we use a different number of tick marks for each. For
example, our diagram above indicates that Z X = W Y , and that these lengths need not be the same as X M and M Y .

Figure 1.4: A Ray

If you're not happy just going from A to B, you can keep going past point B. If you keep going forever, you will make a ray. We
refer to the ray in Figure 1.4 as A B , where the starting point, or origin, of the ray comes first. In the diagram, the little arrow
indicates that the ray continues forever in that direction.

Figure 1.5: A Line

As you might guess, we could continue forever in both directions. The result is a line. Line A B is shown in Figure 1.5. We
sometimes use a lowercase letter to describe a line, such as line k in the figure. We often leave off the little arrows in the
diagrams.

Figure 1.7: Three Concurrent Lines

If three or more points are all on the same line, we say the points are collinear, and if three or more lines all pass through the
same point, we say the lines are concurrent.

Segments, rays, and lines are all one-dimensional figures, since you have only one way you can move along them. Roughly
speaking, any path you can draw with a pencil is one-dimensional, meaning you can either move ‫׳‬forward‫ ׳‬on the path or
‫׳‬backward‫ ׳‬on the path. Once we have the freedom to go off our path and move around on a surface, we're up to two dimensions.
On a surface like this page, we might call our dimensions left-right and up-down. If the page extended forever in every direction,
we'd call it a plane. Most of this book discusses planar figures, which are figures that exist in planes.

However, in Chapter 14, we wander off the page and add a third dimension you might think of as 'above-below.' The physical
space we live in is effectively three-dimensional, and most of what we experience is three-dimensional.

Although it’s much harder to think about, there’s a great deal of math in higher dimensions. But that’s a story for another day.

Exercises

1 .2 .1 : t V

Alice is thinking of a line. How many points on that line does she need to show Bob in order for Bob to know exactly which line
she is thinking about?

Type your solution, notes and/or work here. Show Solution

1.2.2: t V

M is the midpoint of A B and N is the midpoint of B M . If B N = 4, then what is A B ?

Type your solution, notes and/or work here. Show Solution

1.2.3: t V

P, Q, R, S, and T are on line k such that Q is the midpoint of P T , R is the midpoint of Q T , and S is the midpoint of R T . If
P S = 9, then what is P T?

Type your solution, notes and/or work here. Show Solution

1.2.4★ : Source: MATHCOUNTS t V

Points A, B, C, I), and E are five points on a line segment with endpoints A and E . The points are in the order listed above
from left to right such that C D = A B / 2 , B C = C D / 2 . A B = A E /2 , and A E = 12. What is the length of .4/7?

Hint

Type your solution, notes and/or work here. Show Solution


1.3 Round and Round

P roblem s
Problem 1.1 4 Jump to Solution

Mark a point on a piece of a paper and label it O. Use a ruler to find points on your paper that are 1 inch away from the point ().
If you draw all of these points, what figure would you create?

We have many fancy names for things in mathematics. The fancy name we have for a group of points that satisfy certain
conditions is a locus. While you may never have heard of that word, you’ve certainly heard of the first locus well investigate. (And
it’s no big deal if you forget the word ,locus’ until Intermediate Geometry!)

Problem 1.1 t V

Mark a point on a piece of a paper and label it O. Use a ruler to find points on your paper that are 1 inch away from the point O.
If you draw all of these points, what figure would you create?

Solution for Problem 7.7: When we draw all the points that are 1 inch away from 0 , we form
a figure called a circle. The point 0 is called the center of the circle. We often refer to a
circle by its center, writing 'circle O' or '0 (7 , where the O symbol tells us that we’re dealing
with a circle. We say that O A is a radius of the circle because it is a segment connecting
the center to a point on the circle. We know that all points on the circle must be 1 inch from
the center, so O A = 1 inch. The term 'radius' is also used to mean the length of a radius,
so we could write: 'The radius of © O is 1 inch.’

You'll notice that we didn’t use a big dot to mark point A When there's a label near where
two figures meet, the label refers to the point where they meet. Therefore, A is the point
where our radius hits the circle, z

Much of our work in this book involves both lines and circles.

Problem 1.2 t V

Can a line and a circle intersect in 0 points? 1 point? 2 points? 3 points? More?

Solution for Problem 7.2: Given © A , we can clearly find a line that doesn’t hit X anywhere. Line k shown below is such a line.
Imagine sliding line k closer and closer to Q)X until it touches the circle at exactly one point, such as line £ touches © A at point
P. We say that line £ is a tangent line to the circle. We can also use 'tangent' as an adjective, and write, 'Line £ is tangent to © A .'

Lines even closer to the center intersect the circle at two points, such as A' does. A line that hits a circle at two points is a
secant line. A segment that connects two points on a circle is a chord. M N is a chord, while \ 11$ is a secant line. A chord that
passes through the center of a circle is a diameter.

Finally, the portion of a circle that connects two points on a circle is called an arc of that circle. Of course, we have a symbol for

that too: M N is the shorter of the two arcs that connect M and N . We call the shorter of the two arcs that connect two points
on a circle a minor arc of the circle. The longer arc that connects the two points is a major arc of the circle. We usually use three

points to denote a major arc: P N M is the longer arc connecting P to M , while P M is the smaller arc connecting them. □

Exercises

1 .3 .1 : t V

In the figure at right, identify whether each of the following is a secant line, a chord, a radius, a
diameter, or a tangent line of © 0 . (If multiple terms are accurate, list all of the accurate terms.)

(a) CO

Type your solution, notes and/or work here. Show Solution

(b) ÈT

Type your solution, notes and/or work here. Show Solution

(c) CD

Type your solution, notes and/or work here. Show Solution

(d) AB

Type your solution, notes and/or work here. Show Solution

(e) C Î)

Type your solution, notes and/or work here. Show Solution

1 .3 .2 : t V

Suppose point P is outside a given circle. Is it always possible to draw a line through P that is tangent to the circle? (No proof
is necessary now; you’ll have the tools to prove your answer later in the text.)

Type your solution, notes and/or work here. Show Solution

1 .3 .3 : Source: AMC 10 t V

What is the maximum number of possible points of intersection of a circle and a triangle? (A triangle is formed by connecting
three points with line segments.)

Type your solution, notes and/or work here. Show Solution

1 .3 .4 ★ : Source: MATHCOUNTS t V

Two circles and three straight lines lie in the same plane. If neither the circles nor the lines are coincident (meaning the two
circles are different and the three lines are all different lines), what is the maximum possible number of points at which at
least two of the five figures intersect?

Hint

Type your solution, notes and/or work here. Show Solution


1.4 Construction: Copy a Segment
Classical construction problems are sprinkled throughout the book because a deep understanding of constructions usually leads
to a deep understanding of geometry. Construction problems ask us to create precise geometric diagrams with two simple tools.
These tools are a compass, to make circles, and a straightedge, to make straight line segments. Notice that we don't say 'ruler‫ ׳‬to
make line segments. You don't get to use your straightedge to measure lengths of segments - you can only draw lines. Similarly,
you aren't allowed to use your protractor to measure or create angles.

So, what can you do?

That's the goal of these construction sections: to start learning what you can do with only compass and straightedge. The only
operations you can perform with your compass and straightedge are the following:

1. Given a point, you can draw any line through the point.
2. Given two points, you can draw the line that passes through them both.
3. Given a point, you can draw any circle centered at that point.
4. Given a point and a segment, you can draw the circle with its center at that point and with radius equal in length to the length
of the segment.
5. Given two points, you can draw the circle through one point such that the other point is the center of the circle.

That’s not much, but with these simple operations we can construct an enormous range of diagrams.

P roblem s

We start our exploration of construction by learning how to copy a segment.

Problem 1.3 t V

Use your compass to find a point Y on k such that A B = X Y . You cannot simply use a
ruler to measure A B , then use that measurement to find Y \ . ‫־‬B
•A
X

Solution for Problem 7.3: All we can do with a compass is draw circles or parts of circles. To
find a point that is A B from X , we first open our compass to a width of A B by putting the
point of the compass at A and the compass pencil at B (or vice versa). Then we make a
circle with center X and this opening as the radius. Since this Q X has a radius equal to A B ,
the two points where it hits k are A B away from X . We can take either one of these as our
point Y . □

Concept: In nearly all construction problems in which we must make a point, we find that point
I by constructing two figures that the point must be on. The point we seek is then at the
I intersection of these two figures. For example, in Problem 1.3, we have line k and
construct Q X that Y must be on. Their intersection gives us the point Y we seek.

Let's try a slightly more challenging construction.

Problem 1.4 t V

Shown are segments A B and M N . Use straightedge and compass to construct a line segment that
has length A B 4‫ ־‬M N , and a line segment that has length A B — M N .

Solution for Problem 1.4: We start by drawing a line k and choosing a point P on line k. We find a point that is A B 4‫ ־‬M N from
P in two steps. First, we find a point a distance of A B from P using our construction technique in Problem 1.3. We do so by
opening our compass to width A B and using this radius to draw a circle centered at P. We take one of the points where this
circle hits k to be point Q.

Then we find a point that is M N from Q by opening our compass to width M N and using this radius to draw a circle centered
at Q. As shown, this circle hits k at two points, R and R '. To get to point R from P, we go a distance of A B to get to Q, then
M N more to reach R . Therefore, P R = A B 4‫ ־‬M N . Similarly, to get to R ' from P, we first go a distance equal to A B to get
to Q, then head back towards P a distance of M N to get to R'. So, P R ' = A B — M N . □

You might have noticed that we didn't need the entire circles we drew in our
B
constructions. We only needed enough of the circle to tell where the circle would hit
the line. Typically, these little arcs are all we draw in our constructions. Therefore, our
paper when constructing A B — M N in Problem 1.4 might look as shown at right.

Exercises

1 .4 .1 : t V

Given the segments shown, construct segments with the following lengths: /‫־‬

(a) A B 4- C D - E F .

Type your solution, notes and/or work here. Show Solution

(b) 2A B .

Type your solution, notes and/or work here. Show Solution

(c) A B - 2 E F + 3C D .

Type your solution, notes and/or work here. Show Solution


1.5 The Burden of Proof
Earlier we defined a line segment as the direct path that connects two points. It seems obvious that any two points can be
connected by a segment. In fact, it seems so obvious that it should be easy to prove. However, it isn't just hard to prove - it’s
impossible. The statement that any two points can be connected by a straight line segment must be simply accepted as a fact.
We call such a statement that must be regarded as fact without proof an axiom. Axioms are also sometimes called postulates.
When the world's most famous geometer, Euclid, wrote his famous Elements, he stated five axioms:

1. Any two points can be connected by a straight line segment.


2. Any line segment can be extended forever in both directions, forming a line.
3. Given any line segment, we can draw a circle with the segment as a radius and one of the segment’s endpoints as center.
4. All right angles are congruent. (We'll talk about right angles and what we mean by ,congruent' shortly!)
5. Given any straight line and a point not on the line, there is exactly one straight line that passes through the point and never
meets the first line.

In Euclid’s Elements, he combined these axioms to prove ever more complicated mathematical statements. We call such proven
mathematical statements theorems. A mathematical statement that is not an axiom but hasn’t been proved false or true is called
a conjecture.

In this book, we don't start from Euclid's axioms and prove everything that follows step-by-step. It’s a good thing, too! It turns out
that even Euclid missed a few axioms. Mathematicians since have shown that Euclid's arguments, in order to be completely valid,
would need many more axioms added to these five. In other words, there are some things that even the great Euclid didn't realize
are so 'obvious‫ ׳‬that they could not be proved. Often when we reach these items in this text, we give a 'common sense’
explanation of why we accept these statements as facts. We note when these really are axioms, as opposed to statements that
we can prove using previous axioms or theorems. You can use the proofs we present both as guides for writing your own proofs
and as stepping stones to prove interesting theorems of your own.
1.6 Summary

Definitions:
■ A point is, well, a point. Euclid called a point 'that which has no part.’ We can't do
much better than that vague description. We typically denote points with capital
letters.

■ A straight path connecting two points is called a segment, and our original two
points are the endpoints of the segment. We refer to a segment by its endpoints,
such as A B. We remove the bar to denote the length of the segment: A B .

Definitions:
■ The point on a segment that is halfway between the endpoints is the midpoint of
the segment. We also say that this point is equidistant from the endpoints.

■ If we start at a point, then head in one direction forever, we form a ray. Our starting

point is the vertex of the ray, and we denote a ray as A B , where the first point is
the vertex of the ray.

■ If we continue a line segment past its endpoints forever in both directions, we form

a line, which we write as A tf.

■ If this page were continued forever in every direction, the result would be a plane.
Since we can move in two general directions, such as right-left and up-down, on a
plane, we say the plane has two dimensions.

■ If we add a third dimension, we are in three-dimensional space.

The set of all points that satisfy specific conditions is called a locus.

Definitions:
The set of all points that are the same distance from a given point is a circle. The
given point is the center of the circle, and the fixed distance is the radius. We often
refer to a circle by its center using the symbol 0 , so © O refers to a circle centered
atO .

A line that touches a circle at a single point is tangent to the circle, while a line that
hits a circle at two points is a secant line. A segment connecting two points on a
circle is a chord, and a chord that passes through the center of its circle is a
diameter. The portion of a circle that connects two points on the circle is an arc,

which we denote with the endpoints of the arc: M N is the shorter arc that
connects M and N .

When performing constructions with a straightedge and compass, you can only draw line segments and circular arcs. You cannot
use a ruler to measure segments. The operations you can perform are:

1. Given a point, you can draw any line through the point.
2. Given two points, you can draw the line that passes through them both.
3. Given a point, you can draw any circle centered at that point.
4. Given a point and a segment, you can draw the circle with its center at that point and with radius equal in length to the length
of the segment.
5. Given two points, you can draw the circle through one point such that the other point is the center of the circle.

Extra! At the top of the first page of each chapter in this book is an image illustrating an interesting geometric
fact. The image at the start of this chapter is of the Golden Ratio Spiral. A Golden Ratio Spiral is inside a
V golden rectangle, which is a rectangle that can be divided into a square and another rectangle such that
the ratio between the dimensions of the new rectangle equals that of the original rectangle.

P x —1
/1

D
Q x —1

Shown above is golden rectangle A B C D with dimensions 1 and x. P Q divides the rectangle into a
square of side 1 and a rectangle with dimensions x — 1 and 1. Since the ratio of the dimensions of
A B C D equals the ratio of the dimensions of B C Q P , we have

1 x —1
x 1

The positive value of a; that satisfies this equation is

1 + \/5
x — ---------- ~ 1.618034.
2
This number is the golden ratio (also sometimes called the golden mean), and is often referred to by the
Greek letter 0 ('phi’).

When we divide a golden rectangle into a square and a rectangle, the ratio of the dimensions of the
smaller rectangle is the same as that of the original rectangle. Therefore, the smaller rectangle is a
golden rectangle too, so we can split it into a square and another smaller golden rectangle. We can do
this over and over indefinitely, forming the figure shown below.

All of the squares in the diagram together make up our largest golden rectangle. When we omit the
largest square, we get our next golden rectangle. Then we omit the next largest square to find the next
golden rectangle, and so on. If we then draw a quarter-circle in each of the squares, as shown above, we
get the Golden Ratio Spiral.
The Lighthouse Theorem

We're going to turn this team around 360 degrees. - Jason Kidd

CHAPTER 2 1
L ,Angles

2.1 What is an Angle?


When two rays share an origin, they form an angle.

In the diagram at left, rays 0 ) t and O ? share origin O. We can refer to the angle they form as
Z X O Y . The Z symbol tells us were referring to an angle. The common origin is called the vertex of

O the angle, and the rays 0 ) t and O Y are called the sides of the angle. Notice that when we write the
angle as Z X O Y , we put the vertex in the middle. We could also refer to the angle as Z Y O X , but not
as Z X Y O . When it’s very clear what angle we're talking about, we can just name it with the vertex:
ZO.

Of course, two intersecting lines also make angles.

Lines A $ and C l*) at right intersect at P. Here, we can't just write Z P , since there are many
different possible angles this could mean, such as Z A P C , Z A P D , Z D P B , or Z B P C . We
might even be referring to Z A P B .

Now that we know what angles are, we need a way to measure them so we can compare one
angle to another.
2.2 Measuring Angles

Just as we use a ruler to measure the lengths of segments, we can use a protractor to measure angles. Roughly speaking, an
angle's measure is how ‫׳‬open‫ ׳‬the angle is. Our protractor above shows half a circle (which we call a semicircle) divided into 180
equal pieces. Each of these little pieces is considered one degree of the semicircle, so that an entire circle is 360 degrees. We
use the symbol ° to denote degrees, so that a whole circle is 360°.

We use a protractor to measure the number of degrees of a circle between the two sides of an angle whose vertex is the center
of the circle. For example, in Figure 2.1, the vertex of Z Y O Z is placed at the center of the semicircle. There are 62 degrees
between sides o X and 0 $ of Z Y O Z , so we say that Z Y O Z = 62°. Sometimes angles are written with an m before Z to
indicate measure: m Z Y O Z = 62°.

P roblem s
Problem 2.1 4 Jump to Solution

Use your protractor to find Z Y O Z , Z C X D, Z D X E, and Z C X E.

Problem 2.2 4 Jump to Solution

The diagram below shows four common angles. In each case, point O is the center of the circle. Z A O B cuts off 1 /4 of a
circle, Z C O D cuts off 1 /3 of a circle, Z E O F cuts off 1/12 of a circle, and Z G O H cuts off 1 /8 of a circle.

(a) What is the measure in degrees of Z A O B ?

(b) What is the measure in degrees of Z C O I) ?

(c) What is the measure in degrees of Z E O F ?

(d) What is the measure in degrees of Z G O H ?

(e) What's so special about 360; why do we use 360 for the number of degrees in a whole circle?

Do not use a protractor; use what you are told about the angles in the text.

Problem 2.3 4 Jump to Solution

Given that Z W O Y = 60° and Z W O X = 20° below, find Z X O Y .

1/
O

Problem 2.4 4 Jump to Solution

Suppose instead of measuring an angle the ‫׳‬regular’ way, we go the ‫׳‬long‫ ׳‬way around, as shown in the
Py
diagram. The ,regular‫ ׳‬angle P Q R has measure 40°. What is the measure of the ‫׳‬long’ way around
angle?
f /to
\Q _ J ‫־‬/?

Problem 2.5 4 Jump to Solution

Use your protractor to create a 37° angle and a 143° angle.

Solution for Problem 2.7: The protractor itself is half a circle (which we call a semicircle); we use it to measure the number of
degrees of a circle the angle cuts off. Here are the steps we follow to use our protractor to measure angles:

1. Place the protractor on the angle so that the vertex of the angle is exactly where the center of the circle would be if the
protractor were a whole circle. Your protractor should clearly show this center point; it’s near the middle of the straight side.
2. Turn the protractor so that one side of the angle is along the 'zero line; i.e., the line through the center point along the
straight edge of the protractor.
3. Find where the other side of the angle meets the curved side of the protractor. The number there tells you the measure of
the angle.

For Z Y O Z , we put our protractor on the page as shown below. We line up side o X of the angle with the zero line of the
protractor, placing the center point of the protractor over O. We find that side o X hits the curved edge at 90°.

When we follow this procedure with Z C X D, we find that there are two numbers where X I 3 meets the curved edge in the
following diagram. We know to take the smaller of these numbers - clearly there are 40 degrees, not 140 degrees, between X (X
and xX>. We can also note that Z C X I ) is less than half the entire semicircle, so its measure must be the smaller of the two
numbers where X I 3 meets the curved edge of the protractor.

We can also use the above diagram to find the measure of Z C X E. Once again, our angle hits a point on the curved edge with
two numbers, but this time we know the angle is greater than 90° (since the angle is more than half the semicircle). Thus, we
know that Z C X E = 115°.

Finally, we can place the protractor as in the diagram below to find that Z D X E = 75°.

Notice that Z C X D 4‫ ־‬Z D X E = Z C X E . This isn’t an accident! Since Z C X D and Z D X E share a side and a vertex,
putting them together gives Z C X E. □

We saw in Problem 2.1 that knowing whether an angle is greater than or less than 90° is necessary for
finding its measure using a protractor. This 90° is such an important measure that angles that are 90° 7■
have a special name, right angles. We usually mark right angles with a little box as shown in Z . J K L at
right. Two lines, rays, or line segments that form a right angle are said to be perpendicular.. / K and K L
are perpendicular; we can use the symbol _L to write this briefly: J K J_ K L .
I

Angles that are less than 90° are called acute, and those that are greater than 90° but less than 180° are called obtuse.
Sometimes we write the measure of an angle inside the angle as shown above.

Problem 2.2 t V

The diagram below shows four common angles. In each case, point O is the center of the circle. Z A O B cuts off 1/4 of a
circle, Z C O D cuts off 1 /3 of a circle, Z E O F cuts off 1/12 of a circle, and Z G O H cuts off 1/8 of a circle.

(a) What is the measure in degrees of ZAOB?

(b) What is the measure in degrees of Z C O D?

(c) What is the measure in degrees of Z EO F?

(d) What is the measure in degrees of ZG O H ?

(e) Why in the world do we use such a weird number, 360, for the number of degrees in a whole circle?

Do not use a protractor; use what you are told about the angles in the text.

Solution for Problem 2.2: Since a whole circle is 360°, and Z A O B is 1 /4 of a circle, we have

Z A O B = ( I ) (360°) = 9 0 ° .

We can tackle the other three angles in exactly the same way:

ZCOD =
G) (360°) = 120 0

ZEO F =
œ (360°) = 30 0

ZGOH =
G) (360°) = 45c

The number 360 comes from the ancient Babylonians. The Babylonians used a number system with 60 digits, instead of our
decimal system, which only has 10 digits. When choosing a number of degrees for a whole circle, they were likely influenced by
their number system and possibly by astronomy (a year has around 360 days). However, a look at our answers above points to
what might have been the largest factor in choosing 360. The Babylonians, like most people today, probably hated fractions.
Since 360 is divisible by lots of different numbers, many common angles in geometry have integer measures. Had we used 100

degrees instead, we’d have to deal with measures such as 1 2 - degrees, 8 - degrees, and so on. □
2 o

Problem 2.3 t V

Given that Z W O Y = 60° and Z W O X = 20°, find Z X O Y .

1A
/

/
Solution for Problem 2.3: As we just saw in Problem 2.1, we can combine two angles that share a side and a vertex to make a
third whose measure equals the sum of the measures of the first two. Applying this to the diagram in the problem, we see that
Z W O X 4‫ ־‬Z X O Y = Z W O Y . Therefore, 20° 4‫ ־‬Z X O Y = 60°, so Z X O Y = 40° □

We call angles that share a side, like Z W O X and Z X O Y in Problem 2.3, adjacent angles.

Problem 2.4 t V

Suppose instead of measuring an angle the ‫׳‬regular' way, we go the 'long' way around, as shown in the
diagram. The ‫׳‬regular' angle P Q R has measure 40°. What is the measure of the 'long‫ ׳‬way around angle?

Solution for Problem 2.4: If we imagine our ‫׳‬regular‫ ׳‬angle P Q R cutting off a circle, we know it cuts off 40° of the circle. The
‫׳‬long‫ ׳‬way around then must be the rest of the circle. Since a whole circle is 360°, the remainder of our circle is
360° - 40° = 320° □

Angles that are greater than 180° are called reflex angles. They are rarely important in problems.

We finish our discussion of measuring angles by learning how to draw them given angle measurements.

Problem 2.5 t V

Use your protractor to create a 37° angle and a 143" angle.

Solution for Problem 2.5: We start with one side, O A , of the 37° angle, which we can draw anywhere. To create the other side, we
use our protractor to figure out where the other side would have to go in order to make a 37° angle. We place our protractor over
o X as if we are measuring an angle with o X as a side. We then find the 37° point on the curved side, since a 37° angle would
have to go through this point. As we’ve noticed before, our protractor has two 37’s, one on each side. Since a 37° angle is clearly
acute, we know to choose point B in the figure on the next page, thus making an angle that is less than 90°.

Similarly, we have two choices when we build our 143° angle. This time we choose the one that creates the obtuse angle, as
shown in the diagram below.

Exercises

2 .2 .1 : t V

Use your protractor to make angles with the following measures:

(a) 90° (b) 45° (c) 135° (d) 220c

Type your solution, notes and/or work here. Show Solution

Use your protractor to measure the angles shown. Classify each angle as right, acute, or obtuse.

(a) ZX

Type your solution, notes and/or work here. Show Solution

(b) Z A B C and Z D B C .

Type your solution, notes and/or work here. Show Solution

(c) Z P Q R , Z P R Q , and Z R P Q .

Type your solution, notes and/or work here.

Extra! The golden ratio that we discussed in Section 1.6 here does not only appear in
geometry! For example, consider the Fibonacci sequence shown below.
V
1, 1, 2, 3, 5, 8, 13, 21, 34, 55, 89, 144, 233, . . .
The first two terms of the sequence are both 1, and each subsequent term is the sum
of the previous two terms. Calculate the ratio between each term and the term before
it, such as 34/21 ~ 1.619. See anything interesting?
2.3 Straight and Vertical Angles

P roblem s
Problem 2.6 4 Jump to Solution

In the figure below, A O D is a straight line. What is the measure of Z A O D ?

b
~o~

Problem 2.9 4 Jump to Solution

Lines i i V and A f(3 intersect at P. Prove that Z M P N = Z L P O .


.L 'M

‫ ׳‬o

As we have noted, two intersecting lines make angles. Can we make an angle with just one line?

Problem 2.6 t V

In the figure, A O B is a straight line. What is the measure of ZAOB?


O

Solution for Problem 2.6: If we don’t see the answer right away, we can try to figure out what portion of a
circle the angle cuts off. So, we draw a circle with center O as in the diagram to the right. Now we can see
that the angle cuts off half a circle (whichever side of the line we pick). So,
Z A O B = (1 /2 ) (360°) = 180°

This one's easy to remember: we call an angle that is really a straight line a straight angle, z

Straight angles appear too simple to be useful, but often the simplest tools are the best.

Solution for Problem 2.7: Since Z W X Y and Z Y X Z together make Z W X Z , which is a straight angle, we know that
Z W X Y + Z r X Z = 180° Hence,

Z W X Y = 180° - Z K X Z = 125°.

We call two angles that add to 180° supplementary angles. As we have seen, when two lines intersect like \-'Y and in
Problem 2.7, any two adjacent angles thus formed are supplementary because together they make a straight line.

Similarly, we call angles that add to 90° complementary angles.

Problem 2.8 t V

Lines L N and A /(3 intersect at P such that Z M P N = 72°. Find Z L P O . .Z


/ m

7k
yo

Solution for Problem 2.8: Angle L P O sure looks equal to Z M P N , and it ‘makes sense that the two are equal, but ‫׳‬makes
sense‫ ׳‬isn't good enough in mathematics. We need proof, which we’ll tackle in the next problem. For now, well try to compute
ZLPO .

Since it's not obvious how to compute Z L P O , we start by finding angles we can measure. Since Z M P N and Z N P O
together make a straight angle, we have Z N P O 4‫ ־‬Z M P N = 180°. Thus, Z N P O = 180° — 72° = 108°.

Similarly, since Z L P O and Z N P O are supplementary, we have

Z L P O = 180° - Z N P O = 180° - 108° = 72°.

Concept: When you can’t find the answer right away, try finding whatever you can - you might
I discover something that leads to the answer! Better yet, you might learn something
even more interesting than the answer. The best problem solvers are explorers.

Solution for Problem 2.9: What’s wrong with this proof:

Bogus Solution: Suppose Z M P N = 72°. Since Z M P O is a straight angle, we know that


Z N P O = 180° — 72° = 108°. Similarly, we have
y Z L P O = 180° - Z N P O = 72° Therefore, Z M P N = Z L P O .

Every statement in that ,proof is true. However, it is not a complete proof because it only addresses one case; it only shows that
Z M P N = Z L P O when Z M P N = 72°.

WARNING!! An example is not a proof!

While examples aren’t proofs, they can be useful as guides. Looking at our example, we can quickly construct our proof.

Since h p 6 is a line, we have

Z M P N = 180° - Z N P O .

Since Ï P Ê is a line, we have

Z L P O = 180° - Z N P O .

Combining these two equations gives

Z M P N = 180° - Z N P O = Z L P O .

Notice that our proof does not depend at all on the measure of Z M P N . The proof works no matter how the lines intersect.

Pairs of angles such as Z M P N and Z L P O in the diagram below are called vertical angles. As we proved in Problem 2.9,
vertical angles are always equal to each other.

We often use little arcs to mark equal angles. In the diagram to the right, Z M P N and Z L P O
each have a single little arc in them to show that they are equal. Angles Z L P M and Z N P O
also are vertical angles, so they are equal. We put a little hash-mark on the arcs at these angles to
show that these two angles are equal to each other, but not necessarily equal to our first pair of
equal angles (which have arcs without hash-marks).

Important: Supplementary, right, obtuse, vertical, acute... by now the number of new names
must be driving you nuts. Don't memorize what all these names mean now! The
Z names are not that important. Besides, as you work your way through this book, you'll
eventually see them so much you'll just know them anyway.

The concepts are more important than the words for solving problems. ‘Angles like
Z M P N and Z L P O in Problem 2.9 are equal’ means something without any more
information. ‘Vertical angles are equal' doesn’t tell you anything until you reach for
your math dictionary to look up vertical angles.

The words, however, are important for communicating the concepts. For now, though,
focus on the ideas. The words will come naturally.

Exercises

2 .3 .1 : t V

Find x, y, and 2 in the diagram below.

Type your solution, notes and/or work here. Show Solution

2 .3 .2 : » V

Find the measure of an angle that is complementary to each of the following angles:

(a) Z O O M = 30°

Type your solution, notes and/or work here. Show Solution 1

(c) Z K O L = 75°

Type your solution, notes and/or work here. Show Solution |


2.4 Parallel Lines
Having dissected what happens when two lines meet, we should wonder about what happens if they don't. If two lines do not
meet, we say that they are parallel. If lines A fa and C fa are parallel, we write £ 2 1| £ 5 .

Figure 2.2: Two Sets of Parallel Lines

Just as we use little arcs to mark angles that are equal, we can use little arrows to mark lines that are parallel. In the diagram
above, lines j , k, and £ are marked parallel, as are lines m and n. You won’t see us use this notation all the time, though. Those
little arrows can really clutter up a diagram.

P roblem s
Problem 2.10 4 Jump to Solution

Draw a pair of parallel lines like those shown below. Then draw a line that crosses both of the parallel lines. Measure all the
angles formed between your line and both of the parallel lines. Write the angle measures in the angles you form. Try it again
with a different crossing line.

Do you notice anything interesting?

<-------------------------------------------------------------------------- — >

<-------------------------------------------------------------------------- — >

Problem 2.11 4 Jump to Solution

Lines m and n are parallel, and we are given the measure of one angle in the diagram as
shown. Find the values of a, b, c, w, x, y, and 2 .

Problem 2.12 4 Jump to Solution

In the figure, we have *Afa | C fa and fafa | fad. We are also given the measures of four angles as shown in terms of x and
y. Find x and y.

Problem 2.13 4 Jump to Solution

Given that W V \ Y Z and W Z || V Y in the diagram, find x.

Problem 2.14 4 Jump to Solution

In the diagram, £ || m and the angles are as marked. Find x.

Back in Section 1.5 here, we noted that one of our axioms (statements we must accept without proof) is

Given any straight line and a point not on the line, there is exactly one straight line that passes through the point and
never meets the first line.

This common-sense statement is sometimes called the Parallel Postulate. Well start our exploration of parallel lines by taking a
look at the angles formed when a line intersects a pair of parallel lines.

Problem 2.10 t V

Draw a pair of parallel lines, then draw a line that crosses both of the parallel lines. Measure all the ^
angles formed between your line and both of the parallel lines. Write the angle measures in the
angles you form. Try it for a second line (don't worry about the angles where your two lines meet - <------------------------- >
just focus on the angles between the lines you draw and the parallel lines). Do you notice anything
interesting?

Solution for Problem 2.70: In the diagram to the right, we have parallel lines £ and m, and
we have added line n, which meets £ and m at A and B, respectively. We call a line that
cuts across parallel lines a transversal. We measure Z H A G and find that it equals 40°.
Since Z H A G and Z C A B are vertical angles, we don’t even have to measure Z C A B .
We know that Z C A B = Z H A G = 40°.

Since Z H A G and Z B A G together make up a straight angle, we don’t have to


measure Z B A G . We know that

Z B A G = 180° - Z H A G = 180° - 40° = 140°.

Similarly, Z H A C = 140°.

We might wonder if we need our protractor at all, but then we think about those angles around B. They sure look equal to those
around A, and common sense tells us that they are, but we measure to make sure. We find that indeed Z A B F = 40°, from
which we deduce that Z D B E = 40° as well. We can also quickly determine that

Z A B D = 180° - Z A B F = 140°

and Z E B F = 140°.

Seeing that Z H A G = Z A B F , we wonder if it’s always true that a transversal will cut parallel lines at equal angles like
Z H A G and Z A B F . Like the Parallel Postulate, this turns out to be one of those ,obvious‫ ׳‬facts that cannot be proved. It must
be assumed. As we have seen while finding the angles above, once we know that these two are equal, we can quickly use lines
and vertical angles to find the rest of the angles.

Figure 2.3: Angles Between a Transversal and Two Parallel Lines

Thus, we see that when two parallel lines are cut by a transversal, we have two groups of four equal angles. Specifically, in Figure
2.3, we have
a = c = e = y
b = d = f = h

Furthermore, the angles in the first group are supplementary to those in the second.

Pairs of these angles have special names to describe their relationships. These names are not terribly important, but you’ll see
them elsewhere.

a and e are corresponding angles.

d and / are alternate interior angles.

a and y are alternate exterior angles.

c and / are same-side interior angles.

b and y are same-side exterior angles.

Again, the names are not such a big deal. After doing enough geometry, you’ll probably know them anyway. Don’t bother
memorizing them now. Just understand which angles are equal and which are supplementary. □

Problem 2.11 t V

Lines m and n are parallel, and we are given the measure of one angle in the diagram as shown.
Find the values of a, b, c, w, x, y, and 2. w lz mS
< xy
‫״‬ 113‫^ ״‬ n
ti c

Solution for Problem 2.77: We know that when a transversal cuts parallel lines, equal angles come in groups of four as we saw in
Problem 2.10. Therefore, we know that w = y = c = 113°. We also know that each angle in the other ,group of four’ has a
measure that is supplementary to 113°:

x = z = a = b = 180° - 113° = 67°.

Now that we understand the relationships between angles when a parallel line is cut by a transversal, let's try a more challenging
problem.

Problem 2.12 t V

In the figure, we have and *Afa || fad. We are also given the measures of four angles as shown in terms of x and y.
Find x and y.

Solution for Problem 2.12: There’s no obvious way to make an equation for x or y, so we start off by using our parallel lines and
vertical angles to write the measures of all the angles we know in terms of x and y.

After labeling the angles we know in terms of x and y, we look for ways to build equations. We can use angles that together form
straight angles at A and B.

Z E A N 4 Z E A B = (3y 4 15°) 43) ‫־‬x - 15°) = 180°

Z F B A 4 Z F B G 4 Z G B H = (3 y 4 15°) 4 x 4 y = 180°.

Rearranging these gives

x -\- y = 60°
x + Ay = 165°

Subtracting the first from the second gives us 3y = 105°, so y = 35° We can then use substitution to find x = 25°. Of course,
we didn’t have to label every angle above - we could have stopped when we had enough information to set up a pair of
equations to solve for x and y.

Note that we could have used parallel line relationships to set up the equations, too. Since we have
ZD AN = ZKDM , so 3x - 15° = x - \- y . Also, ZHBC and ZBCI are supplementary, so
(3y 4■ 15°) 4 (3# — 15°) = 180° Solving these two equations gives us the same answer as before. (It better!) □

Concept: Solving a problem with two different methods is an excellent way to check your
! answer.

We’ll finish with two more challenging problems that illustrate how useful parallel lines can be when seeking angle measures.

Problem 2.13 t V

Given that W V || Y Z and W Z || V Y in the diagram, find x. x


W V -
/1 4 0 ‫״‬Y *

к
z ^ ^

Solution for Problem 2.13: We start by using what we know about parallel lines to find as many angles as we can. We find
Z V = 180° - 140° = 40° since W Z || V Y . Similarly, Z Z = 40°, and Z Z Y V = 180° - Z Z = 140°.

Since Z P Y V = 3x, we have

Z P Y Z = Z V Y Z - Z P Y V = 140° - 3x.

Since this angle, the 90° angle, and the angle with measure x together give us straight line fa fa, we have

Z P Y Z 4 90° 4 x = 180°.

We then substitute Z P Y Z = 140° — 3x into this equation, and we have 140° — 3x 4 90° 4 x = 180°. We solve this
equation for x to find that x = 25°. □

Using information about angles to find information about other angles is often called angle-chasing. We’ve already learned three
important tools in angle-chasing: straight angles, vertical angles, and parallel lines. Stay tuned. We'll see plenty more!

Concept: Often when we’re angle-chasing, our goal is to build an equation to solve for one of
I the variables in our problem.

Problem 2.14 t V

In the diagram, £ || m and the angles are as marked. Find x.


, , A L.
130‫״‬

SY m
c

Solution for Problem 2.14: We need to relate our desired angle to angles we know, but neither
A B nor B C cuts both parallel lines. However, if we add a line A*through B parallel to £ and m,
we can do some angle-chasing. Since k || £, we have Z A B X = 180° — 130° = 50°. Since
k || m, we have Z X B C = x. Since Z X B C 4 Z A B X = Z A B C , we can now write an
equation for x:

x 4 50° = 90°.

Therefore, x = 40°. □

Concept: Parallel lines are so useful in problems involving angles that sometimes we'll add new
ones to a diagram to help us.
I

Exercises

2 .4 .1 : t V

Find x and y in the diagram below.

Type your solution, notes and/or work here. Show Solution

Show that Z A = Z C and Z B = Z D in the diagram below.

Type your solution, notes and/or work here. Show Solution

Find x in the diagram below.

Type your solution, notes and/or work here. Show Solution

In the diagram below, m n, and the angles are as marked. Find x.

Hint

Type your solution, notes and/or work here. Show Solution

In the diagram at right, k || £ and m || n. If the angles are as marked, find x and y.

Hint

Type your solution, notes and/or work here. Show Solution


2.5 Angles in a Triangle
A

Figure 2.4: A Triangle

When we connect three points with line segments, we form a triangle, as shown in Figure 2.4 above. We will often refer to the
triangle as A A B C , or sometimes just A B C . The points A, B, and C are called vertices of the triangle, and the segments A B ,
B C , and A C are called sides. In this section, however, we will investigate the angles of a triangle.

P roblem s

We start our investigation of angles in a triangle by measuring angles in a few triangles.

Problem 2.15 t V

(a) Find the measure of the three angles in each of the triangles below.

(b) Can you guess a statement that is always true about the sum of the angles in a triangle?

Ü t x
\
\ A
/ \ <?
F Y Z

Solution for Problem 2.15: Using our protractor, we can find the measures of the angles in each triangle as below:

A D E F : Z D = 60° Z E = 90° I F = 30°


A X Y Z : Z X = 40° Z y = 70° Z Z = 70°
A P Q R : Z P = 130° Z Q = 30° Z R = 20°

In each case, we see that the sum of the angles is 180° Hmmm... Is that always true? z

Before tackling the question of whether or not the angles of a triangle always add to 180°, let's try a parallel line problem that
includes a triangle.

Solution for Problem 2.16: We start as we usually do when the answer isn’t immediately obvious
- we find what we can. We can use vertical angles to find x = 70° in the figure to the right. We
can also use either line £ or } j/3 to find w. gives 70° + w + 50° = 180°, so
w = 180° - 70° - 50° = 60°

Now we can use our parallel lines to find the angles we want. Since £ || m, we have
Z B C D = w = 60° and Z B D C = x = 70° □

Notice that we don't have to call Z B C D and Z C B E ‫׳‬alternate interior angles' in order to say they are equal. Once we state that
£ || m, we can note that Z B C D = w without writing ,alternate interior angles.' However, if your teacher tells you that you have
to include ,alternate interior angles,‫ ׳‬you better do it!

Inspired by Problem 2.16, we can now prove that the sum of the angles in a triangle is always 180°

Problem 2.17 t V

Prove that the sum of the measures of the angles in a triangle is always 180 .

Solution for Problem 2.17: We start by drawing a triangle and by writing what we want to prove:

Z A B C 4‫ ־‬Z C A B + Z B C A = 180°.

We don’t know a whole lot about angles yet, but we can start by wondering ' Where have we seen 180° before?1
Answer: A straight angle. So, we'd like to find a straight line that has all three of our angles, just like Z E B C ,
Z C B D , and Z D B F make up line £ in our solution to Problem 2.16. However, we don't have any such line yet, so well have to
add a line somewhere.

We need a line that allows us to use what we know about angles. We don’t know much about
angles, so we don't have too many options to investigate. So far, probably the most useful
angle information we have learned comes from parallel lines. So, we add a line through A
parallel to B C to create the diagram to the right, which looks curiously like the figure from
Problem 2.16.

Since || B C , we have Z X A B = Z A B C and Z B C A = Z C A Y . Since Z X A Y is a straight angle, we have

Z X A B 4‫ ־‬Z C A B + Z C A Y = 180°.

Now we're home! We can now use the equalities we found with the parallel lines to get:

Z A B C + Z C A B 4‫ ־‬Z B C A = 180°.

Important: The sum of the angles in a triangle is always 180°.

Z
There are few geometric relationships you will use more than this one!

Important: Don't view the proof for Problem 2.17 as magic. We see 180°, and that makes us
-j think of finding useful lines. We need a line that has angles equal to the three in
A A B C . Equal angles make us think of parallel lines, which always give lots of equal
angles when cut by a transversal line.
Close the book and try to re-create this proof on your own!

Now let's try solving a problem using the fact that the sum of the angles of a triangle is 180°.

Problem 2.18 t V

One angle in a triangle is twice another angle, and the third angle is 54°. What is the measure of the smallest angle in the
triangle?

Solution for Problem 2.18: We know one angle is 54°, but all we know about the other two is that one is twice the other.
Therefore, we let the smaller angle be x, so the larger is 2x. Since the sum of the angles of a triangle is 180°, we have

54° 4- x 42 ‫־‬x = 180°.

Solving this, we find x = 42°. The angles of our triangle are 42°, 54°, and 84°. (Note we can make a quick check by adding the
three and making sure we get 180°)

Our smallest angle is 42°. z

Concept: The key to tackling word problems in geometry is the same as any other kind of word
problem — turn the words into math. Usually this means defining variables and using
I the words to write equations to solve for the variables.

WARNING!! Your last step should be to make sure you’ve answered the question that is asked.

One of the most common uses of the fact that the angles in a triangle add up to 180° is as an angle-chasing tool. Let's give it a
try.

Problem 2.19 t V

In the diagram, m \ n ,A B _L m, Z A D C = Z B C E = Sx, Z C E B = 50°, and Z B C D = x. Find x.

D B
/\
m

Solution for Problem 2.19: We start with our parallel lines, and see that Z A B E = 90° because m || n and m _L A B . Then,
from A A B E we have

Z E A B + Z A B E 4‫ ־‬Z A E B = 180°,

so

Z E A B = 180° - Z A B E - Z A E B = 40°.
We can then use A A C D to find

Z A C D = 180° - Z D A C - Z A D C
= 180° - 40° - Sx
= 140° - Sx.

Concept: When angle-chasing, it’s best to write the values you find for angles on your diagram
as you find them, even when these values include variables.

Our diagram now looks like the figure to the right. This picture suggests a way to finish the
problem. We have three angles with vertices at C that together make a straight line, so we have

Z A C D 4‫ ־‬Z D C B 4‫ ־‬Z B C E = 180°.

Substitution gives

140° — 3a; + a; + 3x = 180°,

so x = 40°. z

Exercises

2 .5 .1 : t V

Two angles in a triangle have measures 30° and 57° What is the measure of the third angle?

Type your solution, notes and/or work here. Solution

2 .5 .2 : t V

The angles in a triangle are in the ratio 1 : 2 : 3 . What are the measures of the angles?

Type your solution, notes and/or work here. Solution

2 .5 .3 : t V

Find Z Z Y P in the diagram below.

Type your solution, notes and/or work here. Show Solution

One of the angles in a triangle is a right angle. Show that the other two angles are complementary.

Type your solution, notes and/or work here. Show Solution

2 .5 .5 ★ : t V

Using what you know about triangles, find a different solution to Problem 2.14.

Hint

Type your solution, notes and/or work here. Solution


2.6 Exterior Angles

P roblem s

In the previous section, we examined the interior angles of a triangle. In this section, we will take a look at the angles formed
when we extend a side of a triangle past a vertex. We cleverly call these the exterior angles of the triangle. For example, in the
problem below, Z A C D is an exterior angle of A A B C .

Solution for Problem 2.20: From A A B C , we have

35° 4 ‫ ־‬79° 4 ‫ ־‬Z A C B = 180°,

so Z A C B = 66°. From line B D we have

Z A C B + x = 180°,

sox* = 180° - 66° = 1 1 4 °□

That straightforward solution suggests we can prove something general about an exterior angle of a triangle.

Problem 2.21 t V

Prove that Z X 4 Z Y = Z X Z P in the diagram shown.


X

X .
Y Z F

Solution for Problem 2.27: Our solution to Problem 2.20 guides the way. From A X Y Z , we have

z x + z y + z x z y = 180°.

From line Y Z P , we have

Z X Z Y + Z X Z P = 180°.

Subtracting the second equation from the first gives us

Z X + Z Y - Z X Z P = 0,

which we can easily rearrange to the desired Z X 4 Z Y = Z X Z P .

Notice that our proof does not at all depend on the values of the angles! □

And now, we have more things to name. Z X and Z Y are called the remote interior angles of exterior angle Z X Z P of
A X Y Z . This name is really not important. We mostly give it a name so we can briefly write what we just proved:

Important: Any exterior angle of a triangle is equal to the sum of its remote interior angles.

Solution for Problem 2.22: We know we'll probably need to find x to answer the problem. We could label every angle we know in
terms of x, but first we take a minute to look for a faster way to get x.

We have Z B A C and Z A C B of A A B C , so we know the exterior angle

Z G B C = Z B A C 4 Z A C B = 2x 4 10°.

From A B || D E , we know that Z G B C = Z D E F = ! + 55°. Hence, we have 2x 4 10° = x + 55°, so x = 45°. Our
desired angle is the supplement of Z I ) E E , so our answer is

Z C E D = 180° - Z D E F = 180° - (x 4 55°) = 80°.

There are many other ways we could have approached this problem. This is almost always true when we have problems involving
exterior angles. Using exterior angles of a triangle is really just a shortcut for using a line and what we learned in Section 2.5
about the angles of a triangle. □

Solution for Problem 2.23: Well use Z A , Z B , and Z C to refer to the interior angles of A A B C . Using what we just learned
about exterior angles, we have

x = ZB 4 ZC
y = ZA + ZC
z = ZA + ZB.

Adding these, and noting that Z A 4 Z B 4 Z C = 180°, gives:

x + y + z = {Z B 4 Z C ) 4 (Z A 4 Z C ) 4 (Z A 4 Z B )
= 2 (Z A + Z B + Z C )
= 2(180°)
= 360°.

You’ll be seeing this again. Only, next time, well be dealing with a figure that has more sides than a simple triangle! □

Exercises

2 .6 .1 : t V

Find y in the figure at left below.

A
i ? ___ \ y ____

Type your solution, notes and/or work here.

2 .6 .2 : t V

Find Z C in the figure below.

Type your solution, notes and/or work here. Show Solution

2 .6 .3 : t V

123° V Find Z Y in the figure at left.

Type your solution, notes and/or work here. Show Solution

Must an exterior angle of a triangle always be greater than 90°?

Type your solution, notes and/or work here. Show Solution

Extra! For an extra challenge, try to figure out the sum of the interior angles,
and the sum of the exterior angles, in a figure with more sides than a
V triangle, such as the figure at right.
2.7 Parallel Lines Revisited

P roblem s
Problem 2.24 4 Jump to Solution

(a) In the first diagram below, what can we say about lines k and m?

(b) Why? (Hint: What's wrong with A P Q R in the second figure?)

In Section 2.4, we learned some useful relationships about angles when we have parallel lines. In this section, we investigate
whether or not we can 'go backwards' by using angle relationships to prove lines are parallel.

Problem 2.24 t V

In the diagram to the right, what can we say about lines k and m? Why can we say it?

V £ ' . ‫ ■״‬.
X Y

. Q 40‫״‬
z

Solution for Problem 2.24: What's wrong with this solution:

Bogus Solution: If lines k and m were parallel, then / X P Y = / X Q Z . Since we know that
/ X P Y = /.X Q Z , lines k and rti must be parallel.

This ,solution' is exactly the same as saying the following:

If an animal is a cat, the animal must have four legs. My pet Spot has four legs. Spot must be a cat.

Clearly, this is bogus, because Spot could be a dog. All we know about Spot is that she has four legs. Our Bogus Solution to the
problem has the same flaw!

We know for sure that if lines k and m were parallel, then Z X P Y = / X Q Z . It is not logically valid to just run that backwards
and say ,If / X P Y = / X Q Z , then lines k and m are parallel.' We have to prove this second statement separately. In Spot's
case, we saw that the ,backwards’ version of ,If an animal is a cat, then the animal must have four legs' is not even true! We call
this ,backwards' version the converse of the original statement.

WARNING!! Suppose we have a true statement of the form:

0 If this, then that.

The converse of this statement is:

If that, then this.

Even if our original statement is true, the converse doesn’t have to be true. We have to
prove the converse separately.

And now, back to our story. We’d like to prove that lines k and m are parallel, and we can't use our angle relationships because we
don't know they are parallel. The only other thing we know about parallel lines is that they never meet. So, we wonder if it is
possible for the lines to meet if / X P Y = / X Q Z .

Suppose k and m meet at R as shown in the figure at right. We hâve a triangle, so we try to
use what we know about triangles. First, we note that

/ R P Q = 180° - 40° = 140°.

Now that we hâve two angles of A P Q R , we can find the third:

/ P R Q = 180° - / R P Q - / R Q P = 180° - 140° - 40° = 0°.

Uh-oh. If k and m meet at R, then / R must be 0°, which doesn’t make any sense. Therefore, we have shown that it is impossible
for k and m to meet to the right of P(§. (We'll leave the case of k and m meeting to the left of W ) as an Exercise.)

Since it is impossible for k and in to meet, they must be parallel. □

Important: Each of the angle relationships regarding parallel lines that we found in Section 2.4
can be used to prove that two lines are parallel. Using these angle relationships is the
Z most common method of proving that two lines are parallel.

Concept: Our solution to the previous problem is an example of proof by contradiction. To


! prove a statement by contradiction, we start by assuming the statement is false. Then
I we show that this assumption leads us to an impossible statement (such as
/ P R Q = 0° above), which tells us that the assumption itself is false. Having proved
the statement cannot be false, we have shown it must be true.

Problem 2.25 t V

Given that the angles are as marked in the diagram, prove that A B || C D and B C | A D .
A b

Solution for Problem 2.25: We start by finding the angles we can find. Triangles A A C I ) and A A R C te W us that

/ A C D = 180° - 30° - 80° = 70°


/ A C B = 180° - 70° - 80° = 30°.

Therefore, we have / D A C = / A C B , which means that A D || B C . We also have / B A C = / D C A , which tells us


A B II C D .

We also could have noted that / D 4‫ ־‬/ D A B = 180°, so A B || C D . Likewise, / B 4‫ ־‬/ B A D = 180° gives us A D || B C .

Exercises

2 .7 .1 : t V

We only did half of the proof in Problem 2.24. Complete the proof by showing that the two lines cannot meet to the left of

Type your solution, notes and/or work here. Show Solution

For the diagram below, we have proved that if n cuts k and m such that x = y (that is, corresponding angles are equal), then
k |! m. Use this fact to show that if y = z (that is, alternate interior angles are equal), then k |j in.

Type your solution, notes and/or work here. Show Solution

Find x in the diagram below.

Hint

Hint

Type your solution, notes and/or work here. Show Solution

Write the converse of each of the following statements, then identify whether or not that converse is true,

(a) If two teams are playing in the World Cup Finals, then the teams must be playing soccer.

Type your solution, notes and/or work here. Show Solution

(b) If two of the angles of a triangle add to 80°, then one angle of the triangle must be 100C

Type your solution, notes and/or work here. Show Solution

(c) If a river is the longest river in the world, then it must be the Nile.

Type your solution, notes and/or work here. Show Solution

(d) If an animal is a duck, then it must be a bird.

Type your solution, notes and/or work here. Show Solution


3.3 SAS Congruence

P roblem s
Problem 3.5 4 Jump to Solution

In each of the triangles below, two sides have lengths 2.5 cm and 4 cm, and the angle between these sides is 60° Measure the
third side and the other two angles of each triangle.

B E

Problem 3.6 4 Jump to Solution

Investigate why the triangles in the previous problem appear to be congruent by constructing a triangle with side lengths 2.5 cm
and 4 cm such that the angle between these two sides is 60°.

(a) Draw segment .4 B such that A B = 4 cm.

(b) Let B C be the side that is 2.5 cm long; thus, C is 2.5 cm from B. Construct the figure that consists of all points that are
2.5 cm from B.

(c) The angle between A B and B C must be 60°. Where can C be to make Z A B C = 60°? (Forget about B C = 2.5 cm for
this part.)

(d) Draw your answer to (c).

(e) Where can C possibly be located?

(f) Are all triangles with two sides of length 2.5 cm and 4 cm with an angle of 60° between them congruent?

Showing that all the sides of one triangle equal those of another is not the only way to quickly prove two triangles are congruent.
In this section we investigate one way to use angles along with sides.

Solution for Problem 3.5: In each case, the third side is 3.5 cm and the other two angles are approximately 82° and 38°, with the
82° angle opposite the 4 cm side. The triangles sure look congruent, z

Sidenote: Perhaps only the ancient Greeks have contributed as much to geometry as the
German mathematician David Hilbert. Among his wide range of work was Grundlagen
^ der Geometrie, which placed geometry in a formal axiomatic framework. Instead of
Euclid’s 5 axioms, Hilbert used 21 axioms, since Euclid's work included many hidden
assumptions that must be accepted as axioms. Among those axioms is SAS
Congruence, which, in combination with other axioms, he used to prove the other
Congruence Theorems presented in this chapter.

Not only did Hilbert produce much great mathematics, but he also inspired and
challenged others. In 1900 he gave an address to the Second International Congress
in Paris on 23 unsolved problems of great importance. Since then, some of the
problems have been solved, but many still puzzle mathematicians to this day. He also
succinctly phrased the passion that guides many great mathematicians and
scientists when he closed an address with the words, "We must know, we shall know‫׳‬.‫״‬

Problem 3.6 t V

Investigate why the triangles in the previous problem appear to be congruent by constructing a triangle with side lengths 2.5 cm
and 4 cm such that the angle between these two sides is 60°.

Solution for Problem 3.6: We start off just like we did in Problem 3.2, by letting A B = 4 cm. We want B C = 2.5 cm and
Z A B C = 60°. Since C is 2.5 cm from B, it lies on a circle with center B and radius 2.5 cm. Since / . A B C = 60°, we know
that C lies on a ray from B that forms a 60° angle with A B . We can use our compass and protractor to build the circle and our
two possible rays as shown below.

In our diagram there are only two options for C: the two points where one of the rays meets the circle. Just like we saw in
Problem 3.2, these C's are mirror images. A A B C i and A A B C -2 have the same side lengths and angle measures.

Therefore, we find that no matter how we build the triangle given two sides and the angle between them, we always get a triangle
with the same sides and angles, z

Important: We call the principle illustrated in Problem 3.6 the Side-Angle-Side Congruence
^ Theorem, or SAS for short. SAS states:

If two sides of one triangle and the angle between them are equal to the
corresponding sides and angle of another triangle, then the two triangles are
congruent.

As with our illustration of SSS in Section 3.2 here, our solution to Problem 3.6 is not a complete proof of the SAS Congruence
Theorem. It merely provides an intuitive explanation. Let’s see SAS in action.

Problem 3.7 t V

Point E is the midpoint of both A C and B D as shown. Prove that A B CD.


A,____ 7 B

K
Y7e 1

Y \ 5

‫־־‬ C

Solution for Problem 3.7: We saw in Section 2.7 that we can use angle equality to prove that lines are parallel. We can get the
angle equality we need by using congruent triangles. Since A E = E C , B E = E D , and / A E B = / C E D (because they are
vertical angles), we have A A E B = A C E D by SAS. Therefore, / A B E = / C D E , so / A B D = / C D B and we have
‫־‬A B II ‫־‬C D . □

Solution for Problem 3.8: We have to prove that two lines are perpendicular, but we don’t have any information about angles. We
look for congruent triangles since that will give us some angle information. The given side equalities along with the obvious
B D = B D give us A D A B = A D C B by SSS. This doesn’t give us any information about angles at point E, but it does give
us some angle equalities like / A D E = / C D E . This, together with A D = D C and D E = DE, gives us
A A D E = A C D E by SAS. Therefore, / A E D = / C E D . Since these two angles are also supplementary, we have
/ A E D = / C E D = 90°, so‫־‬B D _L AC. □

Exercises

3 .3 .1 : t V

Which two of the triangles below must be congruent and why must they be congruent?

Type your solution, notes and/or work here. Show Solution

In triangle A B C , A B = A C . Let M be the point on B C such that A M bisects / B A C (so that / C A M = / B A M ) .

(a) Show that M is the midpoint of B C .

Type your solution, notes and/or work here. Solution

(b) Show that A M J_ B C .

Type your solution, notes and/or work here. Solution

3 .3 .3 : t V

Find V Z in the diagram below. (Note: The diagram is not to scale.)

Hint

Type your solution, notes and/or work here. Show Solution

In the figure below, A B = C D and / B A C = / D C A . Prove that A D || B C .

Type your solution, notes and/or work here. Show Solution


2.8 Summary

Definitions: Two rays that share an origin form an angle. The common
origin of the rays is the vertex of the angle. We use the
symbol Z to denote an angle, and we use a point on each
side and the vertex, or just the vertex, to identify the angle, o
such as Z X O Y at the right.

We can use a protractor to measure angles (see Section 2.2 here). The semicircular arc of the protractor is divided into 180
degrees, so that a whole circle is 360 degrees.

Definitions: Angles can be classified by their measures.

■ A 90° angle is a right angle. Lines, segments, or rays that form a right angle are
said to be perpendicular.

■ An angle smaller than 90° is an acute angle.

■ An angle between 90° and 180° is an obtuse angle.

■ An angle that measures 180° is a straight angle.

■ An angle of more than 180° is a reflex angle.

Definitions:
■ Two angles whose measures add to 180° are supplementary angles. Angles that
together make up a straight angle form a particularly useful example of
supplementary angles.

■ Two angles whose measures add to 90° are complementary angles.

■ When two lines intersect, they form two pairs of vertical angles, such as Z M P N
and Z L P O below. Vertical angles are equal.

Angles Z L P M and Z N P M together form a straight angle, so they are


supplementary (i.e. add to 180°).

Important: The concepts are more important than the words for solving problems. Angles like
Z M P N and Z L P O above are equal' means something without any more
Z information. ,Vertical angles are equal‫ ׳‬doesn't tell you anything until you reach for
your dictionary to look up vertical angles.

The words, however, are important for communicating the concepts. For now, though, focus on the ideas. The words will come
naturally.

Definitions: Two lines that do not intersect are parallel. A line that cuts across multiple parallel
lines is called a transversal line.

Important: The angles formed when a transversal cuts across two .


‫־‬7 parallel lines come in two groups of four equal angles as *
b
Z. shown: <----- 1t VS------------ >
n x C
a = c = c = g 6>x f
b = d = f = h <— — % ------>

Each of the first set of angles is supplementary to each


of the second set of angles. See Section 2.4 here for all
the special names for pairs of these angles.

Important: The relationships described above when a transversal cuts two lines can also be used
7‫־‬ to show that two lines are parallel.

Definitions: When we connect three points with line segments, we form a ^


triangle. The points are the vertices of the triangle, and the
segments are the sides of the triangle. The angles inside the triangle >B
formed by the sides are the interior angles of a triangle. When we
refer to the angles of a triangle, we mean the interior angles. C

Important: The sum of the angles in a triangle is always 180°.

Definitions: When we extend a side past a vertex of a triangle, we ^


form an exterior angle of the triangle, such as Z X Z P
shown. We call Z X and Z Y the remote interior angles
of exterior angle Z X Z P . /
Y Z P

Important: Any exterior angle of a triangle is equal to the sum of its remote interior angles.

P roblem Solving S trateg ies

Concepts:
When you can't find the answer right away, try finding whatever you can - you
might find something that leads to the answer! Better yet, you might find
something even more interesting than the answer. The best problem solvers are
explorers.

Doing a problem two different ways is an excellent way to check your answer.

Concepts:
Often when we're angle-chasing our goal is to build an equation to solve for one of
the variables in our problem.

Parallel lines are so useful in problems involving angles that sometimes we'll add
new ones to a diagram to help us.

When angle-chasing, it’s best to write the values you find for angles on your
diagram as you find them, even when these values include variables.

The key to tackling word problems in geometry is the same as any other kind of
word problem - turn the words into math. Usually this means defining variables
and using the words to write equations to solve for the variables.

Sometimes using a proof by contradiction is much easier than proving a


statement directly. To prove a statement by contradiction, we start by assuming
the statement is false. Then we show that this assumption leads us to an
impossible statement, which tells us that the assumption itself is false. Having
proved the statement cannot be false, we have shown it must be true.

T h in g s To W atch Out For!

WARNING!!
■ An example is not a proof!
O ■ Your last step should be to make sure you’ve answered the question that is asked.

Proof is at the heart of mathematics. In Section 2.7 here, we saw one of the most common logical errors beginners make.

WARNING!! Suppose we have a true statement of the form

0 If this, then that.

The converse of this statement is

If that, then this.

Even if our original statement is true, the converse doesn’t have to be true. We have to
prove the converse separately.
Review Problems

Using your protractor, determine the following angles in the diagram to the right:

(a) ZAOB

Type your solution, notes and/or work here. Show Solution

(b) ZAOC

Type your solution, notes and/or work here. Show Solution

(c) ZBOC

Type your solution, notes and/or work here. Show Solution

(d) ZDOE

Type your solution, notes and/or work here. Show Solution

How many seconds does it take the second hand of a clock to rotate through an angle of 72°?

Type your solution, notes and/or work here. Show Solution

/‫־‬

Given A B || C D in the diagram, find:

(a) ZCGE

Type your solution, notes and/or work here. Show Solution

(b) ZHGC

Type your solution, notes and/or work here. Show Solution

(c) ZFUB

Type your solution, notes and/or work here. Show Solution

(d) ZB II G

Type your solution, notes and/or work here. Show Solution

Two angles of a triangle are 30° and 70° What is the third angle?

Type your solution, notes and/or work here. Show Solution

The angle Z B in A A B C is 60°. If an exterior angle at A is 170°, what is ZC?

Type your solution, notes and/or work here. Show Solution

Find x in the diagram to the right. A

Type your solution, notes and/or work here. Show Solution

Lines P Q and R S are parallel, and Ÿ Û JL *P(!}. If *T]} intersects W ) at X and h è at Y, find Z R Y X .

Type your solution, notes and/or work here. Show Solution

In the diagram below, the angles are as marked. Find x.

Type your solution, notes and/or work here. Show Solution

The measures of the angles are as marked in the diagram below. Find x.

Type your solution, notes and/or work here. Show Solution

The three angles ZA, ZB, and Z C have the property that Z A is complementary to ZB, Z B is complementary to ZC , and
Z C is complementary to ZA. Prove that Z A = Z B = ZC.

Type your solution, notes and/or work here. Show Solution

Let A A B C have (interior) angles in the ratio 3 : 4 : 5 . What is the measure of its smallest exterior angle?

Hint

Type your solution, notes and/or work here. Show Solution

The exterior angles of a triangle are in the ratio 2 : 3 : 4 . What are the (interior) angles of the triangle?

Type your solution, notes and/or work here. Show Solution

Is it possible for the angles in the diagram to the left to have the measures indicated? Why or why not?

Type your solution, notes and/or work here. Show Solution

Is it possible for two exterior angles of a triangle to be supplementary? Why or why not?

Hint

Type your solution, notes and/or work here. Show Solution

2 .4 0 : Source: MATHCOUNTS t V

Three straight lines intersect at O and Z C O D = Z D O E in the diagram below. The ratio of Z C O B to Z B O F is 7 : 2.
What is the number of degrees in ZCOD?

Type your solution, notes and/or work here. Show Solution

Find w in the diagram below.

Hint

Type your solution, notes and/or work here. Show Solution

One angle of a triangle is 20°. If the largest angle of the triangle has six times the measure of the smallest, what are the angles
of the triangle?

Type your solution, notes and/or work here. Show Solution

The measures of the angles in the diagram below are as marked. Find ZC.

Type your solution, notes and/or work here. Show Solution

The three angles of a triangle have measures Z A = x — 2y ,Z B = 3x + 5y, and Z C = 5x — 3y. Find x

Type your solution, notes and/or work here. Show Solution

It is not possible to find the value of y in the previous problem from the given information. What if you are also told that one
angle of the triangle is 10°? Is it now possible to compute y? What are the possible positive value(s) of y?

Type your solution, notes and/or work here. Show Solution

Show that if a transversal cuts two lines such that the same-side interior angles are supplementary, then the two lines are
parallel.

Type your solution, notes and/or work here. Show Solution


3.4 ASA and AAS Congruence

P roblem s
Problem 3.9 4 Jump to Solution

Shown are three triangles with two angles measuring 60° and 40°, and each triangle has a side of length 3 cm between these
two angles. Measure the other two sides of each triangle Make a guess about triangle congruence given two angles and an
included side.

B h

A D E G

Problem 3.10 4 Jump to Solution

Investigate why the triangles in the previous problem are congruent by constructing a triangle with angles 60° and 40° such
that the side between the two angles has length 3 cm. Start by drawing a segment A B such that A B = 3 cm.

(a) Let the angle between A B and B C be 60°. Where can C be to make / A B C = 60°?

(b) Draw your answer to (a). The angle between A B and A C must be 40°. Where can C be to make / B A C = 40°? (Forget
about / . A B C = 60° for this part.)

(c) Draw your answer to (b). Where can C possibly be located?

(d) Are all triangles with a side of length 3 cm between angles with measures 60° and 40° congruent?

Problem 3.11 4 Jump to Solution

What if the equal sides are not between the equal angles? For example, what if
b
A B = D E — 6, / B = / E = 60°, and / C = / F = 70° as shown? Can we conclude F
that A A B C = A D E F ? Why or why not? 60"
6
\
^ 6 ‫ ^־‬Л
A E

Solution for Problem 3.9: In each triangle, the side opposite the 40° angle has length about 2 cm, and the side opposite the 60°
angle has length around 2.6 cm. The triangles appear to be congruent. □

Problem 3.10 t V

Investigate why the triangles in the previous problem are congruent by constructing a triangle with angles 60° and 40 such that
the side between the two angles has length 3 cm.

Solution for Problem ЗЛО: We start with A B of length 3 cm. We let / A B C = 60°. As in Problem
3.6, this means that point C is on one of two rays from В that make an angle of 60° with A B .
Similarly, we have / В А С = 40°, which means that C is on one of the two rays from A that make
an angle of 40° with A B . Point C must be at one of the two intersections of rays. The two potential
C s, marked C ! and C-i, are mirror images of each other.

Therefore, we find that all the triangles we can build given two angles and the side between them are
congruent. □

C2

Important: We call the principle illustrated in Problem 3.10 the Angle-Side-Angle Congruence
Theorem, or ASA for short. ASA states that

If two angles of one triangle and the side between them are equal to the
corresponding angles and side of another triangle, then the two triangles are
congruent.

For example, in the diagram below, we have A A B C = A D F E .

Problem 3.11 t V

What if the equal sides are not between the equal angles? For example, what if
A B = D E = 6, / B = / E — 60°, and / C = / F = 70°? Can we conclude that
A A B C = A D E l 4'? Why or why not?
ЛГ c 70
f ®

Solution for Problem 3.7 7: If we have two angles of a triangle, we have the third. In the diagram above we can quickly see that
/ D = / A = 50°. Hence, we can conclude that A A B C = A O F F by ASA. □

When we have the situation illustrated above, we don't have to find the third angle and invoke ASA. This procedure always works,
so it gets an unsurprising theorem name of its own.

WARNING!! When using AAS, the equal sides must be adjacent to corresponding equal angles.
For example, the triangles shown below are not congruent because the equal sides
O are not adjacent to corresponding equal angles!

Problem 3.12 t V

In the diagram, A D = BC, A D || BC, and E and F are on A C so that


D C
/ A D E = / C B F . Prove that A B || C D and D F = EB.

V— J
------

A в

Solution for Problem 3.12: We go hunting for equal angles to use to prove that A B and
D
C D are parallel. We start by marking the angles we know are equal. We have a pair in our
original diagram, and can add / D A C = / A C B due to A D || B C , as shown to the left.

Now we can find some congruent triangles. Since A D = B C , / D A C = / B C A , and


A C = A C , we have A C A D = A A C B by SAS. Therefore, / D C A = / B A C , so
A B || C D .

We also have another pair of congruent triangles: A D A F = A B C F by ASA.


D C
It's not immediately clear how this will help us prove B E = D F , so we go ahead
and mark the new equal lengths we know from our new triangle congruence.
Focusing on segments B E and D F , we look for congruent triangles that have
these as sides. We start by looking for triangles we know something about, which
leads us to A A D F and A C B E . We already have a side and an angle, and

AF = AE + EF = CF + E F = CE

gives us a second side. So, A A D F = A C B E by SAS. Finally, we conclude that D F = B E .

We could also have used SAS to show A D F E = A B E F , so D F = B E . □

Concept: In complicated geometry problems, mark side and angle equalities as you find them
I (particularly when you find non-obvious ones!)

Exercises

3 .4 .1 : t V

Find all pairs of triangles below that must be congruent. Write out the appropriate congruence (make sure you have the
vertices in the right order!), and explain why the triangles must be congruent.

D J M

В H

Type your solution, notes and/or work here. Show Solution

In the diagram at right, A B II D C . If M is the midpoint of A C , must it also be the &


midpoint of B D ? Why or why not?

Hint

Type your solution, notes and/or work here. Show Solution

Use ASA Congruence to prove that AAS Congruence is a valid Congruence Theorem. (Do not assume AAS Congruence is a
valid theorem for this part - you are asked here to show that any two triangles that satisfy the AAS criteria are indeed
congruent without using AAS.)

Hint

Type your solution, notes and/or work here. Show Solution

In the figure, P Q = P R and / P Q Y = / P R X . F

(a) Prove that Q Y = R X .

Type your solution, notes and/or work here. Solution

(b) ★ Prove that X N = Y N .

Hint

Hint

Type your solution, notes and/or work here. Show Solution

Extra! Divide the figure at right into four congruent pieces that can
be rearranged to form a square.
V
See the links page mentioned in the Acknowledgements for a
link to the site.
Challenge Problems

2 .4 7 : Source: Mandelbrot t V

Three angles in the diagram to the right are marked as right angles. If (j) = 27°, what is the value of 6?

Type your solution, notes and/or work here. Show Solution

2 .4 8 : Source: MATHCOUNTS t V

What is the number of degrees of the angle formed by the minute and hour hands of a clock at 11:10 PM?

Hint

Hint

Hint

Type your solution, notes and/or work here. Show Solution

Find x in the diagram below.

Hint

Type your solution, notes and/or work here. Show Solution

The angles in the diagram below are as marked. Find / B E A .

Hint

Type your solution, notes and/or work here. Show Solution

One angle of a triangle is equal to the sum of the other two. Show that the sum of two exterior angles of the triangle is 180C
greater than the third.

Hint

Hint

Type your solution, notes and/or work here. Show Solution

One of the angles of the triangle in the previous problem is equal to 40°. What are the measures of the other two angles?

Type your solution, notes and/or work here. Show Solution

The angles of a triangle are in arithmetic progression. If one of the angles is 100°, what are the measures of the other two
angles? (An arithmetic progression is a sequence of numbers in which the difference between each term and its preceding
term is always the same.)

Hint

Type your solution, notes and/or work here. Show Solution

It is possible for the interior angles of a triangle to be in the ratio 1 : 2 : 6 , but is it possible for the exterior angles of a triangle
to be in the ratio 1 : 2 : 6 ? Prove your answer.

Hint

Hint

Type your solution, notes and/or work here. Show Solution

Find / A 4‫ ־‬/ B 4‫ ־‬/ C 4‫ ־‬/ D in the figure below.

Hint

Type your solution, notes and/or work here. Show Solution

Find w 4‫ ־‬x 4‫ ־‬y -F ‫׳‬s in the figure below.

Hint

Hint

Type your solution, notes and/or work here. Show Solution

2 .5 7 ★ : t V

Point Z is on side P R of A P Q R such that Z P Z Q = Z P Q Z , and Z P Q R — Z P R Q = 42°. Find Z R Q Z .

Hint

Hint

Hint

Type your solution, notes and/or work here. Show Solution


Outer Napoleon Triangle

Equality may perhaps be a right, but no power on earth can ever turn it into a fact. - Honoré de Balzac

CHAPTER
I
Congruent Triangles

3.1 Introduction
Two figures are congruent if they are exactly the same - in other words, we can slide, spin, and/or flip one figure so that it is
^ exactly on top of the other figure.

Figure 3.1: Congruent Figures

Figure 3.1 shows four pairs of congruent figures. In each case, we can take one of the figures and slide it, spin it, and/or flip it so
that it exactly coincides with the other.

D Q

Figure 3.2: Congruent Triangles

We use the symbol = to denote that two figures are congruent. For example, to describe the congruent triangles in Figure 3.2, we
write A D E F = A Q P R . Notice that we are careful to put the vertices in the same order: D corresponds to Q, E to P, and F
to R. We would not, for example, write A D E F = A P Q R for the triangles in Figure 3.2.

If we have two congruent triangles, all the corresponding pairs of sides are equal, as are the corresponding angles. Conversely, if
all pairs of corresponding sides of two triangles have equal lengths, and all the corresponding angles of the two triangles are
equal, then the triangles are congruent.

Fortunately, to show that two triangles are congruent, we don't have to go through the hassle of proving each pair of sides and
each pair of corresponding angles are congruent. In this chapter, we'll cover various ways requiring considerably less information
to show that two triangles are congruent.
3.5 SSA Not-Necessarily Congruence

P roblem s
Problem 3.13 4 Jump to Solution

In the diagram below, we have three triangles with two sides of length 2.5 cm and 3 cm, and an angle of 40°. The 40° angle has
the 3 cm segment as a side, but does not have the 2.5 cm segment as a side. Are the triangles congruent?

B
if
< / /
A C D

Problem 3.14 4 Jump to Solution

In this problem we explore why there's no ‘Side-Side-Angle Congruence Theorem’ by building a triangle with A B = 1.8 cm,
B C = 1.5 cm, and /L B A C = 40°.

1. Draw A B = 1.8 cm.


2. We know that B C = 1.5 cm. Draw the points in the diagram that are 1.5 cm from B.
3. Draw the points in the diagram that can be C such that Z B A C = 40°. (Forget B C = 1.5 cm for now.)
4. Use the previous two parts to determine where C can possibly be.
5. Is SSA a valid congruence theorem?

6. Do we ever have a case in which given the lengths of sides A B and BC, and Z B AC, there's only one possible length of
AC?

Solution for Problem 3.13: Urn, no. Clearly / / / is way smaller than E F and B C , so the triangles aren’t congruent. □

SSS works. SAS works. ASA, and AAS, too. But SSA, not so much. Why?

Problem 3.14 t V

Figure out why there’s no ‫׳‬Side-Side-Angle Congruence Theorem’ by building a triangle with A B = 1.8 cm, B C = 1.5 cm, and
Z B A C = 40°.

Solution for Problem 3.14: We start as we have before, drawing A B with length 1.8 cm. Since
B C = 1.5 cm, C must be on a circle with center B and radius 1.5 cm. Also, since Z B A C = 40°,
C must be on a ray from A that makes a 40° angle with A B . The ray and the circle are shown to the
right.

Uh-oh. The ray hits the circle in two points, C ! and C-2. Both A A B C i and A A B C 2 match the
information we are given about A A B C , but they are very obviously not congruent. □

WARNING!! Side-Side-Angle (SSA) is not a valid congruence theorem. You cannot use it to prove
that two triangles are congruent.
O

A
A

You might have noticed that if we try to build a triangle given two sides and a non-included angle as in Problem 3.14, we don't
always get two possible points. The left diagram above shows a case in which we do get exactly one possible C, and the right
figure above shows a case in which no triangles are possible. We'll be revisiting the former case later when we have more tools
to talk about this special case.
3.2 SSS Congruence

P roblem s
Problem 3.1 4 Jump to Solution

In each of the three triangles below, the side lengths are 5, 7, and 8 units. Are the measures of the angles the same in all three
triangles? (Yes, you should use your protractor.)

Problem 3.2 4 Jump to Solution

In this problem we investigate how many different non-congruent triangles have sides with lengths 2.5 cm, 3.5 cm, and 4 cm by
building triangle A B C with these side lengths. Start by drawing segment A B with length 4 cm.

(a) Suppose C is 3.5 cm away from A. What points on your paper are 3.5 cm away from ;4? Draw the figure that consists of
all these points.

(b) We have A B = 4 cm, and our answer to (a) tells us where C must be so that A C = 3.5 cm. We also want C to be 2.5
cm away from B. What points on your paper are 2.5 cm from B l Draw the figure that consists of all these points.

(c) Where in your resulting diagram can C possibly be?

(d) How many different non-congruent triangles have sides with lengths 2.5 cm, 3.5 cm, and 4 cm?

Problem 3.3 4 Jump to Solution

In the figure, the sides and angle have measures as shown. Find Z M L N .

O 17 A/

Problem 3.4 4 Jump to Solution

In this problem we will show that if a radius of a circle bisects (cuts in half) a chord of the circle that is not a diameter, then the
radius is perpendicular to the chord.

(a) Draw a circle with center O and a chord connecting points A and B on the circle.

(b) Draw a radius of the circle that passes through the midpoint, M , of A B , thereby bisecting A B .

(c) Find a pair of congruent triangles and use these triangles to show that Z O M A = Z O M B = 90°.

Often our best way to get started with a new geometric principle is to experiment. So, we start off by examining some triangles
that have the same side lengths and checking if the triangles are indeed congruent.

Solution for Problem 3. V. Using our protractor, we find that the angles of each triangle are approximately 82°, 60°, and 38°. Each
triangle has the largest angle opposite the largest side and the smallest angle opposite the smallest side. Hence, the sides and
the angles appear to be the same in all three triangles, so it looks like they're congruent. □

Let's see if we can get a common-sense explanation for why the triangles of Problem 3.1 are congruent.

Problem 3.2 t V

Investigate how many different non-congruent triangles have sides with lengths 2.5 cm, 3.5 cm, and 4 cm by building triangle
A B C with these side lengths.

Solution for Problem 3.2: It doesn’t matter which side we set to 4 cm, so we just pick A B to be the long side. We start building
A A B C by drawing a 4 cm long segment.

A•--------------------------------•b

Now we have to use our other side lengths to figure out where C can possibly be. Suppose A C = 3.5 cm. Then, C must be 3.5
cm away from A. So, C must be on a circle with center A and radius 3.5 cm. Similarly, since B C = 2.5 cm, C must lie on a
circle with center B and radius 2.5 cm. We draw both circles and end up with the diagram below. Since C must be on both of the
circles, C must be at one of the intersection points. Our two options for C (labeled C ! and C y are merely mirror images of each
other.

If we had instead assumed A C = 2.5 cm and B C = 3.5 cm, we would end up with essentially the same picture, as shown
below.

Notice that if we 'flip‫ ׳‬our first diagram horizontally, we get the second one. In fact, we don't really even need this second diagram.
We could have simply swapped the labels A and B in our first diagram to investigate the possibility that A C = 2.5 cm and
B C = 3.5 cm. □

Concept: Recognizing two seemingly different situations as the same can be a powerful
I simplifying tool in math problems. In geometry, this often involves recognizing
I symmetry, as we did in noting that C ! and C 2 are mirror images of each other in each
diagram above, and in noting that the two diagrams themselves are mirror images of
each other.

Important: We call the principle illustrated in Problems 3.1 and 3.2 the Side-Side-Side
^ Congruence Theorem, or SSS Congruence for short. SSS states:

If the lengths of the sides of one triangle equal the lengths of the
corresponding sides of another triangle, then the triangles are congruent.

For example, in the diagram below, we have A A B C = A X Z Y by SSS


Congruence.

Our ‫׳‬proof that all triangles with side lengths 2.5 cm, 3 cm, and 4 cm are congruent skips over a few very important points. First,
we didn't prove that triangles A A B C i and A A B C 2 are congruent in each of our cases. Nor did we prove that the triangles in
the first case are congruent to the triangles in the second. Intuitively, it seems clear that these triangles are all congruent, but we
haven't proved it. In fact, with the tools we have at this point, we cannot prove SSS Congruence at all!

We must accept as an axiom one of the Congruence Theorems we will study in this chapter. We can then use that one theorem,
and some other tools, to prove the others. Since some of those other tools are considerably more advanced than those we have
now, we’ll stick to our intuitive explanations for all of the Congruence Theorems.

Solution for Problem 3.3: What’s wrong with this:

Bogus Solution: Since L M and N O are parallel, Z M L N = Z L N O = 25°.

y
This Bogus Solution assumes that L M || N O . This might be true, but we have to prove it to use it.

Since L O = M N , O N = L M , and L N = L N , we have A L N O = A N L M by SSS. Since angles Z M L N and Z O N L


are corresponding parts of these two triangles, they must be equal. Therefore,

Z M L N = Z O N L = 25°.

Notice that we can now use Z M L N = Z O N L to prove that L M || N O . □

Important: Problem 3.3 shows us a very important general use of congruent triangles. Once we
determine that two triangles are congruent, we can apply whatever we know about
Z the sides and angles of one triangle to the other triangle. This obvious principle often
goes by the fancy name 'Corresponding Parts of Congruent Triangles are Congruent,'
or CPCTC.

Unless your teacher tells you to, you don’t have to clutter your paper writing CPCTC all over. Once you have proved that
A L N O = A N L M , you can simply write, 'Since A L N O = A N L M , we have Z M L N = Z O N I Z

Let's try using congruent triangles in a proof.

Problem 3.4 t V

Prove that if a radius of a circle bisects (cuts in half) a chord of the circle that is not a diameter, then the radius must be
perpendicular to the chord.

Solution for Problem 3.4: We start with a diagram, drawing circle O, chord A B with midpoint M , and the
radius through M . We then mark the equal pieces of the chord and the equal radii of the circle, and see
that we have congruent triangles. (Radii is the plural of radius.) Specifically, A A M O = A B M O by
SSS. So, ZAM O = ZBM O. Since Z A M O 4‫ ־‬Z B M O = 180°, we must have
Z A M O = Z B M O = 90° □

Important: If a radius of a circle bisects a chord of the circle that is not a diameter, then the
^ radius must be perpendicular to the chord.

As we'll see in Chapter 6, this works in reverse, too! A radius perpendicular to a chord must bisect the chord.

Exercises

3 .2 .1 : t V

E F = G H and F G = E H in the diagram at right, H

(a) Prove that A E F G = A G H E.

Type your solution, notes and/or work here. Show Solution

(b) Show that Z E G F = Z G E H .

Type your solution, notes and/or work here. Show Solution

(c) Show that H E || F G .

Type your solution, notes and/or work here. Show Solution

(d) Show that I I G || E F .

Type your solution, notes and/or work here. Show Solution

In triangle A B C , A B = A C . Let M be the midpoint of side B C .

(a) A segment, line, or ray bisects an angle if it divides it into two angles with equal measure. Show that A M bisects
Z B A C by proving that Z B A M = Z C A M .

Type your solution, notes and/or work here. Show Solution

(b) Show that A M J_ B C .

Type your solution, notes and/or work here. Show Solution

3.2.3: t V

In the diagram at right, A B = 7, A D = 4, C D = 4, and B C = 7. Prove that Z A B D = Z C B D .

Hint

Type your solution, notes and/or work here. Show Solution


3.6 Isosceles and Equilateral Triangles

Problems
Problem 3.15 4 Jump to Solution

In the diagram, A B = A C = 5 and / . C A B = 30°.

(a) Let M be the midpoint of B C . Draw A M . Prove that A A C M = A A B M .

(b) Find / A M B .

(c) Find / A C B .

Problem 3.18 4 Jump to Solution

In the diagram, X Y = X Z = 8 and / X = 60°. Find V Z.

Problem 3.19 4 Jump to Solution

Find / P B D .

Problem 3.15 t V

In the diagram, A B = A C = 5 and / C A B — 30°. Find / А С B. C

Solution for Problem 3.15: We're a little thin on information. We have a side equality that seems C
important. But we only have one triangle, so there's no obvious way to use the side equality. We
might look for congruent triangles, but we'll have to somehow make two triangles. Splitting
A A B C by connecting A to the midpoint, M , of B C gives us our congruent triangles.

Since A B = A C , B M = C M , and A M = A M , we have A A M В 2 A A M C by SSS.


This tells us that / В = / С , which is just enough to finish the problem. Since the angles of
A A B C add up to 180°, we have

180° = Z Д + Z B + Z C = 30° 42 ‫ (־‬/ C ) .

Solving for / C , we have / C = (180° — 30° ) /2 = 75°. □

A triangle in which two sides are equal is called an isosceles triangle. The equal sides are sometimes called the legs of the
triangle, and the other side the base.
Our general approach in Problem 3.15 can be used to show that if two sides of a triangle are equal, then the angles opposite
those sides are equal. These two equal angles are often called the base angles of the triangle, and the other angle the vertex
angle. As we saw in the last problem, if A A B C is isosceles with / B = / C , we have Z B = Z C = (180 — / A ) /2.

We might now wonder if this runs the other way: do equal angles imply equal sides?

Solution for Problem 3.16: What's wrong with this solution?

Bogus Solution: vVe proceed as we did in Problem 3.15, by connecting P to M , the midpoint of QR,
.. as shown below. Since M R = M Q , / R = / Q , and P M = P M , we have
y A P M R ^ A P M Q . Therefore, P R = PQ .

This failed solution uses 'SSA' Congruence, which we have shown in Section 3.5 doesn't work! We have to construct our triangles
in a way that lets us use a valid congruence theorem.

Instead of picking the point on Q R that cuts it into two equal lengths, we make equal angles by
connecting P to point X on Q R such that P X J_ Q R . Now we have A P X R = A P X Q by
AAS. Therefore, P Q = P R , as desired. (Note also that Q X = X R , so that X is the same as
point M in our Bogus Solution. This doesn’t make that Bogus Solution any more correct, though!)

Putting our last two problems together gives us a pair of powerful tools.

We'll try these tools on a couple more problems, and discover another important type of triangle.

Problem 3.17 t V

In the diagram, A C = C D = D B , and Z B = 23°. Find / А . A

Solution for Problem 3.17: Since D C = D B , we have / D C В = / В = 23° We could find / C D B to g e t/ A D C , or we can
note that / A D C is an exterior angle of A B C D , so / A D C = / D C В 4■ / В = 46°. Since A C = D C , we have
/ А = / A D C = 46° □

Problem 3.18 t V

In the diagram, X Y — X Z — 8 and / X = 60°. Find Y Z.


A

rL

Solution for Problem 3.18: We can't do much about Y Z right away, but we can find Z Y and / Z . Since / X = 60°,
/ Y -p / Z = 180° - 60° = 120° Since X Y = X Z . we have / Y = / Z , so / Y = / Z = 60° Hence, all the angles of
A X Y Z are equal. Specifically, Z X = / Y means that Y Z = X Z , so Y Z = 8. □

We call the triangle in Problem 3.18 an equilateral triangle because all of its sides are equal. As Exercises, you will use an
approach a lot like the one above to show:

Important: If all three angles of a triangle are equal, then so are all three sides. Conversely, if all
-7 three sides are equal, then all three angles are 60°.

Therefore, in order to prove a triangle is equilateral, we can either prove all the sides are equal, or prove that all the angles are
equal.

Let's put our isosceles and equilateral triangles to work.

Solution for Problem 3.19: We start by drawing B D so we can see the angle we are after. Since A P A B is
equilateral, / P B A = 60°. A B A D is isosceles, since A B = A D . Since / B A D = 90°, the other two
angles of A B A D equal (180° — 9 0 ° )/2 = 45°. Therefore, we have

/ P B D = / P B A + / A B D = 105°.

Problem 3.20 t V

In the diagram, О is the center of the circle, A B || C D , / A B O \ = 24°, and


\ / ( ) B C = / P C D . Find / В О С . ____ ____ ____ ____

Solution for Problem 3.20: We have parallel lines and an isosceles triangle (O B = O C because they
are radii of the same circle). So, we can do some angle-chasing. We let / O B C = x, so that we also
have / O C D = x from the given information and / O C B = x from isosceles A O C B . Since
A B || C D , we have / A B C 4‫ ־‬/ B C D = 180°. Therefore,

/ A B O 4‫ ־‬/ O B C 4‫ ־‬/ B C O 4‫ ־‬/ O C D = 180°.

Substitution gives 24° 43 ‫־‬x = 180°, so x = 52°. Therefore,/ B O C = 180° — 2x = 76°. □

Concept: Assigning a variable to an angle will often help you with your angle-chasing. If you're
stuck on an angle problem, assign one of the angle measures a variable and find
I other angles in terms of that variable. Hopefully, you'll eventually be able to build an
equation you can use to solve for the variable.

We'll end this chapter with a proof involving equilateral and isosceles triangles.

Problem 3 . 2 1 _________________________ t V

In the diagram at right, A A B C is equilateral and D


I AE = EB = B F = CF = AD = CD

such that A A B C is completely inside A D E F . Prove that A D E F is equilateral.

Solution for Problem 3.21: In order to prove that A D E F is equilateral, we must show that
either all its sides have the same length, or all its angles have the same measure. We start
by marking all the given information. Our most useful tool in showing side lengths are equal
is congruent triangles. Therefore, we look for triangles with D E , E F , and F D as
corresponding sides. The triangles that stand out are A D E A , A E F B , and A F D C . If
we can show these are congruent, we'll have the desired D E = E F = F D .

In order to show that A D E A = A E F B = A F D C , we need only show that


/ D A E = / E B F = / F C D , since we already know that the sides adjacent to these
angles in our three triangles are all equal. To learn more about these angles, we look at the
other angles at A, B, and C.
We already have / C A B = / A B C = / B C A . Furthermore, isosceles triangles A E A B , A F B C , and A D C A are all
congruent by SSS, so their base angles are all equal. Now that we know three of the angles with vertex A equal the
corresponding angles with vertex B, we can show/ D A E = / E B F :

/ D A E = 360° - / E A B - / B A C - / D A C
= 360° - / F B C - / C B A - / E B A
= /E B F .
Similarly, we have/ D A E = / F C D . Therefore, by SAS we have A D E A = A E F B = A F D C , which gives us the desired
D E = D F = E F . So, A D E F is equilateral.

Try finding an alternate solution by showing that all the angles of A D E F are equal, z

We typically prove a triangle equilateral by either proving all its side lengths are equal, or all its angles are equal. This often
involves finding congruent triangles.

Concept: Mark the information you have in a problem on your diagram, particularly equal sides
and equal angles. This will make congruent triangles easier to find.

We've explored isosceles and equilateral triangles, but we're not finished naming triangles yet! We have a name for two sides
equal, and one for all three sides equal. If no two sides of a triangle are equal, the triangle is scalene.

Exercises

t V
3.6.1

In A P Q R , P Q = P R and Z B = 43°. Find ZQ .

Show Solution
Type your solution, notes and/or work here.

O О« t V
W•
U•Æm •

Prove that if A B = A C in A A B C , then / A B C = / A C B . (Note: You cannot simply state that the triangle is isosceles,
so the base angles are equal. You are asked here to prove this fact.)

Show Solution
Type your solution, notes and/or work here.

(a) Prove that if the three sides of a triangle are equal in length, then all three angles of the triangle have measure 60c

Show Solution
Type your solution, notes and/or work here.

(b) Prove that if the three angles of a triangle are equal, then all three sides of the triangle have the same length

Show Solution
Type your solution, notes and/or work here.

Note that for this problem, you are not allowed to state that the triangle is equilateral as your proof. You must prove the facts
about equilateral triangles that you learned in the text.

О Am t V
w •w •

Two angles of an equilateral triangle have measures 3 x 4 2 7 ° ‫ ־‬and 2 y — 4°. Find x 4‫ ־‬y.

Show Solution
Type your solution, notes and/or work here.

Point O is the center of the circle in the diagram below. Find / A O B if / O A B = 70c

Hint

Show Solution
Type your solution, notes and/or work here.

Triangle A B C is equilateral in the diagram below. Points D, E, and F are in triangle A B C such that
/ C A D = / A B E = / B C F . D lies on C F , E lies on A D , and F lies on B E . Prove that triangle D E F is equilateral.

Hint

Hint

Show Solution
Type your solution, notes and/or work here.

t V
3 .6 .7 :

In the diagram at right, V W = V X and W X || Y Z.

(a) Prove that W Y — X Z .

Hint

Hint

Show Solution
Type your solution, notes and/or work here.

(b) Prove that Y X = W Z.

Hint

Show Solution
Type your solution, notes and/or work here.
3.7 Construction: Equilateral Triangle and Perpendicular Bisector
As a reminder, we once more repeat the rules of the game for constructions. The tools of classical construction problems are a
compass and a straightedge. Notice that we don't say ,ruler.‫ ׳‬You don’t get to use your straightedge to measure - you can only
draw lines. Similarly, you aren’t allowed to use your protractor to measure or create angles.

The only operations you can perform with your compass and straightedge are:

1. Given a point, you can draw any line through the point.
2. Given two points, you can draw the line that passes through them both.
3. Given a point, you can draw any circle centered at that point.
4. Given a point and a segment, you can draw the circle with its center at that point and with radius equal in length to the length
of the segment.
5. Given two points, you can draw the circle through one point such that the other point is the center of the circle.

P roblem s
Problem 3.22 4 Jump to Solution

Construct an equilateral triangle with a compass and straightedge.

Problem 3.23 4 Jump to Solution

Draw a segment A B . We call the line that is perpendicular to A B and passes through the midpoint of A B the perpendicular
bisector of A B . Construct the perpendicular bisector of A B .

If you used your protractor to make a 60° angle to construct an equilateral triangle, go back to the beginning of this section and
READ THE RULES!

Problem 3.22 t V

Construct an equilateral triangle.

Solution for Problem 3.22: We start off by drawing a segment A B . We want to find point C
such that A C = B C = A B . Now we have a situation a lot like our SSS 'proof of Problem
3.2. Since A C must equal A B , we know that C must be on the circle with center A and radius
A B . We can construct this circle with our compass. Similarly, C is on the circle with center B
and radius A B . Drawing this circle as well gives us the diagram to the right. Our two circles
meet at C and C . Both triangles A B C and A B C ' are equilateral.

To prove that this construction works, we only have to prove that A C = A B = B C . Since
each of these segments is a radius of a circle that is defined to have radius A B , clearly all
three segments are equal. □

Important: Construction problems are two-part problems. Find the construction, then prove that
it works. The second part is usually pretty easy after you’ve found the construction,
Z but don't forget to do it!

You'll see that in this book we often leave the proof part for the reader. Sometimes we’ll ask you to supply the proof that our
construction works as an Exercise.

Problem 3.23 t V

Given a segment A B , construct a line through the midpoint of A B that is perpendicular to A B . We call this line the
perpendicular bisector of A B .

Solution for Problem 3.23: In our last construction, we found two points that appear to be
directly above and directly below the midpoint of AB. Perhaps connecting these points will
give us the line we want. We start with the same construction we used to make an equilateral
triangle. Then we draw C O , which meets AB at M as shown. By SSS, A C AC' = A C B C ’,
so /.A C C ' = /B C C '. Therefore, /.A C M = / B C M . Since we also have A C = B C
and / C A M = / C B M as we found in the previous problem, we have
A A C M S A B C M by ASA. So, A M = B M and / C M A = / C M B. Since
/ C M A 4‫ ־‬/ C M B = 180°, we must have / C M A = / C M B = 90°. Therefore, C C is
the perpendicular bisector of A B . □

Concept: Always be thinking about what you already know how to do when trying to do
I something new!

Exercises

3 .7 .1 : t V

Given a segment A B , divide the segment into 4 equal pieces with a straightedge and compass.

Type your solution, notes and/or work here.

Is it necessary when constructing the perpendicular bisector of A B to use circles with radius A B ? Suppose we instead draw
two intersecting circles with centers A and B and the same radius. Is the segment connecting the two points where these
circles meet the perpendicular bisector of A B ?

Type your solution, notes and/or work here. Show Solution

Construct a 90° angle.

Hint

Type your solution, notes and/or work here. Show Solution

Construct a 30° angle.

Hint

Type your solution, notes and/or work here.


3.8 Summary

Definition: Two figures are congruent if they are exactly the same - in other words, we can slide,
spin, and/or flip one figure so that it is exactly on top of the other figure.

In this chapter we investigated the following theorems for proving that two triangles are congruent:

■ SSS Congruence: If the sides of one triangle equal the sides of another triangle, then the triangles are congruent. (Section
3.2)

■ SAS Congruence: If two sides of one triangle and the angle between them are equal to those of another triangle, then the
two triangles are congruent. (Section 3.3)

■ ASA Congruence: If two angles of one triangle and the side between them are equal to those of another triangle, then the
two triangles are congruent. (Section 3.4)

■ AAS Congruence: If two angles and a side of one triangle equal the corresponding angles and side in another triangle as
shown below, then the triangles are congruent. (Section 3.4)

Important: Once we determine that two triangles are congruent, we can apply whatever we know
about the sides and angles of one triangle to the other triangle. This obvious principle
Z often goes by the fancy name ,Corresponding Parts of Congruent Triangles are
Congruent,’ or CPCTC.

Definitions: A triangle in which two sides are equal is an isosceles triangle. The
equal sides are the legs of the triangle and the other side is the base.
The angle between the two equal sides is often called the vertex
angle of the triangle, and the other two angles are the base angles,
which are equal to each other.

Important: 1. If A B = A C in A A B C , then Z B = Z C .
2 2. If Z B = Z C in A A B C , then A B = A C

Definition: A triangle in which all three sides are equal is an equilateral triangle. All three angles
of an equilateral triangle are 60°.

Important: If all three angles of a triangle are equal, then so are all three sides. Conversely, if all
three sides of a triangle are equal, then all its angles equal 60°.
Z

Throughout the book we will use congruent triangles to prove many important results. One of the first is:

Concepts:
■ In more complicated geometry problems, mark side and angle equalities as you
find them (particularly when you find non-obvious ones!)

■ Dividing isosceles triangles in half by drawing a segment from the vertex between
the equal sides to the midpoint of the base can be very effective.

■ If you're stuck on an angle problem, assign one of the angle measures a variable
and find other angles in terms of that variable. Hopefully, you'll eventually be able
to build an equation you can use to solve for the variable.

■ Mark the information you have in a problem on your diagram, particularly equal
sides and equal angles. This will make congruent triangles particularly easy to find.

■ Always be thinking about what you already know how to do when trying something
new!

T h in g s To W atch Out For!

WARNING!! SSA (Side-Side-Angle) is not a valid congruence theorem. If two sides of one triangle
are equal to two sides of another, and the two triangles have equal corresponding
O angles that are not the angles between the equal corresponding sides, then the two
triangles are not necessarily congruent!
Review Problems

t V
3 .2 4 :

In each of the diagrams below, name all pairs of congruent triangles you can identify (without drawing more segments or
naming more points). Write the relevant triangle congruences and explain why the triangles are congruent. If there are not any
pairs of triangles in a given diagram that must be congruent, state so.

(a)

/1

Type your solution, notes and/or work here.

(b)
E H

Type your solution, notes and/or work here.

(c)
J M

Type your solution, notes and/or work here.

(d)

Type your solution, notes and/or work here.

(e)

Type your solution, notes and/or work here.

(f)

Type your solution, notes and/or work here.

(g)
B D

Type your solution, notes and/or work here.

(h )

Type your solution, notes and/or work here.

0) K M

Type your solution, notes and/or work here.

Find the vertex angle of an isosceles triangle if one base angle is 6° less than half the vertex angle.

Type your solution, notes and/or work here.

Find x in the diagram shown below.

Type your solution, notes and/or work here.

Find E.A in the diagram below.

Type your solution, notes and/or work here.

Segments A D and C D meet at point P such that A P = C P and D P = D P .

(a) Is it always true that A A P D = A C P D ?

Type your solution, notes and/or work here.

(b) Is it always true that A D || DC?

Flint

Type your solution, notes and/or work here.

Let P Q R be a triangle as shown at right, and let S and T be points on sides P R and QR,
respectively. Triangles PTS, PTQ , and R T S are congruent.

(a) Show that /.P S T = 90°.

Type your solution, notes and/or work here.

(b) Find the measures of the angles of the triangle PQR.

Type your solution, notes and/or work here.

In triangle N O K below, K O = K N . Let / be the point on K N such that O I = O N . Let A be the point on N O such that
IA = IN .

(a) Find three angles in the diagram equal in measure to Z N

Type your solution, notes and/or work here.

(b) Prove that IA || K O .

Type your solution, notes and/or work here.

In the diagram below, O is the center of the circle, OA = C D , and Z A O B = ZO C D .

(a) Show that A D O A = A O C D .

Type your solution, notes and/or work here.

(b) Prove that O C || AD.

Type your solution, notes and/or work here.

(c) Show that Z O C B = 2ZO BA .

Type your solution, notes and/or work here.

(d) Find ZO B A .

Type your solution, notes and/or work here.

In the diagram below, A D = B D = A E and D E = EC. Prove that A C = BE.

Hint

Type your solution, notes and/or work here.

In the figure below, Z B O C = 42°, O is the center of the circle, C D || OA, and Z B AO = 2 /.B C D .

C O

(a) Find A C DO.

Type your solution, notes and/or work here.

(b) Show that Z B O A = 180° — 4 /.B C D .

Type your solution, notes and/or work here.

(c) F in d Z B O A

Hint

Type your solution, notes and/or work here.

A B and X Y are diameters of ©O . Prove that A X = B Y .

Type your solution, notes and/or work here.

In triangle ABC, let M be the midpoint of D C Prove that if M A = M B = M C, then A B A C — 90£

Hint

Type your solution, notes and/or work here. Show Solution

All four angles of a square are 90° and all four sides have the same length. Square E F G H is inscribed in E B
square A B C D as shown at right, meaning that each vertex of E F G H is on a side of A B C D

(a) Show that Z A H E = Z B E F .

Type your solution, notes and/or work here. Show Solution

(b) Prove that triangles A E H , D F E , C G F , and D B G are congruent.

Type your solution, notes and/or work here. Show Solution

In the figure below, A D E F is equilateral, A E = B F = CD, and A D = B E = C F . A A B C is completely inside


A DEF, and no two of the triangles ADE, BEF, and C F D share any points besides the vertices of A D EF.

(a) Prove that A A B C is equilateral.

Hint

Type your solution, notes and/or work here. Show Solution

(b) Would A A B C still necessarily be equilateral if A A B C were not fully inside A D E F I

Type your solution, notes and/or work here. Show Solution

In A B C D E F below, A B = F A , D C = E F . C D = D E, and Z C D A = Z A D E . Prove that Z A D C = Z E F A .

Hint

Hint

Type your solution, notes and/or work here. Show Solution

N P in the diagram bisects both Z M N O and Z M P O . Which of the following statements must be
true? For each statement that must be true, prove that it must be true. For each statement that may not
be true, explain why it may not be true.

(a) N O = M N.

Type your solution, notes and/or work here, Show Solution

(b) N O = OP.

Type your solution, notes and/or work here. Show Solution

(c) M P II N O .

Type your solution, notes and/or work here. Show Solution

(d )*A Z O X N P .

Type your solution, notes and/or work here. Show Solution

t V
3 .4 0 :

In the diagram at right, A is the center of the circle and C D X AD. We are given that
Z A C D = 2Z D C D .
(a) Let Z D C D = x. Find expressions for Z A B C and Z B A C in terms of x.

Type your solution, notes and/or work here, Show Solution

(b) Find Z A B C .

Type your solution, notes and/or work here. Show Solution


5.4 SSS Similarity
We use SSS Similarity less often than AA and SAS.

As we noted, few problems require SSS Similarity. We may, however, consider it in problems in which all we are given is lengths,
but we have to prove something about angles.

Problem 5.11 t V

Given the side lengths shown in the diagram, prove that A E | B C and A B \
DE ‫״‬ X y

/ I / ‫־‬
12/ C 4
3 D
/ 1 0

Solution for Problem 5.11: We need to use angles to show the segments are parallel, but all we have are sides. We look for
similarity, and see that

AB AC BC 1
D E ~ AD ~ A E 2 ‫’ ־‬
so A A B C ~ A D E A by SSS Similarity. Therefore, Z B A C = Z E D A, so A B || D E . Also, Z D A E = Z A C B , so
A E || BC. □

Exercises

5 .4 .1 : t V

Two isosceles triangles have the same ratio of leg length to base length. Prove that the vertex angles of the two triangles are
equal.

Hint

Type your solution, notes and/or work here. Show Solution


Challenge Problems

The measures of two angles of an isosceles triangle are 3x 4 4 ° ‫ ־‬and x 4 1 7 ° ‫־‬. Find all possible values of x.

Hint

Type your solution, notes and/or work here. Show Solution

Triangle A B C is equilateral in the diagram below, and A B D E , B C F G , and C A I I I are squares. Prove that triangle D F H
is equilateral.

Hint

Hint

Type your solution, notes and/or work here. Show Solution

Let A B C D be a rectangle, meaning that all four of its angles are right angles. Construct equilateral triangles A B E and
B C F externally on sides A B and B C as shown below. Prove that triangle D E F is equilateral.

Hint

Hint

Hint

Type your solution, notes and/or work here. Show Solution

3 .4 4 : t V

A A B C has a right angle at Z C . Points D and E are on A B as shown below such that A D = A C and B E = B C . Find
ZDCE.

Hint

Hint

Type your solution, notes and/or work here. Show Solution

3 .4 5 ★ : t V

Let A B C D be a square, and let E, F be points such that D A = D E = D F = D C and Z A D E = Z E D F = Z F D C .


Prove that triangle B E F is equilateral.

Hint

Hint

Hint

Type your solution, notes and/or work here. Show Solution

3 .4 6 : t V

In triangle A B C in the diagram below, D and E are points on side A B , and F and G are points on side A C , such that

A D = DG = G B = BC = C E = E F = FA.

Find Z B A C .

B E D

Hint

Type your solution, notes and/or work here. Show Solution

In the diagram below, Z D Z E = Z E Z Y , Z C Y E = Z A Y E , and E D || Y Z . Prove C D = D Z - C Y .

Hint

Type your solution, notes and/or work here. Show Solution

Describe under what conditions SSA Congruence ‫׳‬works'. In other words, when is it true that if A B = X Y and B C = Y Z
and Z B A C = Z Y X Z , that we can immediately conclude that A A B C = A X Y Z? (No proof is necessary; give a clear
description of when SSA works and an intuitive explanation why it works in these cases.)

Hint

Type your solution, notes and/or work here. Show Solution

In the diagram below, Z X Y Z = Z A X Z = 90°, A X = X Y , and Z X Z D = Z Y Z D . Prove that Z D A. A Y .

Hint

Hint

Type your solution, notes and/or work here. Show Solution

Draw equilateral triangles B C F , C A Q , and A B R outside A A B C as shown in the diagram below. Prove that
A P = B Q = CR.

Hint

Type your solution, notes and/or work here. Show Solution

3 .5 1 ★ : Source: Mandelbrot t V

There are two possible triangles with A B = 13, B C = 10, and Z A = 40°. What is the sum of the two possible values of
Z IP .

Hint

Hint

Hint

Type your solution, notes and/or work here. Show Solution

Extra! Take a look at the nearest globe (or imagine one). A woman at the North Pole, looking for the shortest
path to Quito, Ecuador, just barely south of the equator, would fly straight down the 78° West line of
V longitude. Then, to fly to Bifoun, Gabon (which is a quarter of the way around the world to the East), she
turns 90° to face East, and fly straight along the Equator. She then returns to the North Pole by turning
90° North and flying along another line of longitude, thus completing a triangular trip. She could begin
repeating her trip by turning 90° to the left and then flying to Quito again.

North Pole

At each stop, you can see that she must turn 90° to head towards the next stop. But that’s a total of 270c
in this triangle on the surface of the globe! How can that be?

Research spherical geometry to answer this mystery.


The Pizza Theorem

There is no transfer into another kind, like the transfer from length to area and from area to a solid. - Aristotle

CHAPTER
I
Perimeter and Area

4.1 Perimeter
The perimeter of a closed figure is how far you travel if you walk along its boundary all the way around it once. So, the perimeter
of a triangle is simply the sum of the lengths of the sides of the triangle.

P roblem s
Problem 4.1 4 Jump to Solution

A A B C is equilateral. Find A B given that the perimeter of A A B C is 36.

Problem 4.4 4 Jump to Solution

The length of each leg of an isosceles triangle is three times the length of the base of the triangle. The perimeter of the triangle
is 91. What is the length of the base of the triangle?

Questions about perimeter are usually just questions about segment lengths. Well try a few.

Problem 4.1 t V

A A B C is equilateral. Find A B given that the perimeter of A A B C is 36.

Solution for Problem 4.1: Since A A B C is equilateral, all of its sides are equal. Let each have length s. Then the perimeter is 3«.
Therefore, we have 3.s = 36, so s = 12. □

Problem 4.2 t V

Find the perimeter of A D E F.


U
8
E 30 30^

Solution for Problem 4.2: Since Z E = ZF, we have D E = D F = 8. Therefore, our perimeter is
D E 4- D F 4- E F = 16 43 /\8 ‫־‬. □

Problem 4.3 t V

Given that each little square in the grid is 1 x 1 square, find the perimeter of
A B C D E F G H IJ .

Solution for Problem 4.3: We could simply find the lengths of the 10 sides and add up all the numbers, but there's a slicker
approach. All of our sides are either vertical or horizontal. The horizontal side on the bottom has length 8; therefore, the sum of
the horizontal sides on top, which together cover the same horizontal distance without backtracking, must be 8. Similarly,
A B 4‫ ־‬C D = 5, so the vertical sides on the left have total length 5. The ones on the right have the same total length, so they
contribute 5 to the perimeter as well. So, our perimeter is 2(8 4- 5) = 26. □

Problem 4.4 t V

The length of each leg of an isosceles triangle is three times the length of the base of the triangle. The perimeter of the triangle
is 91. What is the length of the base of the triangle?

Solution for Problem 4.4: Let the base of the triangle have length x. Then the length of each leg is 3.x. Since the perimeter of the
triangle is 91, we must have x 43 ‫־‬x 43 ‫־‬x = 91. Solving this equation for x, we find x = 13. The length of the base of the
triangle is 13. □

Concept: The key to solving most geometric word problems is the same as for most non-
geometric word problems: assign variables to unknown quantities and use the words
I to make equations you can solve.

Exercises

4 .1.1: t V

In the diagram below, X Y = 5\/2, Y Z = 10, and Z Y = Z Z = 45°. Find the perimeter of A X Y Z .

Type your solution, notes and/or work here. Show Solution

4 .1.2: Source: AMC 8 t V

Triangles ABC, A D E, and E F G below are all equilateral. Points D and G are midpoints of AC and AE, respectively. If
A B = 4, what is the perimeter of figure A B C D E F G ?

Type your solution, notes and/or work here. Show Solution

4 .1.3: t V

(a) Must two congruent triangles have the same perimeter? Why or why not?

Type your solution, notes and/or work here. Show Solution

(b) Must two triangles with the same perimeter be congruent? Why or why not?

Type your solution, notes and/or work here. Show Solution

(c) ★ Must two triangles with the same perimeter be congruent if the triangles have one angle measure in common?

Type your solution, notes and/or work here. Show Solution

4 .1.4: t V

Each little square in the grid at right is a 1 x 1 square. Find the perimeter of B C
ABCDEFGH.
F

D t
H

Type your solution, notes and/or work here. Show Solution

4 .1.5: t V

A triangle with perimeter 45 has one side that is twice as long as the shortest side and another side that is 50% longer than
the shortest side. Find the length of the shortest side of the triangle.

Hint

Type your solution, notes and/or work here. Show Solution


Review Problems

Find the area of each figure below,

(a)

Type your solution, notes and/or work here. Show Solution

(b)

Type your solution, notes and/or work here. Show Solution

(c)
H 12 /
□------ ------ n

□______ ______ c
J

Type your solution, notes and/or work here. Show Solution

(d )

Type your solution, notes and/or work here. Show Solution

(e )
O

Type your solution, notes and/or work here. Show Solution

A triangle has area 42 and one side of length 7. Find the length of the altitude to the side of length 7. Is it possible to find the
perimeter of this triangle?

Type your solution, notes and/or work here. Show Solution

Find the perimeter of a square that has an area of 75.

Type your solution, notes and/or work here. Show Solution

Find the perimeters of A A B C and A X Y Z shown below given that [ABC] = 24 and [ X Y Z ] = 30.

Type your solution, notes and/or work here. Show Solution

The length of a given rectangle is 2 less than 4 times the width of the rectangle. Its perimeter is 51. What is its area?

Type your solution, notes and/or work here. Show Solution

I have a rectangular living room that is 24 feet long and 16 feet wide. I want to buy a carpet that costs $2.50 per square foot,
and I want to leave a 2 foot margin between the carpet and the wall all the way around the room. How much will the carpet
cost?

Type your solution, notes and/or work here. Show Solution

Type your solution, notes and/or work here. Show Solution

A rectangle has a perimeter of 28 cm and an area of 48 cm2. Find the dimensions of the rectangle.

Type your solution, notes and/or work here. Show Solution

If the height of a triangle is multiplied by 4, what must we do to the base of the triangle in order to leave the area unchanged?

Type your solution, notes and/or work here. Show Solution

The altitude from A to B C in A /4 /iC h a s the same length as the altitude from B to AC. Prove that B C = A C

Type your solution, notes and/or work here. Show Solution

The area of rectangle A B C D shown below is 36 and D E = 2EC.

(a) What is [BCD]?

Type your solution, notes and/or work here. Show Solution

(b) What is [BED]?

Type your solution, notes and/or work here. Show Solution

Each of the outer triangles in the figure shown below has perimeter 23. The perimeter of B D F H J is 51. What is the
perimeter of star A B C D E F G H I J ?

Hint

Type your solution, notes and/or work here. Show Solution

4 .25: Source: MATHCOUNTS t V

In the figure below, A B = 12 cm and B C = A D = 8cm. B C _L A B and D A A. AB. How many square centimeters are
in the area of the shaded region?

Li
Type your solution, notes and/or work here. Show Solution

In the figure below, [WOZ] = 12, [ZOY] = 18, and [ W X Y Z ] = 50. Find [ W O X \

Type your solution, notes and/or work here. Show Solution


4.2 Area

Figure 4.1 :A Square c. ‫ ״ ״‬A‫״‬


Figure 4.2: A Rectangle

‫״ “ » ״‬
‫ ״‬,,,‫ ״ ״ ״‬w, , ,‫«״‬. ‫ ״‬JS , ,:‫״‬r ,” C ‫ ״ » " ; ׳‬S ‫״‬r
opposite ,ides o f, « ,‫^ ״ ״‬ ‫>•'• ■^ ״ ״‬r »- - ‫־ * ׳‬

= s r r ~ : : ; ; r ; = . - D 1
1
need 3 - squares to cover the région A B C D E F at right. so we say ,he area of A B C D E F is 3±. We 2

2
will often use brackets to denote area, like this: [A B C D E F ] = 3 - B C

P roblem s
Problem 4.5
4 Jump to Solution
Consider rectangle ABC D , which has A B = C D = 4 B C = A D
and D. What is the area of ABCD? = 5, and right angles at A, B, C,

Problem 4.6
4 Jump to Solution

S r e c Î a n g t e 6 * * ^ 8 reC,an9'e iS 4 less than 3 times an adjacent side. The perimeter of the rectangle is 64. Find the area

Problem 4.7

A triangle that has a right angle as one of its angles is called a right triangle.

(a) Find the area of right triangle A A B C , where A B = 4, B C = 6, and A B = 90°.

(b) Find a formula for the area of a right triangle given the lengths of its sides.

Problem 4.8

(a) Find the area of A D E F given that./ is on E F , D .l = 8, ‫־‬D J A EF, and E F = 7.

‫”״‬ r ,L “‫“ ״ ־‬ ’ “ “ ‫• ״ »״‬ ‫■ * ״ ״ ׳״• ׳״‬ ‫״־ > ־ ״ ״‬

Problem 4.9

Given that / Z Z K = A X W Y = 90°, * K = 8, * Z = 6. and Z F = 10, find X W .

Problem 4.5
t V
Consider rectangle ABCD. which has A B = C D = 4 B C - A D - h r• - .
and D. What is the area of A B C D l ' ~ A D ~ and right angles at A, B. C.
C

right. So, the areaT^(4)(5)W= 2a □" A B ° D W‘th f° Ur r0WS of flve 1 x 1 sc*uares each‫ ׳‬as shown to the

Finding the area of a rectangle by taking the product of consecutive sides works even when the sides aren't integers.

Important: A rectangle with two adjacent sides of lengths t and w has area tw. Since a square is
Z just rectangle in which these lengths are the same, the area of a square equals the
square of the length of one of its sides.

Let s apply our knowledge of rectangles to a problem.

Problem 4.6
t V

of №e rectangle16 ^ * 8 reCtan9'e * 4 le#S 3 ‫ ״ “ ״‬timeS a" ■ * « ‫ « ״‬sida‫־‬ perimeter of the rectangle is 64. Find the area

“ « S t° :r ,ï.r “ 1- “ 1‫״ * ״‬ ‫״‬ « ‫ ־ * ״‬% - * — » « .‫ ־‬. « ‫ ־‬. ‫־‬

2‘r + % = 2(3 y - 4) -F 2y = 8y - 8.
We are given that the perimeter is 64. so we have 8 y - 8 = 64 Therefore w = QanH r - _ oq a

rectangle is 9(23) = 207. □ >V v and x — 3(9) - 4 — 23, so the area of the

Now let's use our rectangle formula to find the area of a triangle. We'll start with a special kind of triangle.

Problem 4.7
t V
A triangle that has a right angle as one of its angles is called a right triangle.

(a) Find the area of right triangle A A B C , where A B = 4, BC = (i, and A B = 90°.

(b) Find a formula for the area of a right triangle given the lengths of its sides.

Solution for Problem 4.7:

- p‫ ״‬p .p . m„ k. . S T S : “ 0‫ ״‬Bl™ ‫״‬ h“ • ‫* ״ ״‬

1/ D 6 ‫ ״‬A»ram' A A C D is a c°py of A CAB. Since A B A C + A B + A B C A - 180° ‫־‬,‫״‬h

fr!an‫ ״‬l4‫״‬S C ‫׳‬D iS 8 rectangle Wi,h SideS 4 and 6' we know that [A B C D ] = 24 Our two right
24/2 = 12 C° n9rUem 8nd ^ SUm ° f th6ir areaS iS [A B C D \ *>‫ ־‬each ri9ht triangle has area

<b> f AABf w ‫״ ״ • ״ ״ “ » ■ ■* “ ־ • " " ״ ־‬


sides of lengths a and b out of two congruent copies of AAB*C ThpS ' TT bmld 3 rectan9'e with consecutive
_ trian9'eS h8S 06/ 2 "‫״‬ S° ■ « ‫־‬ a"* triangle is half the p r o d ^ S i ^ Z ^ ‫“״‬

So that takes care of right triangles. But what if the triangle isn't a right triangle?

Problem 4.8
t V
(a) Find the area of A D E F given that./ is on E F , D J = 8,‫־‬D J ± ‫־‬E F, and E F = 7
(b) Find a method to determine the area of any triangle.

E J

Solution for Problem 4.8:

(a) We do know how to tackle right triangles, so we break A D E F into A DE.J and A D .)F :

[D E F ] = [D E J\ + [D F J ]
_ (E J )(D J ) . (J F )(D J )
2 + 2
DJ
= — (E J + J F )

= ~ {E F )

‫(־‬I)
= 28
m

We have already shown that if our base is either leg of a right triangle, then the area is

base X altitude

J e T afT Pr° blem C8n be USed t0 Sh0W ,hat ,hiS f0rmula - rka wh-e v e r the two angles at the base

But what if one of the angles at the base is obtuse? In the diagram to the right, let K Z be the

draw ouTalmudTto Z ™ l 'lne X th8t '* perpendicular t0 ‫ ״‬e extend side F z and
[ X Y Z ] = T x P ) ( Y % T S'0n ^ °n the n6Xt P89e- N° WWe need t0 check whether

In our first part we added areas; here, we subtract:

(XYZ) = [ X P Z ] - [ XPY]
= (.X P ) ( P Z ) (XP)(PY)
2 2
X P (P Z-P Y )
2
= (■ y P )(K Z )
2
We have to tweak our definition of 'altitude' a little, but our formula still works.

We can now summarize our findings about the area of a triangle.

Important: To find the area of a triangle, we select one side to be the base.
z The perpendicular segment from the vertex opposite the base
to the base (extended if necessary) is the altitude We then
have:

base x altitude
A rea = p z

For example, in the triangle shown, we have:

[PQR\ = - Q * R Z .

Problem 4.9
t V
Given that Z Z Z K = Z X IT 'K = 90°. X Y = 8, X Z = 6, and Z F = 10. find X W .

E Ä S C S * ngl‫ ־ ‘ ״ ־ ׳ ״‬d‫ ־■״ “ ״‬- - ‫■ ״ ״׳‬ ‫■״״־״‬

[ Z F Z J = i ^ Y ) ( x z l = 24

S in c e ^ = 24■ - - « «h. e q u a tio n

S p rin k le d th ro u g h o u t th is b o o k w ill be m a n y o th e r u se s o f area as a p ro b le m s o lv in g to o l.

Exercises

Find the area of each figure shown below,


(a)

Type your solution, notes and/or work here.


Show Solution

Type your solution, notes and/or work here.


Show Solution

Type your solution, notes and/or work here.


Show Solution

What is the area of square A B C D if the perimeter of the square is 36?

Type your solution, notes and/or work here.


Show Solution

In the diagram at right, K O = G, LA I = 7, and K M = 12.


(a) Find [K L M ].

Type your solution, notes and/or work here.


Show Solution

(b) Find LN.

Hint

Type your solution, notes and/or work here.


Show Solution

: ,s r 8 rfeetrhigh?
24 feet long and . “ '‫» *״״״־־־‬ ‫ ׳‬- * ‫׳‬ ‫׳ » * * ״‬- « . ‫ ־‬.‫ ־‬P. ‫־ » ״‬

Type your solution, notes and/or work here.


Show Solution

Source: MATHCOUNTS t V

Hint

Type your solution, notes and/or work here.


Show Solution

Source: MATHCOUNTS t V

The perimeter of a square garden is 64 meters. The path surrounding the garden has uniform width

outer edgTof the p a th f H° W m8ny ° f fenCin9 are needed «‫ ״‬round the

Hint

Type your solution, notes and/or work here.


Show Solution
4.3 Same Base/Same Altitude

P roblem s
Problem 4.10 4 Jump to Solution

Given that B C = 40 and C D = 20 in the figure, what are [ A B C ] / [ A C D ] and


A
[A B C ]/[A B D ]?

D 20 C 40 É

Problem 4.11 4 Jump to Solution

Suba and Sam have been hired to paint a triangle with base A B on the weirdly shaped wall in c
the diagram. They are told to choose either C or D as the third vertex of the triangle. Suba J L
thinks using D will make the triangle look cooler, but Sam thinks using C will result in a smaller D
j
triangle to paint. How can Suba convince Sam to choose D l
12
9

n_ ■
E /I В

In this section we examine a particularly useful triangle area concept.

Problem 4.10 t V

Given that B C = 40 and C D = 20 in the figure, what are [ A B C ] / [ A C D ] and


[A B C \/[A B D ]>

D 20 C 40

Solution for Problem 4.10: The altitude from A is the same for both A A C D and A A B C . Let h be the length of this altitude.
Hence, [ A B C ] = ( B C ) ( h ) / 2 = 20h and [ A C D ] = { C D ) ( h ) / 2 = 10h. Therefore, [ A B C ] / [ A C D ] = 2 0 h /1 0 h = 2.
Notice that [ A B C ] / [ A C D ] = ( B C ) / ( C D ) .

Similarly, the altitude from A in each of A A B C and A A B D has length h. Therefore,

[A B C ] (B C )(h )/2 BC 2
[A B D ] ‫־‬ (B D )(h )/2 ‫־‬ BD 3 ‫' ־‬

A key step in our solution is noting that the three triangles have an altitude in common. Notice that in each case the ratio of the
areas of the triangles equals the ratio of the lengths of the sides to which this common altitude is drawn. □

Problem 4.11 t V

Suba and Sam have been hired to paint a triangle with base A B on the weirdly shaped wall in C
the diagram. They are told to choose either C or D as the third vertex of the triangle. Suba thinks J L
using D will make the triangle look cooler, but Sam thinks using C will result in a smaller triangle D
j
to paint. How can Suba convince Sam to choose D l
12
9

~i ■
E A В

Solution for Problem 4.11: To figure out which of A A B C or A A В /Twill use more paint, we must consider the areas of the two
triangles. Since

{A B )(D E ) tA n ^ (A B )(B C )
[ A B D ] = -------^ ------ - and [A B C ] = ------- ^ ------ ‫ ־‬,

we see that

[A B D ] (A B )(D E )/2 DE 3
[A B C ] ‫־‬ (A B )(B C )/2 ‫־‬ BC ‫ ־‬4‫־‬

So, Suba should point out that A A B D will require less paint because its area is smaller than that of A A B C . z

The last two problems illustrate two useful problem solving principles:

Important:
1. If two triangles share an altitude, then the ratio of their areas is the ratio of the
Z bases to which that altitude is drawn. This is particularly useful in problems in
which two triangles have bases along the same line.
2. If two triangles share a base, then the ratio of their areas is the ratio of the altitudes
to that base.

Solution for Problem 4.12: Since A A B X and A C B X share an altitude from B, we have

AX [A B X ] 8
CX [C B X ] 5‫־‬

Turning to triangles A A D X and A C D X , we have

[A D X ] AX 8
[C D X ] “ CX 5 ‫* ־‬

Therefore, [ A D X ] = (8 /5 ) [ C D X ] = 16. □

Exercises

4 .3 .1 : t V

In the diagram below, P T = (>, T R = 3, and Q V = 4.

(a) Find [ P Q R \ [ P T Q \ and [ Q T R \

Type your solution, notes and/or work here. Show Solution

(b) Find T R / P T and [ Q T R ] / [ P T Q \

Type your solution, notes and/or work here. Show Solution

In the diagram below, A B = 2 and B E = 3.

(a) Find [B C D ]/[A C D ].

Type your solution, notes and/or work here. Show Solution

(b) Find [ A D B C ] / [ A D C \

Hint

Type your solution, notes and/or work here. Show Solution

(c) Find [ B C D ] / [ A C B D \

Hint

Type your solution, notes and/or work here. Show Solution

Euclid Apartment Building has a wall that is 30 feet long and 10 feet high. Jean is supposed to paint a triangle with one vertex
at the top of the wall, and a base that runs 8 feet along the bottom of the wall. Jean wants to put the vertex at one corner and
the base at the other side of the wall as shown on the left. George, the building owner, wants to save money on paint. He
insists that the top vertex be right above the middle of the base, as shown on the right, so the triangle won't be so big.
However, to show that he’s a nice guy, he says that she can make the base 10 feet wide instead. Jean argues that George's
design will use more paint than hers. Is she right?

Type your solution, notes and/or work here. Show Solution

Source: MATHCOUNTS t V

By what factor is the area of triangle multiplied if the length of its base is doubled and the height is tripled?

Type your solution, notes and/or work here. Show Solution

4 .3 .5 ★ : t V

Find Q X in the diagram at right given that W X = 8 and [ P Q X ] = [ \ V X Y Z ] / 6 . z q /


P
Hint
иА dX
Hint

Type your solution, notes and/or work here. Show Solution

Extra! Can you cut the figure at right into two congruent pieces that
can be arranged to form a rectangle?
V
Challenge Problems

ABCD
F in d t h e a r e a o f s h o w n a t r ig h t.
Bp
H in t
6

T y p e y o u r s o lu tio n , n o te s a n d / o r w o r k h e re . Show Solution

4 .2 8 : Source: MATHCOUNTS t V

A g a r d e n e r p la n s t o b u ild a f e n c e t o e n c lo s e a s q u a r e g a r d e n p lo t . T h e p e r im e te r o f t h e p lo t is 9 6 fe e t, a n d h e s e ts p o s t s a t t h e
c o r n e r s o f t h e s q u a r e . T h e p o s t s a lo n g t h e s id e s a r e s e t 6 f e e t a p a r t . H o w m a n y p o s t s w ill h e u s e t o f e n c e t h e e n t ir e p lo t?

T y p e y o u r s o lu tio n , n o te s a n d / o r w o r k h e re Show S olution

W e b e g in w it h a n e q u ila t e r a l t r ia n g le . W e d iv id e e a c h s id e in t o t h r e e s e g m e n t s o f e q u a l le n g t h , a n d a d d a n e q u ila t e r a l t r ia n g le
t o e a c h s id e u s in g t h e m id d le t h ir d a s a b a s e . W e t h e n r e p e a t t h is , t o g e t a t h ir d f ig u r e .

G iv e n t h a t t h e p e r im e t e r o f t h e f i r s t f ig u r e is 1 2 , w h a t is t h e p e r im e t e r o f t h e s e c o n d f ig u r e ? W h a t is t h e p e r im e t e r o f t h e t h ir d
fig u r e ?

H in t

H in t

T y p e y o u r s o lu tio n , n o te s a n d / o r w o r k h e re . Show S olution

/e r. W h a t is t h e p e r im e t e r o f t h e r e s u lt in g f ig u r e ?

T y p e y o u r s o lu tio n , n o te s a n d / o r w o r k h e re Show S olution

4 .3 1 Source: Mandelbrot t V

A ll s id e s o f t h e b u ild in g s h o w n a t r ig h t m e e t a t r ig h t a n g le s . I f t h r e e o f t h e s id e s m e a s u r e 2
m e t e r s . 7 m e t e r s , a n d 1 1 m e t e r s a s s h o w n , t h e n w h a t i s t h e p e r i m e t e r o f t h e b u i l d i n g in
m e te rs ?

a
11

T y p e y o u r s o lu tio n , n o te s a n d / o r w o r k h e re . Show Solution

In A . 4 BC b e lo w , CX = 2BX and AY = 3 B Y . F in d \BXY\ g i v e n th a t [ABC] = 1 4 4.

H in t

H in t

T y p e y o u r s o lu tio n , n o te s a n d / o r w o r k h e re . Show Solution

4 .3 3 : Source: ARML t V

T h e d ia g r a m b e lo w is f o r m e d b y p la c in g A lin e a s s h o w n . E a c h s q u a r e h a s a s id e le n g t h t h a t is
1 / 2 t h e s id e le n g t h o f t h e n e x t la r g e r s q u < ju r e is 1 1 5 . W h a t is t h e a r e a o f t h e w h o le fig u r e ?

H in t

T y p e y o u r s o lu tio n , n o te s a n d / o r w o r k h e re . Show S olution

S u e a n d B a r r y a re t r y in g t o f in d t h e a re a o f AAI3C. S u e m is ta k e n ly u s e s AB a n d t h e h e ig h t f r o m A ( in s te a d o f t h e h e ig h t
fro m C), a n d B a r r y m is ta k e n ly u s e s BC a n d th e h e ig h t f r o m C ( in s te a d o f t h e h e ig h t fr o m A).
(a ) S u e f in d s a n a r e a o f 1 2 a n d B a r r y f in d s a n a r e a o f 2 7 . W h a t is t h e a r e a o f A . 4 BC?
H in t

H in t

T y p e y o u r s o lu tio n , n o te s a n d / o r w o r k h e re . Show S olution

(b ) ★ S u p p o s e in s te a d t h a t S u e f in d s a n a r e a o f 1 2 0 a n d B a r r y f in d s a n a re a o f 1 5 0 . N o w w h a t is t h e a re a o f A ABC?
H in t

T y p e y o u r s o lu tio n , n o te s a n d / o r w o r k h e re . Show Solution

4 .3 5 : t V

P o in t P is o n WY in t h e d ia g r a m b e lo w . S h o w t h a t [W PX] = [WPZ].

\Af
vv

7‫־‬
Z / \ 1/

H in t

H in t

T y p e y o u r s o lu tio n , n o te s a n d / o r w o r k h e re .

4 .3 6 : Source: Mandelbrot t V

In t h e d ia g r a m b e lo w , t r ia n g le ABC h a s v e rte x A a t t h e o r ig in , v e r te x B a t th e p o in t ( 2 . 0 ) , a n d v e r te x C o n t h e c ir c le w it h
c e n t e r ( 3 , 2 ) a n d r a d iu s 1 . W h a t is t h e m a x im u m p o s s ib le a r e a f o r s u c h a t r ia n g le ?

• (3 -2 ))

/1 (0 ,0 ) £ R C f)

H in t

H in t

T y p e y o u r s o lu tio n , n o te s a n d / o r w o r k h e re .

4 .3 7 ★ : t V

A D , B E , a n d C F m e e t a t X a s s h o w n a t r ig h t. P r o v e t h a t /1

[A X C ] AF
/T \
[B X C ] ‫־‬ F B *

C -' D ■ß
H in t

H in t

T y p e y o u r s o lu tio n , n o te s a n d / o r w o r k h e re .

4 .3 8 ★ : t V

In t h e d ia g r a m b e lo w , g iv e n [PQRS] = 5 [P Q w 4 ]a n d [PQRS] = l[ P /? S ] ,f in d [ABP]/[PQRS\.

P Q
A

c , \ B/ JR
H in t

H in t

H in t

T y p e y o u r s o lu tio n , n o te s a n d / o r w o r k h e re .

4 .3 9 : Source: ARML t V

In r e c ta n g le ABCD b e lo w , p o in t E is o n s id e CD s u c h th a t

[CBE] - [ADE] = [AEB] - [CBE].


W h a t is t h e r a t io o f t h e a r e a o f t h e la r g e s t r e g io n t o t h e a r e a o f t h e s m a lle s t r e g io n ?

H in t

H in t

T y p e y o u r s o lu tio n , n o te s a n d / o r w o r k h e re . Show S olution

F o r J o e 's b ir t h d a y , W ill h a s b o u g h t a 7 x 7 in c h s q u a r e b ir t h d a y c a k e w it h a f la t t o p . I t is a c h o c o la t e c a k e w it h b a n a n a
f r o s t in g o n t h e t o p a n d o n t h e s id e s . It t u r n s o u t t h a t s e v e n p e o p le w ill b e p r e s e n t w h e n t h e c a k e is c u t, a n d e a c h p e r s o n w ill
b e c o m e q u it e e n v io u s i f a n o t h e r p e r s o n r e c e iv e s m o r e c a k e o r f r o s t in g . F in d a w a y t o d iv id e t h e c a k e a m o n g s e v e n p e o p le , s o
t h a t e a c h r e c e iv e s a n e q u a l a m o u n t o f c a k e a n d f r o s t in g .

H in t

H in t

H in t

T y p e y o u r s o lu tio n , n o te s a n d / o r w o r k h e re . Show S olution

Extra! P e r h a p s s u r p r is in g ly , it is p o s s ib le t o d i s s e c t s o m e r e c t a n g le s in t o s q u a r e s o f d i f f e r e n t s iz e s . T h e f ig u r e
b e lo w s h o w s a d is s e c tio n o f a 3 2 x 3 3 r e c t a n g le in t o s q u a r e s , w h o s e s id e le n g t h s a r e in d ic a t e d .
V
52

I s i t p o s s i b l e t o d i s s e c t a s q u a r e i n t o s q u a r e s o f d i f f e r e n t s iz e s ?

T h e a n s w e r is y e s ! T h e f ig u r e b e lo w s h o w s s u c h a d is s e c tio n o f a 1 1 2 x 1 1 2 s q u a re . (T h e n u m b e rs
in d ic a t e s id e le n g t h s o f t h e s q u a r e s . )

112

27
35

50

19
15
11
17
2 6
9 7 112
24
25 18
29
16

^ 4

42
37
33

A p r o b le m fro m th e 2000 A m e r ic a n In v ita tio n a l M a th e m a tic s E x a m in a t io n a s k s th e f o llo w in g : “T h e


d ia g r a m s h o w s a r e c t a n g le t h a t h a s b e e n d is s e c t e d in t o n in e n o n - o v e r la p p in g s q u a r e s . G iv e n t h a t t h e
w id th a n d th e h e ig h t o f t h e r e c ta n g le a r e p o s it iv e in t e g e r s w it h g r e a t e s t c o m m o n d iv is o r 1 , fin d t h e
p e r im e te r o f t h e r e c ta n g le "

Try to solve it!


4.4 Summary

Definition: The perimeter of a closed figure is how far you travel if you walk along its boundary
all the way around it once.

Definition: The area of a closed figure is the number of 1 x 1 squares (or pieces of squares)
needed to exactly cover the figure. We sometimes use brackets to denote area, so
that [A B C ] means the area of A A B C .

Definition: A rectangle has four sides and four right angles, as shown.
/
Furthermore, opposite sides of a rectangle equal each other in r T
length. w 14
3_______ r
/

Important: The area of a rectangle with consecutive sides of length £ and w is iw . Since a
square is just a rectangle in which these lengths are the same, the area of a square
Z equals the square of the length of one of its sides.

Definitions: A triangle that has a right angle as one of its angles is called a right
triangle. The sides adjacent to the right angle of a triangle are
called the legs of the triangle.

Important: The area of a right triangle is half the product of the legs of the triangle. For example,
in the right triangle shown, we have
z
[ABC\ = {AB)!;B C ).

Important: To find the area of a triangle, we select one side to be the base.
H
The perpendicular segment from the vertex opposite the base
Z to the base (extended if necessary) is the altitude. We then
have:

base X altitude
Area =
2

For example, in the triangle shown, we have:

PQ x RZ
\PQR i=

Area can be a very powerful problem-solving tool. One particularly useful pair of principles is:

Important:
1. If two triangles share an altitude, the ratio of their areas is the ratio of the bases to
Z which that altitude is drawn. This is particularly useful in problems in which two
triangles have bases along the same line.
2. If two triangles share a base, then the ratio of their areas is the ratio of the altitudes
to that base.
5.6 Construction: Angles and Parallels
In this section, we take a look at how to use our knowledge of similar triangles for geometric constructions. But first, since
parallel lines and similar triangles are so closely related, well have to learn how to construct a parallel line.

Don’t forget the construction rules! Straightedge and compass only. No protractor. No measuring with your ruler.

P roblem s
Problem 5.19 4 Jump to Solution

Shown below are angle ZX and line m with point Y on it. Construct a line through Y that makes an angle with m that is equal
to ZX.

m
* v

(a) Draw a circle with center X and another circle with the same radius and center Y.

(b) Use your circles from the first part to construct a point Z on the circle centered at Y such that V à makes an angle with
m equal to ZX.

Problem 5.20 4 Jump to Solution

Given a line n and a point A not on n, construct a line through A that is parallel to n.

Problem 5.21 4 Jump to Solution

Draw a segment AB. In this problem we learn how to divide A B into 3 equal pieces.

(a) Draw a line through A but not through B. Construct a segment on this line that has A as one endpoint. Call the other
endpoint P.

(b) Construct a segment that has A as one endpoint, has P on the segment, and that is 3 times as long as A P. Call the other
endpoint of this segment R.

(c) Draw R B. Construct point X on A B such that A X /A B = 1/3.

Equal angles are very important in our study of similar triangles and parallel lines, so well start our constructions by learning how
to copy an angle.

Problem 5.19 t V

Given ZX and line m with point Y on it, construct a line through Y that makes an angle with m that is equal to ZX.

Solution for Problem 5.79: Since we measure angles by the portion of a circle that the angle cuts off, we start by making a circle
with center X . Let the points where this circle hits the sides of ZX be A and B. We then make a circle with center Y with the
same radius, since we want to cut off the same amount of this circle that ZX cuts off its circle. Let P be one of the points where
this circle hits m.

We can't use a ruler to measure A B to tell how much of circle X that ZX cuts off, but we can use our compass! We open our
compass to a width equal to AB, then draw an arc with center P and radius AB. Call the point where this arc meets circle Y
point Q. Drawing Y (§ gives us angle Z Q Y P equal to ZX.

<

Are you convinced that this construction works? I'm not. Well have to prove that ZY = ZX to see why the construction works.
Since QX and © K have the same radius, we have X B = Y P and X A = YQ. Since we opened the compass to width A B to
make our arc centered at P, we have A B = PQ. Therefore, we have A B X A = A P Y Q by SSS, so we have ZX = ZY\
Now, I'm convinced. □

As we've seen, we can use equal angles to prove that two lines are parallel. Let’s see if we can use equal angles to construct two
parallel lines.

Problem 5.20 t V

Given line n and point A not on n, construct a line through ,4 that is parallel to n.

Solution for Problem 5.20: We know how to copy an angle, and equal angles give parallel
lines, so we try copying an angle along line n to point A But first, well need a line through A
and an angle along n, so we draw a line through A that hits n at point X. We then copy
angle X to point A just as we copied an angle in Problem 5.19. We draw circles with the
same radii with centers A and X. We then set our compass to width UV and draw an arc
centered at P with that radius. This arc hits 0^4 at point Q such that ZPAQ = ZVXU.
Therefore, A (} II n. □

We haven't used parallel lines just for problems involving angles - we’ve also used them in problems involving similar triangles
and ratios of lengths.

Problem 5.21 t V

Given segment AB, construct points X and Y on A B such that A X = X Y = Y B = A B/S.

Solution for Problem 5.21: We know how to divide a segment in half, but cutting it by a ratio like 1 /3 calls for more advanced
tools than simple midpoints. The best geometry tool we have for ratios is similar triangles, but it’s not yet clear how we can use
similar triangles.

Unsure how to deal with trisecting (dividing into three pieces) AB, we try a simpler problem. Can we create a segment with A as
an endpoint such that the segment is cut into three equal pieces? This isn't so hard - we start with a line through A, pick a
different point P on the line, then copy A P twice along the line to get Q and R such that A P = P Q = Q R = AR/S.

So, we have a trisected segment with A as an endpoint, but unfortunately, it isn’t A B that we trisected. However, we do have that
1 /3 ratio, so perhaps we can now use similar triangles. Similar triangles call for parallel lines. We draw R B , then construct lines
through P and Q that are parallel to R B . These lines hit A B at X and Y.

We can use similar triangles to prove that X and Y are the points that divide A B into three equal pieces. Since
P X || Q Y || R B , we have A P A X ~ A QAY ~ A R A B . Therefore A X /A B = A P /A R = 1 /3 and
A Y /A B = A Q /A R = 2/3. Since A X = A B /S and AY = 2A B/S, we have A X = X Y = Y B = A B/S, as desired. □
Notice that our first key step in finding this solution was thinking about an easier related problem.

Concept: When stuck on a problem, try solving an easier related problem. One way to do this
I with a construction problem is relaxing one of the constraints of the problem.

For example, in Problem 5.21, we relaxed the constraint that the trisected segment has both ;4 and B as endpoints. Instead, we
just created a trisected segment with A as an endpoint.

Although this is the end of the similar triangles chapter, this won't be the end of your study of similar triangles. You'll see them
pop up in many more problems, and you’ll find another whole chapter devoted to an application of similar triangles with the lofty
name Power of a Point.

Exercises

5 .6 .1 : t V

Given a segment of length 1, construct a segment with length 1/5. Construct a segment with length 2 -
O

Type your solution, notes and/or work here. Show Solution

Given a triangle A A BC, construct a triangle A X Y Z that is similar to A A BC, but has 9 times the area of A A BC.

Type your solution, notes and/or work here. Show Solution


All cases are unique and very similar to others. - T. S. Eliot

CHAPTER 5 ---------------------------------
I
1___________ Similar Triangles

5.1 What is Similarity?

P roblem s

We call two figures similar if one is simply a blown-up, and possibly rotated and/or flipped, version of the other. Our first problem
gives us an example of similar figures.

Solution for Problem 5.1: Measuring each of the segments in the given ratios, we find that in each case, the ratio is 1/2. When we
measure the angles, we find that the angles of A B C D are equal to those in E H G F (note the orders of the vertices!):

ZA = ZE = 90°
CO
O
o
rH

ZB = ZH =
zc = ZG = 80°
ZD = ZF = 60°

We write the similarity in Figure 5.1 as A B C D ~ E H G F since Z A corresponds to Z E , Z B corresponds to Z H , etc. As


with congruence, we have to be careful about the order of the vertices. For example, we would not write A B C D ~ E F G H to
describe Figure 5.1.

The ratio between corresponding lengths in similar figures is constant, and is equal to the ratio by which one figure is ,blown up‫׳‬
to get the other. In Figure 5.1, we have

AB BC CD DA
E H ‫ ־־‬H G ‫ ־‬G F ‫־‬ FE­

AN corresponding lengths of A B C D and E H G F follow this ratio. For example, we could include B D / H F and A C / E G in
that chain of equalities above.

As we saw in Problem 5.1, corresponding angles in similar figures are equal.

Similar figures do not need to have the same orientation. The diagram to the right shows two
similar triangles with different orientations.

Speaking of triangles, well be spending the rest of this chapter discussing how to tell when
two triangles are similar, and how to use similar triangles once we find them. Below are a
couple Exercises that provide practice using triangle similarities to write equations involving
side lengths.

Exercises

5 .1 .1 : t V

Given that A A B C ~ A Y X Z, which of the statements below must be true?

(a) A B / Y X = A C /Y Z .

Type your solution, notes and/or work here. Solution

(b) A B /B C = Y X /X Z .

Type your solution, notes and/or work here. Show Solution

(c) A B / X Z = B C /Y X .

Type your solution, notes and/or work here. Solution

(d) ( A C ) ( Y X ) = CY Z ) ( B A ).

Type your solution, notes and/or work here. Solution

(e) B C /B A = X Y /Z Y .

Type your solution, notes and/or work here. Solution

5 .1 .2 : Source: MATHCOUNTS t V

A A B C ~ A A D B , A C = 4, and A D = 9. What is .4£?

Hint

Type your solution, notes and/or work here. Show Solution


5.2 AA Similarity
In o u r in t r o d u c t io n , w e s t a t e d t h a t s im ila r f ig u r e s h a v e a ll c o r r e s p o n d in g a n g le s e q u a l, a n d t h a t c o r r e s p o n d in g s id e s a r e in a
c o n s t a n t r a t io . I t s o u n d s lik e a l o t o f w o r k t o p r o v e a ll o f t h a t ; h o w e v e r , j u s t a s f o r t r ia n g le c o n g r u e n c e , w e h a v e s o m e s h o r t c u t s
t o p r o v e t h a t t r i a n g l e s a r e s i m i l a r . W e 'l l s t a r t w i t h t h e m o s t c o m m o n l y u s e d m e t h o d .

W e 'l l e x p l o r e w h y A A S i m i l a r i t y w o r k s i n S e c t i o n 5 . 5 , b u t f i r s t w e l l g e t s o m e e x p e r i e n c e u s i n g i t i n s o m e p r o b l e m s .

Problems
Problem 5.2 4 J u m p t o S o lu t io n

B e lo w a r e t w o t r ia n g le s t h a t h a v e t h e s a m e m e a s u r e s f o r t w o a n g le s .

A E

F in d t h e t h i r d a n g le in e a c h , a n d f in d t h e r a t io s A B / D F. A C /D E . B C /E F b y m e a s u r in g t h e s id e s w i t h a r u le r .

Problem 5.3 4 J u m p t o S o lu t io n

In t h is p r o b le m w e t r y t o e x te n d A A S im ila r it y t o f ig u r e s w it h m o r e a n g le s b y c o n s id e r in g f ig u r e s w it h
f o u r a n g le s . C a n y o u c r e a te a f ig u r e EFGH t h a t h a s t h e s a m e a n g le s a s A B C D a t r ig h t s u c h t h a t
A
Ü
EFGH and ABCD a r e n o t s im ila r ? ( In o t h e r w o r d s , c a n y o u c r e a te E F G H s o t h a t t h e a n g le s o f
EFGH e q u a l th o s e o f AB C D , b u t t h e r a tio o f c o r r e s p o n d in g s i d e s b e t w e e n E F G 11 a n d A B C D
is n o t t h e s a m e f o r a ll c o r r e s p o n d in g p a ir s o f s id e s ? ) B

Problem 5.4 4Jum p t o S o lu t io n

In t h e f ig u r e a t r ig h t, MN || OP. O P = 12, MO = 10, and LM = 5 . F in d M N. P


Â

L
/V
4 ----j ------------ 4
5 M 10
Q
°

Problem 5.7 4 J u m p t o S o lu t io n

G iv e n t h a t D E || B C and A Y || X C , p ro v e th a t
A
EY AD
y
EX ~ DB'

x k ^ \
b C

Solution for Problem 5.2: T h e la s t a n g le in e a c h t r ia n g le is 1 8 0 ° — 5 0 ° — 6 0 ° = 7 0 ° , s o t h e a n g le s o f A ABC m a tc h th o s e o f


A D F E . In t h e s a m e w a y , i f w e e v e r h a v e t w o a n g le s o f o n e t r ia n g le e q u a l t o t w o a n g le s o f a n o th e r , w e k n o w t h a t t h e t h ir d
a n g le s in t h e t w o t r ia n g le s a r e e q u a l.

M e a s u r in g , w e f in d t h a t t h e r a t io s a r e e a c h a b o u t 2 / 3 . I t a p p e a r s t o b e t h e c a s e t h a t i f a ll t h e a n g le s o f t w o t r ia n g le s a r e e q u a l,
t h e n t h e t w o t r ia n g le s a r e s im ila r . □

W e m ig h t w o n d e r i f t w o f ig u r e s w it h e q u a l c o r r e s p o n d in g a n g le s a r e a lw a y s s im ila r . S o , w e a d d a n a n g le a n d s e e i f it w o r k s f o r
f ig u r e s w it h f o u r a n g le s .

Problem 5.3 t V

Does your rule work for figures with more than 3 angles? Can you create a figure E F G II that has the ^
U
same angles as A B C D at right such that E F G H and A B C D are not similar? (In other words, can J Lj
you create E F G H so that the angles of E F G H equal those of A B C D , but the ratio of
corresponding sides between E F G H and A B C D is not the same?) 1 r
B C

Solution for Problem 5.3: W e c a n q u ic k ly f in d s u c h a n EFGH. T h e d ia g r a m t o th e


r ig h t s h o w s a s q u a r e E F G H n e x t t o o u r in it ia l r e c ta n g le C le a r ly t h e s e f ig u r e s
h a v e t h e s a m e a n g le s , b u t w h e n w e c h e c k t h e r a t io s , w e f in d t h a t

AB
EF
< 1< BF CG
ABCD and EFGH a r e n o t s im ila r , s o e q u a l a n g le s a r e n o t e n o u g h t o p r o v e
s im ila r it y h e r e . 3

L e t 's r e t u r n t o t r i a n g l e s a n d t a c k l e s o m e p r o b le m s u s in g A A S im ila r it y .

Solution for Problem 5.4: S e e i f y o u c a n f in d t h e f la w in t h is s o lu t io n :

Bogus Solution: S in c e M N || OP, w e h a v e A L A IN = A LO P a n d A L N M = A L P O


T h e re fo re , A L M N ~ A LO P, s o L M /M O = M N /O P . S u b s t i t u t i n g o u r g i v e n
Y s id e le n g t h s g iv e s 5 / 1 0 = M N / 1 2 , s o M N = 6 .

L M /M O — M N /O P . M O i s
E v e r y t h in g in t h i s s o lu t io n is c o r r e c t e x c e p t f o r n o t a s id e o f o n e o f o u r s im i l a r t r ia n g le s ! T h e
c o r r e c t e q u a t io n is L M /L O = M N /O P . S i n c e LO = L M + M O = 1 5 , w e n o w have 5 /1 5 = M N /12, s o M N = 4 . o

Problem 5.5 t V

The lengths in the diagram are as marked, and W X || Y Z . Find P Y and W X . W

4 7 ‫׳‬

V 2
z

Solution for Problem 5.5 : W h e re d o e s t h is s o lu tio n g o w r o n g :

Bogus Solution: S in c e YVX || Z Y , w e have AW — AZ and A X = AY. T h e re fo re ,


A H ‫׳׳׳‬P X ~ A K PZ, a n d w e have

y PX_ _ w x _ _ W_P
PZ ~ Y Z ~ PY
S u b s t it u t io n g iv e s

3 WX 5

10 ~ 12 PY'
W e c a n n o w e a s ily f in d YP = 5 0 /3 and WX = 1 8 /5 .

T h is s o l u t i o n d o e s n 't g e t t h e v e r t e x o r d e r in t h e s im i l a r t r ia n g le s r ig h t , s o it s e t s u p t h e r a t io s w r o n g ! PX and PZ a re n o t
c o r r e s p o n d in g s id e s . PX in A W PX c o rre s p o n d s to PY in A Z PY because AW = AZ.
H e r e 's w h a t t h e s o l u t i o n s h o u l d l o o k l i k e . P a y c l o s e a t t e n t i o n t o t h e v e r t e x o r d e r i n t h e s i m i l a r i t y r e l a t i o n s h i p .

S in c e WX || ZY , w e have AW = AZ and AX = AY T h e re fo re , A H ? PX ~ A ZPY. H ence, w e have

PX WX WP
PY ~ YZ ~ P Z '
S u b s t it u t io n g iv e s

3 WX_ _ _5_
PY~ 12 10 ‫־־‬

W e c a n n o w e a s ily f in d PY = 6 and W X = 6. □

P e rh a p s y o u s e e a c o m m o n th re a d in t h e la s t t w o p r o b le m s . W h ile y o u w o n 't a lw a y s f in d p a r a lle l lin e s in s im ila r t r ia n g le


p r o b le m s , y o u 'll a lm o s t a lw a y s f in d s im ila r t r ia n g le s w h e n y o u h a v e p a r a lle l lin e s .

Important: P a r a lle l lin e s m e a n e q u a l a n g le s . E q u a l a n g le s m e a n s im ila r t r ia n g le s . T h e f ig u r e s


b e lo w s h o w t w o v e r y c o m m o n s e t- u p s in w h ic h p a r a lle l lin e s le a d t o s im ila r t r ia n g le s .
Z S p e c ific a lly , APQB ~ A P S T a n d A .J K L ~ A M N L .

WARNING!! R e a d t h e B o g u s S o lu t io n s t o P r o b le m s 5 . 4 a n d 5 .5 a g a in . T h e s e a r e v e r y c o m m o n
e r r o r s ; u n d e r s ta n d t h e m s o y o u c a n a v o id t h e m .
o

Solution for Problem 5 . 6 : S i n c e A B A D = A C A B a n d A B D A = A C B A , w e h a v e A B A D ~ A C A B b y A A S im ila r ity .


T h e r e f o r e , w e h a v e B C /B D = A B /A D = 5 / 3 , s o B C = ( 5 / 3 )(B D ) = 2 0 / 3 .

W e c a n u s e t h is s a m e s im ila r ity t o fin d AC, a n d t h e n s u b t r a c t A D t o g e t CD. W e c o u ld a ls o n o te t h a t ABCD = ABCA and


A B D C = ACBA, so ABCD ~ A A C B b y A A S im ila r ity . T h e r e fo r e ,

C D /B D = B C /A B = (2 0 /3 )/5 = 4 /3 ,

so C D = (4 /3 )(B D ) = 1 6 /3 . □

S im ila r t r ia n g le s - t h e y 'r e n o t j u s t f o r p a r a lle l lin e s .

Important: S im ila r t r ia n g le s fr e q u e n tly pop u p in p r o b le m s w it h r ig h t a n g le s T h e d ia g r a m in

z P r o b le m 5 . 6 s h o w s a c o m m o n w a y t h is o c c u r s . M a k e s u r e y o u s e e t h a t

A A B D ~ A BCD ~ AACB.

A s y o u 'll s e e t h r o u g h o u t t h e r e s t o f t h e b o o k , s im i l a r t r ia n g le s o c c u r in a ll s o r t s o f p r o b le m s , n o t j u s t t h o s e w i t h p a r a lle l lin e s a n d


p e r p e n d ic u la r lin e s . T h e y 'r e a ls o a n i m p o r t a n t s t e p in m a n y p r o o f s .

Solution for Problem 5.7: P a r a lle l lin e s m e a n s im i l a r t r ia n g le s . T h e r a t io s o f s id e le n g t h s in t h e p r o b le m a ls o s u g g e s t w e lo o k f o r


s im ila r t r ia n g le s .

A Y || X C , w e h a v e A A Y E ~
S in c e A CXE. N o w w e lo o k a t w h a t t h is m e a n s f o r o u r r a t io s . F r o m A AY E ~ A CXE,
we have E Y /E X = A E /E C . A l l w e h a v e le f t is t o s h o w t h a t A E /E C = A D /D B .

S in c e D E || BC, w e h a v e A A D E ~ A AB C . T h e r e f o r e , A D /A B = A E /A C , w h i c h is a lm o s t w h a t w e w a n t! W e b r e a k
AB and A C i n t o A D + D B a n d A E + EC, h o p i n g w e c a n d o a l i t t l e a l g e b r a t o f i n i s h :
AD AE
AD + D B AE + EC
I f o n ly w e c o u ld g e t r id o f t h e AD and AE in t h e d e n o m in a t o r s - th e n w e w o u ld h a v e A D /D B = A E /E C . F o r tu n a te ly , w e
c a n d o it. W e c a n f lip b o t h f r a c t io n s :

AD + D B AE + EC
AD AE

AD DB AE EC DB EC
T h e re fo re , — + — = — + — so ! + _ = ! + — w h ic h g iv e s u s

DB EC
AD ~ A E '
F lip p in g t h e s e f r a c t io n s b a c k o v e r g iv e s u s A D /D B = A E /E C . T h e re fo re , w e h a v e E Y /E X = A E /E C = A D /D B , as
d e s ir e d . □

O u r s o lu t io n t o t h e p r e v io u s p r o b le m r e v e a ls a n o t h e r h a n d y r e la t io n s h ip in v o lv in g s im ila r t r ia n g le s :

(a ) F in d AC and BC.

T y p e y o u r s o lu tio n , n o te s a n d / o r w o r k h e re , Show Solution

(b ) F in d H J .

T y p e y o u r s o lu tio n , n o te s a n d / o r w o r k h e re , Show Solution

(c ) F in d ON and M N.

L 1 .6 M N

T y p e y o u r s o lu tio n , n o te s a n d / o r w o r k h e re , Show Solution

(d ) F in d HS.

T y p e y o u r s o lu tio n , n o te s a n d / o r w o r k h e re . Show Solution

T y p e y o u r s o lu tio n , n o te s a n d / o r w o r k h e re . Show Solution

5 .2 .3 :

In t h e d ia g r a m , WX YZ is a s q u a r e . M is t h e m id p o in t o f YZ, and A B J_ M X

(a ) S h o w th a t WZ || XY.
H in t

T y p e y o u r s o lu tio n , n o te s a n d / o r w o r k h e re . Show Solution

(b ) P ro v e th a t AZ = Y B.

T y p e y o u r s o lu tio n , n o te s a n d / o r w o r k h e re . Show Solution

(c ) P ro v e th a t X B = X A.

T y p e y o u r s o lu tio n , n o te s a n d / o r w o r k h e re . Show Solution

(d ) P ro v e th a t A . 4 Z M ~ A M Y X. a n d u s e th is fa c t t o p ro v e AZ = X Y /4 .

T y p e y o u r s o lu tio n , n o te s a n d / o r w o r k h e re . Show Solution

In t r ia n g le ABC, A B = A C ,B C = 1, a n d ABAC — 3 6 °. Let D be t h e p o in t o n s id e AC s u c h th a t A A B D — ACBD.


(a ) P ro v e t h a t tr ia n g le s ABC and BC D a r e s im ila r .

T y p e y o u r s o lu tio n , n o te s a n d / o r w o r k h e re . Show Solution

( b ) ★ F in d AB.
H in t

T y p e y o u r s o lu tio n , n o te s a n d / o r w o r k h e re . Show Solution

5 .2 . 5 ★ : t V

F in d x in t e r m s o f y g iv e n t h e d ia g r a m b e lo w .

/ C
y \
\

5
D

E y F
Ÿ
C
N

7 h

H in t

H in t

T y p e y o u r s o lu tio n , n o te s a n d / o r w o r k h e re .

Extra! M y d a d w a s g o in g t o c u t d o w n a d e a d t r e e in o u r y a r d o n e d a y . b u t h e w a s a f r a id it
m ig h t h it s o m e n e a r b y p o w e r lin e s . H e k n e w t h a t i f t h e t r e e w e r e o v e r 4 5 f e e t t a ll, t h e
V t r e e w o u ld h it t h e p o w e r lin e s . H e s t o o d 3 0 f e e t f r o m t h e b a s e o f t h e t r e e a n d h e ld a
r u le r 6 in c h e s in f r o n t o f h is e y e . H e lin e d t h e b o t t o m o f t h e r u le r u p w it h t h e b a s e o f
t h e tr e e , a n d s a w t h a t t h e t o p o f t h e t r e e lin e d u p w it h a p o in t 8 in c h e s h ig h o n t h e
r u le r . H e t h e n k n e w h e c o u ld s a f e ly c u t t h e t r e e d o w n . H o w d id h e k n o w ?
5.7 Summary

Definition: Two figures are similar if one is simply a blown-up, and possibly rotated and/or
flipped, version of the other.

Important: Corresponding angles in similar figures are equal, and the ratio of the lengths of
corresponding sides of similar triangles is always the same.
z

In similar quadrilaterals A B C D and E H G F , we have Z A = ZE , Z B = zu,


Z C = ZG, and Z D = Z F . We also have

AB BC CD DA AC BD
EH IIG GF FE EG HF
We denote these figures as similar by writing A B C D ~ E H G F .

There are three main ways to show that two triangles are similar:

■ AA Similarity. If two angles of one triangle equal two angles of another, then the triangles are similar. This is by far the
most commonly used method to prove two triangles are similar. (Section 5.2)

■ SAS Similarity. If two sides in one triangle are in the same ratio as two sides in another triangle, and the angles between
the sides in each triangle equal each other, then the triangles are similar. (Section 5.3)

■ SSS Similarity. If each side of one triangle is the same constant multiple of the corresponding side of another triangle, then
the triangles are similar. (Section 5.4)

Parallel lines and perpendicular lines are clues to look for similar triangles. Three very common set-ups that contain similar
triangles are shown below.

/1 P

a a b c ~a b d c ~a a d b

Important:
If B C || D E and and c è meet at A as shown, then A
Z AB AC
B /. \ C
BD CE' ATA
D > E

Important: If two triangles are similar such that the sides of the larger triangle are k times the
^ sides of the smaller, then the area of the larger triangle is k 2 times that of the smaller.

This relationship holds for any pair of similar figures, not just for triangles.

P roblem Solving S trateg ies

Concepts:
■ When you’re stuck on a problem, ask yourself, ,What piece of information have I
not used?‫׳‬

■ In many problems, there's more than meets the eye. Extending segments that
seem to end abruptly (particularly in the middle of a triangle) can often yield quick
solutions.

■ When stuck on a problem, try solving an easier related problem. For constructions,
useful easier related problems often involve relaxing one of the constraints of the
problem.

■ Consider using similar triangles in problems involving ratios of segment lengths.

T hin g s To W atch Out For!

WARNING!! Below are shown two common situations that lead to mistakes. The diagram on the
left may lead you to write ‫׳‬A A B C ~ A A D E , so A B / B D = B C / D E . ' The one
O on the right might lead to ‘A .J K L ~ A N L M , so J L / N L = K L /M L .' Both of
these are incorrect! Make sure you see why!
5 . 3 S A S S im ila r it y

P roblem s
Problem 5.8 4 Jump to Solution

(a) Measure B C , E F , and angles Z B , ZC , Z E , and Z F .

(b) Can you make a guess about how to use Side-Angle-Side for triangle similarity?

1.5/ V 25

Problem 5.9 4 Jump to Solution

AB AD 4
In the figure below on the left, we have — = 7 7 7 ‫= ־‬ - , and clearly Z B A D = Z C A E . We wish to prove that
AC A hj o___
A A B D ~ A A C E . (Note that we cannot assume that B D || C E \ We have to prove it.)

D Bl

4
2 / V. \
E C

(a) Suppose we draw a line through B parallel to C E that hits A E at X as in the diagram on the right. What do we know
about A A B X and A A C E ?

(b) Given that A E = 15 in both diagrams above, what is A X ?

(c) What can we conclude about D and X?

(d) What can we conclude about A A B D and A A C E ?

(e) What similarity rule can we create from this investigation?

Problem 5.10 Source: Mandelbrot 4 Jump to Solution

Given A C = 4, C D = 5, and A B = 6 as in the diagram, find B C if the perimeter of A B C D


is 20.

Problem 5.8 t V

(a) Measure B C , E F , and angles Z B, Z C , Z E, and Z F.

(b) Can you make a guess about how to use Side-Angle-Side for triangle similarity?

Solution for Problem 5.8: We aren't surprised to find that B C appears to be half E F : B C is about 2 cm and E F is around 4 cm.
We also aren't shocked to find that Z B appears to equal Z E and Z C appears to equal Z F .

This example suggests that if two sides in one triangle are in the same ratio as two sides in another triangle (as
A B / A C = D E / D F ) , and the angles between these sides are equal (as Z A = ZD ), then the triangles are similar. □

No doubt, you know where this is headed. Time to develop a proof for our guess. As usual, we try to use what we already know,
AA Similarity, to prove our guess for ‘SAS Similarity‫׳‬.

Solution for Problem 5.9: What did we do wrong here:

Bogus Solution: Since B D || C E , we have Z A B D = Z A C E and Z A D B = Z A E C , so


A A B D ~ A A C E by AA Similarity.

There's not a single false statement in that solution. However, the assertion that B D || C E needs to be proved, and our Bogus
Solution merely states it without justification.

In the solution below, we take the clever tactic of considering the point X on A E such that B X || C E . Then we prove that X is
in fact D.

We'd like to prove that B D || C E , but there's no obvious way to even start. We seem stuck,
A
so we try to go a different direction. We create a point X on A E as shown at right, such that
B X || C E . Our goal now is to show that X must be D. Notice that we are not assuming
that B D || C E . We are taking some other point, X , such that B X || C E , then trying to
prove that X must be D.

Since B X || C E , we have Z A B X = Z A C E and Z A X B = ZAEC, so


A A B X ~ A A C E by AA Similarity. Therefore,

AX AB 4
A E ~ A C 5 ‫’־־‬

so A X = ( 4 /5 ) ( A E ) = 12. Hence, X is on A E 12 units from A But that's where point D is! Therefore, D must be the same
point as point X , i.e., D is the point on A E such that B D || C E . Now that we've proved B D || C E , we can conclude that
A A B D ~ A ACE. □

We have established another way to prove two triangles are similar.

Important: (SAS Similarity) tells us that if two sides in one triangle are in the same ratio as two
sides in another triangle (as A B / A C = D E / D F below), and the angles between
Z these sides are equal (as Z A = Z D below), then the triangles are similar.

Ü
A
I\ 0 60= .....6
4
В c
E

Note that we can also write that ratio equality as the ratio of corresponding sides in
the triangles: A B / D E = A C / D F .

You may be wondering how our solution to Problem 5.9 can be used to prove SAS Similarity in general, since Problem 5.9 only
deals with the case of two triangles that share an angle, as A A B D and A A C E share Z A We can use this approach generally
because if an angle in one triangle equals an angle in another, we can always slide (and/or flip) one triangle until it's on top of the
other, as shown in Figure 5.2.

Figure 5.2: Sliding Triangles to Prove Similarity

SAS Similarity is most often used in diagrams like the one shown in Problem 5.9. However, it does come up in less obvious
situations.

Solution for Problem 5.10: Since

AC _ 4 _ 2 AB _ 6 _ 2
and
AB “ 6 3 ‫־‬ AD 3 ‫ ־‬9 ‫’ ־‬

we have A А С В ~ A A B D by SAS (since the angle between the sides in each ratio above is Z;4). Since the sides of A A B D
are 3 /2 the corresponding sides of A A C B , we have B D = S B C /2. Now we can use that perimeter information. Since
B C + C D + D B = 20, we have

BC + 5 + = 20.

Therefore, B C = (i. □

Exercises

5 .3 .1 : t V

Find D E in the figure below.

Type your solution, notes and/or work here. Show Solution

5 .3 .2 : t V

In the figure below, M is the midpoint of E B and of E G . E and F are midpoints of / M and M J , respectively. Prove that
77 II <777.

Type your solution, notes and/or work here. Show Solution

5.3.3: t V

Show that if W Z 1 = ( W X ) ( W Y ) in the diagram below, then Z W Z X = Z W Y Z .

Hint

Type your solution, notes and/or work here. Show Solution

In the diagram below, Z P R Q = Z P Q A = 90°, Q R = Q A, and Z Q P C = Z R P C .

(a) Prove Z Q C B = Z Q B C .

Hint

Type your solution, notes and/or work here. Show Solution

(b) ★ Prove R A || P B .

Hint

Type your solution, notes and/or work here. Show Solution


Review Problems

5 .2 2 : t V

In each of the p imilar triangles or state that there are not any pairs of triangles that are
necessarily simi o t c im i
iu find, state why the triangles are similar.

(a)

Type your solution, notes and/or work here. Show Solution

(b)

Type your solution, notes and/or work here. Show Solution

(c)

Type your solution, notes and/or work here. Show Solution

(d)
О

Type your solution, notes and/or work here. Show Solution

(e )
W

12\ / 9

Type your solution, notes and/or work here. Show Solution

(f)
D

Type your solution, notes and/or work here. Show Solution

5 .2 3 : t V

Find x and у in the diagram at right, given the angle equalities and side lengths shown in
A P Q R and А Л И С .

Type your solution, notes and/or work here. Show Solution

5 .2 4 : t V

Points P and Q are on A P and A C , respectively, such that P Q || P C . Given A B = 12, P P = 9, and A C = 18, find Q A.

Type your solution, notes and/or work here. Show Solution

5 .2 5 : Source: MATHCOUNTS t V

The side I ;rs. One of the side lengths of a similar triangle


is 36 cent !ntimeie perimeter of the second triangle?

Type your solution, notes and/or work here. Show Solution

5 .2 6 : t V

What's wrong with the diagram shown below?

Type your solution, notes and/or work here. Show Solution

5 .2 7 : t V

Find D E in the diagram below.

Type your solution, notes and/or work here. Show Solution

5 .2 8 : t V

Why is the diagram shown below impossible?

Type your solution, notes and/or work here. Show Solution

5 .2 9 : t V

In the diagram below, find W Y and Y V .

IV

Type your solution, notes and/or work here. Show Solution

5 .3 0 : t V

A A P C at right is equilateral. M is on A P and N on A C such that P M = C N .

(a) Prove that A M = A N .

Type your solution, notes and/or work here. Show Solution

(b) Prove that A A M N is equilateral.

Type your solution, notes and/or work here. Show Solution

5 .3 1 : t V

Given A A P C ~ A Y Z X , [ A P C ] = 40, [ Y Z X ] = 360, A P = 9, and P C = 12, find the following:

(a) rZ .

Type your solution, notes and/or work here. Show Solution

(b) The length of the altitude to side X Z of triangle A Y Z X .

Type your solution, notes and/or work here. Show Solution

5 .3 2 : t V

Let A B C D be a rectangle as shown below, with A P = 25 and P C = 12. Let E be a point on A P , such that A E < P E
and triangles A E D and P C E are similar. Find A E .

A t B

Type your solution, notes and/or work here. Show Solution

5 .3 3 : t V

In the diagram below, P W = 6 and W X = 4. Find Q X .

Type your solution, notes and/or work here. Show Solution

5 .3 4 : t V

(Try this without looking back in the text first!) In the diagram below, A P || P Q || C R . Prove that

1 1 1
C R ‫ ־־‬A P + B Q '

Type your solution, notes and/or work here. Show Solution

5 .3 5 : Source: MATHCOUNTS t V

Two of the !ngth 12 cm and 20 cm, as shown. What is the number of centimeters in the
length of X\ ngth 20 cm? (If you know the Pythagorean Theorem, try doing this problem
without it!)

Type your solution, notes and/or work here. Show Solution

Let A B C be a triangle, and let D and E be points on sides A P and A C , respectively, such that D E || P C . Prove that

AD DP
AE ‫־‬ CE'

(Try to do this one without looking back in the text for the proof!)

Type your solution, notes and/or work here. Show Solution


Challenge Problems

5.37: t V

Let A B C be a triangle, and let D and E be points on A B and A C , respectively, such that A D / A E = B D / E C . Prove that
D E || B C . Make sure you see why this differs from the previous problem!

Hint

Hint

Type your solution, notes and/or work here. Show Solution

If the sum of one of the th‫״‬ angle is the same for two different isosceles triangles, must the triangles
be similar?

Hint

Type your solution, notes and/or work here. Show Solution

Source: MATHCOUNTS t V

In the figure below, isosceles A A B C with base A B has altitude C H = 24 cm. D E = G F, H F = 12 cm, and F B = 6
cm. Find the area of C D E F G .

Hint

Type your solution, notes and/or work here. Show Solution

5 .4 0 : t V

In triangle A B C below, D and E are points on sides A C and A B , respectively, such that ( A D ) ( A C ) = ( A E ) ( A B ) .
Prove that Z C D E A Z C B E = 180° and Z A D B A Z B E C = 180°.

Hint

Hint

Type your solution, notes and/or work here. Show Solution

5 .4 1 : t V

D, E , and F are on sides B C , A C , and A B , respectively, of A A B C such that D E A B , D F || A C , and B C || E F .


Prove that D, E, and F are the midpoints of the sides of A A B C .

Hint

Type your solution, notes and/or work here. Show Solution

5 .4 2 : t V

In the diagram below, P S || Q T and P Q || S T . Prove that S U /S P = Q P /Q R .

Type your solution, notes and/or work here. Show Solution

5 .4 3 : Source: AMC 8 t V

The area of triangle X Y Z below is 8 square inches. Points A and B are midpoints of congruent segments X Y and X Z .
Altitude X C bisects Y Z . What is the area of the shaded region?

Hint

Hint

Type your solution, notes and/or work here. Show Solution

U ^^! Source: MATHCOUNTS t V


\3 • ™
T■

The midpoints of the three sides of an equilateral triangle are connected to form a second triangle. A third triangle is formed
by connecting the midpoints of the second triangle. This process is repeated until a tenth triangle is formed. What is the ratio
of the perimeter of the tenth triangle to that perimeter of the third triangle?

Hint

Type your solution, notes and/or work here. Show Solution

5.45: Source: AMC 10 t V

In rectangle A B C D below, A B = 5 and B C = 3. Points F and G are on C D so that D F = 1 and G C = 2. Lines A ^


and h d intersect at E. Find the area of A A E B .

Hint

Type your solution, notes and/or work here. Show Solution

5 .4 6 ★ : t V

In triangle A B C below, A C = 4, B C = 3, A B = 5, and Z A C B = 90°. The infinite sequence of points C\, C-2, C& C 4, . . .
is generated as follows: C ! is the foot of the altitude from C‫ ׳‬to side A B , C -2 is the foot of the altitude from C !to side A C , C$
is the foot of the altitude from C2 ‫ ׳‬to side A B, and so on. Calculate the sum

C C \ A C\C-2 A C 2C :i A Ç 3 C 4 4‫ • • • ־‬.

Hint

Hint

Type your solution, notes and/or work here. Show Solution

5 .4 7 ★ : t V

In the figure shown, we have taken an equilateral triangle and divided each side into four segments of equal length. We have
then connected these points to form smaller equilateral triangles.

Consider instead dividing each side into n segments of equal length, where n is some positive integer, then connecting these
points as before to form smaller equilateral triangles. Use this dissection of the original equilateral triangle to prove that the
sum of the first n positive odd integers is n 2.

Hint

Hint

Type your solution, notes and/or work here. Show Solution

5 .4 8 ★ : Source: Mandelbrot t V

Figure A B C D below has sides A B = 6, C D = 8, B C = D A = 2, and A B II C D . Segments are drawn from the
midpoint P of A B to points Q and R on side C D so that P Q and P R are parallel to A D and B C , as shown. Diagonal
D B intersects P Q at X and P R at Y . Evaluate P X / Y R .

A P B

Hint

Hint

Type your solution, notes and/or work here. Show Solution

5 .4 9 ★ : Source: AMC 12 t V

In rectangle A B C D below, points F and G lie on A B so that A F = F G = G B and E is the midpoint of D C . Also, A C
intersects E F at / / and E G at J . The area of rectangle A B C D is 70. Find the area of A A I I F.

Hint

Hint

Hint

Type your solution, notes and/or work here. Show Solution

5 .5 0 ★ : t V

ABC ;quare D E F G such that D and E are on B C , G


is on A B , and /‫ ׳‬is on A C .

Hint

Hint

Type your solution, notes and/or work here. Show Solution

5 .5 1 ★ : t V

Points P, Q, R, S, T , and U are on the sides of triangle A B C , as shown, such that line
segments U R , Q T , and S P all pass through point X , and are parallel to B C , C A , and A B ,
respectively. Prove that

PQ RS TU
BC + CA + AB ~ L

Hint

Hint

Hint

Type your solution, notes and/or work here. Show Solution

Extra! A polyomino, a term coined by USC mathematics professor Solomon W. Golomb in 1953, is simply a
piece consisting of a number of connected squares. Evidently, it is a generalization of a domino, a piece
V consisting of two connected squares.

The number of squares is indicated by the prefix, so a monomino consists of one square, a domino has
two, a triomino has three, a tetromino has four, a pentomino has five, and so on. The five tetrominoes are
shown below. (You might recognize them from the video game Tetris!)

n c f f 3

The first obvious question to ask is, ',How many polyominoes are there that contain a given number of
squares?‫ ״‬The answer is partially answered by the following table.

Number of squares Number of polyominoes


1 1
2 1
3 2
4 5
5 12
6 35
7 108
8 369
9 1285

At the time of writing, the number of polyominoes is known up to 56 squares, but no general formula is
known.

The next obvious question to ask is, "What interesting shapes can these polyominoes make?‫ ״‬For
example, can the five tetrominoes be used to cover a 4 x 5 chessboard?

The answer turns out to be ,'no.‫ ״‬But how can we prove this? Just saying that we have tried many
different ways and given up is not a very satisfactory answer, and ultimately not rigorous. It turns out
there is a simple proof, using a common problem solving technique. We color the 4 x 5 chessboard as
shown.

f t f l

Let's take another look at those tetrominoes again.

n c f f 3

If we place the first piece on the rectangle, how many black and white squares will it cover? It's not hard
to see that it will always cover two black squares and two white squares.

The same holds for the fourth and fifth pieces.

₪ ^
And there are two different combinations of colored squares the second piece can cover, but both still
have two black squares and two white squares.

□₪ e B
That means that these four pieces, together, will always cover eight black squares and eight white
squares. But what about the third piece? It can be placed two different ways.

* A
One way covers three white squares and one black square, and the other way covers one white square
and three black squares. This means that the five pieces, together, will always cover eleven white squares
and nine black squares, or nine white squares and eleven black squares, which means in particular they
can never cover ten white squares and ten black squares, as shown. Thus, we have used parity (even­
ness and odd-ness) and chessboard coloring to prove that a covering cannot exist.

The twelve pentominoes, on the other hand, can fit snugly inside a 6 x 10 rectangle. See if you can figure
out how. (Answer in Section 11.1 here.)
The Pythagorean Theorem

Choose always the way that seems the best however rough it may be; custom will soon render it easy and agreeable. - Pythagoras

CHAPTER 6
Right Triangles
In this chapter we study right triangles. As a quick refresher, a right triangle is a triangle that has a right
angle among its angles. The side opposite the right angle is called the hypotenuse, and the other two
sides are called legs. In the figure to the right, A B is the hypotenuse, while A C and B C are both legs.
As you'll see throughout the book, many problems are solved by building right triangles and using the
principles you'll learn in this chapter, particularly one of the most famous math theorems of all: the
Pythagorean Theorem.

6.1 Pythagorean Theorem

P roblem s
Problem 6.1 4 Jump to Solution

In this problem we will prove one of the most famous theorems of mathematics. The diagram
c
shows right triangle A A B C with hypotenuse A B = c and legs A C = b and B C = a.

(a) Prove that a2 = cd,

(b) Prove that b2 = ce. B<

(c) Use the first two parts to show that a 2 4 b2 = c2. This is the Pythagorean Theorem.

Problem 6.4 4 Jump to Solution

Isosceles triangle A M N О has M N = N O = 7 and M O = 10. Find [Л7 N O ],


/V

7' 7 X 7

M 10 c

Problem 6.5 4 Jump to Solution

Two vertical poles are 14 feet apart. One is 16 feet tall, and the other is 9 feet tall. A rope extends
straight from the top of one pole to the top of the other pole. How long is the rope?
5
16
9

14 R

Problem 6.6 4 Jump to Solution

Let P be a point and m be a line that doesn't pass through P. Let X be on m such that
P X J_ m. The distance from a point to a line is the length of the shortest segment from the
point to the line. In this problem, we prove that the distance from P to m is P X .

(a) Let B be a point on m besides X . What kind of triangle is A P X B l

(b) Use the Pythagorean Theorem to get an expression for P B 2.

(c) Why does the previous part show that P X is the shortest possible distance from P to line m l

Problem 6.7 4 Jump to Solution

In this problem we investigate whether A A B C must be a right triangle given that B C = a, A C = b, A B = c, and
a2 + b2 = c2

(a) Suppose we have a triangle A D B F such that Z F = 90°, E F = a, and D F = b. (These are the same lengths as B C
and A C in A A B C ) What is D E I

(b) What can we say about triangles A A B C and A D E F ?

(c) Must A A B C be a right triangle?

We start off with a proof of one of the most famous theorems in mathematics.

Solution for Problem 6.7; As we showed in Problem 5.14, we can use A A D C ~ A A C B t o show that b/e = c/b, so b2 = ce.
Similarly, A B D C ~ A B C A gives us a /d = c/a , so a 2 = cd. Adding these expressions for a 2 and b2 gives us

a2 4‫ ־‬b2 = cd 4‫ ־‬ce = c(d 4 e ) = c2.

Important: In any right triangle, the sum of the squares of the legs
equals the square of the hypotenuse. In the figure to the
Z right, we have

a2 4 b2 = c2

This is the famous Pythagorean Theorem.

Problem 6.2 t V

Find the missing sides in the triangles below.

J 8

Solution for Problem 6.2:

(a) A A B C : A B 2 4 B C 2 = A C 2, so 64 4 B C 2 = 128. Therefore, B C 2 = 64 and B C = 8.

(b) A D E F & z A F E X : From A D E F we have D E 2 4 E F 2 = D F 2, so 36 4 E F 2 = 100, and E F = 8. Now we can


tackle E X with A F E X : E X 2 4 F E 2 = F X 2, so E X 2 = F X 2 - E F 2 = 225. So, E X = 15.

(c) A G H I : H I '2 + G I 2 = G H 2. so H I 2 = 196 - 49 = 147. Therefore, H I = s/ Ш =

(d) A .IK L :

K L 2 = L J 2 + J K 2 = 9 + 64 = 73.

So, K L = >/73.

Solution for Problem 6.3: We have a right triangle, so we can apply the Pythagorean Theorem:

72 4 r 2 = (r 4 l ) 2.

Therefore, 49 4 r 2 = r 2 4 2r 4 1, so r = 24. □

Sometimes it's not the sides of a right triangle we are missing, but the whole right triangle itself! In these cases, we have to build
the right triangle ourselves, then apply the Pythagorean Theorem.

Problem 6.4 t V

Isosceles triangle A M N O has M N = N O = 7 and M О = 10. Find [M N O }.

Solution for Problem 6.4: Finding an altitude will allow us to find the area. When we first studied
N
isosceles triangles in Section 3.6 here, we saw that we can cut an isosceles triangle in half with an
altitude. We do so here in the figure to the right, in which altitude N X cuts M O in half so that
M X = X O = 5. The Pythagorean Theorem applied to A M N X gives

M X 2 4 N X 2 = M N 2,

so 25 4 N X 2 = 49. Therefore, N X = \/2 4 = 2 \/6 , and our area is

[M N O ] = <N X * M O ' = lOv/6.


z

In our next example, it’s not as clear initially how to build a useful right triangle.

Problem 6.5 t V

Two vertical poles are 14 feet apart. One is 16 feet tall, and the other is 9 feet tall. A rope extends ^ t

straight from the top of one pole to the top of the other pole. How long is the rope?

16
9
1_____________ c
Q1 14 A

Solution for Problem 6.5: We don’t have a right triangle, but the right angles at Q and R have us
thinking about building our own. We draw S T perpendicular to P Q , and we have a right triangle
with hypotenuse P S \ Since Z.Q 4 /.S T Q = 180°, we have Q R \ \ T S . Therefore,
Z .T S R 4 Z.R = 180°, so Z T S R = 90°, and S T Q R is a rectangle. Since S T Q R is a
rectangle, we have T S = Q R = 14 and T Q = S R = 9. Therefore, P T = P Q — S R = 7,
and we can now use the Pythagorean Theorem to find PS:

P S 2 = P T 2 4 T S 2 = 49 4 196 = 245.

Therefore, the length of our rope is v/ 245 = 7 v/ 5. □

Concept: Building right triangles and applying the Pythagorean Theorem is one of the most
I common ways to find lengths. This is particularly true in problems that already involve
right angles, such as Problem 6.5.

Problem 6.6 t V

The distance from a point to a line is the length of the shortest segment from the point to the line. Prove that this shortest
segment is perpendicular to the line.

Solution for Problem 6.6: This is a great example of proving the obvious. Sometimes the obvious is so obvious that people will
propose solutions like this:

Bogus Solution: The shortest distance from P to the line m will be when we go straight from P to the
line. This would make a right angle with m because we get a right angle when we go
V straight from the point to the line.

This ,proof doesn't say anything at all. There aren't any false statements, but there aren't any useful statements either. It
essentially argues 'the angle is a right angle because that’s what you get when you make a right angle.' Here's why we get a right
angle:

We start by drawing a diagram. Let P be our point and m be our line. Let X be on m such that
P X J_ in. We must show that P X is the shortest segment from P to m, so we must show that P X
is shorter than any other segment from P to m. We let B be another point on m, and compare P X to
PB.

Now
we have a right triangle, A P X B , so we can use the Pythagorean Theorem:
P B 2 = P X 2 4 X B 2. Since X B 2 is clearly greater than 0, we must have P B 2 > P X 2, so
P B > P X . So, the perpendicular segment from P to m is shorter than any other segment from P to m. □

Important: The distance from a point to a line is the length of the segment from the point to the
7‫־‬ line that is perpendicular to the line.

You'll have a chance to use this definition of the distance from a point to a line to prove the following as an Exercise:

Throughout this section we have used the Pythagorean Theorem to find information about the sides of right triangles. Perhaps
you’re wondering if we can use the Pythagorean Theorem 'in reverse' to determine if a triangle is a right triangle.

Problem 6.7 t V

Suppose we have A A B C such that B C = a, A C = b, A B = c, and a 2 4 b2 = c2. Is A A B C necessarily a right triangle?

Solution for Problem 6.7: If a 2 4 b2 = c2, we do know that there is some triangle with side lengths a, b, cthat is a right triangle.
Specifically, consider A D E F with Z F = 90°, E F = a, and D F = b. Then, the Pythagorean Theorem tells us that
D E 2 = E F 2 4 D F 2 = a2 4 b2. Since we know that a2 4 b2 = c2, we have I ) E = c. Therefore, A D E F is a right triangle
with sides of length a, b, and c. Since A A B C has the same side lengths as A D E F , we have A A B C = A D E F by SSS.
Therefore, A A B C must also be a right triangle. □

Important: If the sides of a triangle satisfy the Pythagorean Theorem, then the triangle must be a
right triangle. Any time you see triangle sides that satisfy the Pythagorean Theorem,
Z you should mark the angle opposite the longest side as a right angle.

Sidenote: The ancient Egyptians used this method of determining if a triangle is right in
construction. They used loops of rope consisting of 12 rope segments of equal
length. When pulled taut to form a triangle with sides equal to 3, 4, and 5 of these
segments, they had a right angle opposite the 5-segment side! They could then use
this to make sure angles on buildings or bricks, etc., were right angles.

Let’s put this principle to work in a simple problem.

Solution for Problem 6.8: Since A B 2 4 B C 2 = A C 2, we know that A A B C is a right triangle with right angle at B. Therefore,
A C B D is a right triangle with hypotenuse C D , so we can use the Pythagorean Theorem:

C D 2 = B C 2 4 B D 2 = 16 4 81 = 97,

so C D = \/9 7. □

Exercises

6 .1 .1 : t V

Find the missing side length in each of the diagrams below:

(a) 5

Type your solution, notes and/or work here. Show Solution

(b)

Type your solution, notes and/or work here. Show Solution

(c)

Type your solution, notes and/or work here. Show Solution

(d)

10

Type your solution, notes and/or work here. Show Solution

(e)
M

Type your solution, notes and/or work here. Show Solution

(0

Type your solution, notes and/or work here. Show Solution

6.1.2 : t V

Prove that the two non-right angles of a right triangle are complementary.

Type your solution, notes and/or work here. Show Solution

6 .1 .3 : t V

One leg of a right triangle is 3 cm more than 3 times the other leg, and the hypotenuse is 1 cm longer than the longest leg. Find
the area of the triangle.

Hint

Type your solution, notes and/or work here. Show Solution

6 .1 .4 : t V

Prove that in a right triangle, the hypotenuse is the longest side.

Type your solution, notes and/or work here. Show Solution

6.1.5: t V

Which of the following groups of three numbers can be the side lengths of a right triangle?

(a) 6,8,10.

Type your solution, notes and/or work here. Show Solution

(b) 4,5,6.

Type your solution, notes and/or work here. Show Solution

(c) 9,3 \/3 ,6 \/3 .

Type your solution, notes and/or work here. Show Solution

(d) 5 /8 ,3 /2 ,1 3 /8 .

Type your solution, notes and/or work here. Show Solution

(e) 2 > /2 ,3 \/2 ,5.

Type your solution, notes and/or work here. Show Solution

(9 1.2,3.5,37.

Type your solution, notes and/or work here. Show Solution

6.1.6: t V

Find the area of a triangle with sides of length Уб, y /ï, and УТЗ.

Type your solution, notes and/or work here. Show Solution

6 .1 .7 : t V

Find the area of an isosceles triangle with two legs of length 8 and base of length 6.

Hint

Type your solution, notes and/or work here. Show Solution

6.1.8: t V

In this problem, we show that if *Al^ || c t ) , then A and B are the same distance from C7t).

(a) Let X and Y be on & /3 such that A X J_ b l 3 and B Y J_ c 3 . Show that Z .X A B = 90°.

Type your solution, notes and/or work here. Show Solution

(b) Prove that A A X B = A Y B X , and use this to prove that A and B are equidistant from C7$.

Type your solution, notes and/or work here. Show Solution

6 .1 .9 : t V

Initially, a 25-foot ladder rests against a vertical wall such that the top of the ladder is 24 feet from the ground. Then, Nathan
moves the base of the ladder farther out from the wall so that the top of the ladder slides down until resting against the wall at
a point 20 feet above the ground. Given that the wall is perpendicular to the ground, how far did Nathan move the base of the
ladder?

Type your solution, notes and/or work here. Show Solution

6. 1. 10★ : Source: MATHCOUNTS t V

A triangle has sides measuring 13 cm, 13 cm, and 10 cm. A second triangle is drawn with sides measuring 13 cm, 13 cm and
x cm, where a; is a whole number other than 10. If the two triangles have equal areas, what is the value of x l

Hint

Hint

Type your solution, notes and/or work here. Show Solution

Extra! It has long been a sport among lovers of mathematics to find new proofs of the
Pythagorean Theorem. Most engaging are the proofs that can be expressed simply as
v a diagram, challenging the viewer to fill in the details and learn how the diagram
provides a proof of the Pythagorean Theorem. Pure mathematicians are not the only
people who get to join in this game; proofs have been attributed to President James
A. Garfield and Leonardo da Vinci (among many others). We'll share several of these
'proofs without words’ with you in this book. You can find a long list of Pythagorean
Theorem proofs at Alexander Bogomolny's excellent Interactive Mathematics
Miscellany website and in Roger Nelsen's two Proofs Without Words books.
6.2 Two Special Right Triangles

Problems
Problem 6.9 4 Jump to Solution

A X Y Z is an isosceles right triangle as shown with Z Y = 90°. Given that X Z = 6, find Z X and X Y . ^

Y 2

Problem 6.10 4 Jump to Solution

(a) If a right triangle is isosceles, must it be the two legs that are the equal sides?

(b) Show that in any isosceles right triangle, the length of the hypotenuse is \ f l times the length of a leg.

Problem 6.11 4 Jump to Solution

In this problem we will derive a formula for the area of an equilateral triangle. We'll start with
/9
A A B C , an equilateral triangle with side length 4.

(a) Let A X be an altitude of A A B C as shown. Find B X . \4


y
(b) Find A X .

(c) Find the area of A A B C .


B >f c
(d) Let A P Q R be an equilateral triangle with side length s. Find [P Q R \ in terms of s.

We know a special side relationship when a triangle is isosceles, and another special relationship when a triangle is a right
triangle. What if the triangle is both?

Problem 6.9 t V

A X Y Z is an isosceles right triangle with Z Y = 90°. Given that X Z = G, find Z X and X Y.


X

‫־‬1 1 \
Y 1 2

Solution for Problem 6.9: Since the triangle is isosceles, the angles opposite the equal sides are equal. Therefore, Z X = Z Z .
Since Z Y is a right angle, Z X and Z Z must add to 90° So, Z X = Z Z = 9 0 °/2 = 45°.

To find X Y , we use the Pythagorean Theorem: X Y 2 A -Y Z 2 = X Z 2. Since Y Z = X Y and X Z — 6, we have


2 (X Y '2) = 36, so X Y = \ / l 8 = 3 7 2 . □

This problem suggests we might be able to find a general relationship among the sides in an isosceles right triangle.

Problem 6.10 t V

(a) If a right triangle is isosceles, must the two sides that are equal be the legs of the triangle?

(b) Show that in any isosceles right triangle, the length of the hypotenuse is \f2 times the length of a leg.

Solution for Problem 6.70:

(a) The Pythagorean Theorem quickly shows us that it must be the two legs that are equal in an isosceles right triangle.
Suppose our triangle is A A B C with right angle at Z C . The Pythagorean Theorem gives us A C 2 4‫ ־‬B C 2 = A B 2. So, if
A B equals A C or BC, then the other leg must be 0, which is impossible. Therefore, in any isosceles right triangle, it is the
two legs that are equal.

(b) If legs A C and BC each equal x, then we can find hypotenuse A B with the Pythagorean Theorem:
A B 2 = x 2 4- x 2 = 2 x 2. Therefore, A B = x \ [ 2 , so the hypotenuse must be \f 2 times a leg.

Important: In any isosceles right triangle, the legs are equal and the A
~7 hypotenuse is 7 2 times the leg. Because the angles of an
isosceles right triangle are always 45°, 45°, and 90°, an x 4 y \ A ‫־‬V2
isosceles right triangle is sometimes called a 45-45-90 triangle.
If we know the length of any side of such a triangle, we can n *K
quickly find the others using the side relationships shown in the * É
figure at the right.

Sidenote: Due to the theorem that bears his name, Pythagoras may be an even more well-
known name to the general public than the widely acknowledged 'father' of geometry,
** Euclid. Pythagoras lived during the 6th and 5th centuries BC (nearly 200 years before
Euclid), and was the founder of a secret society known, unsurprisingly, as the
Pythagoreans. It is unclear whether Pythagoras was the first to prove the Pythagorean
Theorem, but he and his organization were definitely responsible for a number of
other discoveries, including one that dramatically shook their understanding of the
world.

The Pythagoreans had a deeply held belief that every phenomenon in nature could be
described by whole numbers. However, their study of isosceles right triangles led
them to the startling conclusion that the ratio of the length of the hypotenuse of an
isosceles right triangle to the length of a leg of the triangle cannot be expressed as a
ratio of two integers. As we have seen in this section, this ratio equals V2.

To show that \P l cannot be written as a ratio of two integers, we use proof by


contradiction. This means we assume the opposite of what we want to prove, and
show that this assumption leads to an impossible conclusion. Here's what the proof
looks like:

We assume that \P l = p /q , where p and q are integers and p /q is in lowest terms.


Squaring \f2 = p /q gives 2 = p 2/ q 2, so we have p2 = 2q2. Therefore, p must be
even, so we can write p = 2m for some integer m. This gives (2 m )2 = 2q2, so
2m 2 = q2. Therefore, q is even, too. But p and q can't both be even if p /q is in lowest
terms!

Since our assumption that \[2 can be written as a ratio of integers in lowest terms
leads to the impossible conclusion that the ratio is also not in lowest terms, we have
found our contradiction that forces us to conclude that our assumption is wrong.
Therefore, \P l cannot be written as a ratio of integers.

We call a number that can be written as a ratio of integers a rational number. A


number like \f2 that cannot be so written is called an irrational number. Some
sources claim that the Pythagorean responsible for proving the existence of irrational
numbers, Hippasus, was rewarded for his insight by being murdered. Mathematical
discoveries are considerably more well-rewarded now!

Problem 6.11 t V

Find the area of an equilateral triangle with side length 4, then find a formula for the area of an equilateral triangle with side
length s.

Solution for Problem 6.77; To find the area, we will need to find the length of an altitude. Therefore, we
A
draw altitude A X as shown. Since Z B = Z C , Z A X B = Z A X C , and A B = A C , we have
A A X B = A A X C . Therefore, B X = C X = B C / 2 = 2. We can now use the Pythagorean
Theorem to find A X . Since A X 2 A B X 2 = A B 2 we have:

A X = V A B 2 - B X 2 = 7 1 6 ^ 1 = 7 1 2 = 273.

The area of A A B C is

lA B C l , iÊ S iâ n , , lV S .

We can use the exact same procedure to derive a formula for the area of an equilateral triangle with side length s. If we let the
sides of A A B C each be s and draw altitude A X as before, we have B X = C X = s/2. The Pythagorean Theorem then
gives

/IX ^ V A B t - B X ^ / s ’ - j = y ç = fA .

Now we can find our area:

{B C ){A X ) 6-(6-v/3/2) s2V?>


[A B C ] =

Important: The area of an equilateral triangle with side length s is 6Jv/3/4. If you understand
how we found this formula, you shouldn't need to memorize it!
Z
Our derivation of the formula for the area of an equilateral triangle suggests a notable relationship among the sides of a triangle
with angles 30°, 60°, and 90°

Problem 6.12 t V

(a) How are the sides of a triangle that have angle measures 30°, 60°, and 90° related?

(b) Suppose A P Q B has sides 3 , 3\/3, and 6. Can we deduce that the angles of A PQ R are 30°, 60°, and 90°?

Solution for Problem 6.72:


2(‫׳‬
(a) We found the area of an equilateral triangle by cutting the triangle into two right triangles that
have acute angles of 30° and 60°. Here we go backwards - we build an equilateral triangle
with two of these right triangles. In the diagram to the right, congruent right triangles A X Y Z
and A X Y D together form equilateral triangle X D Z .

Suppose the length of X Z is s. Since Z D = X Z = s and D Y = Y Z . v je have Y Z = s/2.


As we saw in Problem 6.11, we can use the Pythagorean Theorem on A X Y Z to find that
X Y = 6-\/3/2. Therefore, the sides in any triangle with angles 30°, 60°, and 90° come in the
ratio:

Leg opposite 30° : Leg opposite 60° : Hypotenuse = 1 : \/3 : 2.

In other words, the hypotenuse is double the leg opposite the 30° angle, and the leg opposite the 60° angle is \/3 times the
other leg.

For example, if the hypotenuse is 12, then the leg opposite the 30° angle is 12/2 = 6, and the leg
X
opposite the 60° angle is 6 \/3 as shown in the figure to the right.

We have just discovered the relationship among the sides of another common type of right triangle.

Y 6 2

Important: In a right triangle with acute angles of 30° and 60°, the X
7 sides are in the ratio 1 : 7 3 : 2 as shown to the right. Such
a triangle is often called a 30-60-90 triangle.
3 o \^
S tB

-,
Y s Z

(b) Having found this relationship among the sides given the angles, we wonder if all triangles that have sides in these ratios
are 30-60-90 triangles. We’ll consider one such triangle: A P Q R with sides 3, 3 \/3 , and 6. What's wrong with this 'proof‫׳‬
that A P Q R is a 30-60-90 triangle:

Bogus Solution: We extend R Q to S as shown such that !!u


Q S = Q R - 3. Since QS = QR and
y
\
P Q = PQ, we have A P Q R = A P Q S by SAS
Congruence. Therefore, P S = P R = 6. Clearly, 6/
R S = 6 also, so A P R S is equilateral. So,
Z R = 60° and Z R P Q = 90° - Z R = 30°, 3 V3 \
\
\\
and A P Q R is 30-60-90. /_ r
R 3 (2 3 i

Unfortunately, we have a big problem here: ‫׳‬Since Q S = Q R and P Q = PQ, we have A P Q R = A P Q S by SAS
Congruence.’ This statement assumes that Z P Q R = Z P Q S , which in turn assumes that Z P Q R is a right angle. But we
can’t assume that, because that's what we’re trying to prove!

There are a couple ways we can show that A P Q R is a 30-60-90 triangle.

Solution 7: Patch the hole in the Bogus Solution. We show that A P Q R is a right triangle by showing that its sides satisfy
the Pythagorean Theorem:

32 + (3 v/3 )2 = 9 + 27 = 36 = 62.

Therefore, A P Q R is a right triangle with right angle opposite the side with length 6. From here, we can use the Bogus
Solution to complete our proof that A P Q R is a 30-60-90 triangle.

Solution 2: We can use triangle congruence another way to write an even simpler proof! We compare our A P Q R to 30-60-
90 triangle A A B C , which has hypotenuse (i. Because A A B C is 30-60-90, we know that its legs have lengths 3 and 3 \/3 .
Therefore, A A B C = A P Q R by SSS Congruence, so the angles of A P Q R are the same as those of A A B C . In other
words, A P Q R is a 30-60-90 triangle.

Important: |f the side lengths of a triangle are in the ratio 1 : \/3 : 2, then the triangle is a
30-60-90 7‫ ־‬triangle, with the right angle opposite the longest side and the 30°
angle opposite the shortest side.

You've seen two proofs for one specific triangle above; you'll have a chance to write a proof for all such triangles in the
Exercises. (You knew that was coming!)

Let’s use our knowledge of 45-45-90 and 30-60-90 triangles to find some side lengths in a problem.

Problem 6.13 t V

Find all the missing sides in the diagram.


t
D ^ J \
Ÿ 60’/ \
,45°\ /
B
6
3
A C

Solution for Problem 6.13: Because A A B C is a 30-60-90 triangle and A B is opposite the 30° angle, we have
A C = A B \ / 3 = 6v/3 and B C = 2 ( A B ) = 12.

Because A B D C is a 45-45-90 triangle, we have B D = B C = 12 and C D = B C V 2 = 12\/2.

Because A E I ) C is a 30-60-90 triangle and C D is the longer leg, we have


C D = D E \ / 3 . Since C D = 1 2\/2, we have

^ CD 1 2 \/2 12v/2 v/3 12x/6 J ^


d * = 7 5 = ‫^־‬ = ^ T 7 ‫־‬I = ~ = 4 v /*•

We also have E C = 2E D = 8 \/6 . Finally, A E C F is a 45-45-90 triangle with


hypotenuse E C = 8 \/6 . Therefore, the legs each have length E C /\J 2 = 8 \/3 . All
our lengths are shown in the completed figure at right. =

Exercises

6 .2 .1 : t V

(a) Find A B , Z B, and Z C .

Type your solution, notes and/or work here. Show Solution

(b) Find D E , E E , and Z F .

Type your solution, notes and/or work here. Show Solution

(c) Find G I, Z G, and Z /.

Type your solution, notes and/or work here. Show Solution

(d) Find J K , J L , L M , Z L J M , and Z M .

Type your solution, notes and/or work here. Show Solution

: 7 3 : 2 is a right triangle.

Hint

Type your solution, notes and/or work here. Show Solution

6 .2 .3 : t V

Let A B C D be a square of side length 3, and let E, F , G, and / / be points on sides A B , B C , C D , and A t B
D A , respectively, such that A E = C F = C G = A H = 1.

(a) Prove that E F G H is a rectangle by showing that all of its angles are right angles.

Type your solution, notes and/or work here. Show Solution

(b) Find the area of E F G II.

Type your solution, notes and/or work here. Show Solution

Find the area of A P Q R given that P Q = Q R = 8 and Z P Q R = 120c

Hint

Type your solution, notes and/or work here. Show Solution

Source: MATHCOUNTS t V

In A A B C , A B = 20, Z .4 = 30°. and Z C = 45°. Find B C .

Hint

Type your solution, notes and/or work here. Show Solution


6.3 Pythagorean Triples
A Pythagorean triple is a set of three integers that satisfy the Pythagorean Theorem. For example, {3. 4 ,5 } is a Pythagorean
triple because 352 = 42 ‫ ־‬4 ‫־‬. However, {1. 2 ,3 } is not a Pythagorean triple since there's no way to add two of l 2 22, and 32 to
get the third.

Before proceeding with the problems, play around a little and try to find some Pythagorean triples on your own. Can you find any
patterns in your triples?

P roblem s

Problem 6.14
t V

(a) Find the missing side length in each right triangle shown below.

(b) Given that {a, 6. c ) is a Pythagorean triple, is {2a, 2 6 ,2c} also a Pythagorean triple? How about {n a , nb, nc), where n is
a positive integer?

X
D
A G
10 20 333
3 \ x 6 \
‫ ־‬1 n \ ‫ ־‬1 \ h \
B 4 C E F H 16 / Y 444 2

Solution for Problem 6.14:

(a) We can apply the Pythagorean Theorem to find

AC = >/ AD '2 + B C 2 = \ / 9 + 16 = 5
EF = V D F 2 - D E 2 — \/6 4 = 8
GH = s/ G I 2 - H P = \ / l 4 4 = 12
X Z = \ / X Y 2 + Y Z 2 = \/somo big numbers = 555

OK, we got a little lazy on that last one. We noted in the first three that the sides are always in the ratio 3 : 4 : 5, so we took
a stab at 555 for X Z. We’re right, of course, but why?

There are a couple ways we can prove that we are right. First, we could note that A X Y Z ~ A A B C due to SAS
Similarity. Second, we could use algebra, as we’ll see in the second part of the problem.

(b) We are given that {a , 6 ,c } is a Pythagorean triple, so a 2 + 62 = c2. To test {2a, 2 6 ,2c}, we consider the sum of the
squares of the first two terms:

(2 a )2 4- (26)2 = 4a2 4462 ‫־‬


= 4 (a 2 4- 62)
= 4c2(because a2 462 ‫ = ־‬c2)
= (2 c)2.

Therefore, {2a, 26, 2c} is a Pythagorean triple. There's nothing special about 2; we can run through this with any number:

( n a ) 2 4‫ ( ־‬n b ) 2 = n 2d 2 4- n 262 = n 2( d 2 462 ‫ = )־‬n 2c 2 = ( ti c ) 2.

So, {n a , n6, n c ) is also a Pythagorean triple. Notice also that n doesn't have to be an integer to produce side lengths of a
right triangle! For example, since a triangle with sides {3, 4 ,5 } is a right triangle, a triangle with sides

{3 (1 /7 ), 4 (1 /7 ), 5 (1 /7 )} = { 3 / 7 , 4 / 7 , 5 / 7 }

is a right triangle.

You will often see triangles with sides in this ratio referred to as 3-4-5 triangles. Triangles whose sides are in the ratios of
other Pythagorean triples are sometimes similarly identified, such as 5-12-13 triangles or 7-24-25 triangles.

Important: If {a , 6, c ) are the sides of a right triangle, then so are {n a , nb, n c ) for any positive
7‫־‬ number n.

Therefore, we can generate whole groups of Pythagorean triples from a single triple:

{ 3 ,4 ,5 } —> { 6 ,8 ,1 0 } —> {9 ,1 2 ,1 5 }
- 4 {3 3 3 ,4 4 4 ,5 5 5 } 4 • • •

There's nothing special about integer multiples of our {3, 4 ,5 } triple. We can generate all sorts of other sets of three numbers
that are the sides of a right triangle. Here are a few:

{0 .3 ,0 .4 ,0 .5 } { 3 /1 1 ,4 /1 1 ,5 /1 1 } { 6 \/5 ,8 \/5 ,1 0 v /5 }

Recognizing common Pythagorean triples, or multiples of those triples, is a very useful problem solving tool. The following
problem exhibits some common triples, as well as a neat pattern for generating more.

Problem 6.15
t V

(a) Find the last side in each right triangle.

(b) Do you see a pattern? Can you prove the pattern always works? 3e x 6:; ‫׳‬° $
4 15

12 37
24 1

Solution for Problem 6.15:

(a) We can use the Pythagorean Theorem to find the rest of the sides as shown below.

(b) If we take the smallest even leg, halve it, then square the result, we get the number
that is one more than the other leg and one less than the hypotenuse. Putting this
in math terms, we let our smallest even leg have length 2n. Then the other two
sides are n — 1 and n 4 1 ‫־‬. We can make a table that shows what happens for
different values of the even leg.

Smallest Even Leg Other Leg Hypotenuse


n 2n nz - 1 na+ 1
2 4 3 5
3 6 8 10
4 8 15 17
5 10 24 26
6 12 35 37
7 14 48 50

All the triangles described in the table are indeed right triangles. We can show that any triangle with sides 2n, n 2 - 1, and
n 2 4 1 ‫־‬, where n > 1, is a right triangle with some algebra:

(2 n )2 4‫( ־‬n 2 - l ) 2 4 n 2 - f n 4 - 2 n 2 4-1


r i 4 4 2 ‫־‬n 2 4 1 ‫־‬
( n 2 4‫ ־‬l ) 2.

Sidenote: Choose three positive integers k. m, and n, where m > n. Compute the three
^ numbers k ( n r — n~), 2 kmn, and k (m 2 4‫ ־‬ri2). Try it for several different values of k
.m , and n. You should find that the three numbers you generate in each case form a
Pythagorean triple. See if you can figure out why this works.

Even more surprising is the fact that every single Pythagorean triple fits this mold. In
other words, for every single Pythagorean triple, we can always find some k, m, and n
such that

{ 2 km n , k (rn 2 — ri2), k (rn 2 4‫ ־‬ri2)}

generates that triple.

Problem 6.16
t V

Find the missing side in each of the triangles below without using the Pythagorean Theorem I
directly. Just use your knowledge of Pythagorean triples! A **r 2.5
M 0.7
Q
P- R
20 *
72 -
□__
B K
\ c 78
z

Solution for Problem 6.16: We note that the legs of A A B C are in the ratio 15/20 = 3 /4 . Therefore, our legs are in the same
ratio as the legs in a 3-4-5 triangle. We can then conclude that the hypotenuse is 5 /3 of the smallest leg, or (5 /3 )(1 5 ) = 25.

We might also have found A C by noting that the legs of A A B C are 20 = 4(5) and 15 = 3(5), so this is a 3-4-5 triangle.
Therefore, we know that A C = 5(5) = 25.

In A P Q R , the ratio of the given leg to the hypotenuse is 0 .7 /2 .5 = 7/25. This makes us think of the Pythagorean triple
{7 ,2 4 ,2 5 }. Since 0.7 = 7(0.1) and 2.5 = 25(0.1), we know the other leg is 24(0.1) = 2.4.

In A X Y Z , the ratio of the given leg to the hypotenuse is 72/78 = 12/13, which reminds us of the Pythagorean triple
{5 ,1 2 ,1 3 }. Since our leg is 72 = 12(6) and the hypotenuse is 78 = 13(6), the other leg must be 5(6) = 3 0 .

Notice our general tactic for finding the missing length in each triangle in Problem 6.16 - we take the ratio of the sides we know
and then try to recognize a simple Pythagorean triple that has the numbers in our ratio. □

WARNING!! Make sure you see that it's important that the 2.5 length in A P Q R of Problem 6.16
is the hypotenuse and not a leg. A triangle with legs 0.7 and 2.5 does not have third
u side 2.4. In fact, it doesn’t match any simple Pythagorean triple. Therefore, you must
be careful when using your knowledge of Pythagorean triples to make sure your given
sides match corresponding sides (legs or hypotenuse) of the Pythagorean triple you
would like to use.

Sidenote: Perhaps inspired by Pythagorean Triples, the great mathematician Pierre de Fermat
explored the equation

x n 4 yn = zn

for values of n besides n = 2 (n = 2 gives the Pythagorean Theorem, of course).


That there are no positive integers x, y and z that satisfy this equation when n is an
integer larger than 2 is known as Fermat's Last Theorem. Although it seems like a
simple proposition, Fermat's Last Theorem evaded proof for hundreds of years until
Andrew Wiles proved it in 1994. Alas, his proof is far too long to fit in the margins of
this book.

Exercises

6 .3 .1 : t v

Which of the following are side lengths of a right triangle? (Try using your knowledge of Pythagorean triples!)

(a) 300,400,500.

Type your solution, notes and/or work here. Show Solution

(b) 36,48,60.

Type your solution, notes and/or work here. Show Solution

(c) v/5, V Ï2 , V Ï3 .

Type your solution, notes and/or work here. Show Solution

(d) 20,37.5,42.5.

Type your solution, notes and/or work here. Show Solution

(e) 1.44,1.96,2.4.

Type your solution, notes and/or work here. Show Solution

(f) 15,36, 39.

Type your solution, notes and/or work here. Show Solution

6 .3 .2

(a) Is there a right triangle such that all three side lengths are odd integers?

Hint

Type your solution, notes and/or work here. Show Solution

(b) Is there a right triangle such that two of the side lengths are even integers and the other is an odd integer?

Hint

Type your solution, notes and/or work here. Show Solution

(c) Is there a right triangle in which two side lengths are simple fractions (ratios of integers, such as 2 /3 or 3/7), and the
other side length is an integer?

Type your solution, notes and/or work here. Solution

(d) Is there a right triangle in which one of the side lengths is a simple fraction and the other two side lengths are integers?

Type your solution, notes and/or work here. Solution

6 .3 .3

Susie rides her bike north 3 miles, then east 8 miles, then south 13 miles, then east 16 miles. How far is she from where she
started?

Type your solution, notes and/or work here. Solution

In this problem we find all Pythagorean triples in which 18 is one of the legs.

(a) Let 6 and c be the other two sides, where c is the hypotenuse. Show that c2 — b2 = 182.

Type your solution, notes and/or work here. Solution

(b) Factor the left-hand side of the equation from part (a) and use the factors of 182 to solve for all possible pairs of integers
(6, c)that satisfy the equation.

Type your solution, notes and/or work here. Show Solution

6 .3 .6 ★ : t V

Find the Pythagorean triple that has 97 as the length of the hypotenuse without using a computer or calculator.
Hint

Type your solution, notes and/or work here. Solution


6.5★ Heron’s Formula
As we saw when discussing SSS Congruence, all triangles with the same three side lengths are congruent. Therefore, they all
have the same area. So, we wonder, ‘Since the area is fixed given the three side lengths, is there a way to figure out what the area
is?‫׳‬

In this section well answer this question with the Pythagorean Theorem and a lot of algebra.

Problems
Problem 6.21 4 Jump to Solution

In this problem, we find the area of A A B C , which has sides of length 13 ,14, and 15. We start by
/
drawing altitude A X . Let it have length h and let B X = x.

(a) What is X C in terms of x?


13/
h V 5
(b) Apply the Pythagorean Theorem to A A B X to get an equation with h and x in it.

(c) Apply the Pythagorean Theorem to A A C X to get an equation with h and x in it. /X ! \
(d) Subtract your equation in (c) from the one in (b) to find x. Use x to find h.
в X 14 C
(e) Find the area of the triangle.

(f) How could you use your knowledge of Pythagorean triples to avoid all this algebra?

Problem 6.22 4 Jump to Solution

In this problem we will try to find a formula for the area of a triangle in terms of only its sides. As
usual, we let A B = c, B C = a, and A C = 6. We start again with altitude A X = h, and we let
B X = x. Finally, we let s = (a 4 b 4 c) / 2 to simplify our algebra in the problem. (Warning:
There’s a lot of algebra in this problem. You will use the difference of squares factorization many
times: y2 — z2 = (y - z) ( y 4‫ ־‬z).)

(a) Apply the Pythagorean Theorem to A A B X to get an equation with a, b, c, h, and/or x in it.

(b) Apply the Pythagorean Theorem to A A C X to get an equation with a, b, c, h, and/or x in it.

(c) Subtract your equation in (b) from the one in (a) to get an equation with just x, a, b, and c. Use this equation to show that
x = (a2 A c2 — 62)/(2 a ).

(d) Substitute your expression from (c) into one of your earlier equations to show that

2 ^ /s (s — a )(s — b)(s — c)
h =
a

(e) Show that

[ABC] = yjs(s — a )(s — b)(s — c).

Before we tackle finding a formula for the area of a triangle in terms of its sides, we try finding the area of a specific triangle.

Concept: Often it’s best to try a few examples before attempting to derive a general formula.
I The examples can be a good guide when you try to derive your formula.

Solution for Problem 6.21: We start by drawing altitude A X = h of A A B C as shown. We let


B X = x, so that X C = 14 — x. We have some right triangles, and lots of lengths, so we try the
Pythagorean Theorem. Applying the Pythagorean Theorem to triangles A A X B and A A X C yields:

x 2 4‫ ־‬h2 = 169
(14 — x ) 2 4 ‫ ־‬h 2 = 225

Seeing h 2 on the left of both equations, we think to subtract the second equation from the first to
eliminate h 2. We get

x 2 — (14 — x ) 2 = 169 - 225.

Therefore, x 2 — 196 4 2Sx — x 2 = —56, so x = 5. We can substitute x = 5 into either of our original equations to find
h = 12. Therefore,

(HC)(AX)
[ABC] = Ш М =84
2

Seeing that x and h are integers, we wonder if we could have found them without the algebra. Triangles A A B X and A A C X
share a leg and have 13 and 15 as their respective hypotenuses. This makes us think of the Pythagorean triples 5-12-13 and 9-
12-15, which have the needed hypotenuses and a leg length in common. Fortunately, the two other legs, 5 and 9, add up to 14, the
length of B C . Therefore, when we glue a 5-12-13 triangle to a 9-12-15 along the leg with length 12, we get a triangle with sides
13,14,15.□

Unfortunately, we usually can't find Pythagorean triples that fit together nicely to make our triangle. However, we can follow our
procedure from Problem 6.21 to find a formula for the area of a triangle in terms of its sides.

Put on your algebra h a t...

Problem 6 .2 2 t V

In triangle A B C , let A B = c, A C = b, and B C = a and let s = (a 4 5 + c ) / 2. Find a formula for [A B C ] in terms of s, a,


b, and c.

Solution for Problem 6.22: We set up the problem in the same way as the last problem, drawing
A
altitude A X and letting A X = h and B X = x, so X C = a — x.

We apply the Pythagorean Theorem to A A B X and A A C X to get:

x2 A h 2 = c2
(a - x ) 2 4 h 2 = b2

Again, we can subtract the second equation from the first to find x:

x 2 - (a - x ) 2 = c2 — b2.

Therefore, x 2 - a 2 4 2a x — x 2 = c2 — b2, so 2ax = c2 — b2 4 a 2, and we have

a2 4 c2 — b2
x =
2 a

That wasn't so bad, but finding h looks a little scary. We can make the algebra somewhat nicer by solving x 2 4 b 2 = c2 for / r
and factoring using the difference of squares:

h 2 = c2 — x 2 = (c - x )(c 4 t ).

Substituting our expression for x into this equation gives

a t+ c t-b ^ f a2 + c2 - 6 2
h2 = - c4 —
2a ) \ 2a
2ac — a2 — c2 4 b2 \ ( 2ac 4 a2 4 c2 — 62
4 2a ) \ 2a

Now we recognize —(a — c)2 = —a 2 4 2ac — c2 and (a 4 c)2 = a 2 4 2ac 4 c2 in the numerators:

b2 — (a — c )2 \ / (a 4 c)2 — b2
h2 =
( 2a )(
) \ 2a )
[b - (a. - c)][6 4 (a - c)] \ ( ( a A c - b )(a A b A c )
4 2a ) ) \(

( 6 4 c — a )(a A b — c )(a A c — b )(a 4 6 4 c )
2a

4^2 •

(Make sure you see how we used the factorization y 2 — z 2 = (y — z )(y 4 z) to simplify the algebra.)

We can write h 2 even more briefly with the substitution s = (a 4 6 4 c ) / 2. We call this s the semiperimeter, i.e. one-half the
perimeter. From this, we have 2s = a 4 6 4 c, so 6 4 c — a = 2s — 2a, Similarly, we can substitute for all the terms in the
numerator of our h 2 expression:

2 (2.s - 2a)(2.s - 2c)(2.s - 26)(2.s)


h2 =
4a2
4.s(.s - a )(s - b)(s — c)
a*

Now that we have an expression for h, we can find the area:

a j4 s (s — a )(s — b)(s — c)
= 2V a2
= y j s(s — a )(s — b)(s — c).

It's not always the nicest way to find the area of a triangle, but if we have the lengths of the three sides, we can find the area with:

Important: Herons Formula tells us that if the sides of a triangle are a, 6, and c, and we let
^ $ = (a 4 6 4 c)/2, then the area of the triangle is

yjs(s — a)(.s — 6)(.s — c).

Aside from algebra, the only tool we used in deriving Heron’s formula was the Pythagorean Theorem.

Concept: When in doubt, build right triangles and use the Pythagorean Theorem if you’re trying
I to find a length.

Here are a few triangles on which to try Heron’s formula. You should compare your answers to the areas you have found with
other methods.

Solution for Problem 6.23: In each case, there are other ways to find the area of the triangle. See if you can find the area using
these other ways and compare your result to Heron's formula.

A A B C : s = (6 4 6 4 6 )/2 = 9, so

[A D C ] = ^ 9 ( 9 - 6 )(9 - 6 )(9 - 6) = 9 \/3 .

A D E F : s = (8 + 15 + 1 7 )/2 = 20, so

[ D E F ] = y 60 = (8 - 20)(15 - 20)(17 - 20)20‫׳‬.

A G H I : s = (7 + 7 + 1 0 )/2 = 12, so

[G H I] = ^ /1 2 ( 1 2 - 10)(12 - 7 )( 12 - 7) = 10%/6.

A . I K L : s = (13 + 14 + 1 5 )/2 = 21, so

[ J K L ] = ^ 2 1 (2 1 - 13)(21 - 14)(21 - 15) = 84.

Lest you start to think Heron's is always the way to go, try finding the area of a triangle with sides 2 \/3 , 4, and 2 \/7 , or a triangle
with sides \ / l 3 , \/Т З , and V6. Heron doesn't look so pretty in either case, but the answers can be found rather quickly with
other methods. See if you can figure out how.

Exercises

6 .5 .1 : * V

Extra! Chinese mathematician Liu Hui based a proof of the


Pythagorean Theorem on the diagram at right in the
V third century A.D. (Of course, he probably didn’t call
the result the Pythagorean Theorem!) See if you can
fill in the details!
6.4 Congruence and Similarity Revisited

P roblem s
Problem 6.17 4 Jump to Solution

Are the two triangles shown congruent? Why or why not?

Problem 6.18 4 Jump to Solution

In this problem we will prove that if radius O C of © O is perpendicular to chord A B of ©O, then radius O C bisects chord
‫־‬A B .

(a) Draw a diagram to use in your proof. Include both radius O C and chord A B .

(b) Find a pair of congruent triangles that you can use to complete the proof.

Problem 6.19 4 Jump to Solution

The center of a circle is 4 units away from a chord P Q of the circle. If P Q = 12, what is the radius of the circle?

In this section we discuss a special congruence theorem and a special similarity theorem that only work for right triangles.

Solution for Problem 6.17: What's wrong here:

Bogus Solution: Two sides and an angle of one triangle equal two sides and an angle of the other, so
the triangles are congruent.

The problem here is that SSA is not in general a valid congruence theorem, as we found in Section 3.5. However, if we note that
the angles that are equal are right angles...

We can use the Pythagorean Theorem to find A B and F G . In both cases, the length is \ / l0 0 — 81 = ■\/l9. Therefore, by SSS
(or SAS), we have A A B C = A F G E . □

In the same way, any time the hypotenuse and a leg of one right triangle equal those of another, we can use the Pythagorean
Theorem to show that the third sides of each triangle are equal. In other words, SSA works for right triangles. Rather than give it
an unwieldy name like 'SSA, but just for right triangles’, we call it:

Important: Hypotenuse-Leg Congruence (HL Congruence) states that if the hypotenuse and a
-r leg of one right triangle equal those of another, then the triangles are congruent.

As you might guess, there's a Leg-Leg Congruence (LL Congruence), but that's just SAS Congruence (make sure you see why), so
there’s really no need to give this a special name.

Problem 6.18 t V

Prove that if radius O C of © O is perpendicular to chord A B of ©O, then radius O C bisects chord A B .

Solution for Problem 6.18: As usual, we start with a diagram, including both our radius and our chord. We
C
add the radii to point A and B so we have right triangles to work with. Since O A = O B and
O X = O X , we have A O X A = A O X B by HL Congruence. Therefore, A X = B X , so radius O C
bisects chord A B . □

Problem 6.18, combined with Problem 3.4, gives us a very useful tool:

Important: If a radius of a circle bisects a chord of the circle, it is perpendicular to the chord.
7‫־‬ Conversely, if a radius of a circle is perpendicular to the chord, it bisects the chord.

We can use this tool on a variety of circle problems. For example:

Problem 6.19 t V

The center of a circle is 4 units away from a chord P Q of the circle. If P Q = 1 2 , what is the radius of the circle?

Solution for Problem 6.19: We let point M be the point on P Q closest to the center of the circle, which
we'll call O. Since M is the point on P Q closest to O, we have O M A. P Q . Because O M is part of a
radius that is perpendicular to chord P Q , O M bisects chord P Q . So, we have P M = M Q = 6.
Finally, we apply the Pythagorean Theorem to A O P M to find

OP = VPM 2+ OM 2 = v/36 + 16 = 2\/Î3.


I

Unsurprisingly, the Pythagorean Theorem can be used to give us a Hypotenuse-Leg Similarity Theorem as well.

Problem 6.20 t V

Are the two triangles in the diagram similar? Why or why not?
/4 K
20
12
/1 5
n r
(C P 16 Q

Solution for Problem 6.20: What's wrong with this:

Bogus Solution: Since 20/15 = 16/12, we have two pairs of sides in a common ratio and Z A = Z Q
, so by SAS Similarity, the triangles are similar.

SAS Similarity requires the angle to be between the two sides that have the constant ratios, so we can't use SAS Similarity like
this.

We recognize these two triangles as 3-45‫ ־‬triangles (or we use the Pythagorean Theorem), and we find that A B = 9 and
Q R = 12. Therefore, we have

AB _ AC _ BC
R Q ~ P Q ~ PR '

so A A B C ~ A Q R P by SSS Similarity. □

In much the same way, we can use the Pythagorean Theorem to show that if a hypotenuse and a leg of one right triangle are in
the same ratio as the hypotenuse and leg of another right triangle, then the other legs are also in that ratio. So, the two triangles
are similar. Therefore, we have a new similarity theorem just for right triangles.

Important: Hypotenuse-Leg Similarity (HL Similarity) states that if the hypotenuse and a leg of
one right triangle are in the same ratio as the hypotenuse and leg of another right
Z triangle, then the triangles are similar.

As with congruence, there is a Leg-Leg Similarity Theorem that states that if the legs of one right triangle are in the same ratio
as the legs of another right triangle, then the two triangles are similar. However, this is just the same thing as SAS Similarity.
(Make sure you see why!)

Exercises

6 .4 .1 : t V

Which of the following pairs of triangles are similar? For the pairs that are similar, write both the similarity relationship and why
the two triangles are similar.

(a)

Type your solution, notes and/or work here. Show Solution

(b)

Type your solution, notes and/or work here. Show Solution

(c)

Type your solution, notes and/or work here. Show Solution

(d)

Type your solution, notes and/or work here. Show Solution

Points A and B are on a circle with radius 9. Given that the center of the circle is 3 units from A B , find A B

Type your solution, notes and/or work here. Show Solution

In this problem we prove the HL Similarity Theorem we discovered in Problem 6.20. Let A A B C and A M N O be right
triangles with right angles at Z A and Z M . Furthermore, suppose A B / B C = M N / N O . We wish to prove that
A A B C ~ A M NO.

(a) Use the Pythagorean Theorem to find expressions for A C 2 and M O 2.

Type your solution, notes and/or work here. Show Solution

(b) Use the given equation A B / B C = M N / N O to find an expression for A B . Write an expression equal to A C 2/ M O

Type your solution, notes and/or work here. Show Solution

(c) Substitute your expression for A B into your expression for A C 2/ M O 2. Prove that A C 2/ M O 2 = B C 2/ N O 2.

Hint

Type your solution, notes and/or work here. Show Solution

(d) Prove that A A B C ~ A M N O .

Hint

Type your solution, notes and/or work here. Show Solution

Source: HMMT t V

A D and B C are both perpendicular to A B in the diagram at left below, and C D _L A C . If A B = 4 and B C = 3, find C D .

Type your solution, notes and/or work here. Show Solution

A B C D is a square in the diagram at right above. M is the midpoint of A D and B M J_ M N .

/1 b

(a) Prove that A M D N ~ A B A M .

Type your solution, notes and/or work here. Show Solution

(b) ★ Prove that Z A B M = Z M B N .

Hint

Type your solution, notes and/or work here. Show Solution


6.6 Construction: Perpendicular Lines

P roblem s
Problem 6.24 4 Jump to Solution

Construct a line through A that is perpendicular to line n. (Hint: See Problem 3.23.)

n
<---------------------------------------------->

With all this talk about perpendicular lines and right triangles, we should wonder how to construct perpendicular lines with a ruler
and compass.

Problem 6.24 t V

Given line n and point A, construct a line through A that is perpendicular to n.

Solution for Problem 6.24: Back in Section 3.7 here, we constructed a very specific
perpendicular line given a segment. Maybe we can use what we learned there to make
a line through A perpendicular to n. To do so, we'll have to find two points on n such
that A is directly 'above' the midpoint of the segment connecting the two points. So,
we draw a circle with center A that hits n in two points, X and Y . Therefore,
A X = AY.

We can then mimic our construction in Section 3.7 here by drawing circles with
centers X and Y and radius A Y . Since A Y = A X , these circles will meet at A Let /'ft
point B be the other point where these circles meet. Since A X = A Y , X B = Y B,
and A B = A B , we have A A X B = A A Y B. Therefore, / . M A X = / M A Y . Together with A X = A Y and A M = A M ,
this gives us A M A X = A M A Y by SAS Congruence. Finally, we have / X M A = / Y M A , so these angles are each 90°
because together they make up a straight angle.

Therefore, 5 5 passes through A and is perpendicular to n. □

Now that we can create perpendiculars, we can use right triangles in our constructions. Let's give it a try.

Problem 6.25 t V

Suppose the shown segment has length 1. Construct a segment with length VlO.

Solution for Problem 6.25: Seeing the square root, we think of the Pythagorean Theorem, since we've seen a lot
of square root signs in our right triangle problems. Since l 2 4 1 0 = 32 ‫־‬, we want to make a right triangle with/ Z
legs of length 1 and 3. The hypotenuse of this triangle will have length \ / l0 . We start by extending PQ, then
‫־‬0

r .
*

constructing a line through Q perpendicular to W ) as described in Problem 6.24. Specifically, our steps for
constructing the perpendicular line are: V ✓

Z
1. Draw a circle with center Q and radius QP. Name the second point where this circle hits P Q point X .
2. Draw circles with centers P and X , both with radius P X . Name the points where these circles meet Y and Z.

3. Draw Sf 5 , which is the line through Q perpendicular to PQ. (Make sure you see why this line must go through Q.)

Once we have this perpendicular line, we create a segment of length 3 along it by drawing a circle with center Q
and radius Q P to get point R on the perpendicular such that Q R = 1. Then we can copy Q R a couple times to
get S and T such that R S = S T = 1, so Q T = 3. Segment P T is then the hypotenuse of a right triangle with
legs of lengths 1 and 3, so it has length \ / l 2 - f 32 = y/lQ . □

Exercises

6 .6 .1 : t V

Construct a 45-4590‫ ־‬triangle and a 30-60-90 triangle.

Type your solution, notes and/or work here.

6 .6 .2 : t V

Given a segment of length 1, construct a segment of length \/3 .

Type your solution, notes and/or work here.


6.7 Summary

Definitions: A right triangle is a triangle that has a right angle among its angles. The side opposite
the right angle is called the hypotenuse and the other two sides are the legs.

Important: In any right triangle, the sum of the squares of the legs ^
■y equals the square of the hypotenuse. Or, in the figure to the
right, we have ^ XC

a2 + b2 = c2. c 1‫־‬ \ c
a b
This is the famous Pythagorean Theorem. The Pythagorean
Theorem also runs in reverse: If the sides of a triangle satis! y the Pythagorean
Theorem, then the triangle must be a right triangle. The longest sic e of such a triangle
is the hypotenuse, and the angle opposite it is a right angle.

Two special right triangles, the 45-4590‫ ־‬triangle and the 3 0 9 0 ‫־‬60‫ ־‬triangle, are shown below. The relationships among the sides
of each triangle are as indicated.

Conversely, if you determine that the side lengths of a triangle are in the ratio 1 : 1 : \/2 , you can deduce that the triangle is a 45‫־‬
4590‫ ־‬triangle. If the side lengths are in the ratio 1 : \/3 : 2, then the triangle is a 3 0 9 0 ‫־‬60‫ ־‬triangle with the 30° angle opposite
the shortest side.

Important: We used the Pythagorean Theorem to prove the following useful facts:

Z ■ The distance from a point to a line is the length of the segment from the point to
the line that is perpendicular to the line.

■ The area of an equilateral triangle with side length s is s2 v/3/4. If you understand
how we found this formula, you shouldn’t need to memorize it!

■ Herons Formula tells us that if the sides of a triangle are a, b, and c, and we let
s = (a + b 4‫ ־‬c ) j 2, then the area of the triangle is
co
cc

cc

cc
1

Definition: A Pythagorean triple is a set of three integers that satisfy the Pythagorean Theorem.
Some common Pythagorean triples are:

{ 3 ,4 ,5 } {5 ,1 2 ,1 3 } {7 ,2 4 ,2 5 } {8 ,1 5 ,1 7 } {9 ,4 0 ,4 1 }.

Important: If {a, b ,c} are the sides of a right triangle, then so are {n a , nb, n c ) for any positive
7‫־‬ number n.

We also investigated two new congruence theorems and two new similarity theorems specifically for right triangles:

■ HL Congruence. If the hypotenuse and a leg of one right triangle equal those of another, then the triangles are congruent.

■ LL Congruence. If the legs of one right triangle equal those of another, then the triangles are congruent. (This is the same
as SAS Congruence.)

■ HL Similarity. If the hypotenuse and a leg of one right triangle are in the same ratio as the hypotenuse and leg of another
right triangle, then the triangles are similar.

■ LL Similarity. If the legs of one right triangle are in the same ratio as the legs of another right triangle, then the triangles are
similar. (This is the same as SAS Similarity.)

We used HL Congruence to prove the following useful fact:

Important: If a radius of a circle bisects a chord of the circle, it is perpendicular to the chord.
Conversely, if a radius of a circle is perpendicular to the chord, it bisects the chord.
Z
P roblem Solving S trateg ies

Concepts:
■ Building right triangles and applying the Pythagorean Theorem is one of the most
common ways to find lengths. This is particularly true in problems that already
involve right angles.

■ Often it's best to try a few examples before trying to derive a general formula - the
examples can be a good guide when you try to derive your formula.

T hin g s To W atch Out For!

WARNING!! Be careful when using your knowledge of Pythagorean triples to make sure your given
sides match corresponding sides (legs or hypotenuse) of the Pythagorean triple you
O would like to use.

Extra! The dazzling tiling 'proof without words' of the


Pythagorean Theorem shown at right comes from
V Annairizi of Arabia (circa 900 AD). See if you can figure
out how it works! Source: Proofs Without Words II by
Roger Nelsen
Review Problems

t V

Find the missing side length in each of the triangles below:

(a)

Type your solution, notes and/or work here. Show Solution

(b)

Type your solution, notes and/or work here. Show Solution

(c)

V Ï7

Type your solution, notes and/or work here. Show Solution

t V

Find the missing side length in each of the triangles below by using your knowledge of Pythagorean triples:

(a)

Type your solution, notes and/or work here. Show Solution

(b)

Type your solution, notes and/or work here. Show Solution

(c)

Type your solution, notes and/or work here. Show Solution

t V

Find the missing side lengths in the triangles below:

(a)

Type your solution, notes and/or work here. Show Solution

(b)

Type your solution, notes and/or work here. Show Solution

t V

For each of the following parts, state whether or not the three numbers listed could be the side lengths of a right triangle,

(a) 11,16,19.

Type your solution, notes and/or work here. Show Solution

(b) 1/5,4/15,1/3.

Type your solution, notes and/or work here. Show Solution

(c) v/7 3 ,2 v 2,9‫׳‬.

Type your solution, notes and/or work here. Show Solution

(d) x /o li, \ / L 2 , vT3.

Type your solution, notes and/or work here. Show Solution

(e) 0.77,2.64,2.75.

Type your solution, notes and/or work here. Show Solution

6 .3 0 : t V

Given that A B = 3, B C = 6, and / . A B C = 60°, why must A A B C be a right triangle? (You cannot simply say ,It is a 30-
60-90 triangle; you must prove that it is!')

», K 'l
Type your solution, notes and/or work here.

6 .3 1 : t V

In this problem, we find yet another proof of the Pythagorean Theorem.

a b a

(a) Construct a square of side length a + b, and place four right triangles with legs of lengths a and b in it as shown in the
left diagram. Let K be the area of each shaded triangle. Express the area of the whole square in terms of K and the
areas of the two interior squares.

Type your solution, notes and/or work here. Show Solution

(b) Now place the four triangles as shown in the right diagram and let c be the length of the hypotenuse of each of the
triangles. Express the area of the large square in terms of K and the area of the interior square.

Type your solution, notes and/or work here. Show Solution

(c) What do you conclude from parts (a) and (b)?

Type your solution, notes and/or work here. Show Solution

Source: MATHCOUNTS t V

In the figure at right, A B = B C = l . A D || B C and C D || A E . Find A E .

Type your solution, notes and/or work here. Show Solution

t V

Find the perimeter of an isosceles triangle with base length 10 and area 60.

Type your solution, notes and/or work here. Show Solution

t V

Using any method you like, find the areas of the triangles with the side lengths below:

(a) 6,8,10.

Type your solution, notes and/or work here. Show Solution

(b) 3,4,6.

Type your solution, notes and/or work here. Show Solution

(c) v/6, v/24, v/30.

Type your solution, notes and/or work here. Show Solution

(d) 1,24/7,25/7.

Type your solution, notes and/or work here. Show Solution

(e) 5,6,7.

Type your solution, notes and/or work here. Show Solution

Source: MATHCOUNTS t V

In the figure shown, E is the midpoint of A B , D is the midpoint of A C , A B = 16, and A C = 1 2 .

(a) Show that A E F B ~ A C A B .

Type your solution, notes and/or work here. Show Solution

(b) Find D G and E F .

Type your solution, notes and/or work here. Show Solution

(c) What is the area of A E F G D ?

Type your solution, notes and/or work here. Show Solution

6 .3 6 : t V

Find the area of an equilateral triangle with a height of length 8.

Type your solution, notes and/or work here. Show Solution

6 .3 7 : Source: MATHCOUNTS t V

In A M N O , M N = 13, and N O = 37. P is on M O such that N P _L M O and N P = 12. Find the area of A M N O .

Type your solution, notes and/or work here. Show Solution

6 *3 8 • t V

Let A B C be a right triangle, with / A C B = 90°. Let D be the foot of the perpendicular from C to side A B . Let x = A D
and y = B D . Prove that C D = y/xy.

Type your solution, notes and/or work here. Show Solution

t V

In the diagram below, A B = A C , A F ± B C , F D _L A B , and F E ± A C . Prove that E D = E E .

Type your solution, notes and/or work here. Show Solution

6 .4 0 : Source: MATHCOUNTS t V

In the diagram at right, B D = 6, A B = 3, and D E = 5. What is the length of A E ?

Type your solution, notes and/or work here. Show Solution

6 .4 1 : t V

The length of one leg of a right triangle is 22, and the other two sides also have integer lengths. Find the perimeter of the
triangle.

Type your solution, notes and/or work here. Show Solution

6 .4 2 . t V

Point A is on side X Z of A X Y Z such that X A = X Y . Given that / Y X Z = 90°, / Y Z X = 30° and


Z A = 6 — \ / l2 , find the area of A X Y Z .

Type your solution, notes and/or work here. Show Solution

Source: AMC 10 t V

The sides of a triangle have lengths of 15, 20, and 25. Find the length of the shortest altitude of the triangle

Type your solution, notes and/or work here. Show Solution


Challenge Problems

A right triangle has area 210 and hypotenuse 29. Find the perimeter of the triangle.

Hint

Hint

Type your solution, notes and/or work here. Show Solution

What's wrong with the diagram below?

Hint

Type your solution, notes and/or work here. Show Solution

6 .4 6 : Source: AMC10 t V

In rectangle A B C D , A D = 1, P is on A B , and D B and D P trisect /.A D C . (An angle is trisected when it is divided in
three equal angles.) What is the perimeter of A B D P ?

Hint

Type your solution, notes and/or work here. Show Solution

The hypotenuse of a right triangle has length 8. The triangle’s area is also 8. Find the perimeter of the triangle.

Hint

Type your solution, notes and/or work here. Show Solution

6.48: Source: ARML t V

In the diagram at right / B = / D = 90°, A B — D C = 24, and B C = A D = 32. b u

(a) Prove that A A B E = A C D E .

Type your solution, notes and/or work here. Show Solution

(b) Let M be the midpoint of A C . Prove that E M _L A C .

Type your solution, notes and/or work here. Show Solution

(c) Find the area of A A E C .

Hint

Type your solution, notes and/or work here. Show Solution

6 .4 9 : Source: ARML t V

Triangle A B C has A B = 12, B C = 16, and A C = 20. If D is on A C such that A D = 12, find the area of A + D B .

Hint

Hint

Type your solution, notes and/or work here. Show Solution

6.50: Source: AMC 10 t V

Riders on a Ferris wheel travel in a circle in a vertical plane. A particular wheel has radius 20 feet and revolves at the constant
rate of one revolution per minute. How many seconds does it take a rider to travel from the bottom of the wheel to a point 10
vertical feet above the bottom?

Hint

Type your solution, notes and/or work here. Show Solution

6.51 Source: AMC 12 t V

Shown at right is rectangle A B C D . Angle C is trisected by C F and C E , where E is on


A B , F is on A D , B E = 6, and A F = 2.

(a) Find B C .

Type your solution, notes and/or work here, Show Solution

(b) Find D F .

Type your solution, notes and/or work here. Show Solution

(c) Find [A B C D ].

Type your solution, notes and/or work here. Show Solution

Initially, a fifty-foot ladder rests against a wall. As I start to climb it, the ladder slides down, finally stopping such that it touches
the wall at a point 8 feet below where it originally touched the wall. During the slide, the base of the ladder slid 16 feet from its
original position. How far is the top of the ladder from the ground after the slide, given that the wall is perpendicular to the
ground?

Hint

Type your solution, notes and/or work here. Show Solution

t V

Using only a pencil and paper, find the Pythagorean triple with 73 as the length of the hypotenuse.

Hint

Hint

Type your solution, notes and/or work here. Show Solution

6 .5 4 ★ : t V

If the lengths of the three sides of a right triangle are whole numbers, how many such distinct noncongruent right triangles
exist having one leg with a length of 24 units?

Type your solution, notes and/or work here. Show Solution

6 .5 5 : t V

In A R S T , R S = 13, S T = 14, and R T = 15.

(a) Find the length of the height from R to ST.

Type your solution, notes and/or work here. Show Solution

(b) ★ Let M be the midpoint of S T . Find R M .

Hint

Type your solution, notes and/or work here. Show Solution

6 .5 6 ★ : t V

We begin with an equilateral triangle with side length 1. We divide each side into three segments of equal length, and add an
equilateral triangle to each side using the middle third as a base. We then repeat this, to get a third figure.

If we continue this process forever, what is the area of the resulting figure?

Hint

Hint

Type your solution, notes and/or work here. Show Solution

6 .5 7 ★ : Source: AMC 10 t V

Let A X O Y be a right-angled triangle with Z X O Y = 90°. Let M and N be the midpoints of legs O X and O Y ,
respectively. Given that X N = 19 and Y M = 22, find X Y .

Hint

Hint

Type your solution, notes and/or work here. Show Solution

6 .5 8 ★ : t V

In triangle A B C , A B = A C , B C = \ / 3 — 1, and Z B A C = 30°. Find the length of A B .

Hint

Type your solution, notes and/or work here. Show Solution

6 .5 9 ★ : t V

In A A B C , A B = 6 and B C = 8. Find ,4 C in each of the following cases:

(a) Z B = 30°

Hint

Type your solution, notes and/or work here. Show Solution

(b) Z B = 45°

Hint

Type your solution, notes and/or work here. Show Solution

(c) Z B = 135°

Hint

Type your solution, notes and/or work here. Show Solution

Extra! The great modern mathematician Paul Erdos was fond of describing particularly beautiful proofs as
being from ‫׳‬the book.‫ ׳‬For sport, many aesthetically inclined math fans find beautiful ‫׳‬proofs without
V words' by using diagrams designed to ‫׳‬show‫ ׳‬the truth of a mathematical statement rather than ‫׳‬saying‫ ׳‬it.
In our various proofs without words of the Pythagorean Theorem, we've seen proofs without words
applied to the most natural area of mathematics for them, geometry. However, the diagrams below show
that the tools of geometry can be used to ‫׳‬prove without words' statements from other fields of
mathematics, as well. Perhaps you'll agree that these proofs are indeed ‫׳‬from the book.‫׳‬

□EB

Match the images above with the following mathematical statements:

+ n 2 = n(n + l)(2n + 1)
l 2 + 22 + 32 + 42 +

H + G M + - - 4

n ( n 41 ‫)־‬
1 + 2 + 3 + 4 4 ‫ •••־‬+ ™=

1 + 3 + 5 + 7 + 9 + ••• + (2 n — 1) = n 2.

Make sure you see how each statement is illustrated by its matching image!

Source: Proofs Without Words by Roger Nelsen


Why isn't there a special name for the tops o f your feet? - Lily Tomlin

CHAPTER
I
I Special Parts of a Triangle
Most of mastering geometry is mastering triangles. In this chapter we explore several special points, lines, and circles that can
be found in any triangle.

In general, any line segment from a vertex of a triangle to a point on the opposite side is called a cevian. Before we dive into the
special parts of a triangle, well investigate a couple of special lines.

7.1 Bisectors
The perpendicular bisector of a line segment is the line passing through the midpoint of the segment such that the line is
perpendicular to the segment. In Figure 7.1, line k is the perpendicular bisector of A B .

Figure 7.1: A Perpendicular Bisector

The angle bisector of an angle is the ray that divides the angle into two equal angles. In Figure 7.2, ray m is the angle bisector of
ZAOB.

P roblem s
Problem 7.1 4 Jump to Solution

Line m is the perpendicular bisector of A B . Use congruent triangles to show that if C is on m, then C A = C B .

Problem 7.2 4 Jump to Solution

In this problem, we will show that if A C = B C , then C is on the perpendicular bisector of A B .

(a) Let C be a point such that A C = B C and let M be the midpoint of A B . Draw a diagram.

(b) Draw C M , then use congruent triangles to prove that C M _L A B . Why does this mean that C is on the perpendicular
bisector of A B ?

Problem 7.3 4 Jump to Solution

In this problem we show that any point on the angle bisector of Z X Y Z is equidistant from Y X and Y%.

(a) Let C be on the angle bisector of Z X Y Z . Draw Z X Y Z and segments from C to the sides of Z X Y Z that are
perpendicular to Y ) t and Y~%, respectively.

(b) Use congruent triangles to prove that the two segments from C to the sides of the angle are equal in length.

Before we start tackling problems involving special lines in triangles, we will use what we already know about congruent triangles
to learn about these special lines. Well start with perpendicular bisectors.

Problem 7.1 t V

Line m is the perpendicular bisector of A B . Show that if C is on m, then C A = C B .

Solution for Problem 7.1: Drawing A C and C B , we see that A A M C = A B M C by SAS


Congruence since A M = M B , Z A M C = Z B M C , and C M = C M . Therefore, A C = C B . □

Having shown that every point on the perpendicular bisector of a segment is equidistant from the endpoints of the segment, we
should wonder if every point that is equidistant from these endpoints has to be on the perpendicular bisector of the segment
connecting them.

Problem 7.2 t V

Show that if A C = B C , then C is on the perpendicular bisector of A B .

Solution for Problem 7.2: To show that C is on the perpendicular bisector of A B , we connect C to the
midpoint of A B , which well call M , then show that C M _L A B . Since M is the midpoint of A B , we
have A M = M B . We are given A C = C B , and obviously C M = C M , so we have
A A C M = A B C M by SSS Congruence. Therefore, Z A M C = Z B M C . Since these two angles
must add to 180°, they must each equal 90°. So, C M is the perpendicular bisector of A B . (This proof
doesn’t address the possibility that C is the midpoint of A B . We take care of this by noting that the
midpoint of A B is on the perpendicular bisector of A B by definition.) □

Putting these last two problems together tells us something very important about the perpendicular bisector of a segment.

Important: The perpendicular bisector of a segment is the straight line consisting of all points
that are equidistant from the endpoints of the segment.
Z
We can prove something very similar for angle bisectors.

Problem 7.3 t V

Show that any point on the angle bisector of Z X Y Z is equidistant from Y and Y %.

Solution for Problem 7.3: We start by drawing the lengths we need to show equal. These are the
perpendicular segments C P and C Q from C to Y j l and Y 2 as shown. By AAS Congruence,
we have A C P Y S A C Q Y , so C P = C Q .

Therefore, any point on the angle bisector of an angle is equidistant from the sides of the angle. □

As with the perpendicular bisector, we can 'run this backwards,‫ ׳‬deducing that any point that is equidistant from the sides of an
angle must be on the angle bisector.

Problem 7.4 t V

C is a point inside Z X Y Z such that C is equidistant from Y $ and Y as shown. Show that C
must be on the angle bisector of Z X Y Z .

Solution for Problem 7.4: We have A C P Y = A C Q Y by HL Congruence because C P = C Q


and C Y = C Y . Therefore, Z C Y P = Z C Y Q , so Y (5 bisects Z X Y Z . □

Putting the last two problems together tells us a little more about angle bisectors.

Important: The angle bisector of an angle consists of all points that are equidistant from the
7‫־‬ sides of the angle.

Exercises

7 .1 .1 : t V

Line m is the perpendicular bisector of both A B and C D . Which of the following must be true and why (or why not)?

(a) A B = CD.

Type your solution, notes and/or work here. Show Solution

(b) A B || C D .

Type your solution, notes and/or work here. Show Solution

(c) * A C = B D .

Hint

Type your solution, notes and/or work here. Show Solution

7 .1 .2 : t V

Describe all the points that are equidistant from two intersecting lines.

Type your solution, notes and/or work here. Show Solution

7 .1 .3 : t V

Describe all the points that are equidistant from two parallel lines.

Type your solution, notes and/or work here. Show Solution


7.2 Perpendicular Bisectors of a Triangle

Problems
Problem 7.5 4 Jump to Solution

Let k and £ be perpendicular bisectors of the sides A B and A C of A A B C as shown. b


(a) From what two points must every point on k be equidistant?
k /
(b) From what two points must every point on £ be equidistant?

(c) Let O be the intersection of k and £. Why must O be on the perpendicular bisector of / O
BC? 4 -----H------
/4
(d) If we draw a circle with center O and radius O A, will the circle go through B and C as 1
well? Why or why not?

Problem 7.6 4 Jump to Solution

Line m is the perpendicular bisector of side X Y of right triangle A X Y Z , and m meets


X
X Z at L as shown.

(a) Show that A X K L ~ A X Y Z . m : \ L


K
(b) Use part (a) to show that L is the midpoint of X Z .
‫־‬1
(c) Where do the perpendicular bisectors of the sides of a right triangle meet? Y Z

(d) What is the radius of the circle that passes through X , Y , and Z?

Problem 7.7 4 Jump to Solution

Ariel and Zappa are playing 'NAME THAT CIRCLE.' In ,NAME THAT CIRCLE,' Ariel imagines a circle drawn on her sheet of paper.
Zappa can then ask for points that are on the circle. Zappa wins when he can draw the whole circle.

(a) Can Zappa win the game if Ariel only gives him two points?

(b) Can Zappa win if Ariel gives him three points?

(c) Zappa asks for four points. Ariel isn't really paying attention and just picks out four points at
random on the piece of paper and ends up with points shown in the diagram at right. Zappa
tries and tries, but he can't NAME THAT CIRCLE. Finally, he insists that there is no such circle.
Could he possibly be right or should he just try harder?

Ariel's points

Problem 7.8 4 Jump to Solution

In this problem we find the radius of a circle that passes through all three vertices of an equilateral triangle. Let A A B C be our
triangle, with A B = 6. Let P be the midpoint of B C and Q the midpoint of A C , and let O be the center of the circle through
the vertices of A A B C .

(a) Draw a diagram. Include the perpendicular bisectors of A C and B C . Where must these perpendicular bisectors
intersect?

(b) Prove that A O C P is a 30-60-90 right triangle.

(c) Find C P , then find the radius of the circle.

Now that we've learned a little bit about perpendicular bisectors, we‫׳‬re ready to study the perpendicular bisectors of the sides of
a triangle.

Problem 7 .5 t V

Show that the three perpendicular bisectors of the sides of a triangle are concurrent, meaning they meet at a single point. Show
that there is a circle centered at this point that passes through all three vertices of the triangle.

Solution for Problem 7.5: We start by thinking about just two of the perpendicular bisectors, k
b
and £, shown in the figure at left. Let k and £ meet at O. We wish to show that O is on the
perpendicular bisector of B C .

Since O is on the perpendicular bisector of A B , it is equidistant from A and B, i.e. O A = O B.


Similarly, since O is on the perpendicular bisector of A C , we have O A = O C . So,
O A = O B = O C . Since O B = O C , O must be on the perpendicular bisector of B C (see
Problem 7.2), so the perpendicular bisectors of the sides of a triangle are concurrent.

Seeing O A = O B = O C , we note that if we draw a circle with center O and radius O A , the
circle will pass through B and C as well. □

The point O and the circle described in the last problem are so useful in geometry problems that they have their own names.

Important: The perpendicular bisectors of the sides of a triangle


are concurrent at a point called the circumcenter. The
Z circle centered at the circumcenter that passes
through the vertices of the original triangle is called
the circumcircle of the triangle because it is
circumscribed about the triangle (meaning it passes
through all the vertices of the triangle).

Finally, the radius of this circle is called the


circumradius, the circumcenter is usually labeled with
the letter O, and the circumradius is usually called R .

In right triangles, the circumcircle is particularly easy to describe.

Problem 7 .6 t V

Where is the circumcenter of a right triangle? What is the circumradius of a right triangle?

Solution for Problem 7.6: If we draw a few right triangles and their perpendicular bisectors, we
X
will begin to suspect that the midpoint of the hypotenuse is the circumcenter.

We start our proof by drawing the perpendicular bisector of one of the legs, such as line m in
the diagram. Let m meet X Z at L. Since m and Y Z are both perpendicular to X Y , we have
m || Y ‫־‬Z. Therefore, A X K L ~ AXYZ, so X L /X Z = X K /X Y . Since
X K / X Y = 1/2, we have X L / X Z = 1/2, so L is the midpoint of X Z . Clearly, L is also
on the perpendicular bisector of X Z (since L is the midpoint of X Z ) , so L must be where
the perpendicular bisectors of A X Y Z intersect. Therefore, L is the circumcenter of A X Y Z .

The endpoints of the hypotenuse are vertices of the triangle, so by definition they are on the circumcircle. Since the midpoint of
the hypotenuse of our right triangle is the circumcenter, the hypotenuse is a diameter of the circumcircle. Therefore, the
circumradius is half the length of the hypotenuse of the right triangle, z

Back in Problem 1.2.1, we saw that two points are enough to define a line. All this talk about circles going through the vertices of
a triangle should make us wonder - how many points of a circle do we need to define a circle? Clearly, one won't do, since we
can draw all sorts of circles through a given point.

Problem 7 .7 t V

Ariel and Zappa are playing ,NAME THAT CIRCLE.' In 'NAME THAT CIRCLE,' Ariel imagines a circle drawn on her sheet of paper.
Zappa can then ask for points that are on the circle. Zappa wins when he can draw the whole circle.

(a) Can Zappa win the game if Ariel only gives him two points?

(b) Can Zappa win if Ariel gives him three points?

(c) Zappa asks for four points. Ariel isn't really paying attention and just picks out four points at random on the piece of paper.
Zappa tries and tries, but he can't NAME THAT CIRCLE. Finally, he insists that there is no such circle. Could he possibly be
right or should he just try harder?

Solution for Problem 7.7:

(a) Clearly two points isn’t enough - we can make all sorts of different circles given two points. This is
shown at right.

(b) Given three points, Zappa can NAME THAT CIRCLE! He can connect the points to make a triangle.
Then, he can draw the perpendicular bisectors of two of the sides of the triangle. Where they meet is
the circumcenter. He can then make the circumcircle. Notice that the three points must be
noncollinear, meaning that it’s impossible to draw a single line that contains all three points.

The center of any circle that passes through all three points must be on all three perpendicular
bisectors of the sides of the triangle with these points as vertices. Therefore, the circumcenter is the only possible center of
a circle through the three points. Therefore, the circumcircle is the only circle that passes through all three points.

(c) Let the points be A, B, C, and I). As described in the previous part, there is only one circle
•Ü
through A, B, and C. But this circle doesn't have to go through D\ Therefore, if we pick out
four points at random, there doesn’t have to be a circle that passes through all four points.
Zappa should give up once he sees that the circumcircle of A A B C doesn’t pass through
I). One such arrangement of points is shown at right.

Important: Just as two points determine a line, we have now shown that three noncollinear
points determine a circle. This means that given any three noncollinear points, there is
Z exactly one circle that passes through all three.

Now let's try a problem involving the circumradius.

Problem 7 .8 t V

Find the circumradius of an equilateral triangle with side length 6.

Solution for Problem 7.8: Let A A B C be our equilateral triangle. We connect its circumcenter to a
A
vertex and draw a couple perpendicular bisectors as shown. Since C Q = C P and O C = OC, we
know that A O C P = A O C Q by HL Congruence. Therefore, Z O C Q = Z O C P , so O C must
bisect Z A C B . Hence, Z O C P = Z A C B / 2 = 30° and A O C P is a 30-60-90 triangle. Since P is
the midpoint of B C , we have C P = 3, so

“ ‫ ־‬c,’ S ) ‫ ־‬r ‫־‬A

Exercises

7 .2 .1 : t V

Where is the circumcenter of an acute triangle - in, on, or outside the triangle? How about an obtuse triangle? (You do not
need to prove your answer - you'll learn tools later that will make these proofs easier.)

Type your solution, notes and/or work here. Show Solution

Find the radius of the shown circle with center O given that B C = 8 and Q O = 2.

Type your solution, notes and/or work here. Show Solution

Find the circumradius of an equilateral triangle with side length 18.

Type your solution, notes and/or work here. Show Solution

What is the circumradius of a right triangle with legs of length 6 and 8?

Type your solution, notes and/or work here. Show Solution


7.3 Angle Bisectors of a Triangle

P roblem s
Problem 7.9 4 Jump to Solution

Let k be the angle bisector of / . C A B and m be the angle bisector o f / A B C . Let k and m meet at
c
/.

(a) Every point on k must be equidistant from what two lines?

(b) Every point on m must be equidistant from what two lines?

(c) Why must / be on the angle bisector of / C l

Problem 7.10 4 Jump to Solution

The angle bisectors of A X Y Z meet at /, and perpendicular segments from / to the sides of
the triangle are drawn as shown.

(a) Why is I R = I Q = I P i

(b) Suppose we draw a circle with center / and radius / P. Does the circle hit Y Z at any point
besides P i

(c) Show that the circle from (b) is tangent to sides X Z and X Y at Q and R, respectively.

Problem 7.11 4 Jump to Solution

Segments G H , I I I , and I G are tangent to the circle with center C as shown. Given that
/ G H I = 70° and / G I H = 50°, find the following:

(a) /C IH .

(b) /H C I.

Problem 7.12 4 Jump to Solution

In the diagram, B P is the angle bisector of / B . In this problem, we will prove the Angle Bisector
B
Theorem, which states that if B E is an angle bisector as shown, then A B / A E = C B /C E .

(a) Ratios make us think of similar triangles, which make us think of parallel lines. Draw a line
through C parallel to A B , then extend B E past E to hit this new line at X . What triangles
are similar?

(b) What type of triangle is A B C X l

(c) Use your similar triangles from (a) and your observation in (b) to prove that A B / A E = C B / C E .

Problem 7.13 4 Jump to Solution

Find A C and X Q in the triangles shown. { X Z = 12)

C Y
6 D 7 8 Z

Problem 7.14 4 Jump to Solution

Back in Problem 7.10, we discovered a circle that is tangent to all three sides of A X Y Z . In this
problem, we will discover a formula for the area of A X Y Z in terms of the radius of this circle
and the perimeter of the triangle.

(a) Break A X Y Z into three triangles: A X Y /, A Y Z I , and A Z X /. Find the area of each
triangle in terms of one of the sides of A X Y Z and r, the radius of the circle in the
diagram.

(b) Find the desired formula for [ X Y Z ] by adding [ X Y I \ [Y Z I], and [ Z X I \

Problem 7.15 4 Jump to Solution

In this problem we find the radius of a circle that is tangent to all three sides of A A B C given that the sides of A A B C have
lengths 7, 24, and 25.

(a) What kind of triangle is A A B C l What are the area and perimeter of A A B C l

(b) What is the radius of the circle tangent to all three sides of A A B C l

Having found that the perpendicular bisectors of the sides of a triangle are concurrent, we wonder if a similar proof can be used
to show that the bisectors of the angles of a triangle are also concurrent.

Problem 7.9 t V

Prove that the angle bisectors of a triangle are concurrent.

Solution for Problem 7.9: We start much as we did with the perpendicular bisectors. We consider the
C
intersection of two of our angle bisectors, point / in the diagram. Since / is on the bisector of / C A B ,
it is equidistant from 5(3and .4 A Since it is on the angle bisector of / A B C , it is equidistant from
5 5 and 5 (3 . Therefore, / is equidistant from all three sides of the triangle. Specifically, since / is
equidistant from 5 (3 and 5(3, / is also on the angle bisector of / A C B , as we proved in Problem 7.4.

We found a special circle centered at the intersection of the perpendicular bisectors. Let's see if the intersection of the angle
bisectors is the center of a special circle, too!

Problem 7.10 t V

The angle bisectors of A X Y Z meet at I, and perpendicular segments from / to the sides of
the triangle are drawn as shown.

(a) Why is I R = I Q = I P i

(b) Suppose we draw a circle with center / and radius / P. Does the circle hit Y Z at any point
besides P i

(c) Show that the circle from (b) is tangent to sides X Z and X Y at Q and R, respectively.

Solution for Problem 7.7 0:

(a) In the previous problem, we found that /, the intersection of the angle bisectors of A X Y Z , is equidistant from the sides of
A X Y Z . Since / R, IQ , and / P are the distances from / to the sides of A X Y Z, we have I R = I Q = I P .

(b) One way we can show that the circle with center / and radius I P doesn't hit Y Z anywhere else is to show that all the other
points on Y Z are outside the circle.

As shown, point V is a point on Y Z besides point P. From right triangle A I P V , we have


X
I V 2 = I P 2 4‫ ־‬V P 2, so I V > I P . Our circle consists of all points exactly I P from /.
Since V is farther than I P from /, it must be outside our circle. There’s nothing special
about V; all points on Y Z besides P are outside our circle. Therefore, it is impossible for the
circle with radius / P to hit Y Z at a second point. So, the circle must be tangent to Y Z .

(c) Since I P = I Q = IR , the circle with center / and radius I P goes through Q and R, too.
For the same reason as in the previous part, this circle is tangent to all three sides of A X Y Z

We have found the center of a special circle at the intersection of the angle bisectors of a triangle.

Important: The angle bisectors of a triangle are concurrent at a


X
point called the incenter. This point is equidistant from
Z the sides of the triangle. This common distance from
the incenter to the sides of a triangle is called the
inradius, because the circle with center / and this
radius is tangent to all three sides of the triangle. This
circle is unsurprisingly called the incircle because it is
inscribed in the triangle (meaning it is tangent to all the
sides of the triangle). The incenter is usually denoted /, and the inradius is usually
written as r.

Each triangle has exactly one incircle. (You’ll be asked to prove this in Problem 12.43.) Therefore, if a circle is tangent to all three
sides of a triangle, its center is the intersection of the angle bisectors of the triangle.

Solution for Problem 7.77: Since the circle is tangent to all three sides of A G I I I, it is the incircle of A G I I I . Therefore, C is the
incenter of A G 111, so C l and C H are bisectors of angles / G i l l and / G I I I, respectively.

/ C H I = ( / G H I ) /2 = 35° and / C I H = ( / G I H ) / 2 = 25°, so we have

/ H C I = 180° - 35° - 25° = 120°.

The incenter and the incircle aren't the only useful aspects of angle bisectors.

Problem 7.12 t V

Given that E is on A C such that B E is the bisector of / A B C , prove that A B j A E = C B / C E .

Solution for Problem 7.72: Seeing ratios, we think of similar triangles. Similar triangles make us think of
parallel lines. We draw a line through C parallel to A B and we extend B E past E to hit this new line
at X . We pick this parallel line to draw because it lets us use the angles that are formed by the angle
bisector. Specifically, we have / B X C = / A B X . / A B X also equals / X B C , so A X C B is
isosceles and C B = C X . A

Our parallel line gives us similar triangles: A A B E ^ A C X E . This similarity gives us


A B / A E = C X /C E . Since C B = C X , we have the desired A B / A E = C B /C E . □
X

Important: The Angle Bisector Theorem states that if E is on A C B


such that B E is the angle bisector of / B in A A B C , then
Z
AB CB / /\ \
AE ~ C E' A *-
We should think of this any time we have a problem
involving lengths and angle bisectors.

For example, try using the Angle Bisector Theorem on this problem.

Solution for Problem 7.13: From the Angle Bisector Theorem applied to A A B C , we have A B / B D = A C / C D , so
1 2 /6 = A C / 7 and A C = 14.

Finding X Q is a little more challenging. We let X Q = x, so Q Z = 12 — x. The Angle Bisector Theorem tells us

XY ZY
XQ ~ ZQ’

so we have

9 _ 8
x 12- x '

Cross-multiplying gives 9(12 — a:) = 8a:, so x = 108/17. Therefore, X Q = x = 108/17. Note that
X Q = ( Y X ) ( X Z ) / ( Y X 4‫ ־‬Y Z ) . Is this a coincidence? □

But wait, there’s more. We can even relate an incircle to the area of its triangle!

Problem 7.14 t V

Find the area of A X Y Z in terms of its inradius and its side lengths.

Solution for Problem 7.14: Those radii of the incircle perpendicular to the sides make us think of
X
altitudes. Indeed, if we break A X Y Z into three triangles with / as a vertex, these radii are
altitudes. Adding the areas of these three triangles gives us [ X Y Z \

[ X Y Z ] = [ X Y I ] 4‫ [ ־‬Y Z I ] 4‫ [ ־‬X Z I ]
(IR)(XY) , (IP)(YZ) , (IQ)(XZ)
««‫־־‬ ‫*־‬ * /‫\־‬ 'V *

Since I R = I P = I Q = r, the inradius, and X Y 4- Y Z 4‫ ־‬X Z = 2s, where s is the


semiperimeter of A X Y Z , we have:

|Y V ~ r(XY) r(YZ) r(XZ)


lX Y Z ' ~ ‫ ־‬T ‫ ־־‬+ ‫ ־‬1 ‫־‬ + ‫ ־‬1 ‫־־‬

7‫ ־‬A 2 j
= TS.

Important: The area of a triangle equals its inradius times its semiperimeter.

Z
This [A B C ] = r.s can be a useful tool in problems involving the area of a triangle or the inradius (or both).

Problem 7.15 t V

Find the radius of a circle that is tangent to all three sides of A A B C given that the sides of A A B C have lengths 7, 24, and
25.

Solution for Problem 7.15: Since 72 4252 = 242 ‫־‬, the triangle is a right triangle with legs of length 7 and 24. (This problem is an
example of why it's useful to recognize Pythagorean triples.) So, we can easily find [ A B C ] = (7 )(2 4 )/2 = 84. Since we can
also find the semiperimeter, we can find the inradius. Because s = (7 4 2 8 = 2 /(2 5 ‫ ־‬24 4‫־‬, we have

[A B C ] 84 0

Exercises

7 .3 .1 : t V

A B , A C , and B C are tangent to QO . If / A = 70°, / B = 72°, and / C = 38°, what is the measure &
of / O A B l

Hint

Type your solution, notes and/or work here. •i . *•L » . * * .« .

Is the incenter of an obtuse triangle inside, outside, or on the triangle?

Type your solution, notes and/or work here. Show Solution

Can the incenter and the circumcenter of a triangle ever be the same point?

Type your solution, notes and/or work here. •i f v * • 11 l i

P S and R T are angle bisectors of A P Q R , and / P I R = 130°.

(a) Given/ R P Q = 30°, find / P R T , / P R Q , and ZQ.

Type your solution, notes and/or work here. •Î fv*•. I I •

(b) Given/ R P Q = 50°, find / P R T , / P R Q , and ZQ.

Type your solution, notes and/or work here. •i . *•L » . * * .« .

(c) Given/ R P Q = 80°, find / P R T , / P R Q , and ZQ.

Type your solution, notes and/or work here. •. »I •

(d) Do you notice anything unusual in your answers for ZQ? Why does this unusual pattern occur?

Type your solution, notes and/or work here. •i f v * • 11 l i

Find R Z in the figure at right if P R = 9, Q Z = 4, P Q = 6, and P Z bisects / Q P R .

Type your solution, notes and/or work here.

In A A B C , A B = 10, A C = 12, and B C = 8. Point M is on side B C such th a t/ B A M = / C A M . Find B M .

Type your solution, notes and/or work here. Show Solution

For each part below, find the length of the inradius of a triangle with the given numbers as side lengths,

(a) 3,4,5

Type your solution, notes and/or work here. Show Solution

(b) 6,6,6

Type your solution, notes and/or work here. Show Solution

(c) 7,7,10

Type your solution, notes and/or work here. Show Solution

(d) 5,6,7

Type your solution, notes and/or work here. Show Solution


7.4 Medians
A median of a triangle is a segment from a vertex to the midpoint of the opposite side. In the figure below, A D , H E , and C F
are all medians.

In each of the problems in this section, you can assume the medians of a triangle are concurrent. Well prove that the medians of
a triangle are concurrent in Chapter 17. You’ll also have a chance to do it yourself in the Challenge Problems. (Hint: We suggest
re-reading Section 4.3.)

P roblem s
Problem 7.16 4 Jump to Solution

In this problem we will prove that the medians of a triangle divide the triangle into 6 triangles
of equal area. We will make heavy use of the same base/same altitude principle we studied in
Section 4.3.

(a) Let [D G B ] = x, [C G E \ = y, and [A G E ] = z as shown in the diagram. Prove that


[D G C ] = x, [A G E ] = y, and [D G F ] = z.

(b) Show that [A I3 D ] = [A C D ] and use this to prove that y = z.

(c) Prove that the six triangles formed by drawing the medians of A .4 D C have equal area.

Problem 7.17 4 Jump to Solution

Use triangles A A G H and A D G B in the previous problem to prove that the centroid cuts each median in a 2 : 1 ratio,
specifically A G / G D = 2.

Problem 7.18 4 Jump to Solution

A P Q R is an equilateral triangle with side length 12.


(a) Show that each angle bisector of an equilateral triangle is also a median.

(b) Find the length of the inradius of A P Q R .

Problem 7.19 4 Jump to Solution

Points D, E, and F a re the midpoints of the sides of A A B C as shown below.


C
(a) Show that A B D F ~ A B C A.

(b) Show that A D E F ~ A A B C .

(c) Show that E F || C B .

(d) Show that each of the four small triangles in the diagram has area equal to 1/4 the area of
A ABC.

Problem 7.20 4 Jump to Solution

A A B C has sides of length A B = 9, B C = 12, and C A = 15. Find the length of the median from B to the midpoint of
AC.

Hint

Hint

Medians create triangles with equal bases; therefore, they create triangles with equal areas. We can use these equal areas to
learn even more about the medians of a triangle.

Problem 7.16 t V

Prove that the medians of a triangle divide the triangle into six triangles with equal area.

Solution for Problem 7.16: Let our triangle be A A B C with midpoints D, E, and F, and with
C
centroid G, as shown. We have [B G D ] = [C G D ] because these triangles have the same
altitude from G to equal bases B D and C D . Similarly, [A G E ] = [C G E ] and
[A E G ] = [B E G ]. We assign variables to these areas as shown in the diagram at right.

The little triangles aren’t the only triangles that share altitudes. Since A A B D and A A C D
have the same altitude from A to equal bases B D and D C , we have [A B D ] = [A C D ].
Therefore, x + 2 z = x + 2y, so y = z. Similarly, we can use [A F C ] = [B F C ]to show that
x = y, so we have x = y = z and all six little triangles have the same area, z

Important: The medians of a triangle divide the triangle into six little triangles of equal area.

Z_______________________________________
We can use this property of medians to discover an interesting property of the centroid.

Problem 7.17 t V

Prove that the centroid of any triangle cuts each of the triangle’s median in a 2 : 1 ratio, with the longer portion being the
segment from the centroid to the vertex.

Solution for Problem 7.17: Our work with areas gives us the solution right away. Since A A G B
C
consists of two of the little equal-area triangles and A G D B is only one of them, we have
[A G B ] = 2 [G D B \ These two triangles share an altitude from B to bases A G and G D , so

AG [A G B ] o
G D ~ [G D B ] ‫־‬ '

Clearly this works for any median, c

This simple fact can be useful in a variety of problems. Here’s an example.

Problem 7.18 t V

A P Q R is an equilateral triangle with side length 12. Find the length of the inradius of A P Q R .

Solution for Problem 7.18: Let P L and R N be angle bisectors of A P Q R . Since the angle
P
bisectors of an equilateral triangle are also the medians of the triangle, L is the midpoint of R Q and
the incenter, /, is also the centroid. So, / divides median P L such that P I / 1 L = 2/1. Therefore,
I L = P L / 3.

A P L R is a 30-60-90 triangle, so R L = P R / 2 = 6 and P L = 7?L\/3 = 6\/3. Finally, we have


I L = P L / 3 = 2\/3. (Note: We could also have solved this problem by noting that A I R L is a 30-
60-90 triangle.) z

Instead of connecting the midpoints to the opposite vertices, suppose we connect them to each other, as shown in Figure 7.3.
A D E F is called the medial triangle of A A B C .

Problem 7.19 t V

Prove that the four small triangles in Figure 7.3 are congruent, and that each is similar to A A B C .

Solution for Problem 7.19: A F A E and A B A C have an angle in common, but we don’t know anything about the other angles
of the triangles. So, we look to SAS Similarity to prove similarity. Since A F / A B = A E / A C = 1/2, we have
A F A E ~ A /F 4 C ‫ ׳‬bySAS Similarity. Similarly, we can show A F B D ~ A A B C and A C E D ~ A C A B .

Since each of the small triangles above have lengths that are 1 /2 the sides of A A B C , we have

FE DE DF 1
B C ~ A B ~ A C 2 ‫'־־‬

Therefore, A D E F ~ A A B C by SSS Similarity.

Since all four little triangles are similar to A A B C with the same side ratio, the four little triangles must be congruent. □

Notice that we can also use A A F E ~ A A B C to show that Z A F E = /.A B C , from which we have F E || B C . Similarly
we can show that the other two sides of the medial triangle are parallel to their respective sides of the original triangle.

Important: Given that A D E F is the medial triangle of A A B C as shown below, we have:

Z C

1. A D E F ~ A A B C , A D E F £ A F D D £ A A F E 2 A E D C .
EF DE DF 1
2 .

‫־‬B C ‫ ־ ־‬A B ‫־‬ AC ~ 2


3. D F || A C . E F || B C .a n d D E || A B .
Together, facts II and III are sometimes called the Midline Theorem.

Let’s try a couple problems using the properties of medians we have just found.

Problem 7.20 t V

A A B C has sides of length A B = 9, B C = 1 2 , and C A = 15. Find the length of the median from B to the midpoint of A C .

Solution for Problem 7.20: First we notice that A A B C has side lengths in the ratio 3 : 4 : 5, so
it is a right triangle with hypotenuse A C . Therefore, we are looking for the length of the median
to the hypotenuse of a right triangle. Since the midpoint of the hypotenuse of a right triangle is
also the circumcenter of the right triangle (see Problem 7.6), the median to the midpoint of the
hypotenuse is a radius of the circumcircle as shown in the diagram. Therefore, the length of our
median equals the circumradius, which is half the hypotenuse. So, our answer is
C A /2 = 15/2. □

Concept: When given the lengths of the sides of a triangle in a problem, always take the time to
check if it is a right triangle. Special properties of right triangles often simplify
I problems.

Important: The median to the hypotenuse of a right triangle is equal in length to half the
hypotenuse.
Z
Problem 7.21 t V

In A J K L , J K = J L = 10 and K L = 16.
J
(a) Find the length of median J M . N

(b) Find the length of median K N .

Solution for Problem 7.21:

(a) Since J K = JL, median J M is also an altitude (since A .J K M = A J L M ) . Therefore, A J M K is a right triangle with
hypotenuse J K = 10 and one leg of length K M = 8. So, J M = 6.

(b) Let J M and K N meet at G, which is the centroid of A J K L . It’s not obvious how to get J
K N , but we do have right triangle A K G M , which we can use to find K G . Since G is the
centroid and J M is a median, G M = J M / 3 = 2. Therefore,

K G = y/GAP + K M 2 = v/4+2/17 = 64‫״‬.


Since KN is a median and G the centroid, K G / K N = 2 /3 . Therefore,
K N = (3 /2 ) K G = 3 ^ 1 7 .

Exercises

7 .4 .1 : t V

D and E are the midpoints of sides B C and A C of A A B C in the diagram at right. A D and B E
meet at G, A G = 9, and G E = 4. Find G I ) and B G .

Type your solution, notes and/or work here. Show Solution


7.5 Altitudes
W e ’v e a l r e a d y s e e n a l t i t u d e s i n o u r i n v e s t i g a t i o n o f t r i a n g l e a r e a . Y o u c a n p r o b a b l y g u e s s b y n o w w h a t w e l l p r o v e f i r s t a b o u t
a ltitu d e s .

Problems
P r o b le m 7 .2 2 4 J u m p t o S o lu t io n

In t h is p r o b le m w e w ill p r o v e t h a t t h e lin e s c o n t a in in g t h e a lt it u d e s o f a n y t r ia n g le a r e c o n c u r r e n t . T h is is a p r e t t y s lic k p r o o f .


B e f o r e t r y in g it, y o u m i g h t w a n t t o r e v ie w w h a t w e le a r n e d a b o u t t h e m e d ia l t r i a n g l e in S e c t io n 7 . 4 h e r e .

(a ) O u r d ia g r a m s h o w s a ltitu d e s AD, B E , a n d C F o f AABC. D ra w A


a lin e t h r o u g h
p a r a lle l t o BC, a lin e t h r o u g h B p a r a l l e l t o AC, a n d a lin e t h r o u g h C AB T h e
p a r a lle l t o
in t e r s e c t io n s o f t h e s e t h r e e lin e s f o r m a n o t h e r t r ia n g le . C a ll t h is t r ia n g le A JK L , w it h A
on KL and B on J L.
(b ) S h o w th a t A C M K = AACB.

(c ) U s e p a r t ( b ) a n d s im ila r c o n g r u e n c e s t o p r o v e t h a t A, B, a n d C a re th e m id p o in ts o f K L,
J L , a n d J K , r e s p e c t iv e ly .

(d ) H o w a r e lin e s . 4 / 3 , UÈ, a n d f c f i r e la te d t o A J K L?
(e ) P ro v e t h a t th e a ltitu d e s o f AABC a re c o n c u rre n t.

P r o b le m 7 .2 3 4 J u m p t o S o lu t io n

W h e r e d o a lt itu d e s o f a r ig h t tr ia n g le in te r s e c t - in s id e , o u t s id e , o r o n t h e t r ia n g le ? H o w a b o u t a n a c u t e t r ia n g le ? A n o b t u s e
t r ia n g le ? ( I f n e c e s s a r y , e x t e n d t h e a lt it u d e s t o t h e p o in t w h e r e t h e y a ll m e e t . )

P r o b le m 7 .2 4 4 J u m p t o S o lu t io n

A lt itu d e s QZ and XP in t e r s e c t at N as show n. G iv e n th a t Z Y X Z — 10 ° and


ZX ZY = 4 5 ° , fin d e a c h o f t h e fo llo w in g :

(a ) ZZXP.
(b) ZX Z Q .
(c ) ZYXP.
(d )*Z N Y Z .
H in t

P r o b le m 7 .2 5

T h e a ltitu d e s o f A A B C m e e t a t p o in t / / . A t w h a t p o in t d o th e a ltitu d e s o f A ABU m e e t? H o w a b o u t t h e a ltitu d e s o f


A AC.117 A B C ID ( A s u s u a l , e x t e n d t h e a l t i t u d e s i f n e c e s s a r y . )

W e s t a r t , a s y o u p r o b a b ly g u e s s e d , b y p r o v in g t h a t t h e a lt it u d e s o f a t r ia n g le a r e c o n c u r r e n t . T h is is a b it o f a m a g ic a l p r o o f , a n d it
u s e s w h a t w e h a v e a lr e a d y le a r n e d a b o u t p e r p e n d ic u la r b is e c t o r s a n d t h e m e d ia l t r ia n g le .

Problem 7.22 t V

Prove that the lines containing the altitudes o f a triangle are concurrent.

Solution for Problem 7.22: O u r a lt it u d e s m a k e u s t h in k o f p e r p e n d ic u la r lin e s . C o n c u r r e n t p e r p e n d ic u la r lin e s m a k e u s t h in k o f


p e r p e n d i c u l a r b i s e c t o r s . U n f o r t u n a t e l y , t h e r e ’s n o s i m p l e s o l u t i o n l i k e w e h a d w i t h p e r p e n d i c u l a r b i s e c t o r s . S o . w e p u t o u r w i s h f u l
th in k in g h a t o n , a n d w o n d e r if w e c o u ld p o s s ib ly s h o w t h a t t h e a ltitu d e s o f o u r tr ia n g le a re a ls o t h e p e r p e n d ic u la r b is e c to r s o f th e
s id e s o f s o m e o t h e r t r ia n g le . I f s o , w e w o u ld k n o w t h e y a r e c o n c u r r e n t , s in c e t h e p e r p e n d ic u la r b is e c t o r s o f o u r 'o t h e r t r ia n g le ’
m u s t b e c o n c u r r e n t . B u t w h a t ’s o u r ’o t h e r t r i a n g l e ?

W e w a n t o u r .4 , B, and C t o b e t h e m id p o in t s o f t h e s id e s o f o u r o t h e r t r ia n g le ( s in c e t h e n t h e a lt it u d e s o f AABC w o u ld b e
p e r p e n d ic u la r b is e c t o r s o f t h e s id e s o f t h e o t h e r t r ia n g le ) . T h is m e a n s w e w a n t AABC t o b e t h e m e d ia l tr ia n g le o f s o m e o th e r
t r ia n g le . S in c e w e k n o w t h a t t h e s id e s o f a t r ia n g le a r e p a r a lle l t o t h e s id e s o f it s m e d ia l t r ia n g le , w e k n o w h o w t o m a k e a t r ia n g le
s ta r t in g w it h it s m e d ia l t r ia n g le .

W e d r a w l i n e s J l i , *KL, a n d JL A B C p a r a lle l t o t h e s id e s o f
t h r o u g h t h e v e r tic e s o f A
AABC A A B C i s i n d e e d t h e m e d i a l t r i a n g l e o f A .JKL, w e
a s s h o w n . T o p ro v e th a t
m u s t s h o w t h a t t h e v e r t ic e s o f A A B C a r e t h e m id p o in t s o f t h e s id e s o f A . / K L . S in c e
‫־‬A B || J K , Z C A B = Z A C K . S i m i l a r l y , Z C A K = ZB C A , s o A C A K 2 A A C B .
L ik e w is e , A B L A = A A C B , s o A B L A = A C A K . T h e r e f o r e , A L = A K , s o A is
th e m id p o in t o f L K .

In t h e s a m e w a y . w e f in d t h a t B and C a re th e m id p o in ts o f L J a n d J K, r e s p e c t i v e l y .
T h e re fo re . A ABC is t h e m e d ia l t r ia n g le o f A J KL. S in c e K L || BC, a l t i t u d e A D o f
A A B C is perpendicular to K L . Since .4 /3 is perpendicular to K L and passes through
its m id p o in t , it is t h e p e r p e n d ic u la r b is e c to r o f KL. S im ila r ly , t i p and H a re a ls o

p e r p e n d ic u la r b is e c t o r s o f t h e s id e s o f A JKL. L in e s . 4 / 3 . H , and a re c o n c u r r e n t b e c a u s e th e y a re t h e p e r p e n d ic u la r
b is e c t o r s o f t h e s id e s o f A JKL. T h e s e lin e s a ls o c o n t a in t h e a lt it u d e s o f A ABC, s o w e h a v e p r o v e d t h a t t h e lin e s c o n ta in in g
th e a ltitu d e s o f AABC a re c o n c u rre n t. □

Im p o rta n t: T h e a ltitu d e s o f a n y tr ia n g le a re c o n c u r r e n t a t a p o in t c a lle d t h e o r t h o c e n t e r . W e


7‫־‬ u s u a l l y d e n o t e t h e o r t h o c e n t e r w i t h t h e l e t t e r 11.

T o g e t a f e e l f o r t h e o r t h o c e n t e r , w e lo o k f o r i t in a f e w d i f f e r e n t t y p e s o f t r ia n g le s .

Problem 7.23 t V

Where do altitudes o f a right triangle intersect — inside, outside, or on the triangle? How about an acute triangle? An obtuse
triangle?

Solution for Problem 7.23: W e s t a r t w i t h r i g h t t r i a n g l e A A B C w i t h r i g h t a n g l e at B P o in t B is t h e f o o t o f t h e a lt it u d e f r o m . 4 t o


B C a n d t h e f o o t o f t h e a l t i t u d e f r o m C t o . 4 B, t h e r e f o r e , t h e o r t h o c e n t e r o f a r ig h t t r ia n g le is t h e v e r t e x o f t h e r ig h t a n g le .

In a c u t e A DEF, w e s e e t h a t a ll t h e a lt it u d e s g o in s id e t h e t r ia n g le , s o t h e o r t h o c e n t e r is in s id e t h e t r ia n g le . In o b t u s e A PQR,
t h e a lt it u d e s f r o m t h e t w o a c u te a n g le s a r e e n t ir e ly o u t s id e t h e t r ia n g le ( e x c e p t w h e r e t h e y p a s s t h r o u g h a v e r te x ) . T h e r e fo r e , t h e
o r t h o c e n t e r o f a n o b t u s e t r ia n g le is o u t s id e t h e t r ia n g le . □

L e t 's t r y u s in g t h e o r t h o c e n t e r in a p r o b le m .

Problem 7.24 t V

Altitudes Q Z and X P o f A X Y Z intersect at N . Given that Z Y X Z — 7 0 ° and Z X Z Y — 4 5 ° X


,find the following:
j \
(a) ZZXP. C
(b) ZX ZQ . j
2__
(c) ZYXP. Y P

(d)* Z N Y Z .

Solution for Problem 7.24:


(a ) F r o m r ig h t tr ia n g le A X PZ, w e have ZZXP = 90° — ZXZP = 4 5 °.

(b ) F r o m r ig h t tr ia n g le AXZQ, w e have

ZXZQ = 90° - ZZXQ = 90° - Z Z X K = 2 0 °.

(c ) W e c o u ld u s e e ith e r r ig h t tr ia n g le A X Y P, o r w e c o u ld u s e ZZX P and ZYXZ:


ZYXP = Z r X Z - ZZXP = 7 0 ° - 4 5 ° = 2 5 °.

(d ) S in c e t w o a lt it u d e s m e e t a t N, w e k n o w th a t N is t h e o r t h o c e n t e r . T h e r e f o r e , i f w e c o n t in u e
X
YN X Z a t II,
to m e e t w e k n o w th a t YR is a ls o a n a lt it u d e . F r o m r ig h t t r ia n g le ARYZ, we
fin d t h a t ZNYZ = 9 0 ° - ZRZY = 4 5 °.

N o t ic e t h a t r e c o g n iz in g t h e o r t h o c e n t e r w a s a k e y s t e p in t h e s o l u t i o n t o o u r la s t p a r t . T h is is j u s t o n e s m a ll e x a m p le o f h o w t h e
a w a r e n e s s o f t h e s p e c ia l p o in t s , lin e s , a n d c ir c le s o f a t r ia n g le c a n h e lp .

C o n c e p t: I f w e h a v e t h e a ltitu d e s f r o m t w o v e r t ic e s o f a t r ia n g le , t h e n w e k n o w t h a t t h e lin e
• t h r o u g h t h e in t e r s e c t io n p o in t o f t h e s e t w o a lt it u d e s a n d t h e t h ir d v e r te x is a ls o a n
' a lt itu d e . O n c e y o u h a v e t w o a lt itu d e s , y o u s h o u ld a lm o s t a lw a y s t h in k a b o u t t h is t h ir d
a lt itu d e .

T h is , o f c o u r s e , a ls o w o r k s w it h o u r o t h e r s p e c ia l lin e s - i f y o u h a v e t w o m e d ia n s , y o u
h a v e t h e t h ir d ; i f y o u h a v e t w o a n g le b is e c t o r s , y o u h a v e t h e t h ir d , e tc . W it h t h e s e
o t h e r s p e c ia l lin e s y o u a ls o g e t a lit t le m o r e , s in c e w e h a v e s p e c ia l k n o w le d g e a b o u t
t h e c e n tr o id ( th e 2 : 1 r a t io ) , t h e in c e n te r ( c e n te r o f th e in s c r ib e d c ir c le ) , a n d t h e
c ir c u m c e n t e r ( c e n t e r o f t h e c ir c u m s c r ib e d c ir c le ) .

A g a in , i f y o u h a v e t w o o f a n y o f t h e s e s p e c ia l lin e s , y o u s h o u ld c o n s id e r t h e t h ir d lin e
a n d r e m e m b e r w h a t y o u k n o w a b o u t t h e in t e r s e c t io n p o in t.

H e r e 's a n i n t e r e s t i n g t i d b i t a b o u t a l t i t u d e s a n d o r t h o c e n t e r s :

Problem 7.25 t V

The altitudes o f A A B C meet at point II. At what point do the altitudes o f A A B U meet? How about A AC 11? A B C 111

Solution for Problem 7.25: O u r d ia g r a m s h o w s t h a t t h e a lt itu d e s o f AABH a re Ü ( p e r p e n d ic u la r C


t o s id e AB), *AÔ ( p e r p e n d i c u l a r t o 3//I), a n d *BÔ ( p e r p e n d i c u l a r t o *A ll) T h e s e l i n e s c l e a r l y a l l
p a s s th ro u g h C, so C is t h e o r t h o c e n t e r o f AABH. S im ila r ly , B is t h e o r t h o c e n t e r o f AAHC
a n d . 4 is t h e o r t h o c e n t e r o f A B B C . □

A n d h e r e 's a n i n t e r e s t i n g f a c t a b o u t h o w t h e o r t h o c e n t e r , c ir c u m c e n t e r , a n d c e n t r o i d o f a t r i a n g l e a r e r e la t e d :

S id e n o t e : In a n y t r ia n g le , t h e c e n tr o id . G, i s o n t h e lin e s e g m e n t c o n n e c t in g t h e o r t h o c e n t e r , / 7 .
t o t h e c ir c u m c e n t e r , O, s u c h th a t 2 OG — G lI T h is lin e t h r o u g h t h e s e t h r e e p o in t s
n is c a lle d t h e E u le r lin e , a f t e r t h e g r e a t m a t h e m a t ic ia n L e o n h a r d E u le r . O n e s u c h lin e is
s h o w n b e lo w .

E u le r w a s o n e o f t h e m o s t p r o l i f i c m a t h e m a t i c i a n s e v e r , p r o d u c in g n o t a b le w o r k in
n e a r ly e v e r y f ie ld o f m a t h e m a t ic s , a n d c r e a t in g n e w f ie ld s h im s e lf . T h e E u le r lin e is
j u s t o n e o f m a n y m a t h e m a t i c a l r e s u l t s t h a t h a v e b e e n n a m e d a f t e r E u le r . H is i m p a c t
o n m a t h e m a t ic s w a s s o g r e a t t h a t m u c h o f o u r n o t a t io n t o d a y w a s in s p ir e d b y h im .
such as f(x) f o r fu n c tio n s , i fo r T, e fo r th e base of com m on lo g s , £ fo r
s u m m a tio n , a n d even 7 r f o r . w e ll, p i. He lin k e d th re e o f th e s e w ith h is fa m o u s
s ta te m e n t en' = —1.

Exercises

7 .5 .1 : t V

RC and QB a re a ltitu d e s o f A PQ R as show n, ZQRP = 2 0 °, a n d ZQ PR = 3 2 ° . F in d t h e c


f o llo w in g : Q
(a ) ZPQB.

T y p e y o u r s o lu tio n , n o te s a n d / o r w o r k h e re . Show S olution

(b ) ZCRP.

T y p e y o u r s o lu tio n , n o te s a n d / o r w o r k h e re . Show Solution

(c ) ZCQR.

T y p e y o u r s o lu tio n , n o te s a n d / o r w o r k h e re . Show Solution

(d ) ZCQB.

T y p e y o u r s o lu tio n , n o te s a n d / o r w o r k h e re . Show Solution

In t h e d ia g r a m , Y Z = 15, Y B = 12, and X A = 10.

(a ) F in d [X Y Z \

T y p e y o u r s o lu tio n , n o te s a n d / o r w o r k h e re . Show Solution

(b ) F in d BZ.

T y p e y o u r s o lu tio n , n o te s a n d / o r w o r k h e re . Show Solution

(c ) F in d X Z .

T y p e y o u r s o lu tio n , n o te s a n d / o r w o r k h e re . Show Solution

(d ) F i n d X 13.

T y p e y o u r s o lu tio n , n o te s a n d / o r w o r k h e re . Show S olution

(e ) F in d XY.

T y p e y o u r s o lu tio n , n o te s a n d / o r w o r k h e re . Show Solution

AABC is is o s c e le s w it h A B — AC. M u s t th e a ltitu d e s f r o m B and C h a v e t h e s a m e le n g th ? W h y o r w h y n o t?

T y p e y o u r s o lu tio n , n o te s a n d / o r w o r k h e re . Show S olution


7.6★ Challenging Problems
In this section we tackle several challenging problems in which we use what we've learned in this chapter about the special
points, lines, and circles of a triangle. You may wish to try your hand at the Review Problems at the end of this chapter before
attempting the problems in this section.

P roblem s
Problem 7.26 4 Jump to Solution

In this problem, our goal is to find the area of A B C D , given that A B = B C = 6, C D = 3 \/2 , Z A B C = 90°, and that
there is a circle passing through all four vertices of A B C D .

(a) Where is the center, O, of the circle that passes through all four vertices?

(b) What is the radius of this circle?

(c) What kind of triangle is A ()C D ?

(d) What are the angles in A O A D ? What is / . C D A?

(e) Find the area of A B C D .

Problem 7.27 4 Jump to Solution

Given A B = B C = 10 and A C = 12, find the circumradius and the inradius of A A B C .

Hint

Hint

Hint

Problem 7.28 4 Jump to Solution

In the diagram, A D bisects / B A C and / is the incenter of A A B C . Furthermore,


/I
A B = 7, B C = 8, and A C = 11. Find A I / I D .

(a) Use the Angle Bisector Theorem to find another pair of segments that have a ratio
equal to A I / I D .

(b) Use the Angle Bisector Theorem again to find another pair of lengths (besides A I
and / D) that have a ratio equal to the ratio you found in part (a).

(c) Let B D = x. Set up and solve an equation for x using the ratios you found in the
previous two parts. Then finish the problem by finding A I / ID .

Problem 7.29 4 Jump to Solution

Is it possible for there to be points E and F on side B C of A A B C such that B E = E F = F C and


/ B A E = / E A F = / F A C ? Why or why not?

Hint

Problem 7.30 4 Jump to Solution

Point E is the midpoint of A C , and A D is a median of A A B C . F is on A B such that A F = A B / A


/1
E F and A D meet at X . Find A X / A D .

Hint

Problem 7.26 t V

Find the area of A B C D , given that A B = B C = 6, C D = 3^/2, / A B C = 90°, and that there is a circle passing through
all four vertices of A B C D .

Solution for Problem 7.26: We don't know much about figures with four sides at this point, but we now know a whole lot about
triangles. We start with right triangle A A B C . The area of this triangle is ( A B ) ( B C ) / 2 = 18, so all we have to find to finish is
[A C D \ Unfortunately, we don't have much information about A A C D .

We do, however, know that the circumcenter of A A B C is the midpoint of A C . Therefore the
center of the circle, O, is the midpoint of A C . Breaking the problem into triangles worked well
once, so we try again by drawing O D . Since A A B C is an isosceles right triangle,
A C = 6 \/2 . So, the radius of the circle is 3 \/2 . We label all the lengths we know, and we see
that A ( ) C D is an equilateral triangle.

Since A ( ) C D is equilateral, its angles are all 60°. Furthermore,

/ A O D = 180° - / C O D = 120°.

Since A A O D is isosceles, / O A D = / O D A = 30°. Now, we see that A A C D is a 30-


6090‫ ־‬triangle, so A D = C D \ / 3 = 3 \/6 . Finally, [A C D ] = ( C D ) ( A D ) / 2 = 9 \/3 .

Finally, we have

[A B C D ] = [A B C ] + [A C D ] = 18 + 9 \/3 .

In Section 12.1, we will learn a much faster way to deduce th a t/ A D C is a right angle. □

In this solution, we seemed to stumble on a surprising equilateral triangle and a useful right angle. We were able to find them
because we kept track of everything we learned in our diagram.

Concept: Label your diagram with all the lengths and angles you can find. This will help you find
I relationships in the diagram that might be hard to find otherwise.

Problem 7.27 t V

Given A B = B C = 10 and A C = 1 2 , find the circumradius and the inradius of A A B C .

Solution for Problem 7.27: Since we're looking for the circumradius and the inradius, were thinking
В
about perpendicular bisectors and angle bisectors. Our triangle is isosceles, so well start with the
special lines from the vertex angle ( / B ) since they’re all the same line.

B P splits our triangle in two congruent halves, so A P = 6. Since A B P A is right, we have


B P = 8 from the Pythagorean Theorem. To find the circumradius, we build a right triangle with the
circumradius as a side by connecting the circumcenter, O, to A. Since O B = x and B P = 8, we
have O P = 8 — x and we can apply the Pythagorean Theorem to A O A P to find
O A 2 = O P 2 4‫ ־‬A P 2. Substituting our expressions for the sides, we have

x 2 = (8 - x f 4 3 6 ‫־‬.

A little algebra then yields 16.T = 100, so x = 25/4.

Since the area of a triangle is its semiperimeter times its inradius, we can quickly find the inradius of A A B C :

r _ [A B C ] _ (A C )(B P )/2 _ 48 _
r ~ s ~ (1 0 + 1 0 + 1 2 )/2 “ 16 “

Challenge: See if you can find the inradius using a tactic like the one we used for the circumradius! □

Concept: Need to find a length in a problem? Try building right triangles.

Solution for Problem 7.28: We’re looking for a ratio and we have an angle bisector, so we try using the Angle Bisector Theorem.
Since / is the incenter of A A B C , B E bisects / A B C . Therefore, B I is an angle bisector of A A B D . From the Angle
Bisector Theorem, we have

A I _ AB
I D ‫ ־־‬B D '
We know A B = 7, so if we can find BD, we’re finished. The Angle Bisector Theorem also gives us A B / B D = A C /C D
when applied to angle bisector A D of A ABC. Since B D 4‫ ־‬D C = B C = 8, we have:

7 11
‫־‬B D 8 ‫־־‬- B D ‫׳‬
Cross-multiplying gives 7(8 — B D ) = 11 BD, from which we find B D = 28/9. Therefore, A I / I D = A B / B D = 9/4. □

Concept: Keep your eye on the ball! Often, working backwards from what you want will guide
I you to the solution.

Problem 7.29 t V

Is it possible for there to be points E and F on side B C of A A B C such that B E = E F = F C and


/ В А Е = / E A F = / F A C ? Why or why not?

Solution for Problem 7.29: We have angle bisectors and lengths, so we try the Angle Bisector Theorem.
A
A E and A F trisect / B A C , but we can also look at them as the angle bisectors of / B A F and
/ E A C , respectively. The Angle Bisector Theorem applied to A B A F and A E A C gives us

AB BE л AE _ EF
— = — —= 1
— and
AF EF AC ~ FC

Therefore, we have A B = A F and A E = A C .

Marking these equalities in our diagram, congruent triangles jump out. Specifically, SSS tells us
A
A A B E = A A F E = A A F C . However, this means that / А Е В = / A E F = 90° and
/ A F E = / A F C = 90°. Therefore A A E F has two right angles, which is impossible.

So, it is not possible for there to be points E and F on side B C of A A B C such that
B E = E F = F C and / В А Е = / E A F = / F A C . □

Problem 7.30 t V

Point E is the midpoint of A C , and A D is a median of A A B C . F is on A B such that A F = A B / A E F and A D meet at


X . Find A X / A D .

Solution for Problem 7.30: We start with a diagram. We go ahead and draw in the other two
A
medians, hoping we can use what we know about the centroid. Since К is the midpoint of A В
and A F = A B / 4, F is the midpoint of A K . So, A A F E ~ A A K C by SAS Similarity.
Therefore, / A F E = / А К С , so F E || К С . This gives us A A F X ~ A A K G , so

AX _ AF _ 1
A G ‫ ־־‬A K 2 ‫' ־‬

Since G is the centroid of A A B C , we have A G / A D = 2/3, so

AX (Л Х \ (A G \ 1
AD ‫־‬ \A G J \ A d ) ~ 3'

Exercises

7 .6 .1 : t V

P, Q, and R are the midpoints of sides Y Z , X Z , X Y , respectively, and Z A is the altitude to side
XY.

(a) Show that A P = RQ.

Hint

Type your solution, notes and/or work here.

(b) ★ Show that / P R Q = / P A Q .

Hint

Type your solution, notes and/or work here. Show Solution

In right triangle A B C , let F be the midpoint of hypotenuse A B , and let D be the foot of the altitude from C to A B . Let E be
on A B such that C E is the angle bisector of / A C B . Prove that / D C E = / E C F .

Hint

Type your solution, notes and/or work here. Show Solution

In A P Q R , P Q = 8, P R = 1 0 , and Q R = 9. P D bisects / Q P R , Q A 1 P D , and B C


passes through A such that B C || P R .

(a) Prove that A C is a median of A P A Q .

Type your solution, notes and/or work here. Show Solution

(b) ★ Find D B .

Hint

Hint

Hint

Type your solution, notes and/or work here. Show Solution

Extra! The incircle and the circumcircle are not the only notable circles that can be found in
a triangle. For any triangle, a single circle passes through the midpoints of the sides,
V the feet of the altitudes, and the midpoints of the segments connecting the
orthocenter to the vertices. This circle is cleverly called the nine-point circle of the
triangle.

The nine-point circle is tangent to the incircle, has a radius equal to half the
circumradius (you have the tools to prove this - see if you can!), and its center is the
midpoint of the segment connecting the orthocenter and the circumcenter, which we
discussed in Section 7.5 here. We'll prove the existence of the nine-point circle, and
some of its properties, during our deeper investigations of triangles in Intermediate
Geometry.
7.8 Summary

Definitions:
■ The perpendicular bisector of a line segment is the line through the midpoint of
the segment that is perpendicular to the segment.

Important: The perpendicular bisector of a segment is a straight line consisting of all points
that are equidistant from the endpoints of the segment.
Z
■ The angle bisector of an angle is the line that divides the angle into two equal
angles.

Important: The angle bisector of an angle consists of all points that are equidistant from the
sides of the angle.
Z

Definition: A cevian is a line segment from a vertex of a triangle to a point on the line containing
the opposite side of the triangle.

We explored important properties of several sets of cevians:

Definitions: The perpendicular bisectors of the sides of a triangle


b
are concurrent at a point called the circumcenter. The
circle centered at the circumcenter that passes
through the vertices of the original triangle is called
the circumcircle of the triangle because it is
circumscribed about the triangle (meaning it passes
through all the vertices of the triangle).

Finally, the radius of this circle is called the


circumradius, the circumcenter is usually labeled with
the letter O, and the circumradius is usually called R.

Definitions: The angle bisectors of a triangle are concurrent at a


X
point called the incenter. This point is equidistant from
the sides of the triangle. This common distance from
the incenter to the sides of a triangle is called the
inradius, because the circle with center I and this
radius is tangent to all three sides of the triangle. This
circle is unsurprisingly called the incircle because it is
inscribed in the triangle (meaning it is tangent to all the
sides of the triangle). The incenter is usually denoted /, and the inradius is usually
written as r.

Definitions: A median of a triangle connects a vertex of a triangle to the


C
midpoint of the opposite side. The medians of a triangle are
concurrent at a point called the centroid of the triangle. The
centroid of the triangle is usually labeled G.

Important: The medians of a triangle divide the triangle into six little triangles with equal area.

The centroid of a triangle cuts its medians into a 2 : 1 ratio. For example, for the
triangle shown, we have

AG _ BG _ CG _ 2
GD ~ G E ~ GF ~ 1‫׳‬

Definition: The altitudes of a triangle (extended if necessary) are concurrent


C
at the orthocenter of the triangle, which is usually denoted // .

Important: The orthocenter of an acute triangle is inside the triangle. The orthocenter of a right
triangle is the vertex of the right angle. The orthocenter of an obtuse triangle is
z outside the triangle.

Important: Right triangles offer a few important relationships.

Z ■ The circumcenter of a right triangle is the midpoint of


the hypotenuse, and the circumradius equals one-half
the hypotenuse. /1

■ The median to the hypotenuse of a right triangle is equal


in length to half the hypotenuse.

Our exploration of these special points, lines, and circles uncovered a few more important facts:

Important: Given any three noncollinear points, there is exactly one circle that passes through all
three.
Z
Important: The Angle Bisector Theorem states that if E is on A C
such that R E is the angle bisector of Z R in triangle
Z A A R C , then
/ k
AD CR j Z 1 \ c
E C
AE CE'

We should think of this any time we have a problem involving lengths and angle
bisectors.

Important: The area of a triangle equals its inradius times half its perimeter.

Z
Important: If we connect the midpoints of the sides of a triangle, we divide the triangle into four
congruent triangles, each of which is similar to the original triangle. The central
Z triangle is called the medial triangle of our original triangle.

P roblem Solving S trateg ies

Concepts:
If we have the altitudes from two vertices of a triangle, then we know that the line
through the intersection point of these two altitudes and the third vertex is also an
altitude. Once you have two altitudes, you should almost always think about this
third altitude.

This, of course, also works with our other special lines - if you have two medians,
you have the third; if you have two angle bisectors, you have the third, etc. With
these other special lines you also get a little more, since we have special
knowledge about the centroid (the 2 : 1 ratio), the incenter (center of the inscribed
circle), and the circumcenter (center of the circumscribed circle).

Again, if you have two of any of these special lines, you should consider the third
line and remember what you know about the intersection point.

When given the lengths of the sides of a triangle in a problem, always take the time
to check if it is a right triangle. If it is, this fact may simplify the problem
considerably.

Label your diagram with all the lengths and angles you can find. This will help you
find relationships in the diagram that might be hard to find otherwise.

Need to find a length in a problem? Try building right triangles.

Keep your eye on the ball! Often working backwards from what you want will guide
you to the solution.
7.7 Construction: Bisectors
In this section well construct some of the special points, lines, and circles of a triangle.

P roblem s

We already know how to make perpendicular lines, midpoints, and perpendicular bisectors. But we haven't learned how to make
angle bisectors, so well start there.

Problem 7.31 t V

Given A X , construct a ray from X that bisects A X .

Solution for Problem 7.31: Since we measure angles as a portion of a circle, we


start with a circle centered at X . We call the points where the circle meets the
sides of the angle points A and B. Since X A = X B , A X B A is isosceles.
This makes us happy because we know that the angle bisector of the vertex
angle of an isosceles triangle is also a median, an altitude, and a perpendicular
bisector.

Therefore, all we have to do is construct the perpendicular bisector of A B to


have the angle bisector of A X . We can quickly construct this by drawing arcs
centered at A and B with radius A X . These arcs meet at X and a point we'll
call Y. X 'Y bisects A A X B . Well leave the proof that this construction works
as an Exercise. □

Now that we can construct the special lines of a triangle, we can construct special circles. For example:

Problem 7.32 t V

Given a triangle A A B C , construct the incircle of the triangle.

Solution for Problem 7.32: To construct a circle, we need both its center and its radius. We can find the
C
center of the incircle by constructing the bisectors of two of the angles of the triangle. We can perform
these constructions exactly as described in Problem 7.31. Where these rays meet is our center, point I.

Now that we have the center, we just need the radius. The radius of the incircle drawn to a point where
the incircle touches a side of the triangle is perpendicular to that side. Therefore, we can find the radius
by constructing this perpendicular segment from /. Specifically, we construct a line through / that is
perpendicular to A B as described in Problem 6.24. Suppose this line meets A B at Z. / Z is the /1
length of our radius.

We then construct our incircle by drawing a circle with center I and radius I Z . □

We’ll finish this chapter with two clever solutions to a challenging construction problem.

Problem 7.33 t V

Given a circle but not its center, construct the center of the circle.

Solution for Problem 7.33: Solution 1: We don’t know a whole lot about constructing parts of
a circle, but we know a ton about constructing parts of a triangle. Specifically, we know how
to construct the circumcircle of a triangle - its center is just the intersection of the
perpendicular bisectors of the sides.

Therefore, we just pick three points A, B, and C on the circumference of our circle. We then
construct the perpendicular bisectors of two of the sides of A A B C as shown. The
intersection of these perpendicular bisectors, point O, is the circumcenter of A A B C . Since
our starting circle is the circumcircle of A A B C , the circumcenter of A A B C is the center
of the starting circle.

Solution 2: Another useful circumcenter fact we know is that the circumcenter of a right triangle
is the midpoint of its hypotenuse. Inspired by this observation, we create right A P Q R by
choosing point P on the circle, drawing line n through P to meet the circle again at Q, then
constructing line m through P perpendicular to n. Where line m hits the circle again gives us
point R.

Since A P Q R is a right triangle, the midpoint of its hypotenuse is the center of its
circumcircle. Therefore, we find the center of our original circle by constructing O, the midpoint
of R Q . □

Exercises

7 .7 .1 : t V

Given A A B C , construct the circumcircle of the triangle.

Type your solution, notes and/or work here. Show Solution

Construct a 30° angle.

Type your solution, notes and/or work here. Show Solution

Construct a 45° angle.

Type your solution, notes and/or work here. Show Solution

Construct a 120° angle.

Type your solution, notes and/or work here. Show Solution

Given A A B C , construct the orthocenter of the triangle.

Type your solution, notes and/or work here. Show Solution

Prove that the construction described in the solution to Problem 7.31 really does produce an angle bisector.

Type your solution, notes and/or work here. Show Solution


Review Problems

In triangle A P Q R , P Q = 12, Z Q = 90°, and Q R = 16. Find the following:

(a) The area of A P Q R .

Type your solution, notes and/or work here. Show Solution

(b) The circumradius of A P Q R .

Type your solution, notes and/or work here. Show Solution

(c) The inradius of A P Q R .

Type your solution, notes and/or work here. Show Solution

Points T and U are on Q/C such that K is 2 units from T U . If T U = 14, what is the radius of the circle?

Type your solution, notes and/or work here. Show Solution

A X Y Z is an equilateral triangle with side length 12. M is the midpoint of side Y Z and N is the midpoint of X Z . Y N and
X M meet at E.

(a) What kind of triangle is A X Y M l

Type your solution, notes and/or work here. Show Solution

(b) Find X M .

Type your solution, notes and/or work here. Show Solution

(c) Find [X Y Z ].

Type your solution, notes and/or work here. Show Solution

(d) Find X E / E M .

Type your solution, notes and/or work here. Show Solution

(e) Find the inradius of A X Y Z .

Type your solution, notes and/or work here. Show Solution

(f) Find the circumradius of A X Y Z .

Type your solution, notes and/or work here. Show Solution

Altitudes A D and C F of acute triangle A A B C meet at / / . Prove that Z C H D = Z A B C .

Type your solution, notes and/or work here. Show Solution

If the altitudes of a triangle all have the same length, must the triangle be equilateral? Why or why not?

Type your solution, notes and/or work here. Show Solution

Point I ) is on side Y Z of A X Y Z such that Z Y X I ) = Z Z X D . Given that X Z = 8, Y D = 3, and D Z = 4, find X Y .

Type your solution, notes and/or work here. Show Solution

Medians A D , B E , and C F meet at G. The area of A A B C is 48. Find the areas of:

(a) A ADC

Type your solution, notes and/or work here. Show Solution

(b) AAGC

Type your solution, notes and/or work here. Show Solution

(c) A GFB

Type your solution, notes and/or work here. Show Solution

(d) A DEF

Type your solution, notes and/or work here. Show Solution

(e) AAEF

Type your solution, notes and/or work here. Show Solution

A T U V is isosceles with T U = U V = 30 and T V = 36. U Y bisects Z T U V and T M is a median of A T U V . U Y and


T M meet at X .

(a) Show that Y is the midpoint of T V .

Type your solution, notes and/or work here. Show Solution

(b) Show that U Y A -T V .

Type your solution, notes and/or work here. Show Solution

(c) Find T Y and U Y .

Type your solution, notes and/or work here. Show Solution

(d) Find X Y and X U .

Type your solution, notes and/or work here. Show Solution

(e) Find X T and X M .

Type your solution, notes and/or work here. Show Solution

In Problem 7.8 we found the circumradius of an equilateral triangle with side length 6, while in Exercise 7.2.3, we found the
circumradius of an equilateral triangle with side length 18. Given that the first circumradius is 2 \/3 , how could we have quickly
found the second without going through the same laborious procedure we used to solve Problem 7.8?

Type your solution, notes and/or work here. Show Solution

In triangle A B C , altitude A D intersects angle bisector B E at point X . If Z B A C = 117° and Z A C B = 35°, then
determine Z D X E .

Type your solution, notes and/or work here. Show Solution

Point O is the circumcenter of A P Q R , Z Q P R = 45°, and Z Q P O = 23c

(a) Find Z R P O a n d Z O Q P .

Type your solution, notes and/or work here. Show Solution

(b) ★ Find Z O Q R .

Type your solution, notes and/or work here. Show Solution

M and N are the midpoints of C A and C B , respectively, as shown at right. B M and A N meet at C
X . Given that X M = 3.5 and X A = 7.2, find X N and X B.

Type your solution, notes and/or work here. Show Solution

Medians A X and B Y of triangle A A B C are perpendicular at point O. A X = 12 and B C = 10.

(a) Find A O and B Y .

Type your solution, notes and/or work here. Show Solution

(b) ★ Find the length of median C Z .

Type your solution, notes and/or work here. Show Solution

Cevians A D and B E meet at point X inside A A B C . C X extended meets A B at F . If

[ A E X ] = [ C E X ] = [ C D X ] = [ B D X ] = [ B X F ] = [ A X F ],

must X be the centroid of A A B C ?

Type your solution, notes and/or work here. Show Solution

B E and C F are angle bisectors that meet at / as shown at right. C E = 4, A E = 6, and A B = 8.

ZA
(a) Prove that Z E I C = 9 0 ° ----- — .

Type your solution, notes and/or work here. Show Solution

(b) Find B C .

Type your solution, notes and/or work here. Show Solution

(c) Find B F .

Type your solution, notes and/or work here. Show Solution

(a) Show that if the centroid and the orthocenter of a triangle are the same point, then the triangle is equilateral

Type your solution, notes and/or work here. Show Solution

(b) Show that if the incenter and the centroid of a triangle are the same point, then the triangle is equilateral

Type your solution, notes and/or work here. Show Solution


Challenge Problems

The angle bisector of Z G of A G 111 passes through the circumcenter of A G 11I. Show that G H = G I.

Hint

Type your solution, notes and/or work here. Show Solution

Let I I and O denote the orthocenter and circumcenter of acute triangle A B C , respectively. * A li meets at P and A O
meets B C at Q. Prove that Z B A P = Z C A Q .

Hint

Hint

Type your solution, notes and/or work here. Show Solution

In A . I K L, we have J K = J L = 25 and K L = 30. Find the following:

(a) the area of A J K L.

Type your solution, notes and/or work here. Show Solution

(b) the inradius of A .J K L .

Type your solution, notes and/or work here. Show Solution

(c) the circumradius of A .J K L .

Hint

Type your solution, notes and/or work here. Show Solution

Let / be the incenter of triangle A B C , and let jÛ meet B C at A'.

(a) Prove that A ' l / I A = B A r/A B .

Type your solution, notes and/or work here. Show Solution

(b) ★ Prove that A ' l / I A = B C / ( A B - f A C ).

Hint

Type your solution, notes and/or work here. Show Solution

7 .5 4 : Source: AHSME t V

In triangle A B C , Z A = 100°, Z B = 50°, and Z C = 30°. I I is on B C and M on A C such that A H is an altitude and
B M is a median. Find Z M I I C .

Hint

Type your solution, notes and/or work here. Show Solution

Is it possible for two angle bisectors in a triangle to be perpendicular? Why or why not?

Hint

Type your solution, notes and/or work here. Show Solution

7 .5 6 : Source: MATHCOUNTS t V

In A A B C , Z C is a right angle. Point M is the midpoint of A B , point N is the midpoint of A C , and point O is the midpoint
of A M . The perimeter of A A B C is 112 and O N = 12.5. What is the area of M N C B ?

Type your solution, notes and/or work here. Show Solution

For a given triangle A B C , extend A B and A C as shown below. We will prove that there is a circle tangent to B C and the
extensions of A B and A C . We start by drawing the bisectors of two of the exterior angles of A A B C as shown with the grey
lines below.

(a) Let P be a point on the external angle bisector of angle Z B . Prove that P is equidistant from A B and

Type your solution, notes and/or work here. Show Solution

(b) Let Q be a point on the external angle bisector of angle Z C . Prove that Q is equidistant from 5 5 and S 5 .

Type your solution, notes and/or work here. Show Solution

(c) Let I a be the intersection of the external angle bisectors of angles Z B and Z C , and let X , Y, and Z be the feet of the
altitudes from / ‫ ״‬to B C , A B , and A C . Prove that l aX = I aY = I aZ . What does this tell us about the circle with
center I a and radius I aX ? This circle is called an excircle of A A B C ] I a is called an excenter, and r a is an exradius.
Every triangle has three excircles, one tangent to each side of the triangle.

Type your solution, notes and/or work here. Show Solution

(d) Prove that I a lies on the internal angle bisector of angle Z A .

Type your solution, notes and/or work here. Show Solution

[A B C \
(e) ★ Let r a = I aX . Prove that r a = -------——, where s is the semiperimeter of A A B C .
s —BC
Hint

Hint

Type your solution, notes and/or work here. Show Solution

7 .5 8 : Source: AMC 10 t V

Points M and N are the midpoints of sides P A and P B of A P A B . As P moves along a line that is parallel to side A B ,
which of the four quantities listed below change?

(a) the length of segment M N .

Type your solution, notes and/or work here. Show Solution

(b) the perimeter of A P A B .

Type your solution, notes and/or work here. Show Solution

(c) the area of A P A B .

Type your solution, notes and/or work here. Show Solution

(d) the area of A B N M .

Type your solution, notes and/or work here. Show Solution

7 .5 9 : Source: (b) HMMT t V

Let A X Y Z have Z X = 60° and Z Y = 45°. A circle with center P passes through A and B on side X Y , C and I ) on side
Y Z , and E and F on side Z X . Suppose A B = C D = E F .

(a) Prove that P is the incenter of A X Y Z .

Type your solution, notes and/or work here. Show Solution

(b) Find Z X P Y .

Type your solution, notes and/or work here. Show Solution

7 .6 0 ★ : t V

A ladder is initially resting vertically against a wall that is perpendicular to the ground. It begins to slip and fall to the ground,
with the bottom of the ladder moving directly from the wall, and the top of the ladder always touching the wall. What path does
the midpoint of the ladder trace? (Don't forget to prove that the midpoint of the ladder really does hit every point on the path!)

Hint

Hint

Type your solution, notes and/or work here. Show Solution

7 .6 1 : Source: Mandelbrot t V

Triangle A A B C has area 48. Let P be the midpoint of median A M and let N be the midpoint of
side A B . If G is the intersection of M N and B P , find the area of A M G P .

Hint

Hint

Type your solution, notes and/or work here. Show Solution

7 .6 2 ★ : t V

In this problem we will prove that the medians of a triangle are concurrent. Let A A B C be our triangle. Medians A D and B E
meet at point G. C Ô meets A B at point F .

(a) Show that [A C G ] = [G C B ] = [A G B \

Hint

Type your solution, notes and/or work here. Show Solution

(b) Use your result from part (a) to show that A F = F B .

Type your solution, notes and/or work here. Show Solution

(c) Do the previous parts show that the medians of any triangle are concurrent?

Type your solution, notes and/or work here. Show Solution

7 .6 3 ★ : t V

Point N is on hypotenuse B C of A A B C such that Z C A N = 45°. Given A C = 8 and A B = 6, find A N .

Hint

Hint

Hint

Type your solution, notes and/or work here. Show Solution

7 .6 4 ★ : Source: Mandelbrot t V

The circle at right has radius 1 and is circumscribed about equilateral triangle A B C . If X is the midpoint of
A C and Y is on arc A C such that Z Y X A is right,then find length X Y .

Type your solution, notes and/or work here. Show Solution


T he E ight Poi nt C i rd e T heorem

It is easier to square the circle than to get round a mathematician. - Augustus De Morgan

CHAPTER
i
Quadrilaterals

8.1 Quadrilateral Basics

Figure 8.2: A Concave Quadrilateral

Figure 8.1 shows a quadrilateral, namely A B C D . A quadrilateral has four sides, four vertices, and four angles. All the
quadrilaterals we will study in this text have all four vertices in the same plane. Nearly all quadrilaterals we deal with in this book
are convex, like A B C D above, meaning that all of the interior angles of the quadrilateral are less than 180°. There are also
concave quadrilaterals, like E F G H in Figure 8.2. In this section we address basic definitions and facts that hold for all
quadrilaterals. In later sections, we turn to specific types of quadrilaterals.

P roblem s
Problem 8.1 4 Jump to Solution

Use what you know about the sum of the angles in a triangle to find Z D in the figure below.

As you've probably already considered, we can connect two more pairs of vertices in a quadrilateral by connecting the opposite
vertices.

At left, we have added A C and B D to quadrilateral A B C D . We call these segments connecting the
opposite vertices of a quadrilateral the diagonals of a quadrilateral.

Let’s see how we can use a diagonal in a problem.

Solution for Problem 8.1: We don't yet know anything about angles of a quadrilateral, but we know
about triangles, so we add diagonal A C , thus making two triangles, A A B C and A A D C . The sum
of the angles in each of these triangles is 180°, so the sum of the angles in the whole quadrilateral
A B C D is 2(180°) = 360°. Now we can quickly find Z D :

Z D = 360° - Z D A B - Z A B C - Z B C D = 70°. D

Important: The interior angles of any quadrilateral add to 360°.

Z
Concept: Triangles, triangles, triangles! As you’ll see in the coming sections, nearly everything
! we can derive for specific types of quadrilaterals stems from triangles.

Exercises

8 .1 .1 : * V
8.2 Trapezoids

A trapezoid is a quadrilateral in which two sides are parallel. Figure 8.3 shows trapezoid W X Y Z . in which W X || Z Y .

Figure 8.3: A Trapezoid

Figure 8.4: Is This a Trapezoid?

trano
Ä 7‫״‬ÄiH?• 5 r .•S , , , ?,
-----------------siues
r ” ** < ‫ ״‬mat are parallel III ‫ « ״ « ״ ״ ״ ׳ « * “ ״ ׳ ״‬m m » . » a 4 ; » ,‫ ״‬, M, ,
s r s - z ‫־‬: r r - ? ‫־‬r * d'f‫״״״<*־■״־״״״' ״־־״״•״‬.‫׳״‬
to these people. Figure 8.4 would be considered s ir .‫״‬
be considered a trapezoid. ‫״‬ ‫״‬ ‫״‬ .
answer. Some people define a tra
aeople define a traoezoiri as ha‫ ״‬i
P 6 me " rape‫־‬old as havln9 at ‫׳‬east one pair of parallel sides:

P roblem s
Problem 8.2
4 Jump to Solution
^trapezoid A D C D , we have A D || C D , Z A = 150°, and Z B = 80° Find Z C and
A

Problem 8.3
4 Jump to Solution

C
G NN, D
^mN, V n Cf rDn are
and ' Aas
B shown
\ C D 'india9° nalS 130Find
the diagram. andAANC and
meet
A Bat
. N ' and side lengths U N

In trapezoid E F G H . we have E F || G i l , E F = 6, F G = 10, G H = 12, and E H = 8


Our goal
is to find the length of median M N .

(a) Extend/*,// and F G so that the two meet at X .

(b) Use similar triangles to find X E and X F , then find X N and X M .

(0 Show ,hat A X E F ~ A X N M . Is ,he median parallel to ,he bases of ,he trapezoid?


(d) Find X I N .

Problem 8.5
4 Jump to Solution
Bases P Q and S R of trapezoid P Q R S have lengths 5 and 9, respectively, and are 8 units apart.
(a) Find [P Q S ] and [QBS],

(b) Find [P Q R S ],

(0 Can you find a general formula for ,he area of a trapezoid given its base lengths and the distance between the bases?

Problem 8.6
4 Jump to Solution

^ T x y z Y Z ■‫”״ '׳ ־‬ W X 11^ V Z - n ^ X Z , 20. ,‫ ״‬,hi


this problem we seek the

(a) We build right triangles by drawing an altitude from X to H . How do we know this altitude meets H
between Y and

— x. Use

(c) Find the area of W X Y Z .

Problem 8.7
4 Jump to Solution
In trapezoid A B C D , we have A D || B C , Z B = Z C . and Z B <
90° Prove that A B = C D .
Hint

Problem 8.8
4 Jump to Solution
.‫ ״‬trape,oid C H U . Ö H || 7 J G H . & H I . CJ - & ‫״‬ - 10. ,‫ ״‬,‫״‬,‫ ״‬plopt m « ‫״‬ flnd H J

(a) Draw heights H X and G Y . Find J Y , Y X , and X I .


(b) Find M X , then find H J .

Many trapezoid problems are solved using the basic properties of parallel lines.

Problem 8.2
t V
In trapezoid A B C D , we have A B || C D , Z A = 150° and Z B = 80°. Find Z C and Z D .

0‫׳‬, S I Since D
A
|| .we h‫ ״‬e
D
C

Problem 8.3
t V
V x ^ 'r n BCD' 11 C D ' dia9onals B D and A C meet at N , and lengths B N C N
D N , and C D are as shown in the diagram. Find A N and A B . ‫־‬ A B

16 12

So/ut/on for Problem 8.3: Parallel lines mean similar triangles: A B || C D gives A A B N ~ A C D N ,
so we have
AN BN AB
CN DN CD'
Since we have B N / D N = 1/4, we find A N = C N / 4 = 3 and A B = C D /4 = 5. □

legs*of thetrapezoWen,i° n * ^ medi3" ^ ‫“ ״ “״‬ ° f 3 trapeZOid The median of a trapezoid connects the midpoints of ,he

Problem 8 .4
t V

Ä E Ä * ‫ ״ ־׳ ׳ ״‬- & « ‫״‬ ‫« “־״‬

Solution for Problem 8.4: Make sure you understand why this solution is incomplete:

Bogus Solution:
hi"7 ! » : ‫־‬b6tWeen the " neS' and 80 is N ■ M N must be Parallel to the
y bases and midway between the lengths of ,he bases. Therefore, M N is the average
of the bases, so M N = ( E F + G H ) / 2 = 9 a

complete solution looks like: " 0166 3t ‫ ״‬h thiS inspira,ion as a starting point, here's what a

Af
XE EF
A
XH ~ GH'
So, substitution gives c/ \ f
n ! \ M
XE 1 J - V
A
XOE + 5
8 2 ‫־‬Z '

S S , 1; il~ x V 3 2 № 2 Z ° , - ■‫ * ׳‬F; r r
“ ‫׳‬f ™ ‫ ־‬w - - 7
N M - (3 / 2 ) ( E F ) = 9. Notice also that Z X N M = Z X E F from our similarity, so N M || E F . a

Important: The!median of a trapezoid is parallel to ,he bases of a trapezoid, and equal in length
^ to the average of the lengths of the bases. 9

( « ‘ 0 ‫״ • ״ «״‬ ‫ ״ ״ “ ״ ״‬° ‫ " » ״ • ■ » « “ ״ » ״ ״ ״ “ * ״ ״ ׳‬Pf8.4 ,0 ™ » ‫ ־‬p m ., these ,‫״‬

We used triangles to l‫ ״‬m ,boa, m«di,b; ‫״ ״ ״‬ ‫ ״ ״‬th‫ ־‬m ‫ ״‬, . ‫< ״ ״‬b‫ ־‬ut f t ‫ ־‬m ‫־ (־‬

Problem 8.5
t V
Bases P Q and SR have lengths 5 and 9, respectively, and are 8 units apart. Wha, is the area of trapezoid PQRS?

Solution for Problem 8.5: We know how to find ,he area of triangles, so we divide ,he trapezoid into two
triangles by drawing diagonal Q S . We then draw the altitudes Q X and S F of the two triangles Since

:: ^ ‫ ־‬s w5 3 “ - «‫־ ־״׳‬ ~ Ç S;


[PQÄ5] = [PQS] + [QPS]
= (p Q )(S Y ) t (S R )(Q X )
2 + 2
, 8

= 5G.

S S * ‫“ ״‬ “ - “ •> - • ‫— ־׳‬ ‫״׳‬

‫ ׳ ״ ׳ ״ • ״ ״ ״ " ■ ״ ״ ־‬, h‫׳ ־ ״ ׳ ״ ״ ־ ״ ׳ » ״ ״ ״ ״ ״ “ • ״ ״ ־‬

Problem 8.6
t V
In trapezoid W X Y Z . we have || 7 2 . W X = 17, X Y = 13, Y Z = 21, and * Z = 20.
Find the area of W X Y Z.

So/utron fo rP ro ie m 8.6: We needthe height, so we try to add a heigh, to the the diagram in a way that forms useful right

T h f^ ^ 9 fr° m " t0 H " the m° St Pr° miSin9 Candidat6' bU< We f'rSt h3Ve ,0 f,9Ure ° ut where the altitude from X
S S, n0t immediately Clear ' I ^ e f00t ° f the al,itUde' - ' ‫ ״‬call H . is on F Z or not. (It might be beyond Y on

and H Y are both less than Y Z . H must be on Y Z . Now we can confidently draw our diagram with H o n V Z .

T h e o re m " A X H Z a n d A A T /y • 1‫ ^ ״‬Z " = 21 ~ X and we can now Pythagorean


W ------ -- ,X
x 2 + H1 = 169
(21 - a:)2 + h 2 - 400

Subtracting the first equation from the second allows us to eliminate h and find x. We get ^

[(21 - a;)2 + h 2} - ( x 2 + h 2) = 400 - 169.


Therefore,

x 2 - 42x + 441 + h 2 - x 2 - h 2 = 231,


so —42 x — —210, which gives x = 5.

‫״‬ « * > ‫ ־ ״ ״‬goossed * . « .11. . ‫„ ־ ״‬ of


nicel, doos‫׳״‬, m a n that h most bo 12. W , still b „ a » cbccl t h T z / ^ l ' ( 2 1 “ ■ “ “ , ' » , “ ‫" ׳‬ h,P° " " ‫ “ ״‬s
Now that we have h = 12, we can find

[ W X Y Z ] = ( h ) ( W X + Y Z ) / 2 = (1 2 )(3 8 )/2 = 228.

C7 ' ‘ Ä S t S s m ‫“־‬ ” “ ‫ " ״ “ ״ ״‬S‫ ״‬, ‫ ״‬,n ‫ " ־‬S “ ’

We II now take a look at a special class of trapezoids.

Problem 8.7
t V
In trapezoid A B C D , we have A D \\ B C , Z B = Z C . and Z B < 90°. Prove that A B = C D .

n T l x = D v ml 7: Onc!m t ‫׳‬- ^ introduce ri9bt triangles by drawing al,itudes from A and D


Therefore, A B = C D ™ * * " " C $° ^ M S Con9ruence, we have A A B X St A D C Y .

Notice ,hat we can easily deal with ,he case of Z B and Z C being obtuse by noting that if Z B = Z C ,

Z f4 = 180° - Z B = 180° ~ Z C = Z D .

legs of ,he trapezoid are equahn ten^h"□ ^ ^ aCUt<5’ 3nd ^ 030 US6 essen,ially the same Proof as above to prove that the

I ” ,”. S d F l r S “ g ,r.m r r i d ' “ r ° “ r ‫ * ״‬, 0‫״ ־‬ h‫ ־‬r b~ " ‫— • ״ ׳ ־ • " ־‬ *


diagonals of an isosceles trapezoid are congruent Finally in S e J m T z i* trar h ° ' d ^ 30 ExerClse' you,‫ ״‬also Prove that the
that if ,he legs are equal in length (and not parallel) o if the d a a o n a ls ^ t 1 ^ rela,ionshiPs run 'backwards'; i.e.,
angles are equal 9 dragonals of a trapezoid are congruent, then both pairs of base

Important: In an isosceles trapezoid:

z (a) The base angles come in two pairs of equal angles


as shown at right.

(b) The legs are equal.

(c) The diagonals are equal.

If ‫־‬my one of these items is true for a trapezoid with exactly one pair of parallel sides
then all the others must be true for that trapezoid.

Problem 8.8
t V
Find / / . / given that G H I J is an isosceles trapezoid with G H = 8, H I = G.I = 6, and I J =

7 * Ä K S r . 7 ,. r 2 r / x S S Ä
eongruence. Therefore, J Y = X I . Next, we see that Y X = G H because G Y

Now we find H X from A H X I H X = J ‫־‬H D - TY? - , / 5 ^ c• ‫״‬


to get H J : V 1 1 X - V 35. Finally, we use right triangle H X J

H J = S H X * + X J * = y / 3 5 T 8 T = v^ T 6 = 2F29.
Building right triangles to find lengths strikes again. □

‫ ״־‬d » . ‫״‬ . ‫ ־‬PP‫ ־‬, « , « d „ . tb , £. "

Exercises

trapezok|a,eral PQRS' PQ "Ä5'^ "n0tPara‫״‬e' t0 QR = P S . and Z P = 83°. Fi


Find the rest of the angles of the

Type your solution, notes and/or work here.


Show Solution

Find the area of a trapezoid with bases 44 and 24 and with height 18.

Type your solution, notes and/or work here.


Show Solution

length of the shorter base. i! 96‫ ׳‬o n ‫־‬ ‫“• ״‬ * 6 ‫•־ ״ ״‬ ‫״ ״ ״ «־ » ׳‬

Type your solution, notes and/or work here.


Show Solution

A B C D shown at right is a trapezoid with B C || AD, Z A B D = 88° Z A


= 72°, and
Z C = 126°. Find Z A D C and ZCBD.

Type your solution, notes and/or work here.


Show Solution

Prove that the diagonals of an isosceles trap!

Type your solution, notes and/or work here.


Show Solution

pna ! a l ' l e l ^ SUCfh tdat A B » ^ a" d A B < C D , then the median of A B C D is


that this proof covers any trapezoid in which the6leg^are n^t p^ralTelT ^ ^ traPeZ° ,d‘ ^ Pr° b,em 8 8 8 4‫ ־‬your guide• (Note

W A X D C ’* Z c AB*m“ P° i'‫’ ״‬ ® m " № l r 4 ‫ ״ * ־‬D and to m « , „ X . P,o‫״‬

Type your solution, notes and/or work here.


Show Solution

(b) Prove that A D / X A - B C /X B , and use this to prove that A M / X A = B N / X B .

Type your solution, notes and/or work here.


Show Solution

(c) Prove that X M / X A = X N / X B .

Type your solution, notes and/or work here.


Show Solution

(d) Prove that M N || A B .

Type your solution, notes and/or work here.


Show Solution

(e) *Prove that M N = ( A B + C D ) /2.

Hint

Type your solution, notes and/or work here.


Show Solution

The bases of a trapezoid have lengths 50 and 75. Its diagonals have lengths 35 and 120. Find ,he area of the trapezoid.
Hint

Hint

Type your solution, notes and/or work here.


Show Solution

Extra! Consider a quadrilateral with perpendicular


diagonals as shown at right. The Eight-Point Circle
V
Theorem tells us that the midpoints of the sides and
the feet of the perpendiculars from these midpoints
to the opposite respective sides of the quadrilateral
all lie on the same circle.

If you can’t prove the Eight-Point Circle Theorem


now, try coming back to it after you've tackled
Problem 8.19. You might find the result of Problem
7.6 useful, as well.

Extra! If we construct an equilateral triangle on the outside of


each side of any triangle, the triangle formed by rv7‫ * " ־‬- . .
V
connecting the centroids of these equilateral triangles
is called the Outer Napoleon Triangle. As the diagram >
strongly suggests, the outer Napoleon triangle is
always equilateral. As you might guess, we can
construct the equilateral triangles 'inward' and connect
their centers to build an Inner Napoleon Triangle. Draw
a triangle, then construct the Inner Napoleon Triangle.
Does it look equilateral, too?

While the French emperor Napoleon Bonaparte is generally given credit for
discovering these triangles, there’s no solid proof that he actually did so.
8.3 Parallelograms

Figure 8.5: A Parallelogram

^ r S t diS^ T ^ ° 9ram ‫ ״‬b° ,h P9irS ° f ° PP0Si,e Sid6S are Para‫״‬el■A B C D in R9ure 8 5 is » parallelogram because

P roblem s
Problem 8.9
4 Jump to Solution

7 a n d Z a A ~ X + % Z I 1 = :h '■ and Z D = 30° in parallelogram A B C D as shown, find x,


x+

Problem 8.10
___ 4 Jump to Solution
In this problem we show that if A B = C D and A D = B C , then A B C D is a parallelogram.

(a) Draw a quadrilateral A B C D with A B = C D and A D = B C . and draw diagonal A C . Prove that A B C A * A D A C .
(b) Prove that A B || C D .

(c) Prove that A B C D is a parallelogram.

‫״‬ t t 4™ ‫״‬ " A D = B C >‫ ״‬a A B ‫' ־‬C D J M‫ ־‬k• * ‫ ״ ״ ״ ״‬- ‫ « » • ״ ״ ״ ״‬- ‫ ״ « ״ ׳‬i ‫״ ־ » ־ ״‬

Problem 8.11
4 Jump to Solution
(a) Prove that if E F G H is a parallelogram, then diagonals ‫־‬E G and 7 7 7 bisect each other.

<B> * Y ‫ ״ ״‬a■' r v r 7 and of W X V Z b is .» ..o h

Problem 8.12

Sides E U and N T of parallelogram E N T U are 5 units apart as shown. E U = 12 and E N = 8

W tmpde z ï!d )rea ° f E N T U ■ (Him: Rememb6r ,he diSSe° ti0n We USed t0 ^ e area of a

(b) How far apart are sides E N and T V ?

‫“״ “ • ״ ״ ״ “״‬ ‫״•״ • “ * ״ ״ * ״ ״ ״‬ ‫—״‬ — •0‫־״‬. « ‫ ״‬.

Problem 8.9
t V
Given that the measures of the angles of parallelogram A B C D are as shown, find x. y. and Z C .

Y * + ? w B

Solution for Problem 8.9: Since A D II B C we have / n ! / r — ic n ° ___ ___

Notice that in our solution we found that Z B = / / ) and / 4 — / C At th« ^ +u


opposite angles of any parallelogram are equal. ‫־‬ d ° f ,he SeCt'° n' y0U " be asked t0 Prove * * the

Problem 8.10
t V
Show that if A B = C D and A D = B C , then A B C D is a parallelogram.

Solution for Problem 8.70. Where did we go wrong here:

Bogus Solution: We_Cu1 A B C D into triangles by drawing A C . Since


V A D || B C . we have Z D A C = Z B C A . Combining
1 *his with A D = B C and A C = A C . we have
A D A C = A B C A by SAS Congruence. Therefore,
Z B A C - Z A C D , so A B || C D . Similarly, we can
show that A D || B C . so A B C D is a parallelogram.

Fortunately m g ^ e s S J o n S e ! ^ ^ ^ ^ d° n,t k" ° W ' hat ™ 11 ^ 80 we ca" ’* use ‫■** ״‬

= ° Z A C D eJ ° i h r ^ T m f I1 1 ‫ ״‬10‫ ׳‬f ° Ve‫ ־‬/ ‫ ^ ״‬aVe A D / l C “ by SSS « « Therefore,


parallelogram. □ ‫׳״‬ ‫ ״‬° * ^ Z D A ° = Z A C B 50 A D II
as well. Thus, A B C D is a

We can also quickly showj^at in any parallelogram, the opposite sides must be equal. We do so
by again using diagonal A C of parallelogram A B C D . We have A .4 J3 C “ A C D A bv ASA
Congruence. These congruent triangles give us A B = C D and B C = A D . Make sure vou see
why this addresses a different question from the one initially asked in this problem!
D
We've looked at the sides and the angles of a parallelogram: let's check out the diagonals.

Problem 8.11
t V
(a) Prove that if E F G H is a parallelogram, then diagonals E G and 7 7 7 bisect each other.

(b) MAYY Z a'parallelogram ?? 9iVen dia9° na'S W and * * ° f * ‫ ״ * « ״ * ״‬I bisect each ‫ ״‬th e , is

Solution for Problem 8.1 1 :

(a) Z F F r FG/ l n u Paralle'° 9ram; we have E F = G H and E F || G H . Therefore,


ZFEG = ZEG H and Z E F H = Z F H G . so A E F O ^ A G H O by ASA
S o o n T Cef r r r u T E ° = O G and F ° = ° H ■ 50 ° is the midpoint of both
diagonals of E F G H . Thus, the diagonals of a parallelogram bisect each other.

(b)
° " Ce a‫ ״‬,a'n' We markAthe information we're given - the fact that the diagonals bisect each
other. We seejhat A .W O Z SSA Y O X by SAS Congruence, so ZZ W Y = l w Y X
^ h means W Z || X Y . Similarly A W O X 2 A Y O Z . which we can use to show
| Y Z Hence, if the diagonals of a quadrilateral bisect each other then the
quadrilateral is a parallelogram.

u r . _

vve^rtoniP trre'Triem rO iniai,‫ " ״‬from the last three problems to see parallelograms the way an experienced geometer
sees

Important: In parallelogram A B C D . the opposite sides are equal, the opposite angles are equal
^ and the diagonals bisect each other. '

Figure 8.6: Side and Angle Equalities in Parallelograms


Conversely, A B C D is a parallelogram if any one of the following are true:
(a) .4J3 = C D and A D = B C .

(b) Z A — Z C and Z B — Z D .

(c) Diagonals A C and B D bisect each other.

Proving one of these means the other two are true.

ea^er approach" 9 * ^ ‫ ״ ״‬Paral‫׳‬el° 9ram■We could ‘ he same tactic as we used an even

Problem 8.12
t V
Sides E U and N T of parallelogram E N T U are 5 units apart as shown. E U = 12 and E N =

(a) Find the area of parallelogram E N T U .

(b) How far apart are sides ‫־‬E N and T V ?

Solution for Problem 8.12:

[E N T U ] = [E U T ] + [N E T ]

_ ( E U ) ( 5) , ( N T ) ( 5)
2 + 2
_ (E U )(5 ) , (£ 1 0 (5 )
2 + 2
= ( E U ) ( 5)
= 60.
Notice that we can easily use this approach to show that the area of a parallelogram is its base times its height to that base.

(b) K / T 6 the h336' EN‘ the are9 fr° m the Pr6Vi0US Part' SinCe tMe area ° f ENTU is 60' the hei9ht between E N and

[E N T U ] _ 60 _ J5
EN 8 ‫־ ־‬2 ‫׳‬

7 Y U I is a parallelogram with T Y = (j and Y U = 8.

(a) What is the perimeter of T Y U I?

Type your solution, notes and/or work here.


Show Solution

(b) Do we have enough information to find the area of T Y U /?

Type your solution, notes and/or work here.


Show Solution

W X Y Z is a parallelogram. Prove that ZYV = Z Y

Type your solution, notes and/or work here.


Show Solution

In quadrilateral W O R K . W O = R K and ‫־‬W O || R K . Prove that W O R K is a parallelogram.

Type your solution, notes and/or work here.


Show Solution

rimes itsehe,ghXSeC,i° n * 9 Para‫״‬el° 9ram t0 tur" « in‘ ° a ™ ‘ angle and prove that the area of the parallelogram is its base

Hint

Type your solution, notes and/or work here.


Show Solution

Is it possible for a parallelogram E N T U to have E N


or why not? 4, E U = 12, and for sides E U and N T to be 5 units apart? Why

Hint

Type your solution, notes and/or work here.


Show Solution

In this Exercise, we give another way of deriving the length of the median and area of a trapezoid.

Let A B C D be a trapezoid with bases ~AB and ‫־‬C D .

(a) A A D D is a
P9St ' 1 1° A ' SUCh that B A ' = C D ■and ex,end m
parallelogram. Pas‘ C D ' - c h that C D ’ = A B . Prove that

Type your solution, notes and/or work here.


Show Solution

‫״‬ - n a a - — *

Hint

Type your solution, notes and/or work here.


Show Solution

(C) that
that M
MNN - 9"Mu MS /2
o . the m!dPOin,S — A B 4‫ ־‬C D , and prove
that °Mf A
N D=■B( C ■and A 'DD)'■
Conclude AB + C /2.resPective|y■ Prove that M M '

Type your solution, notes and/or work here.


Show Solution

Ä X t T Ä ‫ * ־ ״‬- - — - - * — “‫ ׳‬A « «
Hint

Type your solution, notes and/or work here.


Show Solution
8.4 Rhombi
+ B

Figure 8.7: A Rhombus

A quadrilateral is a rhombus if all of its sides are equal.

P roblem s
Problem 8.13 4 Jump to Solution

Prove that every rhombus is a parallelogram.

Problem 8.14 4 Jump to Solution

A B C D is a rhombus such that its diagonals A C and B D have lengths A C = 30 and B D = 16. In this problem, we will
find the area of A B C D and the side length of the rhombus.

(a) Let the diagonals intersect at X . Show that A A X B, A C X B, A C X D, and A A X D are congruent.

(b) Prove that A C _L B D .

(c) Find the area of A B C D .

(d) Find A B .

We can prove a whole lot about rhombi in one simple step.

Problem 8.13 t V

Prove that every rhombus is a parallelogram.

Solution for Problem 8.13: Let A B C D be a rhombus. Since A B = C D and A D = B C (because all the sides of A B C D are
equal), we know that A B C D is a parallelogram. □

Important: Every rhombus is a parallelogram. Therefore, all that is true about parallelograms is
true about rhombi.
Z
WARNING!! (Bet you saw this one coming.) Every parallelogram is not a rhombus. Therefore, if we
prove a property of rhombi, this property is not necessarily true for all parallelograms
O - we'd have to prove the property for parallelograms separately.

To see some of the properties that distinguish a rhombus from a parallelogram that is not a rhombus, let's try a problem.

Problem 8.14 t V

A B C D is a rhombus such that its diagonals A C and B D have lengths A C = 30 and B D = 16. Find A B and [A B C D \

Solution for Problem 8.14: We start with a diagram, in which we note that all the sides of
A B C D are equal, and that the diagonals bisect each other since rhombus A B C D is also a
parallelogram. All four little triangles in the diagram are congruent by SSS Congruence. Since
all four angles at X must be equal and they add to 360°, they must be right angles. Hence,
the diagonals of a rhombus not only bisect each other, but they are also perpendicular. Since
each of these right triangles have legs of length 8 and 15, their hypotenuses (which are the
sides of the rhombus) each have length 17.

All those congruent right triangles give us a quick approach to finding the area.

(A X )(B X )
[A B C D } = 4 [A B X ] = 4 = 2 (A X )(B X )

Notice that since A X and B X are each half a diagonal of A B C D , we have

‫ ( ־‬f ) ( f ) - u s m .

Our solution gives us a few special rhombus facts to add to all the facts we know about rhombi because they are parallelograms.

Important: The diagonals of a rhombus are perpendicular.


The area of a rhombus is half the product of its
z diagonals (of course, it is also base times height
since a rhombus is a parallelogram).

(A C )(B D )
[A B C D ] =
D

Exercises

8 .4 .1 : t V

P Q R S is a rhombus with diagonals P R = 6 and Q S = 12. Find the area and the perimeter of P Q R S .

Type your solution, notes and/or work here. Show Solution

8 .4 .2 : t V

W X Y Z is a rhombus with W X = 50 and W Y = 96.

(a) Find X Z.

Type your solution, notes and/or work here. Show Solution

(b) Find [W X Y Z ].

Type your solution, notes and/or work here. Show Solution

(c) Find the distance between W X and Y Z .

Hint

Type your solution, notes and/or work here. Show Solution

8 .4 .3 : t V

Diagonals .4Cand 13D of quadrilateraL4£C\D are perpendicular. Prove that [A B C D ] = ( A C ) ( B D ) /2 .

Hint

Type your solution, notes and/or work here. Show Solution

8 .4 .4 : t V

T U V W is a rhombus with T V = 10 and Z .T U V = 60°.

(a) Show that Z .T U W = 30°.

Type your solution, notes and/or work here. Show Solution

(b) Find [T U V W ].

Type your solution, notes and/or work here. Show Solution


8.5 Rectangles
We've looked at quadrilaterals in which all the sides are equal. Now we consider what happens if all the angles are equal. Such a
quadrilateral is called a rectangle.

Problems

Problem 8.17 4 Jump to Solution

Prove that the diagonals of a rectangle are congruent.

Problem 8.18 4 Jump to Solution

Find the length of a diagonal of a rectangle given that its perimeter is 44 and one side has length 10.

Problem 8.19 4 Jump to Solution

In this problem, we find two different proofs that the quadrilateral formed by connecting the midpoints of a rhombus is a
rectangle.

(a) Start with a diagram. Draw rhombus A B C D and let E F G H be the quadrilateral formed by connecting the midpoints of
its sides, with E on A B , E on B C , and so on.

(b) Solution 1: Show that Z A E H = Z A H E and that A A E I I = A C G F . Use this as a starting point to show that the
angles of E F G I I are all equal.

(c) Solution 2: Show that E F || A C . Show that the angles of E F G I I are right angles.

We know the angles of a rectangle are equal, but to what?

Problem 8.15 t V

What are the measures of the angles in a rectangle?

Solution for Problem 8.15: The angles must add to 360° and they're all equal, so each of the four angles must be a right angle. □

As we did with rhombi, we can quickly get a wealth of information about rectangles.

Problem 8.16 t V

Prove that all rectangles are also parallelograms.

Solution for Problem 8.16: Let A B C D be our rectangle. Since Z A 4‫ ־‬Z B = 180°, we know that
A D || B C . Similarly, Z B 4 Z C = 180°, so A B || C D . Therefore, A B C D is a parallelogram. □

Among the many useful aspects of knowing that every rectangle is a parallelogram, the one that is probably most used is the fact
that opposite sides are equal in length. When talking about rectangles, these two dimensions are often called the length and
width of the rectangle. We have already seen that the area of a rectangle is the product of these two dimensions.

We know that the diagonals of a rhombus are perpendicular, so we wonder if there’s a similar special property of the diagonals of
a rectangle.

Problem 8.17 t V

Prove that the diagonals of a rectangle are congruent.

Solution for Problem 8.17: As we have in many other problems, we turn to congruent triangles for a
solution. Drawing our two diagonals, we have A P Q R = A Q P S by SAS since P S = QR,
P Q = PQ, and Z P Q R = Z Q P S . Therefore, P R = QS. □

Problem 8.18 t V

Find the length of a diagonal of a rectangle given that its perimeter is 44 and one side has length 10.

Solution for Problem 8.18: Let W X Y Z be our rectangle and let X Y = 10. We know the diagonals
have the same length, so we don’t have to worry about which diagonal to find. Since X Y = 10, we
have W Z = 10 as well. Since the perimeter is 44, we have

\V X + X Y + Y Z + Z W = 44.

Since W X = Y Z and X Y = W Z = 10, we have W X 4 ‫ ־‬10 4 ‫ ־‬W X 4 4 4 = 10 ‫־‬, so


W X = 12. Now we can use the Pythagorean Theorem to find

X Z = \ / W X 2 + W Z 2 = n/ 100 + 144 = 2v/61.

Important: The diagonals of a rectangle are equal to each other and


equal to the square root of the sum of the squares of the
Z length and the width of the rectangle.

Q S = P R = s jQ R 2 + S R 2

You shouldn’t have to memorize how to find the diagonals of a rectangle: the right angles of a rectangle should make it clear to
use the Pythagorean Theorem with the sides to get the diagonal.

Now we'll take a look at one way rectangles and rhombi are related, before moving on to a quadrilateral that is both a rhombus
and a rectangle.

Problem 8.19 t V

Prove that the quadrilateral formed by connecting the midpoints of the sides of a rhombus is a rectangle.

Solution for Problem 8.19: We draw rhombus A B C D and let E F G I I be the quadrilateral formed by connecting the midpoints
of the sides of A B C D , with E on A B , E on B C , and so on. We will take two different approaches to solving the problem.

(a) Since we want to show that E F G I I is a rectangle, we look for a way to find the
measures of each angle of E F G I I . Since its vertices are the midpoints of
A B C D and A B C D is a rhombus, the vertices of E F G H divide the sides of
A B C D into 8 equal segments as shown.

Since A B C D is a parallelogram, Z A = Z C and Z B = Z D . Therefore,


A A E I I = A C G F and A B E F = A D G H . Furthermore, each of these
small triangles is isosceles, so we can let Z A E H = x and Z D G H = y and
identify equal angles as shown. Finally, we see that each angle of E F G H has
measure 180° — x — y. Thus, the angles of E F G H are equal and E F G H D
must be a rectangle.

(b) For another approach, we might note that we want to prove that the angles of
E F G H are right angles. This gets us thinking about where we can find right Б
angles in A B C D . Since A B C D is a rhombus, its diagonals are perpendicular.
To relate the sides of the rectangle to these diagonals, we find that
A A E I I ~ A A B D by SAS Similarity since A E / A B = A H / A D = 1 /2 and
Z E A H = Z B A D . Therefore, Z A E H = Z A B D , so E H || B D . Similarly,
we can show that F G || B D . Since A C is perpendicular to B D , it must be
perpendicular to E I I and E G as well, because both E H and F G are parallel to
‫־‬B D .

We can use exactly the same approach to show that E F || A C || G H , from


which we can conclude that E F and G I I are perpendicular to B I) . Now we can tie all these parallel and perpendicular
lines together. Since E F J_ B D and F G || B D , we have E F J_ F G . In exactly the same way, we can show that each of
the other three angles of E F G H is a right angle. Thus, E F G H is a rectangle.

Exercises

8 .5.1: t V

P O S T is a rectangle with P O = 8 and O S = 1 2 .

(a) Find the perimeter of P O S T .

Type your solution, notes and/or work here. Show Solution

(b) Find PS.

Type your solution, notes and/or work here. Show Solution

(c) Find [P O S T ].

Type your solution, notes and/or work here. Show Solution

8.5.2: t V

The length of a rectangle is one less than twice its width. If the perimeter of the rectangle is 36, what is the area of the
rectangle?

Type your solution, notes and/or work here. Show Solution

8.5.3: t V

Diagonals W R and E T of rectangle W E R T meet at Y. Given that Z W Y E = x, find Z E R Y and Z Y R T in terms of x.

Type your solution, notes and/or work here. Show Solution

8 .5.4: Source: MATHCOUNTS t V

A semicircle with center O has a radius of 9 cm. What is the number of centimeters in the length of
RQ, a diagonal of the rectangle shown?

Type your solution, notes and/or work here. Show Solution

8.5.5: t V

I have a 36 inch by 24 inch rectangular painting. I would like to place a frame that is 2 inches wide around the painting. If the
material for the frame costs $1.50 per square inch, how much will the frame cost?

Type your solution, notes and/or work here. Show Solution

8.5.6: t V

What kind of quadrilateral do we get when we connect the midpoints of the sides of a rectangle? (Prove your answer!)

Type your solution, notes and/or work here. Show Solution

8 .5.7★ : t V

E F G H is a rectangle with area 48. If EG.11 is a rectangle such that I I is on ./ /, what is the area of EG.11?

Hint

Type your solution, notes and/or work here. Show Solution


8.6 Squares

We've studied quadrilaterals with all their sides equal, and quadrilaterals with all their angles equal. What if a quadrilateral has
both all sides equal and all angles equal? Such a quadrilateral is called a square, an example of which is shown in Figure 8.8.

As we’ll see, squares are the easiest quadrilaterals to work with since everything that is true about rectangles, rhombi, and
parallelograms is also true about squares.

P roblem s

Problem 8.21 4 Jump to Solution

Find the perimeter and area of a square that has a diagonal of length 8.

We’ve already studied all the tools we need for squares. Flow we use these tools in the following two problems should be
unsurprising.

Problem 8.20 t V

Find the perimeter and area of a square with side length 9.

Solution for Problem 8.20: Since all the sides of the square are equal, the perimeter of our square is 4 x 9 = 36. Since a square
is a rectangle, its area is its length times its width. These dimensions are the same in a square, so the area is just the, urn,
‫׳‬square' of its side length, or 92 = 81. (Now we see why multiplying a number by itself is called 'squaring' the number.) □

Problem 8.21 t V

Find the perimeter and area of a square that has a diagonal of length 8.

Solution for Problem 8.21: A square is a rectangle, so its diagonals are congruent. It is also a rhombus, so its area is half the
product of its diagonals, or (82) /2 = 32 in this case.

We can find a side of the square by noting that a diagonal of a square splits the square into two 4 5 9 0 ‫־‬45‫ ־‬triangles, so each side
equals the diagonal divided by V2, or 8 /V 2 = 4>/2. Alternatively, since the area is just the square of the side length, we can
take the square root of the area we already found: ‫־‬v/32 = 4 \/2 . Thus, the perimeter is 4 (4 \/2 ) = l6 \ /2 . □

These two examples give us the most useful length and area relationships for a square.

Important: Let s be the side length of a square, P be the perimeter, and A


the area. We have
Z
P = 4s A = s2.

Drawing a diagonal creates two 4 5 9 0 ‫־‬45‫ ־‬triangles. Letting the


length of a diagonal be d, we have

d = sV 2 P = d( 2 n/2 )

We often encounter squares in problems together with circles, since both involve segments with equal lengths (sides for the
square, radii for the circle).

Solution for Problem 8.22: There doesn't seem any easy way to get / A O C , so we try splitting it into
pieces by drawing O D . Since A A Y O = A D Y O by SAS, we have A O = O D . Furthermore,
A D = A O because they are radii of the circle. Thus, A A O D is equilateral, which implies
/ A O D = 60°. Now we only have to find /.D O C .

Since O D equals the side length of the square, we have O D = D C . Therefore, A D O C is


isosceles, so / D O C = / D C O . Furthermore, we already know

/ O D C = / A D C - / A D O = 90° - 60° = 30°.

From A D O C we have

/ D O C + / D C O -F / O D C = 180°,

so 2/ D O C 4 1 8 0 ° = 30° ‫־‬. Therefore, / D O C = 75°. Finally, we have

/ A O C = / A O D + / D O C = 135°.

Note that you might also have found / A O D = 60° by seeing that A O = A D = 2 A Y and / A Y O = 90°, so A A O Y is a
3 0 9 0 ‫־‬60‫ ־‬triangle (and likewise for A D O Y .) □

Concept: Breaking a desired angle, length, or area into parts often makes it easier to find.

Exercises

8 .6 .1 : t V

The area of square E F G H is 80. Find E F and EG .

Type your solution, notes and/or work here. Show Solution

8 . 6 .2 : t V

M is the midpoint of A B on square A B C D . If A C and B D meet at O, and M 0 = 4, what is the area of square A B C D ?

Type your solution, notes and/or work here. Show Solution

8 .6 .3 : t V

The diagonals of square T Y U I meet at M . Point K is on side T Y such that T K = T M . Find/ M T K and / T M K .

Type your solution, notes and/or work here. Show Solution

8 .6 .4 : t V

Show that a rectangle with perpendicular diagonals must be a square.

Type your solution, notes and/or work here. Show Solution

8 .6 .5 : t V

A B C D and A C F G are squares. Find [A C E G ]/[ A B C D \

Flint

Type your solution, notes and/or work here. Show Solution

8 .6 .6 : t V

Point E is inside square A B C D such that A A B E is equilateral. Given that A B = 4, find the following:

(a) AE.

Type your solution, notes and/or work here. Show Solution

(b) [A B C D ].

Type your solution, notes and/or work here. Show Solution

(C ) [A B E ],

Type your solution, notes and/or work here. Show Solution

(d) / D A E and/ D E A.

Type your solution, notes and/or work here. Show Solution

(e) The area inside A B C D but outside A A B E .

Type your solution, notes and/or work here. Show Solution

(f) CE.

Type your solution, notes and/or work here. Show Solution


8.7 If and Only If
In this section, we learn what ‫׳‬if and only i f means by exploring what facts we need to classify a quadrilateral, and what facts we
know given a quadrilateral's classification.

We already have a good idea of what 'if means. For example, we can write 'An animal has four legs if it is a dog.' However, 'only if
is a little trickier - for example, the statement 'An animal has four legs only if it is a dog' is clearly false. Cats have four legs, too.

Many mathematical statements put 'if and 'only i f together. Here's what it looks like in a non-mathematical context:

A month has fewer than thirty days if and only if the month is a February.

This statement is equivalent to saying both of the following statements at the same time:

Every month with fewer than thirty days is a February.

Every February has fewer than thirty days.

In other words, to prove an 'if and only i f statement, we must prove two different things. In our example above, we would have to
show that every month that has fewer than thirty days is a February, and we would have to show that every February has fewer
than thirty days. Make sure you understand that these are two different statements to prove!

Important: Proving ,if and only i f statements requires proving two different statements.

Z
P roblem s
Problem 8.23 4 Jump to Solution

Prove that A B C D is a parallelogram if and only if A A = A C and A B = A D .

Problem 8.24 4 Jump to Solution

(a) Prove that if A B C D is a square, then its diagonals bisect its angles.

(b) Is it true to say that A B C D is a square if and only if its diagonals bisect its angles?

Problem 8.25 4 Jump to Solution

Prove that the diagonals of a trapezoid are congruent if and only if the trapezoid is isosceles.

Hint

Problem 8.26 4 Jump to Solution

In this problem, we prove that the circumcenter of a triangle is the same point as the incenter if and only if the triangle is
equilateral.

(a) First, we tackle the 'if' part: Show that in an equilateral triangle, the circumcenter and the incenter are the same point.

(b) In the next three parts, we tackle the 'only if part of the problem by showing that if the incenter and the circumcenter are
the same point, then the triangle is equilateral. Let X be this common point and A A B C be our triangle. What does X
being the circumcenter tell us about X I

(c) What does X being the incenter tell us about X?

(d) Combine your answers from the previous two parts to find congruent triangles. Use these triangles to deduce that
A A B C is equilateral.

Now we'll try our hand at a few proofs (or disproofs) of ,if and only i f statements

Problem 8.23 t V

Prove that A B C D is a parallelogram if and only if / . A = A C and A B — A D .

Solution for Problem 8.23: What's missing from this solution:

Bogus Solution: Since A B C D is a parallelogram, we have A D || B C , so


.. A A 4‫ ־‬A B = 180°. Similarly, A B || C D means
y A B 4‫ ־‬A C = 180°. Therefore, A A = A C . Similarly, we
can show A B = AD.

This solution only does half the problem. 'If and only i f problems are two-part problems! We must prove two statements. First,
we show:

A B C D is a parallelogram if A A = A C and A B = A D .

Since the angles of a quadrilateral add to 360°, we have A A 4‫ ־‬A B 4‫ ־‬A C 4‫ ־‬A D = 360°. Since A C = A A and A D = A B ,
we have 2(A A 4‫ ־‬A B ) = 360°, so A A 4‫ ־‬A B = 180°. Therefore, A D || B C . Since A A = A C , we have A B 4‫ ־‬A C = 180°
also. Therefore, A B || C D and A B C D is a parallelogram.

Next, we must show:

A B C D is a parallelogram only if A A = A C and A B = A D .

In other words, we must show that every parallelogram A B C D has A A = A C and A B = A D . This is what we did in our
Bogus Solution above, so we have already tackled this part.

Therefore, A B C D is a parallelogram if and only if A A = A C and A B = A D . n

Sometimes one part of 'if and only i f is clearly true, but the other half isn't so obvious. And sometimes the other half isn't even
true!

Problem 8.24 t V

(a) Prove that if A B C D is a square, then its diagonals bisect its angles.

(b) Is it true to say that A B C D is a square if and only if its diagonals bisect its angles?

Solution for Problem 8.24:

(a) We know that each diagonal of a square cuts the square into two isosceles right triangles. In other words, each diagonal
cuts the right angles at the vertices into two 45° angles. So, if A B C D is a square, its diagonals bisect its angles.

(b) We’ve already proved one direction of this statement. The other direction is

Prove that A B C D is a square if its diagonals bisect its angles.

Unfortunately, try as we may, we can’t prove this! For example, the diagonals of any rhombus
bisect the angles of the rhombus. We can show this quickly by noting that
A A E D = A C E D in rhombus A B C D at right. Hence, A A D E = A C D E and
diagonal B D bisects A A D C . Similarly, each of the four angles of rhombus A B C D is
bisected by a diagonal of A B C D . Therefore, if the diagonals of a quadrilateral bisect its
angles, we cannot deduce that the quadrilateral is a square, since the quadrilateral may be a
rhombus that is not a square.

Important: To prove a statement is true, you must have a proof that covers all possibilities. To
disprove a statement, all you have to do is find one case in which the statement is
Z false. It’s much easier to break something than to build it!

Let's try a couple more challenging ,if and only i f statements.

Problem 8.25 t V

Prove that the diagonals of a trapezoid are congruent if and only if the trapezoid is isosceles.

Solution for Problem 8.25: Our diagram shows trapezoid A B C D with A B || C D . We'll assume that
A C < 90°, because if A C > 90°, then we note A B < 90° and we can then use essentially the
same proof as for A C < 90°. (What happens if A C = 90°?) Since we have an if and only if
statement, we must prove two items. First, we must show:

The diagonals of a trapezoid are congruent if the trapezoid is isosceles.

We must show that A C = B D if A A D C = A B C D . Back in Problem 8.7, we showed that


A A D C = A B C D implies that A D = B C . Since we also have A A D C = A B C D and C D = C D , we have
A A D C = A B C D by SAS. Therefore, A C = B D .

Next, we must show:

The diagonals of a trapezoid are congruent only if the trapezoid is isosceles.

In other words, we must show that if the diagonals are equal, the trapezoid is isosceles. So, we must
start from A C = B D and show that A A D C = A B C D . We would like to show that
A A C D = A B D C , so we draw altitudes A X and B Y as shown. Since A C = B D and
A X = B Y (because A B and C D are parallel), we have A A X C = A B Y D by HL Congruence.
Hence, we have A A C D = A B D C . Since we also have A C = B D and C D = C D , we have
A A C D = A B D C by SAS Congruence. Therefore, A A D C = A B C D .

Therefore, the diagonals of a trapezoid are congruent if and only if the trapezoid is isosceles, z

You might be wondering, 'How in the world would we think to draw the altitudes we drew in the second part?’ We think to add
these because we have to somehow use the fact that A B || C D . We could also have used this fact by finding some equal
angles (this path is a little longer), but we know that drawing altitudes often simplifies trapezoid problems.

Concept: When stuck on a problem, think 'What information haven't I used yet?' Then, try to find
I some way to use that information.

Enough with quadrilaterals; let’s go back to triangles.

Problem 8.26 t V

Prove that the circumcenter of a triangle is the same point as the incenter if and only if the triangle is equilateral.

Solution for Problem 8.26: What's wrong with the following solution:

Bogus Solution: Let A B C be our triangle. We can easily take care of showing that the circumcenter
and incenter are the same point in an equilateral triangle since the angle bisectors of
y an equilateral triangle are the same lines as the perpendicular bisectors of the sides
of the triangle.

Going the other way is tougher. Let B E be the angle bisector


of A B . Since A B = B C , A A B E = A C B E , and
B E = B E , we have A E B C 9* A E B A . So, A E = E C
and A A E B = A C E B = 90°. Thus, angle bisector B E is
part of the perpendicular bisector of A C . Similarly, we can
show that all the angle bisectors of A A B C are the same as
the perpendicular bisectors, so the triangle must be equilateral.

This 'solution' has several missteps. First, we should be more explicit in the first part about showing why the angle bisectors and
perpendicular bisectors are the same lines in an equilateral triangle. Second, and far more serious, is that our second part of the
proof is logically flawed. We assume as a step in our 'proof that A B = B C : however, we can’t assume this because we don’t
know anything about the sides of A A B C in the second part yet - we are trying to prove that these are equal!

WARNING!! Be very careful that you do not assume what you are trying to prove as part of a proof!

Finally, our 'proof ends by stating that since the angle bisectors and the perpendicular bisectors are the same, the triangle must
be equilateral. This statement, while true, is not proved in our solution - it's what we are asked to prove!

Let's try again:

Let A A B C be our triangle, O be the circumcenter, and / be the incenter.

First, we show that if a triangle is equilateral, then / and O are the same. We draw angle bisector A D .
Since A A B C is equilateral, we have A C A D = A B A D by SAS. So, C D = D B and
A C D A = A B D A = 90° Therefore, A $ is the perpendicular bisector of B C . Similarly, each angle
bisector of A A B C is also a perpendicular bisector of one of the sides. Thus, the intersection of the
angle bisectors, the incenter, is the same point as the intersection of the perpendicular bisectors, the
circumcenter.

Next, we turn to the second part, proving that if the incenter and the circumcenter are the same
point, then the triangle must be equilateral. We let this common point be X . Since X is the
incenter, it is equidistant from all three sides. Since it is the circumcenter, it is equidistant from
all three vertices. These two sets of length equalities are shown in the diagram. We quickly see
that all six little right triangles are congruent by HL Congruence. Hence, we have

AV = V B = B T = TC = CU = UA,

so A B = B C = C A and the triangle is equilateral. (Note: There are plenty of other ways to
do this part; perhaps you can find another way!) □

Another phrase mathematicians will sometimes use to mean 'if and only i f is 'necessary and sufficient.' For example:

It is necessary and sufficient for a quadrilateral's diagonals to bisect each other in order for the quadrilateral to be a
parallelogram.

This is the same as saying:

The diagonals of a quadrilateral bisect each other if and only if the quadrilateral is a parallelogram.

Unsurprisingly, we even have a symbol for 'if and only if,

The diagonals of a quadrilateral bisect each other <=> the quadrilateral is a parallelogram.

Usually this symbol is only used in brief statements. For example: x 2 = 4 & x = ±2. Finally, sometimes the phrase 'if and only
i f is shortened to 'iff.'

Exercises

Prove or disprove each of the following statements.

8 .7 .1 : t V

E E C 1 1 is a rhombus if and only if its diagonals are perpendicular.

Type your solution, notes and/or work here. Show Solution

8 .7 .2 : t V

A X Y Z is equilateral if and only if X Y = Y Z and A X = 60c

Type your solution, notes and/or work here. Show Solution

8 .7 .3 : t V

A quadrilateral is a parallelogram if and only if it is a rhombus.

Type your solution, notes and/or work here. Show Solution

8 .7 .4 : t V

A rhombus is a square if and only if it is also a rectangle.

Type your solution, notes and/or work here. Show Solution

o • 7# • wc ••
o
t V

P Q R S is a parallelogram if and only if P Q = R S and S P = Q R .

Type your solution, notes and/or work here. Show Solution

8 .7 .6 : t V

Trapezoid A B C D with A B || C D is isosceles if and only if A A B D = A B A C .

Hint

Type your solution, notes and/or work here. Show Solution

o 7 • t V

W X Y Z is a rectangle if and only if W X = Y Z , A W X Y = A X Y Z , and W X || K Z .

Type your solution, notes and/or work here. Show Solution

8 .7 .8 : t V

Let M be the midpoint of A B . A A B C is isosceles with A C = B C if and only if C M bisects A A C B .

Hint

Type your solution, notes and/or work here. Show Solution

8 .7 .9 : t V

A B C D is a square if and only if its diagonals are perpendicular and congruent.

Type your solution, notes and/or work here. Show Solution


8.8★ Quadrilateral Problems
In this section we will tackle a few more challenging problems using the quadrilateral principles we have learned.

P roblem s
Problem 8.27 4 Jump to Solution

Diagonal B D of square A B C D is drawn. Square E F G H is then inscribed in A B D C with two


vertices on B D as shown. If A B = 6, find the area of E F G I I .

Problem 8.28 4 Jump to Solution

1 have a rectangular painting that has a length 10 inches longer than its width. 1frame it with a rectangular frame that is two
inches wide all the way around. Given that the area of the frame is 152 square inches, what is the area of the painting?

Problem 8.29 4 Jump to Solution

Find the area of trapezoid A B C D given that A B || C D . A B = 16, B C = 15, C D = 30, and A D = 13.

Problem 8.30 4 Jump to Solution

Isosceles trapezoid A B C D with A B || C D is inscribed in a circle with radius 10 such that the center of the circle is inside
A B C D . Isosceles trapezoid C D E F , with C D || E F , is inscribed in the same circle, but the center of the circle is not inside
C D E F . Given that A B = E F = 12 and C D = 16, find the areas of A B C D and C D E F .

Problem 8.31 4 Jump to Solution

Prove that the sum of the squares of the diagonals of a parallelogram equals the sum of the squares of the sides of the
parallelogram.

Problem 8.27 t V

Diagonal B I ) of square A B C D is drawn. Square E F G H is then inscribed in A B D C with two A


/A
U
7‫ ־‬D
vertices on B D as shown. If A B = 6, find the area of E F G I I . E/

H / F
D1
-------------------
G

Solution for Problem 8.27: We need the side length of E F G I I to get its area, so we let the side length of E F G I I be x. Since
A E B F is a 45-4590‫ ־‬triangle, we have B F = E F \f2 = x \P l. We also have B E = E F = x (and from A D U G ,
D H = x). Because B D || F G , we have Z F G C = 45°, so A F C G is also a 45-45-90 triangle. Therefore,

F C = F G / V 2 = x /V 2 = x \/ 2 /2 .

We know B C = (j, so we have B F 4- F C = B C = 6. Substitution gives

r- x \p l
x v 2 4-----— = 6.
z

3 \/2
Therefore, x = 6, so x = 2 \/2 . So, the area of E F G I I is E F 2 = x 2 = ( 2 \/2 ) 2 = 8. Alternatively, we could have

noted that B D = 3x = 6 \/2 , s o x = 2 \P l. □

I have a rectangular painting that has a length 10 inches longer than its width. I frame it with a rectangular frame that is two
inches wide all the way around. Given that the area of the frame is 152 square inches, what is the area of the painting?

Solution for Problem 8.28: We start by making a sketch of our painting with its frame. We let
14
the width of the painting without frame be x, so the length is x 4 1 0 ‫־‬. Since the frame is two
2
inches thick, we find that the painting with frame is x 4 1 4 ‫ ־‬inches long and x 4- 4 inches
wide. We can then find the area of the frame in terms of x by subtracting the small rectangle 2 A 10+‫׳‬
X x+4
(the picture) from the large one (picture plus frame). The area of the picture plus frame is *2 ”
(x 414 ‫()־‬:r 4- 4) and the area of the picture alone is x ( x 4 1 0 ‫)־‬, so we have:
:2

(a: 4- I4 ) ( x 4 4) ‫ ־‬x ( x 4-10) = 152.

Expanding the left side gives

x 2 4‫ ־‬IS x 4 5 6 ‫ — ־‬x 2 — lO x = 152,

so x = 12.

Now we can find the area of the painting without the frame: x ( x 4264 = (22)12 = (10 ‫ ־‬square inches. □

Concept: A picture is worth a thousand words — making a sketch can greatly help you
I understand a geometric problem.

Problem 8 .2 9 t V

Find the area of trapezoid A B C D given that A B | C D , A B = 16, B C = 15, C D = 30, and A D = 13.

Solution for Problem 8.29: We have the bases, so we just need the height. We draw altitudes from A
and B, thus forming a rectangle and two right triangles. (We like right triangles!) Since
X Y = A B = 16 and C D = 30, we know that D X 4- C Y = 14. At this point, there are a couple
approaches we could take to find the height.

Wishful thinking. We have the hypotenuses of A A X I ) and A B Y C , and we know they each have a
leg of length h. The 15 and 13 make us think of the Pythagorean triples 5-12-13 and 9-12-15. These
each have a leg of length 12, so perhaps our height is 12. This would make D X = 5 and and C Y = 9
, so D X 4- C Y = 14, as desired. Our wishful thinking has been fruitful! The height must be 12, so we can now find the area.

Algebra. Perhaps we don't see the Pythagorean triples, or maybe they don't work out (the answers won't always be integers!);
then, we have to do a little more work. We let D X = x, so C Y = 14 — x. Now we can use the Pythagorean Theorem on
A A D X and A B Y C t o build two equations in terms of h and x:

x 2 4‫ ־‬h 2 = 132
(14 — x )2 4‫ ־‬h2 = 152

We can subtract the first from the second to eliminate h 2:

(14 — x ) 2 — x 2 = 152 — 132.

Factoring both sides as a difference of squares gives

(14 - x - x )(1 4 - x 4‫ ־‬x ) = (15 - 13)(15 4 -13),

so (14 — 2 x ) (14) = 56, which gives x = 5. Now we can find h = 12 with either of our original equations.

Finally, we find our area: h ( A B -P C D ) / 2 = 276. z

Concept: Rectangles and right triangles are easier to work with than trapezoids. When stuck on
I a trapezoid problem, consider dropping altitudes.

Problem 8 .30 t V

Isosceles trapezoid A B C D with A B || C D is inscribed in a circle with radius 10 such that the center of the circle is inside
A B C D . Isosceles trapezoid C D E F , with C D || E F , is inscribed in the same circle, but the center of the circle is not inside
C D E F . Given that A B = E F = 12 and C D = 16, find the areas of A B C D and C D E F .

Solution for Problem 8.30: We’ll tackle A B C D first. In our diagram, we connect the center of the
circle, O, to the vertices of the trapezoid, since we somehow want to use the fact that the trapezoid
is inscribed in a circle. We have O A = O B = O C = O D = 10. We need a height, so we draw
the height through O, since this will build right triangles in which we already know the hypotenuses.
We let X and Y be the feet of the altitudes from O to A B and C D , respectively.

Since A O = B O and X O = X O , we have A A X O = A B X O by HL Congruence. Therefore,


X is the midpoint of A B . Similarly, Y is the midpoint of C D . Hence, A X = 6 and D Y = 8. We
can now use the Pythagorean Theorem on A X O A and A D O Y to determine that X O — 8 and
Y O = 6. Thus, the height is X O 4- Y O — 14, and our area is (X Y ) ( A B 4- C D ) / 2 = 196.

Now we find the area of C D E F . We could break the problem into right triangles as before, or we could be a little more clever
and apply what we learned about A A O B to A E O F . Since A O = EO, A B = E F , and B O = FO , we have
A A O B = A E O F . Therefore, the altitudes from O to A B and to E F are congruent. Letting Z be the foot of the altitude
from O to E F , we have O Z = O X = 8. Moreover, since C D || E F and O Z _L E F , we have O Z _L C D . Since Y is the
foot of the altitude from O, O Z passes through Y . So, the height of C D E F is Y Z = O Z — O Y = 2. Finally, the area of
C D E F is ( Y Z ) { C D 4- E F ) / 2 = 28. □

We’ll finish with a challenging proof.

Problem 8.31 t V

Prove that the sum of the squares of the diagonals of a parallelogram equals the sum of the squares of the sides of the
parallelogram.

Solution for Problem 8.3T: Let A B C D be our parallelogram. We wish to prove


B
A C 2 4‫ ־‬B D 2 = A B 2 4‫ ־‬B C 2 4- C D 2 4- D A 2.

Since A B = C D and A D = B C , we can simplify this to

A C 2 4- B D 2 = 2 ( A D 2 4- C D 2).

Seeing the sums of squares of sides, we reach for the Pythagorean Theorem. We have no right
triangles, so well have to build them. We draw altitudes from .4 and B, thus building right triangles with our diagonals as
hypotenuses. From right triangles A A P C and A B Q D , we have

AC2 = P C 2 -P A P 2
BD2 = DQ2 + BQ2

We can add these to get an expression for the sum of the squares of the diagonals:

A C 2 4- B D 2 = P C 2 4- A P 2 4- D Q '2 4- B Q 2.

That looks like what we want, but we‫׳‬re not quite there. We have to get A D and C D in the equation somehow. We start by
noting that D Q = C D -P C Q and P C = C D — D P . We also note that A P = B Q and A D = B C , so
A A D P = A B C Q by HL Congruence. Thus, D P = CQ. Now our equation becomes:

,4<72 4‫ ־‬B D 2
= PC2 + AP2 + DQ2 + BQ2
= ( C D - D P )2 + A P 2 + ( C D + C Q )2 + B Q 2
= C D 2 - 2(C D )(D P ) + D P 2 + A P 2 + C D 2
+ 2(C D )(C Q ) + C Q 2 + B Q 2
= C D 2 - 2(C D )(D P ) + D P 2 + A P 2 + C D 2
+ 2(C D )(D P ) + D P 2 + A P 2
= 2 ( C D 2) + 2 ( D P 2 + A P 2).

Since D P 2 + A P 2 = A D 2 from right triangle A D A P , we have the desired

A C 2 + B D 2 = 2 ( C D 2 + A D 2).

As we noted earlier, C D = A B and A D = B C , so we've shown that the sum of the squares of the sides of a parallelogram
equals the sum of the squares of its diagonals. □

Exercises

8 .8 .1 : t V

In rectangle A B C D , I I is the midpoint of B C , E lies on A D , and F lies on A B . In rectangle C E F G , I I lies on F G and


I I G = 3. Given Z D E C = 45°, what is the positive difference between the areas of these two rectangles?

Type your solution, notes and/or work here. Show Solution

8 .8 .2 : t V

Find the area of trapezoid A B C D shown at right. B 12 C

Type your solution, notes and/or work here. Solution

I
8 .8 .3 : Source: MATHCOUNTS t V

Quadrilateral A B C D is a trapezoid with A B C D . We know A B = 20 and C D = 12. What is the ratio of the area of
A A C B to the area of A B C D ?

Hint

Type your solution, notes and/or work here. Show Solution

8 .8 .4 : t V

The diagonals of E F G H are perpendicular. Prove that E F 2 4‫ ־‬G H 2 = F G 2 4- E H 2. (A quadrilateral with perpendicular
diagonals is sometimes referred to as orthodiagonal.)

Hint

Type your solution, notes and/or work here. Show Solution

8 .8 .5 ★ : t V

In A A B C , A B = 6, B C = 7, and ,4(7 = 8. Given that M is the midpoint of A B , find C M .

Hint

Hint

Hint

Type your solution, notes and/or work here. Show Solution


8.9 Summary

Definitions: A quadrilateral, such as A B C D at the right, has four segments


as sides, four vertices, and four angles. The segments connecting
opposite vertices are called the diagonals of a quadrilateral. ^ /

/1 v D

Important: The interior angles of any quadrilateral add to 360°.

Definitions: A trapezoid is a quadrilateral with two parallel sides. The


W r ^ —\X
segment connecting the midpoints of the non-parallel sides is
the median of the trapezoid, and the distance between the two
parallel sides is the height of the trapezoid.

Important: The median of a trapezoid is parallel to the bases of a trapezoid, and equal in
length to the average of the lengths of the bases.
Z
The area of a trapezoid equals the height of the trapezoid times the length of the
median of the trapezoid.

Important: In an isosceles trapezoid:

Z (a) The base angles come in two pairs of equal angles


as shown at right.

(b) The legs are equal.

(c) The diagonals are equal.

If any one of these items is true for a trapezoid with exactly one pair of parallel sides,
then all the others must be true for that trapezoid.

Definitions: A parallelogram is a quadrilateral in which both pairs of opposite sides are parallel.

Important: ■ The area of a parallelogram is the product of a side length (the base) and the
distance between that side and the opposite side of the parallelogram. This
Z distance between opposite sides is called a height of the parallelogram.

■ In parallelogram A B C D , the opposite sides are equal, the opposite angles are
equal, and the diagonals bisect each other.

A A

Conversely, A B C D is a parallelogram if any one of the following are true:

(a) A B = C D and A D = B C .

(b) / . A = Z C and Z B = Z D .

(c) Diagonals A C and B D bisect each other.

Therefore, proving one of these means the other two are true.

Definition: A quadrilateral is a rhombus if all of its sides are equal.

Important: ■ Every rhombus is a parallelogram. Therefore, everything that is true about


parallelograms is true about rhombi.
Z
■ The diagonals of a rhombus are perpendicular. The area of a rhombus is half the
product of its diagonals (and also equals its base times its height).

Definition: A quadrilateral in which all angles are equal is a rectangle.

Important: ■ All rectangles are parallelograms, so all that is true of parallelograms is true of
rectangles.
Z
■ Let two consecutive sides of a rectangle have lengths £ and w. The area of the
rectangle is £w, and the diagonals of the rectangle both have length \ / £ 2 4- w 2.

Definition: A quadrilateral in which all sides are equal and all angles are equal is a square.

Important: ■ Each square is a parallelogram, a rectangle, and a rhombus so all that is true of a
parallelogram, a rectangle, or a rhombus is true of a square.
Z
■ If the side length of a square is s and its diagonal is d, then d = s \ / 2 and the area
of the square is s2, or d2 / 2.

In proving various facts about quadrilaterals, we encountered the phrase ,if and only if.‘ Proving ,if and only i f statements requires
proving two different statements.

P roblem Solving S trateg ies

Concepts:
■ Triangles, triangles, triangles. Although this chapter was about quadrilaterals,
looking back you'll see that a great many of our solutions revolved around breaking
the problems into triangles on which we can use all our triangle tools.

■ Rectangles and right triangles are easier to work with than trapezoids. When stuck
on a trapezoid problem, consider dropping altitudes to build rectangles and right
triangles.

■ Breaking a desired angle, length, or area into parts often makes it easier to find.

■ When stuck on a problem, think ,What information haven’t I used yet?’ Then, try to
find some way to use that information.

■ Don't just stare at a diagram. Set sides you seek equal to variables and find other
lengths in terms of those variables. Label the diagram when you find these new
lengths.

■ A picture is worth a thousand words - making a sketch can greatly help you
understand a visual problem.

T h in g s To W atch Out For!

WARNING!!
Although every rhombus is a parallelogram, every parallelogram is not a rhombus.
O Therefore, if we prove a property of rhombi, this property is not necessarily true for
parallelograms. (The same is true of rectangles - rectangles are parallelograms,
but not all parallelograms are rectangles, etc.)

Be very careful that you do not assume what you are trying to prove as part of a
proof!
Review Problems

8 .3 2 : t v

Fill in each of the boxes with the answer "Yes", ‫״‬Sometimes", or "No".

Fact Parallelogram Rhombus Rectangle Square


Opposite sides have equal length
All sides have equal length
Opposite sides are parallel
Opposite angles are equal
All angles are equal
Diagonals bisect each other
Diagonals have equal length

Type your solution, notes and/or work here. Show Solution

8 .3 3 : t V

In quadrilateral A B C D , / . A = Z B = 128°, and Z I ) is 10° less than 5 times Z C . Find Z C and Z D .

Type your solution, notes and/or work here. Show Solution

8 .3 4 : t V

In isosceles trapezoid A B C D , Z A = a; and Z B = 2x — 45°. Find Z C and Z D in the following cases:

(a) A B || C D .

Type your solution, notes and/or work here. Show Solution

(b) A D || B C .

Type your solution, notes and/or work here. Show Solution

8 .3 5 : t V

In which of the following quadrilaterals does a diagonal always divide the quadrilateral into two regions of equal area:
Rhombus, Square, Rectangle, Trapezoid, Parallelogram?

Type your solution, notes and/or work here. Show Solution

If the numbers denoting the perimeter and area of a square are equal, what is the length of its diagonal?

Type your solution, notes and/or work here. Show Solution

If A , B , C , D are midpoints of the sides of square E F G H , what is the ratio of the area of triangle A B C to the area of
square E F G H ?

Type your solution, notes and/or work here. Show Solution

EThe bases of a trapezoid are 2x and Ax. The height is 2x. If the area is 48, what is x?

Type your solution, notes and/or work here. Show Solution

One of the angles of a rhombus is 120°. If the shorter diagonal is 2, what is the area?

Type your solution, notes and/or work here. Show Solution

What is the number of centimeters in the length of a longer side of a rectangle that has a perimeter of 64 centimeters and an
area of 192 square centimeters?

Type your solution, notes and/or work here. Show Solution

8 .4 1 : t V

Quadrilateral X V X Y Z is a parallelogram. Given that XVX = 2 x — 3, X Y = x A 7, and Y Z = 3 x — 8, find the perimeter


o iW X Y Z .

Type your solution, notes and/or work here. Show Solution

XVX Y Z is a rhombus with Z X = 90°. Prove that W X Y Z is a square.

Type your solution, notes and/or work here. Show Solution

One less well-known type of quadrilateral is the kite. The four sides of a kite can be split into two pairs
of consecutive equal sides. For example, the figure at right shows kite A B C D with A B = B C and
C D = D A . The diagonals of A B C D meet at E as shown. Solve the following problems about kite
ABCD. D

(a) Prove that Z A B D = Z C B D .

Type your solution, notes and/or work here. Show Solution

(b) Prove that A C _L B D .

Hint

Hint

Type your solution, notes and/or work here. Show Solution

Which of the following quadrilaterals is (are) a special case of a kite: parallelogram, trapezoid, rhombus, rectangle, square?
(More than one answer may be correct.)

Type your solution, notes and/or work here. Show Solution

8.45: Source: MATHCOUNTS t V

Each side of an equilateral triangle is 8 inches long. An altitude of this triangle is used as the side of a square. What is the
number of square inches in the area of the square?

Type your solution, notes and/or work here. Show Solution

Shown at right is rectangle E F G H . Given that Z G H F = 31° and E Y = H Y , solve the E


following problems:

(a) Find Z X E H .

Type your solution, notes and/or work here. Show Solution

(b) Find Z Y E F .

Type your solution, notes and/or work here. Show Solution

(c) ★ Prove that Ë Ÿ passes through G.

Hint

Hint

Type your solution, notes and/or work here. Show Solution

E F G H is a rhombus. Given that E F = E G = (),find the area of the rhombus.

Hint

Type your solution, notes and/or work here. Show Solution

EProve or disprove this statement: A B C D is a square if and only if its sides are equal in length and A C = B D

Type your solution, notes and/or work here. Show Solution

The angles of quadrilateral W X Y Z are such that ZXV > Z X > Z Y > Z Z . The angles are in an arithmetic progression,
meaning that

ZW - Z X = Z X - ZY = ZY - ZZ.

(a) IfZ X V is four times the measure o i Z Z , what is Z X V l

Type your solution, notes and/or work here. Show Solution

(b) If Z W is two times Z Z , what is ZXV?

Type your solution, notes and/or work here. Show Solution

8 .5 0 : t V

In quadrilateral A B C D , let E, F, G, and / / be midpoints of A B , B C , C D , and D A , respectively. Prove that E F G H is a


parallelogram.

Type your solution, notes and/or work here. Show Solution

8 .5 1 : t V

A square and an equilateral triangle have the same perimeter. What is the ratio of the area of the square to the area of the
equilateral triangle?

Type your solution, notes and/or work here. Show Solution

8 .5 2 : Source: ARML t V

Trapezoid A B C D is divided into four congruent trapezoids as shown. Given A B = 4 and


C D = 8, find the sum of the lengths of all line segments in the figure.

Type your solution, notes and/or work here. Show Solution

8 .5 3 : Source: AMC 8 t V

What is the ratio of the area of the shaded square to the area of the large square in the figure at right? (The
figure is drawn to scale.)

Type your solution, notes and/or work here. Show Solution

8 .5 4 : Source: AMC 10 t V

A street has parallel curbs 40 feet apart. A crosswalk bounded by two parallel stripes crosses the street at an angle. The
length of the curb between the stripes is 15 feet and each stripe is 50 feet long. Find the distance between the stripes.

Type your solution, notes and/or work here. Show Solution


Challenge Problems

In the diagram at right, A X = X Y = 6, Z A X Y = 70°, Z W = 125°, Z X Z A = 21°, w л‫׳‬


and Z X Z W = 34°.

(a) Find Z Z X A .

Type your solution, notes and/or work here. Show Solution

(b) Find W Z .

Type your solution, notes and/or work here. Show Solution

(c) Prove that W Y = X Z .

Hint

Hint

Type your solution, notes and/or work here. Show Solution

8.56: Source: MATHCOUNTS t V

In rectangle A B C D , A B = 16 and A D = 5. F is on A B and G on C D such that F G , B D , and A C are concurrent at


point O. Find [F O B ] 4‫[ ־‬G O C \

Hint

Type your solution, notes and/or work here. Show Solution

Each angle of rectangle A B C D is trisected by a pair of segments. The angle trisectors meet at P,
Q, R, and S, as shown in the diagram at right. Prove that P Q R S is a rhombus.

Hint
D

Type your solution, notes and/or work here. Show Solution

8.58: Source: MATHCOUNTS t V

In rectangle A B C D , M is the midpoint of B C . Points P and Q lie on A B and D C , respectively, such that
P B = 4 /3 • B C and Z P M Q is a right angle. What is the ratio P M : M Q ?

Hint

Type your solution, notes and/or work here. Show Solution

8.59: Source: ARML t V

In the diagram, A B C D is a parallelogram and E is on the extension of B C past C. A E and C D


meet at F . Given [A D F ] = 64 and [C E F ] = 4, determine the following:

(a) C F /D F .

Type your solution, notes and/or work here. Show Solution

(b) [B F C \

Hint

Type your solution, notes and/or work here. Show Solution

(c) [A B C D ]

Hint

Type your solution, notes and/or work here. Show Solution

8 .6 0 : t V

See Problem 8.43 for the definition of a kite.

(a) Find the area of a kite that has diagonals of length 10 and 12.

Hint

Hint

Type your solution, notes and/or work here. Show Solution

(b) ★ Find the area of kite A B C D if A C = 6 \/2 , A B = 4 \/3 , and B C = 2 \/3 .

Hint

Hint

Hint

Type your solution, notes and/or work here. Show Solution

8.61 Source: HMMT t V

Consider a square of side length 1. We draw four lines that each connect a midpoint of a side with a
corner not on that side, such that each midpoint and each corner is touched by only one of these lines as
shown at right. Find the area of the shaded region.

Hint
ж

Type your solution, notes and/or work here. Show Solution

Given M P || N O , P O | M N , M A = A P , and N C = C O as in the diagram at right, prove M A P


that [A B C D ] = [ M N O P ] / 4.

Hint

Type your solution, notes and/or work here. Show Solution

The diagonals of W X Y Z meet at A. Prove that the quadrilateral formed by connecting the incenters of A W X A , A X Y A,
A Y Z A , and A Z W A has perpendicular diagonals.

Hint

Hint

Type your solution, notes and/or work here. Show Solution

E F G H is a rectangle with E F = 12 and area 192. E G J I is a parallelogram such that / / is on . / / . What are the possible
values for the area of E G J I ?

Hint

Type your solution, notes and/or work here. Show Solution

8 .6 5 : Source: AMC 10 t V

In rectangle A B C D , we have A B = 8, B C = 9, I I is on B C with B I I = 6, E is on A D with


D E = 4, line intersects line k / 1 at G, and F is on line .4/3 with G F _L A F . Find the length of
GF.

Hint

Hint

Type your solution, notes and/or work here. Show Solution

8 . 66 : Source: HMMT t V

In trapezoid A B C D , A D || B C . Z A = Z D = 45°, while Z B = Z C = 135°. If A B = 6 and the area of A B C D is 30,


find B C .

Hint

Type your solution, notes and/or work here. Show Solution

8 .6 7 : Source: Mandelbrot t V

A circle is drawn through the vertices of square A B C D , and point X is on minor arc A B . Given that [ X A B ] = 1 and
[ X C D ] = 993, find [ X A D ] + [ X B C ]

Hint

Type your solution, notes and/or work here. Show Solution

8 .68 ★ : Source: MATHCOUNTS t V

The lengths indicated on the rectangle shown are in centimeters. What is the number of
square centimeters in the area of the shaded region?

Hint

Type your solution, notes and/or work here. Show Solution

8 .6 9 ★ : Source: HMMT t V

P is inside rectangle A B C D . P A = 2, P B = 3, and P C = 10. Find P D .

Hint

Hint

Type your solution, notes and/or work here. Show Solution

8 .7 0 ★ : Source: HMMT t V

E In trapezoid A B C D shown, A D II B C , A B = 6, B C = 7, C D = 8, and A D = 17.


Л
' 4
/ \ The sides .4 B and C D are extended to meet at E . Prove that Z E = 90°.

Hint

Type your solution, notes and/or work here. Show Solution

Extra! One of the oldest known puzzles is the Chinese ch'i ch'iao t‫׳‬u, which means 'ingenious seven-piece plan.'
In English, these are known as tangrams.
V

Tangrams involve rearranging the seven pieces that together make the square shown above into other
given shapes. Some shapes that can be formed by these pieces are shown below. See if you can figure
out how!

Perhaps you'll notice a couple things tangrams have in common with math problems. First, with
experience, they get a lot easier. Second, often the simplest looking shapes end up being the hardest
ones to solve! Source: Martin Gardner's Mathematical Puzzles \& Diversions
Construction of a Regular Pentagon

Bees.. .by virtue o f a certain geometrical forethought.. .know that the hexagon is greater than the square and the triangle, and will hold more honey for the same
expenditure of material.

- Pappus

CHAPTER
I
I Polygons

9.1 Introduction to Polygons


We’ve tackled three sides, and we’ve handled four sides. Why stop there? Figure 9.1 shows several different polygons, which are
closed planar figures with line segments as boundaries.

Figure 9.1: Some Polygons

All of the names of the parts of a general polygon are the same as in the polygons we've learned about so far (triangles and
quadrilaterals). The segments that form the boundary are sides, which meet at vertices to form the interior angles. If we connect
two vertices that are not adjacent on the polygon, we form a diagonal, such as diagonal A C in Figure 9.2.

Figure 9.2: Polygon Diagonal

Polygons are classified by the number of sides they have. Here are some names given to different polygons.

Number of Sides Polygon Name


3 triangle
4 quadrilateral
5 pentagon
6 hexagon
7 heptagon
8 octagon
9 nonagon
10 decagon
12 dodecagon

A polygon in which all the sides are equal and all the angles are equal is called a regular polygon. We've already seen two
examples of regular polygons: equilateral triangles and squares. Figure 9.3 shows a few more regular polygons.

Some Regular Polygons

As with quadrilaterals, we’ll be focusing on convex polygons in this chapter unless otherwise stated, which means that the
interior angles of the polygons are all less than 180°.
9.2 Angles in a Polygon
In this section, we explore not only the interior angles of a polygon, but also the exterior angles. Exterior angles are formed by
extending the sides of the polygon. For example, Z B A X in the figure below is an exterior angle of A B O D E .

P roblem s
Problem 9.1 4 Jump to Solution

Find the sum of the measures of the interior angles of pentagon A B O D E shown without measuring
D
the angles. (Flint: See how we did Problem 8.1.)

Problem 9.2 4 Jump to Solution

In this problem we find a formula for the sum of the interior angles of a convex polygon with n sides. (By convex, we mean that
all interior angles have measure less than 180°.)

(a) Draw all the diagonals from a single vertex of a polygon with n sides. In terms of n, how many triangles are formed?

(b) Use your dissection from part (a) to find a formula for the sum of the measures of the interior angles of a polygon.

Problem 9.5 4 Jump to Solution

A B C D E F G H I J K L M N O is a regular 15-gon.

(a) Find Z A C B .

(b) Find Z A C D .

Flint

(c) Find Z A D E .

Flint

We found the sum of the interior angles of a quadrilateral by using a diagonal to cut the quadrilateral into triangles. This strategy
works for all polygons.

Solution for Problem 9.7: We don't know anything about pentagons, but we know plenty about triangles.
D
Therefore, we use diagonals to cut the pentagon into triangles. The sum of the angles in the pentagon
is the same as the sum of the angles in the three triangles A A B O , A A C D , and A A D E . Each
triangle contributes 180° to the total, so the sum of the interior angles in a pentagon is 3(180°) = 540°
E
.□

Problem 9.2 t V

Find a formula for the sum of the interior angles of a convex polygon with n sides.

Solution for Problem 9.2: As we did with the pentagon, we can break any polygon into triangles by drawing
all the diagonals from one of the vertices. The sum of the angles in these triangles equals the sum of the
angles in the polygon. Therefore, all we have to do is figure out how many triangles we form when we draw
the diagonals. We can use our pentagon example for guidance. Let A be the common vertex of all our
triangles. Each side of the polygon, except the two sides connected to A, is the side opposite ;4 in exactly
one of our triangles. So, there must be n — 2 triangles formed. Since there are 180 degrees in each triangle
the sum of the angles in an n -gon is 180(n — 2) degrees. (It’s a little more complicated to prove, but this
result also holds for concave polygons.) □

Important: The sum of the interior angles in an n-sided polygon is 180(n — 2) degrees,
y Therefore, the measure of each of the n interior angles in a regular n -gon is
^180 ‫(־־‬n — 2 ) / n degrees.

Now we turn our attention from the interior angles to the exterior angles.

Solution for Problem 9.3: Each of the exterior angles is supplementary to one of the interior angles, so we know that

Sum of interior angles -F Sum of exterior angles = 10(180°)


= 1800°,

since together, the interior and exterior angles make up 10 straight angles. Since the sum of the interior angles of a decagon is
(180°)(10 — 2) = 1440°, we have

Sum of exterior angles = 1800° — 1440° = 360°.

In the Exercises at the end of the section, you'll be asked to work through this problem for a general ??.‫־‬gon. You'll thereby prove:

Important: The sum of the exterior angles in a convex polygon with n sides is 360°. Therefore,
7 ‫־‬ the measure of each exterior angle in a regular ??.-gon is 360° /n .

Let's try using our new knowledge about the angles of polygons on a couple problems.

Problem 9.4 t V

Find the number of sides in a regular polygon in which each interior angle measures 172°.

Solution for Problem 9.4: We let the polygon have n sides. There are two general approaches we can take to this problem:

Focus on the interior angles. Since the measure of one angle is 172°, and the polygon is regular, the sum of all angles is 172??.
degrees. Since we know the sum is also given by 180(r?. — 2) degrees, we can solve the equation 172n = 180(n — 2) to find
8n = 360, so n = 45.

Focus on the exterior angles. Since the measure of each interior angle is 172°, each exterior angle is 180° — 172° = 8°. The
sum of the exterior angles is 360°, so there must be 3 6 0 /8 = 45 of them. □

Concept: Sometimes thinking about the exterior angles of a polygon offers a simpler approach
I than thinking about the interior angles!

Problem 9.5 t V

A B C D E F G H I J K L M N O is a regular 15‫־‬gon. Find Z A C B , Z A C D , and Z A D E .

Solution for Problem 9.5: We start with a diagram, then we find whatever angles we can
determine. Since the polygon is a regular 15‫־‬gon, each exterior angle is 360/15 = 24 degrees.
Therefore, each interior angle is 180° — 24° = 156°. Since A A B C is isosceles,
Z B A C = Z A C B . The sum of the angles of A A B C gives us

Z A B C 4‫ ־‬Z B A C + Z A C B = 180°,

so

156° + Z A C B + Z A C B = 180°.

Therefore, 2 Z A C B = 24°, so Z A C B = 12°. We also have

Z A C D = Z B C D - Z A C B = 156° - 12° = 144°.

Turning to Z A D E , we try the same strategy: find Z A D C , then subtract it from Z C D E . We note that A B C D is an isosceles
trapezoid in which B C || A D (make sure you see why), so Z A D C = 180° — Z B C D = 24°. So,
Z A D E = 156° - 24° = 132° □

Exercises

9 .2 .1 : » V

Complete the table below for regular polygons.

Name # of Sides Sum of Interior Zs Int. Z Measure Ext. Z Measure


orH
00
O

pentagon 5 540° 72°


hexagon 6
heptagon 7
octagon 8
nonagon 9
decagon 10
dodecagon 12
pentadecagon 15
icosagon 20
triacontagon 30

Type your solution, notes and/or work here. Show Solution

9 .2 .2 : t V

Does the formula for the sum of the interior angles in an ??-gon still work if the polygon is concave,
as in the figure shown at right?

Type your solution, notes and/or work here. Show Solution

9 .2 .3 : t V

Each interior angle of a certain regular polygon has measure 160° Flow many sides does the polygon have?

Type your solution, notes and/or work here. Show Solution

Source: MATHCOUNTS t V

The measures of the angles of a pentagon are in the ratio of 3 : 3 : 3 : 4 : 5. What is the number of degrees in the measure of
the largest angle?

Flint

Type your solution, notes and/or work here. Show Solution

9 .2 .5 : t V

The sum of the interior angles of a polygon is three times the sum of the exterior angles. Flow many sides does the polygon
have?

Type your solution, notes and/or work here. Show Solution


9.3 Polygon Area

P roblem s
Problem 9.6 4 Jump to Solution

Find the area of pentagon A B O D E given the side lengths and right angles shown.
c
Hint

Problem 9.7 4 Jump to Solution

In this problem we find the area of a regular hexagon with side length 8.

(a) Draw all three long diagonals of the hexagon; what kind of triangles do you form?

(b) Find the area of the hexagon by finding the areas of the triangles from (a).

Problem 9.8 4 Jump to Solution

In this problem we will find the area of regular octagon A B C D E F G H with side length 4.

(a) Extend sides A B , C D , E E , and G I I to meet at points W , X , Y , Z. What type of quadrilateral is W X Y Z ?

(b) Find the area of W X Y Z .

(c) Use [ W X Y Z \ to find the area of A B C D E F G H .

Problem 9.9 4 Jump to Solution

The center of a regular polygon is the point inside the regular polygon that is equidistant from all of the vertices of the polygon.
Let the distance from the center of a regular ??‫־‬gon to one of its sides be k and the length of each side of the n-gon be s. Break
the polygon into triangles by connecting the center of the polygon to each vertex. Find a formula for the area of the n -gon in
terms of k, n, and 5 .

We've seen all sorts of problems involving triangle and quadrilateral areas. You won't be at all surprised that we rely heavily on
triangles and quadrilaterals when we find the areas of polygons with more sides.

Solution for Problem 9.6: We start by drawing B E to separate right triangle A A B E from quadrilateral
C
B C D E . We have [ A B E ] = ( v 3 ‫() ׳‬V C )/2 = 3 ^ 2 / 2 , so all we need is [B C D E ]. From the
Pythagorean Theorem, we have B E = y /A B 2 + A E 2 = 3, so B C D E is a rhombus. Since one of
its angles is a right angle, it must also be a square. Therefore, [B C D E ] = 9 and

[A B O D E ] = [B C D E ] + [A B E ] = 9 + 3%/2/2.

Concept: Break complicated areas into more manageable pieces.

Let's find the areas of a couple regular polygons.

Problem 9.7 t V

Find the area of a regular hexagon with side length 8.

Solution for Problem 9.7: We start by breaking the hexagon into pieces we can handle. Since the angles of a
regular hexagon are each 120°, the long diagonals split each angle into two 60° angles. Therefore, each of
the little six triangles formed by drawing all three long diagonals is equilateral. Thus, the area of the
hexagon is 6 times the area of an equilateral triangle with side length 8. As we saw in Problem 6.11, the
area of such a triangle is 82\/ 3 / 4 = l6 \ / 3 . So, the area of our regular hexagon is 6 ( l6 \ / 3 ) = 9 6 \/3 . □

Concept: Many problems involving regular hexagons can be tackled by dissecting the hexagons
I into equilateral triangles.

Problem 9.8 t V

Find the area of regular octagon A B C D E F G H with side length 4.

Solution for Problem 9.8: We can find the area with a dissection (try drawing A F , B E , C H , and D G ),
X B A W
but we can find a more elegant approach. We know each angle of a regular octagon is 135°. Since these
angles are supplements of 45° angles, we start thinking about 4 5 9 0 ‫־‬45‫ ־‬triangles. This makes us think
about squares, too. We extend the sides of the octagon, and we form square W X Y Z . Our octagon is
just W X Y Z with the four corners (the four 4 5 9 0 ‫־‬45‫ ־‬triangles) lopped off.

We can handle squares and 4 5 9 0 ‫־‬45‫ ־‬triangles. Since A H = 4, we have W H = 4 /\/2 = 2\/~2.
Similarly, each of the legs of the little right triangles has length 2 \/2 . Therefore, the area of each little
triangle is ( 2 \/2 ) ( 2 \/2 ) /2 = 4. Since

[W X Y Z ] = W Z 2
= ( W H + H G 4‫ ־‬G Z f

= (4 + 4 v/ 2)2
= 48 4 ‫ ־‬32 ^ 2 ,

the area of our octagon is

[A B C D E F G H ] = [ W X Y Z ] - 4 [W A H ] = 32 + 3 2 \/2 .

Sidenote: The figure below shows how an equilateral triangle can be dissected into four pieces
that can be reassembled to form a square.
n

Similarly, a regular dodecagon can be dissected into six pieces that can be
reassembled to form a square.

Can you find more regular polygons that can be dissected to form other regular
polygons?

We now have another area-finding tool:

Concept: Sometimes we can view our desired area as the 'leftover‫ ׳‬portion from having simple
I pieces taken away from a simple starting figure.

Now we turn to the general problem of finding the areas of regular polygons.

Problem 9.9 t V

The center of a regular polygon is the point inside the regular polygon that is equidistant from all of the vertices of the polygon.
Let the distance from the center of a regular ??-gon to one of its sides be k and the length of each side of the ??‫־‬gon be s. Find a
formula for the area of the ??‫־‬gon in terms of k, n, and s.

Solution for Problem 9.9: Let O be the center of the polygon. (You will be asked to prove that such
A P Z
a point exists as a Challenge Problem.) We can cut the regular polygon into n triangles by
connecting the center to every vertex. One such triangle is A O A Z , shown in the diagram. The
altitude to side A Z of this triangle is the distance from O to a side of the polygon, given in the
problem as k. Since A Z is also given as s, we have [A Z O ] = ( A Z ) ( O P ) / 2 = ks/2 . Finally,
the polygon consists of ??of these triangles, so the area of the whole polygon is n k s /2 . □

The distance from the center to the sides of a regular polygon is sometimes given the obscure name of the apothem. Noting that
the perimeter of the polygon is ns, we can write our area formula as:

Important: The area of a regular polygon is half its perimeter times the distance from the center
of the polygon to a side.
Z
As we have seen in earlier problems, this isn't always the fastest way to find the area. (Try it on the dodecagon above if you want
to torment yourself.)

Sidenote: One particularly fascinating dissection is that of a


regular dodecagon shown at right. See if you can
n figure out how to use the dissection to prove that the
area of the dodecagon is 3 times the square of the
radius of the dodecagon's circumcircle, which is
shown in bold in the diagram. This dissection of a
regular dodecagon is used as the logo for the Art of
Problem Solving Foundation, which supports problem
solving activities for eager middle and high school
students. You can find more information on the
Foundation (and a graphical proof of the relationship
described above) at www.artofproblemsolving.org.

Exercises

9 .3 .1 : t V

A B C D E F is a regular hexagon with side length 9.

(a) Find A D .

Type your solution, notes and/or work here. Show Solution

(b) Find [A B C D E F ].

Type your solution, notes and/or work here. Show Solution

(c) ★ Find A C .

Type your solution, notes and/or work here. Show Solution

Find a formula for the area of a regular hexagon with side length s.

Type your solution, notes and/or work here. Show Solution

9 .3 .3 : t V

What's wrong with pentagon A B O D E shown at right? c

Type your solution, notes and/or work here. Show Solution

A B C D E F G H is a regular octagon with side length 8. Diagonals A E and C G meet at X . Point M is the midpoint of A B .

(a) Find the area of A B C D E F G H .

Type your solution, notes and/or work here. Show Solution

(b) Find X M , the apothem of the octagon.

Type your solution, notes and/or work here. Show Solution

(c) F in d X C .

Hint

Type your solution, notes and/or work here. Show Solution

Q Q E•
^ • w • w •
Source: MATHCOUNTS t V

Points A, B, C, D, E, and F are the vertices of a regular hexagon and also trisect the sides of the large
equilateral triangles shown. Given that the area of A B C D E F is 24, what is the total area of the shaded
regions?

Type your solution, notes and/or work here. Show Solution

9 .3 .6 : t V

We solved Problem 9.8 by extending the sides of a regular octagon to form a square. We didn't, however, prove that we form a
square when we connect the points where these extensions meet. Fix this oversight by providing the proof.

Hint

Type your solution, notes and/or work here. Show Solution

9 .3 .7 ★ : t V

In this section we assumed that the long diagonals of a regular hexagon are concurrent. In this
problem we fix this oversight by proving that these diagonals are concurrent.

(a) Let the hexagon be A B C D E F and let point O be the intersection of the bisectors of Z A and
Z B . Prove that A A O B is equilateral.

Type your solution, notes and/or work here. Show Solution

(b) Draw O C . Prove that A B O C is equilateral.

Type your solution, notes and/or work here. Show Solution

(c) Prove that A C O D is equilateral and that AC3 goes through D.

Type your solution, notes and/or work here. Show Solution

(d) Prove that the long diagonals of A B C D E F all meet at the same point.

Type your solution, notes and/or work here. Show Solution

Sidenote: Back in this section here we challenged you to find pairs of regular polygons such
that the first can be dissected and rearranged to form the other. If you didn't find any,
n keep looking! Not only are there more than the pair we showed you, but in fact any
two polygons with the same area can be dissected into the same set of pieces.
Therefore, if we have two regular polygons with the same area, we can find some way
to dissect one and rearrange the pieces to form the second. Of course, some of these
dissections are very hard to find and may involve many pieces!

Many of these dissections are shown at Wolfram Research’s MathWorld page. You
can see this page at h ttp ://m a th w o rld .w o lfra m .co m /D isse ctio n .h tm l.
9.4 Polygon Problems

P roblem s
Problem 9.10 4 Jump to Solution

Diagonals B D and C E of regular pentagon A B C D E meet at Q. Find / B Q E .


A

E<C 8

Problem 9.11 4 Jump to Solution

In this problem we will find a formula for the number of diagonals in a polygon with n sides.

(a) How many diagonals does a square have?

(b) How many diagonals does a pentagon have?

(c) How many diagonals does a hexagon have?

(d) Let A be a vertex of a polygon with n sides. How many diagonals of the polygon have A as a vertex?

(e) Use your answer to the previous part to find a formula for the number of diagonals in a polygon with n sides. Be sure to
compare your result to your answers in the first 3 parts to check your formula.

Problem 9.12 4 Jump to Solution

A B C D E F G H is a regular octagon. Find the ratio A E / B H .

Hint

Problem 9.13 4 Jump to Solution

In the star diagram, find the sum / A + / C + / E + / G + / 1 . (You cannot assume the
pentagon in the middle of the star is regular!)

,<t x
Problem 9.14 4 Jump to Solution

A B C D E F G H is a regular octagon with side length 12. In this problem we will find the area of A B D G .

(a) What kind of quadrilateral is A B D G ?

(b) Consider E G D E . Draw altitudes from F and E to G D to find G D . (And see if you can find G D in some other clever
way!)

(c) Use your work from the last part as a guide to find the distance from B to G D .

(d) Find [A B D G ].

Solution for Problem 9.10: We start by finding the angles we can find. Each interior angle of a regular pentagon is 108°. Since
A E D O is isosceles,

/ D C E = / . D E C = (180° - 1 0 8 °)/2 = 36°.

Similarly, we have / C D B = 36°. Therefore,

/ C Q D = 180° - / Q D C - / Q C D = 108°,

so we have / B Q E = / C Q D = 108°. □

Problem 9.11 t V

Derive a formula for the number of diagonals in a polygon with n sides.

Solution for Problem 9.11: We consider a single vertex, A A is connected to 2 other vertices by sides, leaving n — 3 vertices to
which A can be connected by diagonals. Since there are n vertices and each can be connected to n — 3 others by diagonals,
there appear to be n (n — 3) diagonals. However, this counts each diagonal twice, once for each endpoint. Therefore, we must
divide by two to count each diagonal only once. This gives us n (n — 3 )/2 diagonals in an n-gon. □

Problem 9.12 t V

A B C D E F G H is a regular octagon. Find the ratio A E / B H .

Solution for Problem 9.12: It's not immediately obvious how to compare B H to A E , but there are several
В A
other diagonals with the same length as B I I . Considering these, we see that four of them form square
A G E C . Since A E is a diagonal of this square, we have A E / B H = A E / A G = \/2 . □

Solution for Problem 9.13: All we have to work with is what we know about the sums of the
angles in pentagons and in triangles. We label the angles in the pentagon as shown. We can find
the non-star angles of the outer triangles in terms of these (for example, / I J H = 180° — a).
However, if we use a little ingenuity, we don’t even need the exterior angles of B D F H .J .
/ Consider the angles in each triangle like A A H E. These must sum to 180°, so we have:

/ A 4‫ ־‬e -F / E = 180 °

/C + a + /G = 180 °

/ E + b+ / I = 180 °

/G + c + Z A = 180 °

/ iA d A /C = 180°

We’re looking fo r/ A 4‫ ־‬/ C 4‫ ־‬/ E 4‫ ־‬/ G 41 / ‫־‬, and we know that

a + b + c + d + e = (180)(5 - 2) = 540°,

since these angles together make up the central pentagon. Therefore, our triangle equations beg to be added together, giving:

2( / A A / C A / E A / G A / I ) A ( a A b A c A d A e ) = 5(180°).

Therefore,

2( / A A / C A / E A / G A / 1 ) A 540° = 900°,

so

/А A /C a / E a /G a / I = 180c

Problem 9.14 t V

A B C D E F G H is a regular octagon with side length 12. Find the area of A B D G .

Solution for Problem 9.14: Our A B D G is a trapezoid (you’ll prove this explicitly in the Exercises). We'd
like to dissect the octagon into pieces we can handle, and we know altitudes of trapezoids can often be
£■ useful, so we draw diagonals A F and B E . These are each perpendicular to G D and F E (you'll prove
this fact in the Exercises, too).

^ Since G D || F E , we have / F G D = 180° — / G F E = 45°, so A G X F is a 4 5 9 0 ‫־‬45‫ ־‬triangle.


Therefore, G X = G F / \ / 2 = 6 \/2 . Similarly, Y D = 6 y/2. XYEF is a rectangle, so
X Y = F E = 12. Therefore, base G D of our trapezoid has length 1 2 + 12\/2. Before we go
through tricky computations to get A X , we can note that by symmetry we have A F = G D (alternatively, we can use HL
Congruence to show A A X G = A D X F), so

A X = A F - X F = D G - G X = 12 + 6 \/2 .

Therefore, our area is:

( A B A D G )(A X )
[ABDG] =
2
(24 + 12>/2)(12 + 6>/2)
2
7 2 (2 + V 2 )(2 + V 2 )

= 36(6 + 4v144 + 216 = (2‫ ׳‬V 2.

Concept: Most polygon problems are really quadrilateral or triangle problems. When stuck with
a polygon, try dissecting it into smaller pieces.
I

In our last solution, we used symmetry to note that A F = D G . By symmetry, we essentially mean that the general properties of
all the vertices are the same. For example, the distance from one vertex to a vertex three vertices away on the regular polygon
(such as A F or D G ) is the same no matter which initial vertex we are talking about. However, we couldn’t say that ,A C = B E
by symmetry‫ ׳‬in the last problem, because A C is the distance between two vertices with one vertex between them, while B E is
the distance between two vertices with two vertices {C and D) between them.

Concept: Symmetry can be a very useful tool in problems involving regular polygons!

In the Exercises that follow, you'll be asked to solve several problems that could be solved using symmetry. In addition to
solutions involving symmetry, try to find explicit solutions using tools such as congruent and similar triangles.

Exercises

9 .4 .1 : t V

A B C D E at right is a regular pentagon. Show that A E Q B is a rhombus.

Type your solution, notes and/or work here. Show Solution

U V W X Y Z is a regular hexagon with center (). U O W A B C is also a regular hexagon with A B = 12.

(a) Find [U O W A B C ].

Type your solution, notes and/or work here. Show Solution

(b) Find [U V W O \

Type your solution, notes and/or work here. Show Solution

All of the interior angles of octagon E F G H IJ K L are 135°, E F = G H = I .J = K L = 2 and


F G = H I = J K = L E = 6.

(a) Find the perimeter of the octagon.

Type your solution, notes and/or work here. Show Solution

(b) ★ Find the area of the octagon.

Hint

Type your solution, notes and/or work here. Show Solution

(c) ★ Find E l .

Hint

Type your solution, notes and/or work here. Show Solution

Source: ARML t V

Let A B C D E F be a regular hexagon. Find the ratio of the area of A B C E to the area of the hexagon.

Hint

Type your solution, notes and/or work here. Show Solution

A B C D E F G H is a regular octagon. Prove that A C E G is a square.

Type your solution, notes and/or work here. Show Solution

Explicitly prove that A B D G of Problem 9.14 is a trapezoid.

Type your solution, notes and/or work here. Show Solution

Explicitly prove that / F X G is a right angle in Problem 9.14.

Type your solution, notes and/or work here. Show Solution

9 .4 .8 : Source: AMC 12 t V

The midpoints of the sides of a regular hexagon A B C D E F are joined in order to form a smaller regular hexagon. What
fraction of the area of A B C D E F is enclosed by the smaller hexagon?

Type your solution, notes and/or work here. Show Solution


9.5 Construction: Regular Polygons

Problems

Problem 9.16 4 Jump to Solution

Construct a regular hexagon.

Problem 9.17 4 Jump to Solution

Construct a regular octagon.

We already know how to construct a regular triangle, so well move on to a regular quadrilateral.

Problem 9.15 t V

Construct a square.

Solution for Problem 9.15: Solution 1: A natural solution is to start with one side, A B , then construct
the others. The other two vertices must be on lines through A and B that are perpendicular to A B ,
so we construct lines m and n perpendicular to A B as shown. We can then find C on m such that
A B = B C by constructing a circle with center B and radius A B . Where this circle meets m gives
us C. Similarly, we construct a circle with center A and radius A B to find D on n such that
A D = AB.

We'll leave the proof that this construction does indeed result in A B C D being a square for an
Exercise.

Solution 2: We can find an even slicker construction by thinking a little more about squares. For
example, we know that the diagonals of a square are perpendicular and they meet at a point that is
equidistant from all four vertices. In other words, the intersection of the diagonals of a square is the
center of the circumcircle of the square. We can use this observation to start with a circle to make our
square. We draw a circle with center O. We then draw two perpendicular lines through O and label the
points where these lines hit the circle A, B, C, and D. Since the four triangles that meet at O are
congruent 45-4590‫ ־‬triangles, we have A B = B C = C D = D A . Moreover, each angle of A B C D
equals 45° + 45° = 90°. Therefore, A B C D is a square. □

Let's try using this "circumcircle of a regular polygon" idea on another polygon.

Problem 9.16 t V

Construct a regular hexagon.

Solution for Problem 9.16: Solution 1: We already know how to make an equilateral triangle, and a
A В
regular hexagon can be built from six equilateral triangles. Therefore, we can start with equilateral
A A O B , then construct equilateral Л O B C on O B , then construct equilateral A O C D on O C ,
and so on. After making 6 equilateral triangles, well have regular hexagon A B C D E F .

But that’s an awful lot of work. There must be a faster way.

Solution 2: Since the circumcircle worked so well with the square, we'll try it with the hexagon. Since
the long diagonal of a regular hexagon is equal both to twice the side length of the hexagon and to
the diameter of the circumcircle of the hexagon, the radius of the circumcircle equals the length of
a side of the regular hexagon. Therefore, if we draw a circle with radius O A , the vertices of a
regular hexagon inscribed in the circle will be at intervals of length O A around the circle.

So, we draw an arc with center A and radius O A to find point B on the circle. Then we draw an arc
with center B and radius O A to get C, and so on around the circle. The resulting A B C D E F is a
regular hexagon.

We can prove A B C D E F is regular by noting that each of the triangles formed by connecting consecutive vertices to the center
is equilateral. For example, A O = B O = A B means A A O B is equilateral. Similarly, so is A B O C , so
/ . A B C = 2(60°) = 120°. In the same way, we see that each of the angles A B C D E F is 120°, and each of the sides is
equal (to the radius of the circle). Therefore, A B C D E F is indeed regular. □

Concept: The circumcircle of a regular polygon can be extremely useful in working with the
I polygon.

We’ve already solved a few regular octagon problems, so you should be ready to construct one now.

Problem 9.17 t V

Construct a regular octagon.

Solution for Problem 9.17: Solution 1: Looking at two consecutive sides of a regular octagon,
we see that if we can construct a 135° angle, we can construct a regular octagon. Since
135° = 90° + 45° and we know how to make both a 90° and 45° angle, we can make a
135° angle.

Specifically, we start with A B , which we continue to point P. We then draw a line through B
perpendicular to A B and let point X be on this line inside the octagon as shown. We bisect
/ X B P and let C be the intersection of this bisector and the circle centered at B with
radius B A . Therefore, A B = B C and

/ A B C = / A B X + / X B C = 135°,

so C is another vertex of our octagon. We can then copy / A B C to make / B C D , and so


on, completing the regular octagon.

But again, that's a lot of work!

Solution 2: Rather than look at the octagon from the point of view of the sides, let’s look at it
from the center's perspective. Again, we can use the circumcircle. We start by creating W
square W X Y Z by constructing perpendicular lines that meet at the center of the circle.
Where these lines meet the circle give us the vertices of square W X Y Z . The other four
vertices of our regular octagon must also form a square, with its vertices at the midpoints of
the arcs connecting the vertices of W X Y Z . To construct these points, we just bisect the
right angles formed by W Y and X Z . Where these bisectors meet the circle give us the
remaining vertices of our regular octagon, W P X Q Y R Z S . □

Exercises

9 .5 .1 : t V

Prove that our second solution to Problem 9.17 does produce a regular octagon.

Type your solution, notes and/or work here.

9 .5 .3 : t V

Construct a regular dodecagon.

Type your solution, notes and/or work here.


9.6 Summary

Definitions: A polygon is a closed planar figure with line segments as boundaries. As with
triangles and quadrilaterals, the segments that form the boundaries are the sides,
which meet at the vertices of the polygon. A diagonal is a segment that is not a side,
but connects two vertices of a polygon. A regular polygon is a polygon in which all
the angles have the same measure and all the sides have the same length.

Important: ■ The area of a regular polygon is half its perimeter times the distance from the
^ center of the polygon to a side.

■ In a polygon with n sides, the measures of the interior angles have a sum of
(n — 2)(180°) and the measures of the exterior angles have a sum of 360°.

The names of common polygons and the interior and exterior angle measures for common regular polygons are below:

Name # of Sides Int. Z Measure Ext. Z Measure


triangle 3 60° 120°
quadrilateral 4 90° 90°

ot‫־‬H
CO
O
pentagon 5 72°
hexagon 6 120° 60°
octagon 8 135° 45°
o
o

Oo
i‫־‬H
TT
nonagon 9
decagon 10 144° 36°

O'o
CO
dodecagon 12 150°

P roblem Solving S trateg ies

Concepts:
■ Sometimes thinking about the exterior angles of a polygon offers a simpler
approach than thinking about the interior angles!

■ Break complicated areas into pieces you can handle. Sometimes we can view a
desired area as the ‫׳‬leftover’ portion from having simple pieces taken away from a
simple starting figure.

■ Many problems involving regular hexagons can be tackled by dissecting the


hexagons into equilateral triangles.

■ Symmetry can be a very useful tool in problems involving regular polygons.

■ Most polygon problems are essentially quadrilateral and triangle problems. When
stuck with a polygon, try dissecting it into quadrilaterals and triangles.

■ The circumcircle of a regular polygon can be extremely useful in working with the
polygon.
Review Problems

How many sides does a regular polygon with interior angles that measure 140° have?

Type your solution, notes and/or work here. Show Solution

What is the measure of each interior angle of a regular polygon that has 36 sides?

Type your solution, notes and/or work here. Show Solution

Find the area of a regular hexagon with side length 4.

Type your solution, notes and/or work here. Show Solution

How many sides does a regular polygon with exterior angles that each measure 6° have?

Type your solution, notes and/or work here. Show Solution

Find the area of a regular octagon with side length 6.

Type your solution, notes and/or work here. Show Solution

The measure of each exterior angle of a regular polygon is 1 /8 the measure of an interior angle. How many sides does the
polygon have?

Type your solution, notes and/or work here. Show Solution

E E G H / . / is a regular hexagon. Find E H /E G .

Hint

Type your solution, notes and/or work here. Show Solution

9 .2 5 : Source: HMMT t V

A square and a regular hexagon are drawn with the same side length. If the area of the square is \/3 , what is the area of the
hexagon?

Type your solution, notes and/or work here. Show Solution

In a given octagon, four angles have measure x degrees and two have measure 2x degrees. The other two angles are right
angles. Is the octagon concave or convex?

Type your solution, notes and/or work here. Show Solution

A given polygon has 27 diagonals. How many sides does the polygon have?

Type your solution, notes and/or work here. Show Solution

A B C D E F G H is a regular octagon with A B = 8. Find A C , A D , and A E .

Type your solution, notes and/or work here. Show Solution

9 .2 9 : Source: MATHCOUNTS t V

A regular pentagon and a regular hexagon share a common side A D , as shown. What is the degree
measure of Z B AC?

Type your solution, notes and/or work here. Show Solution

9 .3 0 : Source: MATHCOUNTS t V

The number of diagonals in a regular polygon is equal to the number of sides. What is the number of degrees in the sum of all
the interior angles of the polygon?

Type your solution, notes and/or work here. Show Solution

A B C D E F G H I J is a regular decagon. Find appropriate congruent triangles (and prove they're congruent) to prove each of
the following:

(a) A C = A I.

Type your solution, notes and/or work here. Show Solution

(b) A D = AH.

Type your solution, notes and/or work here. Show Solution

(c) A E = CG.

Type your solution, notes and/or work here. Show Solution

Let A B C D be a square and let A B E F G be a regular pentagon in the diagram at right. Find Z B C E . F

Type your solution, notes and/or work here. Show Solution

A B C D E F G H is a regular octagon. Diagonals A E and C G meet at X . Point M is the midpoint of A B .

(a) Find Z X A B .

Type your solution, notes and/or work here. Show Solution

(b) Find Z M X A .

Type your solution, notes and/or work here. Show Solution

(c) Find Z M X D.

Type your solution, notes and/or work here. Show Solution


Challenge Problems

9 .3 4 : Source: (b) ARML t V

A \ A -2A h • ‫ ־ ־‬. 4 ‫ ״‬i s a r e g u l a r ? 7 - g o n .

(a ) P r o v e t h a t A ! A ^ A - aA a i s a t r a p e z o i d .

T y p e y o u r s o lu tio n , n o te s a n d / o r w o r k h e re . Show Solution

(b ) P r o v e t h a t Z . A 2A ! Æ ! = 3 6 0 ° /n .

T y p e y o u r s o lu tio n , n o te s a n d / o r w o r k h e re . Show S olution

In o u r d is c u s s io n o f r e g u la r p o ly g o n s , w e a s s u m e d t h a t e v e r y r e g u la r p o ly g o n h a s a p o in t t h a t is e q u id is t a n t f r o m a ll t h e
v e r t ic e s . W e c a lle d t h i s p o i n t t h e 'c e n t e r * o f t h e p o ly g o n .

(a ) P r o v e t h a t in s id e e v e r y r e g u la r p o ly g o n V th e re is a p o in t O th a t is e q u id is t a n t f r o m a ll o f t h e v e r t ic e s o f V.
H in t

T y p e y o u r s o lu tio n , n o te s a n d / o r w o r k h e re . Show Solution

(b ) Let O b e t h e c e n te r o f r e g u la r p o ly g o n V, a n d .4 b e o n e o f th e v e r tic e s o f V. P ro v e th a t 0 . 4 b is e c t s a n in t e r io r a n g le o f
V.

T y p e y o u r s o lu tio n , n o te s a n d / o r w o r k h e re . Show Solution

T h e p e n t a g o n A B O D E a t r ig h t h a s a r ig h t a n g le a t .4 , A B = AE, and E D = D C = CB = L If
B E — 2 a n d B E || C D , w h a t i s t h e a r e a o f t h e p e n t a g o n ?

H in t B

T y p e y o u r s o lu tio n , n o te s a n d / o r w o r k h e re . Show Solution

9 .3 7 : Source: MATHCOUNTS t V

T h e t o t a l n u m b e r o f d e g r e e s in t h e s u m o f t h e in t e r io r a n g le s o f t w o r e g u la r p o ly g o n s is 1 9 8 0 . T h e s u m o f th e n u m b e r o f
d i a g o n a ls in t h e t w o p o ly g o n s is 3 4 . W h a t is t h e p o s it iv e d if f e r e n c e b e t w e e n t h e n u m b e r s o f s id e s o f t h e t w o p o ly g o n s ?

H in t

T y p e y o u r s o lu tio n , n o te s a n d / o r w o r k h e re . Show S olution

F in d t h e s u m o f t h e a n g le s a t t h e p o in t s o f t h e 7 ‫־‬p o in t e d s ta r A B C D E F G s h o w n . D o n o t a s s u m e t h a t t h e h e p t a g o n in t h e
c e n t e r is r e g u la r .

H in t

H in t

T y p e y o u r s o lu tio n , n o te s a n d / o r w o r k h e re . Show S olution

A r e g u l< a m e p e r im e t e r . W h a t is t h e r a t io o f t h e a r e a o f t h e h e x a g o n t o t h e a r e a
o f th e tr

T y p e y o u r s o lu tio n , n o te s a n d / o r w o r k h e re . Show Solution

9 .4 0 : Source: MATHCOUNTS t V

In a p a t io , a p a t t e r n is d e t e r m in e d b y r e g u la r h e x a g o n a l t ile s , s q u a r e t ile s a n d e q u ila t e r a l t r ia n g u la r t ile s a s


s h o w n i n t h e d i a g r a m . I f t h e a r e a o f e a c h h e x a g o n a l t i l e i s 9 6 i n ’2 , w h a t i s t h e n u m b e r o f s q u a r e i n c h e s in
t h e a r e a o f e a c h s q u a r e t ile ? W h a t is t h e a r e a o f t h e w h o le t ile d r e g io n s h o w n ?

H in t

T y p e y o u r s o lu tio n , n o te s a n d / o r w o r k h e re . Show Solution

G iv e n t h a t A B C D E F is a r e g u la r h e x a g o n w it h s id e le n g t h 1 2 , f in d t h e a r e a o f A A C E .

T y p e y o u r s o lu tio n , n o te s a n d / o r w o r k h e re . Show S olution

9 .4 2 : Source: USAMTS t V

T h e in t e r io r a n g le s o f a c o n v e x p o ly g o n f o r m a n a r it h m e t ic p r o g r e s s io n w it h a c o m m o n d if f e r e n c e o f 4 ° . D e te r m in e t h e
n u m b e r o f s id e s o f t h e p o ly g o n i f it s la r g e s t in t e r io r a n g le is 1 7 2 ° .

H in t

T y p e y o u r s o lu tio n , n o te s a n d / o r w o r k h e re . Show Solution

O p p o s it e s id e s o f h e x a g o n A B C D E F s h o w n a r e p a r a lle l. G iv e n t h a t A D = 2 5 a n d t h e s id e
le n g th s o f th e h e x a g o n a re a s s h o w n , f in d [ A B C D E F ] .

H in t

T y p e y o u r s o lu tio n , n o te s a n d / o r w o r k h e re . Show Solution

9 .4 4 : Source: USAMTS &AHSME t V

D e t e r m in e w it h p r o o f t h e n u m b e r o f p o s it iv e in t e g e r s n s u c h t h a t a c o n v e x r e g u la r p o ly g o n w it h n s id e s h a s i n t e r io r a n g le s
w h o s e m e a s u r e s , in d e g r e e s , a r e in t e g e r s .

H in t

T y p e y o u r s o lu tio n , n o te s a n d / o r w o r k h e re . Show Solution

(a ) W h a t is t h e la r g e s t p o s s ib le n u m b e r o f in t e r io r a n g le s o f a c o n v e x p e n t a g o n t h a t c a n h a v e m e a s u r e 9 0 ° ?

H in t

T y p e y o u r s o lu tio n , n o te s a n d / o r w o r k h e re . Show Solution

(b ) W h a t is t h e la r g e s t p o s s ib le n u m b e r o f in t e r io r a n g le s o f a c o n v e x d e c a g o n t h a t c a n h a v e m e a s u r e 9 0 ° ?

T y p e y o u r s o lu tio n , n o te s a n d / o r w o r k h e re . Show Solution

(c ) F in d a f o r m u la in t e r m s o f n f o r t h e la r g e s t n u m b e r o f in t e r io r a n g le s o f a c o n v e x ? 7 -g o n t h a t c a n h a v e m e a s u r e 9 0 c

T y p e y o u r s o lu tio n , n o te s a n d / o r w o r k h e re . Show Solution

I t is p o s s ib le t o s u r r o u n d a p o in t w it h t h r e e r e g u la r h e x a g o n s w it h s id e le n g t h 1 a s s h o w n a t le f t b e lo w s u c h t h a t e a c h h e x a g o n
s h a r e s a s id e w it h t h e o t h e r t w o h e x a g o n s . A t r ig h t b e lo w , w e s e e t h a t i t is a ls o p o s s ib le t o s u r r o u n d a p o in t w it h a r e g u la r
h e x a g o n , a s q u a r e , a n d a r e g u la r d o d e c a g o n s u c h t h a t e a c h h a s s id e le n g t h 1 a n d e a c h o f t h e p o ly g o n s s h a r e s a s id e w it h t h e
o th e r tw o .

F in d a ll o t h e r g r o u p s o f t h r e e r e g u la r p o ly g o n s w it h s id e le n g t h 1 t h a t c a n s u r r o u n d a p o in t s u c h t h a t e a c h p o ly g o n s h a r e s a
s id e w it h t h e o t h e r t w o . P r o v e t h a t y o u h a v e f o u n d a ll p o s s ib le g r o u p s o f t h r e e p o ly g o n s .

H in t

H in t

T y p e y o u r s o lu tio n , n o te s a n d / o r w o r k h e re . Show Solution

L e t A B C b e a r ig h t t r ia n g le , w it h Z A C B = 90° L e t P a, P b , a n d P c - b e r e g u la r p e n t a g o n s , w it h s id e le n g t h s B C , G 'A a n d
A B , r e s p e c t i v e l y . P r o v e t h a t [ P A] + [P B\ = [P c \

H in t

H in t

T y p e y o u r s o lu tio n , n o te s a n d / o r w o r k h e re . Show Solution

9 .4 8 ★ : Source: AIME t V

T h e t w o s q u a r e s s h o w n s h a r e t h e s a m e c e n t e r O a n d h a v e s id e s o f le n g t h 1 . T h e le n g t h o f A B
is 4 3 / 9 9 . F in d t h e a r e a o f o c t a g o n A B C D E F G H fo r m e d b y c o n n e c tin g th e 8 p o in ts w h e r e
t h e t w o s q u a r e s in t e r s e c t a s s h o w n .

H in t

H in t

H in t

T y p e y o u r s o lu tio n , n o te s a n d / o r w o r k h e re . Show Solution

E x tra ! H a v e y o u e v e r s e e n a b a th ro o m f lo o r t ile d w it h r e g u la r p e n t a g o n s ? H o w a b o u t w it h r e g u la r h e x a g o n s ?
H e p ta g o n s 9 W h y d o s q u a r e s , t r ia n g le s , a n d h e x a g o n s s e e m to show u p e v e r y w h e r e , w h ile t h e o th e r
V r e g u l a r p o l y g o n s d o n ’t g e t m u c h u s e ?

L ik e lo t s o f o t h e r t h in g s in g e o m e t r y , it a ll b o ils d o w n t o a n g le s . N o w t h a t y o u k n o w h o w t o f in d t h e a n g le
in a r e g u la r p o ly g o n , y o u c a n a n s w e r t h is p r o b le m f o r y o u r s e lf ! T a k e a lo o k a t a t ilin g t h a t d o e s w o r k , s u c h
a s c o v e r in g t h e p la n e w it h s q u a r e s ( lik e a p ie c e o f g r a p h p a p e r ) . A t e a c h v e r te x , e x a c t ly f o u r s q u a r e s
m e e t. T h e a n g le o f e a c h s q u a r e is 9 0 ° , s o t h e f o u r a n g le s a t e a c h v e r t e x a d d u p t o 3 6 0 ° .

I f y o u s o lv e d P r o b le m 9 .4 6 , y o u s h o u ld b e a b le t o q u ic k ly id e n t if y w h ic h r e g u la r p o ly g o n s c a n b e u s e d t o
t ile a p la n e a ll b y t h e m s e lv e s . W e c a ll s u c h a r e g u la r t ilin g o f th e p la n e a te s s e lla tio n The th re e
t e s s e lla t io n s t h a t u s e o n ly a s in g le r e g u la r p o ly g o n o v e r a n d o v e r a r e s h o w n b e lo w . T h e s e a r e c a lle d
r e g u la r te s s e lla tio n s .

A s w e s a w in P r o b le m 9 .4 6 , w e h a v e m a n y m o r e p o s s ib ilit ie s i f w e a llo w o u r s e lv e s t o u s e m o r e t h a n o n e
ty p e o f p o ly g o n Show n b e lo w a re tw o s e m ir e g u la r t e s s e lla t io n s , in w h ic h th e sam e s e t o f r e g u la r
p o ly g o n s s u r r o u n d e a c h v e r t e x in t h e t e s s e lla t io n . T h e r e a r e e ig h t s u c h t e s s e lla t io n s ( n o t in c lu d in g t h e
r e g u la r t e s s e l l a t i o n s ) ; s e e i f y o u c a n f i n d t h e o t h e r s ix .

I f w e n o lo n g e r r e s t r ic t o u r s e lv e s t o h a v in g t h e s a m e p o ly g o n s a r o u n d e a c h p o in t , w e h a v e d e m ir e g u la r
t e s s e lla t io n s , o n e o f w h ic h is s h o w n b e lo w

O f c o u r s e , w e n e e d n o t o n ly u s e r e g u la r p o ly g o n s in b u ild in g t ilin g s . M o r e o v e r , t ilin g s n e e d n o t b e p e r io d ic


( m e a n in g h a v in g t h e s a m e p a t t e r n o v e r a n d o v e r ) P e n r o s e t ilin g s c o n s is t o f t w o q u a d r ila te r a ls t h a t a re
u s e d a s t ile s t o m a k e f a s c in a t in g n o n - p e r io d ic t ilin g s . A n e x a m p le is s h o w n b e lo w .

W e d o n ’t h a v e t o r e s t r i c t o u r s e l v e s t o s i m p l e g e o m e t r i c f i g u r e s t o t i l e a w h o l e p l a n e . T h e D u t c h a r t i s t M .
C. E s c h e r p ro d u c e d th o u s a n d s o f te s s e lla tio n s u s in g v a r io u s a n im a l s h a p e s and o th e r w h im s ic a l
d e s ig n s . R a th e r t h a n b e in g s a t is f ie d w it h ju s t a t e s s e lla t io n , E s c h e r o f t e n w o v e h is t e s s e lla t io n s in t o v e r y
e n g a g i n g p i e c e s o f a r t . Y o u ’ ll f i n d l i n k s t o E s c h e r ’s a r t a t h t t p : / / w v z w . m c e s c h e r . c o m / .

T e s s e lla tio n s h a v e e v e n m a d e t h e ir w a y in t o p u z z le s a n d g a m e s , a s t h e t w o p ic tu r e s o f t h e t e s s e lla t io n


g a m e Bats and Lizards b e l o w s h o w . P e rh a p s y o u c a n s e e h o w th e s e p a tte r n s w e re c re a te d fro m th e
r e g u la r t e s s e lla t io n s o n t h e p r e v io u s p a g e !
Morley s First Triangle

The worst form o f inequality is to try to make unequal things equal. - Aristotle

CHAPTER
I
Geometric Inequalities
In this chapter we will explore three geometric inequalities involving triangles. First, we will examine how the order of the lengths
of the sides is related to the order of the measures of the angles in a triangle. Then, we will learn how to tell if a triangle is acute
or obtuse just by considering its side lengths. Finally, we will explore the most widely used geometric inequality of all - the
Triangle Inequality. Like many of the most useful mathematical tools, the Triangle Inequality is so simple it’s almost obvious, but
it can be used to develop many non-obvious solutions to complex problems.

10.1 Sides and Angles of a Triangle

P roblem s
Problem 10.1

Draw a few triangles. Label each triangle A A /1C, and compare the order of the lengths A B , B C , and A C (from shortest to
longest) to the order of the measures of the angles Z A , Z B , and Z C . Make a guess about how the order of the lengths of the
sides is related to the order of the measures of the angles in a triangle. Try to prove your guess always works before continuing
this section! (Don't forget, you have to consider acute, right, and obtuse triangles.)

Problem 10.2 4 Jump to Solution

Prove that the hypotenuse of a right triangle is longer than each of the other two sides of the triangle.

Problem 10.3 4 Jump to Solution

In this problem, we will prove that in an obtuse triangle, the side opposite the obtuse angle is the
A v;-....
longest side of the triangle. We start with A A B C with obtuse Z C as shown.

(a) We already have information about the longest side in a right triangle. Try building a right triangle C В
with A B as a hypotenuse to show that A B > B C . Can you use a similar approach to show
A B > AC?

(b) Try finding a second solution by drawing a right triangle with B C as a leg.

Problem 10.4 4 Jump to Solution

Let A A B C be acute with Z B > Z C . We will show that A C > A B .

(a) Draw altitude A X and find the point D on faÔ such that A D = A C (C and D are different points). Show that
Z D A X > Z B A X . Use this inequality to show that D is beyond B on X è .

(b) Prove that A C > A B .

Problem 10.5 4 Jump to Solution

Let A A B C be acute. Show that if A C > A B , then Z B > Z C .

After looking at hundreds, if not thousands, of triangles while working to this point in the book, you probably think that the largest
side of a triangle is opposite the triangle's largest angle, and the shortest side is opposite the smallest angle. Good instincts!
We'll have to work through a few cases to prove it, though.

Problem 10.2 t V

Prove that the hypotenuse of a right triangle is longer than each of the other two sides of the triangle.

Solution for Problem 10.2: Let A C be the hypotenuse of A A B C . The Pythagorean Theorem tells us A C 2 = A B 2 4‫ ־‬B C 2.
Since A B 2 and B C 2 are both positive, we must have A C > A B and A C > B C . Hence, the hypotenuse of a right triangle is
longer than each of the other two sides of the triangle. □

Taking care of the longest side in a right triangle was pretty easy. Now let's take a look at obtuse triangles.

Problem 10.3 t V

Prove that in an obtuse triangle, the side opposite the obtuse angle is the longest side of the triangle.

Solution for Problem 10.3: Let Z C in A A B C be obtuse. We already know that the
hypotenuse of a right triangle is the longest side of the triangle, so we try building a right
triangle we can use. We want to show that A B is the longest side of our triangle, so we make
it the hypotenuse of a right triangle by drawing altitude A D from A as shown. Since A B is
the hypotenuse of A A B D , we have A B > B D . Since B D > B C , we have
A B > B D > B C . Similarly, we can draw an altitude from B to show that A B > A C .

We could also have built a right triangle with B C as a leg by drawing a line through C
perpendicular to B C as shown. Since A B C B is a right triangle, we have E B > B C . Since
A B > E B , we have A B > E B > B C . Similarly, we can show that A B > A C .

Therefore, in an obtuse triangle, the side opposite the obtuse angle is the longest side of the
triangle. □

Now we're ready to deal with acute angles.

Problem 10.4 t V

Let A A B C be acute with Z B > Z C . Show that A C > A B .

Solution for Problem 10.4: See if you can find what's missing in this ‫׳‬proof:

Bogus Solution: We start by drawing altitude A X . Since Z B > Z C , we have A


ZB A X < ZCAX. Therefore, B X < CX. Since a
y A B 2' = A X 2' + B X 2' and A C 2' = A X '2 + C X 2, our /
B X < C X tells us that A B < A C . /
‫־‬1 \
B yr c

The problem here is that we haven't justified that B X < C X claim. Why does Z B A X < Z C A X mean that B X < C X
(and no, you can’t just say ‫׳‬It's obvious!‫?)׳‬

It's not clear how we can build a right triangle to directly compare A B and A C , but we can build an
/1
obtuse one. We draw altitude A X and locate point D on such that A D = A C . Since
A D = A C , we have Z A D X = Z C . Therefore, Z D < Z A B C . Since Z D = 90° — Z D A X and
Z A B C = 90° - Z B A X , we have 90° - Z D A X < 90° - Z B A X , so Z B A X < Z D A X .
Therefore, D is beyond B on C È as shown at right.

Since Z A B C is acute, Z A B D is obtuse. This tells us A D > A B . Since A D = A C , we have the


desired A C > A B . □

Problem 10.5 t V

Let A A B C be acute. Show that if A C > A B , then Z B > Z C .

Solution for Problem 10.5: What's wrong with this solution:

Bogus Solution: If Z B > Z C , then A C > A B . Since we know that A C > A B , we have
Z B > ZC .
y__________________________________________
The problem with that ‘proof is that it is exactly the same as saying ,If I live in California, I live in America. Since I live in America, I
live in California.‫ ׳‬As we've noted before, we cannot assume that the converse of a true statement is true! We must prove a
statement and its converse separately. Here, we will present two solutions.

Solution 1: Although our Bogus Solution was indeed quite bogus, we can still use what we already know about angles and sides.
Specifically, consider three cases:

Case 1: Z B > ZC . As we have seen, if Z B > Z C , then A C > A B .

Case 2: Z B = ZC . If Z B = Z C , then A C = A B .

Case 3: Z B < ZC . As we have seen, if Z B < Z C , then A C < A B .

We are given that A C > A B . The only one of these cases that leads to A C > A B is Case 1, so we can deduce that
Z B > Z C . Make sure you see why this solution works, but the bogus one doesn’t!

Solution 2: We can backtrack through our solution to Problem 10.4. We find point D on c i such that A
A D = A C . To show that D is beyond B on C & , we first draw altitude A X . Since we are given
A C > A B and we have both A C 2 = A D 2 = A X 2 4‫ ־‬X D 2 and A B 2 = A X 2 4 X B 2, we must
have X D > X B .

Now we can use A A B D to show that Z C < Z A B C . From A A B D , we have

Z A D B + Z A B D = 180° - Z D A B .

Therefore, Z A D B 4 Z A B D < 180°. Since ZA D B = ZC and Z A B D = 180° — Z A B C , substitution into


Z A D B 4 Z A B D < 180° gives us

Z C 4 (180° - Z A B C ) < 180°,

so Z C < Z A B C as desired. □

We can summarize all of our discoveries thus far in this section very simply:

Important: In any triangle, the longest side is opposite the largest angle and the shortest side is
opposite the smallest angle. The middle side, of course, is therefore opposite the
Z middle angle.

In other words, in A A B C , A B > A C > B C if and only if Z C > Z B > Z A .

Let's try using these facts on a couple problems.

Solution for Problem 10.6: First we focus on A A B C . Since Z A > Z B > Z C , we have B C > A C > A B . Then, we turn to
A B D C . Since Z B > Z C > Z D , we have C D > B D > B C . We got lucky! B C is the smallest in one inequality string and
the largest in the other, so we can put the inequalities together:

C D > B D > BC > AC > AB.

Problem 10.7 t V

In A A B C the median A M is longer than B C / 2. Prove that Z B A C is acute.

Solution for Problem 10.7: We start with a diagram. We’d like to prove something about an angle, but all
A
we are given is an inequality regarding lengths. So, we use the length inequality to get some angle
inequalities to work with. Specifically, since A M > B M in A A B M and A M > M C in A A C M
(because A M > B C j 2 and B M = M C = BC /2),\N e have

ZB > ZBAM
ZC > ZCAM .

We want to prove something about Z B A C , which equals Z B A M 4 Z C A M , so we add these two inequalities to give
Z B 4 Z C > Z B A C . Since Z B 4 Z C 4 Z B A C = 180°, we can write Z B 4 Z C > Z B A C as

180° - Z B A C > Z B A C .

This gives us 180° > 2 Z B A C , so 90° > Z B A C . Therefore, Z B A C is acute. □

Exercises

1 0 .1 .1 : t V

In each of the parts below, order all the segments in the diagram from longest to shortest. (The diagrams are not drawn to
scale!)

В
(a)

Type your solution, notes and/or work here. Show Solution

(b)

Type your solution, notes and/or work here. Show Solution

(c)

Type your solution, notes and/or work here. Show Solution

10.1.2: t V

In isosceles triangle A P Q R , we have Z P = 54° and P Q = P R . Which is longer, P Q or Q R ?

Type your solution, notes and/or work here. Show Solution

1 0 .1 .3 ★ : t V

Point X is on A B . Point C is given such that Z A X C = Z A C B = 100°. Show that X is on segment A B , rather than
beyond B on A & or beyond ;4 on b X.

Hint

Hint

Type your solution, notes and/or work here. Show Solution


10.2 Pythagoras — Not Just For Right Triangles?
Back in Section 6.1, we learned that if A C 2 4‫ ־‬D C 2 = A D 2, then Z A C D is a right angle. But what if A C 2 4‫ ־‬D C 2 doesn’t
equal A D 2?

P roblem s
Problem 10.8

Draw lots of different triangles. Make a conjecture about what must be true if A D 2 4‫ ־‬D C 2 > A C 2, and what must be true if
A D 2 4‫ ־‬D C 2 < A C 2. Try to prove your conjectures before continuing!

Problem 10.9 4 Jump to Solution

Let Z C in A ADC be acute as shown. We will prove that


>4
A C 2 -F D C 2 > AD2.

(a) Draw altitude A X . Find an expression for AD2.

(b) Show that AC > AX. B c

(c) Show that A B 2 < A C 2 4 ‫ ־‬B C 2.

(d) Our diagram assumes that Z D is also acute. What if it isn’t?

Problem 10.10 4 Jump to Solution

Let Z C in A A D C be obtuse as shown. We will prove that

A B 2 > A C 2 + B C 2. V '' \ _
C B
(a) Draw altitude .4 D to fe<3.

(b) Find an expression for A B .

(c) Find A D in terms of ,4C and C D , and find D D in terms of D C and C D .

(d) Use the previous two parts to show that A B 2 > A C 2 4 ‫ ־‬B C 2.

Problem 10.11 4 j ump to Solution

Prove that the converses of the facts we proved in Problems 10.9 and 10.10 are also true. In other words, prove that if
AD 2 > A C 2 -f DC2, then Z C is obtuse, and prove that if AD 2 < A C 2 4‫ ־‬DC2, then Z C is acute.

We know that if A A D C is a right triangle with hypotenuse A D , we have A C 2 4‫ ־‬D C 2 = A D 2. Intuitively, it seems that if
A A D C is acute, then A C 2 4‫ ־‬D C 2 > A D 2, and if it is obtuse with Z C > 90°, then A C 2 4‫ ־‬D C 2 < A D 2. But intuitively
isn't good enough for us, is it?

Solution for Problem 70.9: Those sums of squares of sides send us hunting for right triangles. Drawing
altitude A X gives us a couple. From right triangle A A D X , we have A D 2 = A X 2 4‫ ־‬D X 2. Clearly
D X < D C , and right triangle A A C X gives us A X < A C . Therefore, we have

A D 2 = A X 2 4 ‫ ־‬D X 2 < A C 2 4‫ ־‬D C 2.

You might be wondering ‫׳‬Where did we use the fact that Z C is acute?’ Look closely - why must X be on
D C ? If Z D and Z C are both acute, then altitude A X must meet D C . Our proof, therefore, doesn’t
address the case where Z D is obtuse. However, in this case we have A C > A D , so obviously A C 2 4‫ ־‬D C 2 > A D 2. □

Concept: One of the most useful approaches to solving new problems is to try to think of
I similar-looking problems you know how to handle. For example, our main step in the
' solution to Problem 10.9 was thinking to build right triangles once we saw the sum of
squares of sides.

Problem 1 0.1 0 \ \j

Let Z C in A A D C be obtuse. Prove that A D 2 > A C 2 4‫ ־‬D C 2.

Solution for Problem 70.70: Once again, we start by drawing an altitude ( A D ) to build right
triangles. From right triangle A A D D , we have A D 2 = A D 2 -F D D 2. In order to get a D C
term and an A C term in our equation, we note that ,4 D 2 = A C 2 — C D 2 from A A D C ,
and D D = D C + C D . Therefore, we have

AD2 = A D 2 -F D D 2
= A C 2 - C D 2 + ( D C + C D )2
= A C 2 - C D 2 4‫ ־‬D C 2 4 2 ‫ (־‬D C ) ( C D ) 4‫ ־‬C D 2
= A C 2 - f D C 2 -P 2 ( D C ) ( C D )

Since 2( D C ) ( C D ) must be positive, we have A D 2 > A C 2 -F D C 2. □

Make sure you see where in our solution we used the fact that Z A C D is obtuse.

Concept: If you don’t use all the given information in a solution, proofread it closely to make
! sure you haven't made a mistake! Sometimes you won't need all the information
' you're given, but often if you haven't used it all, it's because you made a mistake
somewhere.

Furthermore, when you're stuck on a problem, read it again and see if there is any
information you haven’t used yet!

In Problem 6.7 we proved the converse of the Pythagorean Theorem; namely, that if the sides of a triangle satisfy a 2 4‫ ־‬b2 = c2,
then the triangle is right. Now we investigate if the converses of the inequalities we have just discovered are true.

Problem 10.11 t \j

Prove that the converses of the facts we proved in Problems 10.9 and 10.10 are also true. In other words, prove that if
A D 2 > A C 2 -F D C 2, then Z C is obtuse, and prove that if A D 2 < A C 2 -F D C 2, then Z C is acute.

Solution for Problem 70.7 7: We can mirror our first solution to Problem 10.5 by considering three different cases:

Case 7: Z C is acute. If Z C is acute, then A D 2 < A C 2 4‫ ־‬D C 2.

Case 2: Z C is right. If Z C is right, then AD 2 = A C 2 -F D C 2.

Case 3: Z C is obtuse. If Z C is obtuse, then A B 2 > A C 2 4‫ ־‬D C 2.

Each possible triangle A A D C must fall under exactly one of these cases. Only in the first case is A B 2 < A C 2 -F D C 2, so if
A B 2 < A C 2 4‫ ־‬D C 2, we can conclude that Z C is acute. Similarly, only in the third case is A D 2 > A C 2 -F D C 2, so we can
conclude that Z C is obtuse whenever A D 2 > A C 2 4‫ ־‬B C 2.

In the Exercises, you'll have a chance to provide a second ’geometric’ argument, as we did in Solution 2 to Problem 10.5. □

We can now relate the squares of the sides of a triangle to the angles of the triangle:

Important: Z C of A A D C is acute if and only if A B 2 < A C 2 4‫ ־‬B C 2.

Z Z C of A A D C is right if and only if A B 2 = j4 C 2 4‫ ־‬B C 2.

Z C of A A D C is obtuse if and only if A D 2 > A C 2 -F D C 2.

Let's try using some of this information on a problem.

Problem 1 0.1 2 t \j

In A X Y Z, we have A Y = 11 and Y Z = 14 For how many integer values of X Z is A X Y Z acute?

Solution for Problem 10.12: In order for A X Y Z to be acute, all three of its angles must be acute. Using the given side lengths
and our inequalities above, we see that we must have

121 + 196 > XZ


196 + X Z 2 > 121
121 + X Z 2 > 196

Simplifying these three yields:

317 > XZ2


XZ2 > -7 5
XZ2 > 75

The middle inequality is clearly always true. (We really didn’t even have to include it - clearly 11 couldn’t ever be the largest side!)
Combining the other two and noting that we seek integer values of X Z, we have 9 < X Z < 17 (since 82 < 75 < 92 and
182 > 317 > 172). So, there are 9 integer values of X Z such that A X Y Z is acute. □

Exercises

1 0 .2 .1 : t V

Each of the following groups of three numbers are the lengths of the sides of a triangle. Identify each triangle as acute, right,
or obtuse.

(a) 6,8,10

Type your solution, notes and/or work here. Show Solution

(b) 6,8,11

Type your solution, notes and/or work here. Show Solution

(c) 6,8,9

Type your solution, notes and/or work here. Solution

(d) / Ï 3 . 3 / 3 . / Ï 5

Type your solution, notes and/or work here. Show Solution

(e) 2.1,1.8,3.1

Type your solution, notes and/or work here. Show Solution

(f) 5/7,9/14,1

Type your solution, notes and/or work here. Show Solution

10.2.2: t V

In A A D C , A D = 17 and D C = 27. For how many integer values of A C is A A D C acute?

Type your solution, notes and/or work here. Solution

1 0 .2 .3 ★ : t V

In this problem we will find a geometric proof that Z A C D is obtuse if A D 2 > A C 2 - f B C 2.

(a) Let X be the point on *DC such that A X is an altitude of A A D C . Find an expression for A B 2 in terms of other
segment lengths in the resulting diagram. (Note that at this point, we do not know if X is on D C or not. It might be on
the extension of the segment instead.)

Type your solution, notes and/or work here. Solution

(b) Why must A A D C be obtuse if X is not on D C , but is instead on f a c such that C is between D and X?

Type your solution, notes and/or work here. Show Solution

(c) If X is on D C , then how are D X , C X , and D C related?

Type your solution, notes and/or work here. Show Solution

(d) Solve your expression from part (c) for D X and substitute the result into your equation for part (a). Is it possible for
A D 2 to be greater than A C 2 - f B C 2?

Hint

Type your solution, notes and/or work here. Show Solution

(e) Complete the proof by concluding that X must be on h S such that C is between X and D, so Z A C D must be obtuse.

Hint

Type your solution, notes and/or work here. Show Solution


10.3 The Triangle Inequality
The Triangle Inequality answers the question ,When can three given segment lengths be the side lengths of a triangle?’ The
answer to this question is both more obvious and more powerful than the other inequalities we have explored in this chapter.
We'll start with a proof of this simple inequality, then show why the inequality is so powerful by using it to solve a variety of
problems.

P roblem s
Problem 10.13 4 Jump to Solution

In this problem we will prove the Triangle Inequality. Don’t skip over this problem because it looks obvious! Try to find a
mathematical proof for each part.

(a) Use the inequalities from the previous sections to prove that in any triangle, the sum of two sides is greater than the third
side. This is the Triangle Inequality.

(b) The Triangle Inequality is often written as ‫׳‬the sum of two sides is greater than or equal to the third side/ Why - where
does the ,or equal to’ come from?

(c) If a, b, and c are positive numbers such that a 4‫ ־‬b > c, a 4‫ ־‬c > b, and b 4‫ ־‬c > a, then show that there exists a triangle
with side lengths a, b, and c.

Problem 10.14 4 Jump to Solution

In how many ways can we choose three different numbers from the set {1, 2 , 3 , 4 , 5 , 6 } such that the three could be the sides
of a triangle? (Note: The order of the chosen numbers doesn't matter; we consider {3, 4 ,5 } to be the same as {4, 3 ,5 }.)

Problem 10.15 4 Jump to Solution

Can the lengths of the altitudes of a triangle be in the ratio 2 : 5 : 6? Why or why not?

Problem 10.17 4 Jump to Solution

Circle O and circle P are tangent at point T such that neither circle passes through the interior of the other. In this problem we
will prove that O, P, and T are collinear (i.e., that a line passes through all three).

(a) Prove that O T 4‫ ־‬T P > O P.

(b) Show that if O T 4‫ ־‬T P > O P, then © O and © P meet at a second point besides T.

(c) Show that O, P, and T must be collinear.

We’ll start by ,proving the obvious/

Problem 10.13 t V

(a) Prove that in any triangle, the sum of two sides is greater than the third side. This is the Triangle Inequality.

(b) The Triangle Inequality is often written as 'the sum of two sides is greater than or equal to the third side/ Why - where
does the ,or equal to* come from?

(c) If a, b, and c are positive numbers such that a 4‫ ־‬b > c,a 4‫ ־‬c > b, and b + c > a, then show that there exists a triangle
with side lengths a, b, and c.

Solution for Problem 70.7 3:

(a) What's wrong with this solution:

Bogus Solution: Suppose we draw A B first. Clearly the farthest C could be from A is if we just
.. draw B C in the same direction, so that A B 4‫ ־‬B C = A C . Any other way we
y draw B C will result in C being closer to A. Therefore, A B 4‫ ־‬B C > A C if
A A B C is a real triangle.

The second sentence is true, but we haven't proved it. Since were dealing with an inequality, we reach for the inequalities
we've already proved to tackle the Triangle Inequality.

We’ve had so much success building right triangles, we do so again here, drawing altitude B D as b
shown. We have two cases to consider:

Case 7; A A B C is acute. We draw altitude B D , forming two right triangles. We thus have
A B > A D and B C > C D . Adding these gives the desired A B 4‫ ־‬B C > A D 4‫ ־‬D C = A C .
We can do the same for each angle, showing that each side is less than the sum of the other two
sides.

Case 2 : A A B C is right or obtuse. If Z B > 90° as shown, then clearly A C > A B and A C > B C
, so we definitely have A B < A C 4‫ ־‬B C and B C < A C 4‫ ־‬A B . All we have left to prove is
A C < A B 4‫ ־‬B C . Once again, right triangles come to the rescue. Drawing altitude B E gives us
A B > A E and B C > E C , so A B 4‫ ־‬B C > A E 4‫ ־‬E C = A C .

(b) We have seen above that if A, B, and C are not collinear, then there cannot be equality. However, when A, B, and C are
collinear such that B is between A and C, we have A B 4‫ ־‬B C = A C . If A, B, and C are collinear, we call the resulting
'triangle' A A B C a degenerate triangle. (Typically the term ,triangle only refers to nondegenerate triangles, i.e. those in
which the vertices are not collinear.)

Note that when B is on A C , we still have A C 4‫ ־‬B C > A B and A C 4 ‫ ־‬A B > B C in addition to A B 4‫ ־‬B C = A C , so
the Triangle Inequality is still satisfied with a small modification:

Important: The Triangle Inequality states that for any three points, A, B, and C, we have

Z A B 4- B C > A C,

where equality holds if and only if B is on A C . Therefore, for nondegenerate


triangles, A B 4‫ ־‬B C > AC.

(c) Suppose a > b > c, and let our triangle be A A B C with A B = c, A C = b, and
B C = a. We can start building our triangle by drawing B C with length a. We know
that A must be on the circle with center B and radius A B = c, and on the circle
with center C and radius A C = b. We draw both circles. Because a > b > c, we
know that B C > A C > A B . Therefore, the radius of ©£? is less than B C , so
© Z i intersects B C . We call this point of intersection E. Similarly, © C meets B C
at point D as shown.

Since b 4‫ ־‬c > a, we know that B E 4‫ ־‬D C = b 4‫ ־‬c > a = B C . Since


B E 4‫ ־‬D C > B C , we know that D cannot be on E C . In other words, our two circles must meet! Where these circles
meet gives us our final vertex of A A B C .

Note that we started off our proof to the last part with ,Suppose a > b > c ! But what if that’s not the case? Does our whole
proof fall apart?

No, it doesn’t. The whole proof still works; we just have to rearrange the letters a little. All the same logic still works.

Important: Mathematicians have a special way of saying ,All the different cases are essentially
the same, so proving it for this one proves it for all of them/ To say this in our solution
Z to the last part of the previous problem, a mathematician would have written, ‘
Without loss of generality, let a > b > C .. Then, the mathematician doesn’t need
a whole separate proof for essentially equivalent cases such as 6 > a > c.
Sometimes ,without loss of generality‫ ׳‬is shortened to ‫ ״‬WLOG‫׳‬.

Now well use the Triangle Inequality to determine sets of specific side lengths that can be the sides of a triangle.

Problem 10.14 t V

In how many ways can we choose three different numbers from the set {1, 2 ,3 ,4 , 5 ,6 } such that the three could be the sides
of a nondegenerate triangle? (Note: The order of the chosen numbers doesn't matter; we consider {3. 4 ,5 } to be the same as
{4 ,3 ,5 }.)

Solution for Problem 10.14: We first notice that if we have three numbers to consider as possible side lengths of a triangle, we
only need to make sure that the sum of the smallest two is greater than the third. (Make sure you see why!) We could just start
listing all the ones we see that work, but we should take an organized approach to make sure we don't miss any. We can do so by
classifying sets of three numbers by the smallest number.

Case 7; Smallest side has length 7. No triangles can be made with three different lengths from our set if we include one of length
1.

Case 2: Smallest side has length 2. The other two sides must be 1 apart, giving the sets {2. 3 ,4 }, {2, 4 ,5 }, and {2, 5 ,6 }.

Case 3: Smallest side has length 3. There are only three possibilities and they all work: {3. 4 ,5 }, {3, 4 ,6 }, {3. 5 ,6 }.

Case 4: Smallest side has length 4. The only possibility is {4. 5 ,6 }, which works.

Adding them all up, we have 7 possibilities. □

We sometimes have to use some other tools in addition to the Triangle Inequality.

Problem 10.15 t V

Can the lengths of the altitudes of a triangle be in the ratio 2 : 5 : 6? Why or why not?

Solution for Problem 10.15: We don’t know anything about how the lengths of the altitudes of a triangle are related to each other.
We do, however, know a whole lot about how the lengths of the sides of a triangle are related to each other. Therefore, we turn
the problem from one involving altitudes into one involving side lengths. We let the area be K , let the side lengths be a, b, c, and
the lengths of the altitudes to these sides be ho, hb, h a respectively. Therefore, we have K = (ih a/2 = bhb/2 = c.hc/2, so the
sides of the triangle have lengths

2К 2 К 2 К

If our heights are in the ratio 2 : 5 : 6, then for some x, our heights are 2x, 5.t , and 6x. Then, our sides are

2 /f 2 /f 2 /f
2 x ' 5x ' 6x

However, the sum of the smallest two sides is then

2К 2 K 1 2 K 10 К 22 К
Sx 6x 30x 30x 30a; ’

which is definitely less than the largest side, which is 2 K / 2 x = K / x . Therefore, the sides don't satisfy the Triangle Inequality,
which means it is impossible to have a triangle with heights in the ratio 2 : 5 : 6. □

Concept: When facing problems involving lengths of altitudes of a triangle, consider using area
! as a tool.

The Triangle Inequality is probably the most commonly used tool in geometric inequality proofs; here's an example of the
Triangle Inequality in action.

Problem 10.16 t V

Let A M be a median of A A B C . Prove that A M > (A B 4- A C — B C ) / 2.

Solution for Problem 10.16: We can apply the Triangle Inequality to both A A B M and A A C M , giving:
/1
AM + BM > AB
A M 4‫ ־‬C M > AC.

Noting that B M 4- C M = B C , we think to add these inequalities, which gives


2 A M 4‫ ־‬B C > A B 4‫ ־‬A C . Subtracting B C from both sides and dividing by 2 yields

A B 4- A C - B C
AM >

Concept: If you don’t see the path to the solution immediately, don't just sit and stare at the
I problem! Make some observations. Write down statements you can prove that might
I be helpful. Perhaps you’ll be able to combine these observations to complete your
proof.

In Problem 10.16, even if we didn't see the solution immediately, the problem involves an inequality that has sums of lengths.
This makes us think of using the Triangle Inequality to make observations. However, we don't simply write Triangle Inequality
relationships blindly. We use the problem as a guide to make the observations. Specifically:

Concept: When trying to solve geometric inequalities, pay attention to which side of the
• inequality you want to prove each length is on. For example, we wouldn't want to start
I Problem 10.16 by observing A M < A B 4‫ ־‬B M , since this inequality has A M on
the smaller side, but what we want to prove has A M on the larger side.

We end the chapter with an important application of the Triangle Inequality.

Problem 10.17 t V

Circle O and circle P are tangent at point T such that neither circle passes through the interior of the other. Such circles are
said to be externally tangent. Prove that O, P, and T are collinear.

Solution for Problem 10.17: We don’t know a whole lot about tangent lines at
this point. However, in this section we have discovered a way to show that three
points are on a line. Specifically, if O T 4- T P = O P, then T is on O P ,
because otherwise the Triangle Inequality guarantees O T 4- T P > O P .

There doesn't seem to be any easy way to approach showing that


O T 4- T P = O P. Instead, we try to show that it's impossible to have
O T 4- T P > O P and still have tangent circles. If we have O T 4- T P > O P
, then T cannot be on O P . Therefore, there is some point R such that O P is
the perpendicular bisector of T R . To find point R, we draw altitude T X from
T to O P , then we draw R X such that R X = T X and R X J_ O P . Since
O X = O X , Z T X O = Z R X O , and R X = T X , we have A T O X = A R O X . Therefore, T O = RO, so R is on ©O.
Similarly, we have A P T X = A P R X , so P T = P R and R is on © /4 Therefore, if O T 4- T P > OP, it is impossible for
the circles to be tangent because there is a second point at which the circles meet. However, our circles must be tangent, so we
can't have O T 4- T P > OP. The Triangle Inequality tells us we can't have O T 4- T P < OP. Therefore, we must have
O T 4- T P = OP, which means О, T, and P are collinear. □

Important: If © O and © P are tangent at point T, then O, P, and T are collinear. We’ll often use
this fact in problems with tangent circles by connecting the centers of the circles,
Z knowing this line also goes through the point of tangency.

Notice that Problem 10.17 only tackles the case in which the circles are externally
tangent. It’s also true for circles that are internally tangent, i.e., when one circle is wholly
inside the other except at the one point at which they are tangent. In the diagram at right,
© Z i and © C are internally tangent, while ©Z7 is externally tangent to both of the other
circles.

Exercises

1 0 .3 .1 : t V

Which of the following sets of three numbers could be the side lengths of a triangle?

(a) 4,5,6

Type your solution, notes and/or work here. Show Solution

(b) 7,20,9

Type your solution, notes and/or work here. Show Solution

(c) 1/2,1/6,1/3

Type your solution, notes and/or work here. Show Solution

(d) 3.4,11.3,9.8

Type your solution, notes and/or work here. Show Solution

(e) V b .V Ï4 .V Î9

Type your solution, notes and/or work here. Show Solution

1 0 .3 .2 : Source: MATHCOUNTS t V

The lengths of two sides of a triangle are 7 cm and 3 cm. If the number of centimeters in the perimeter is a whole number,
what is the number of centimeters in the positive difference between the greatest and least possible perimeters?

Type your solution, notes and/or work here. Show Solution

1 0 .3 .3 : t V

Prove that the sum of the diagonals of a quadrilateral is less than the quadrilateral's perimeter.

Type your solution, notes and/or work here. Show Solution

1 0 .3 .4 ★ : t V

A B C D is a square and O is a point. Prove that the distance from O to A is not greater than the sum of the distances from O
to the other three vertices, no matter which point we take to be point O.

Hint

Type your solution, notes and/or work here. Show Solution

1 0 .3 .5 ★ : t V

Prove that if a, b, and c are the sides of a triangle, then so are \Ja., \fb, and \fc. What about a 2, b2, and c2?

Hint

Hint

Type your solution, notes and/or work here. Show Solution


10.4 Summary

Important: In any triangle, the longest side is opposite the largest angle and the shortest side is
opposite the smallest angle. The middle side, of course, is therefore opposite the
Z middle angle.

In other words, in A A B C , A B > A C > B C if and only if Z C > Z B > /.A .

Important: Z C of A A B C is acute if and only if AB'2 < A C 2 + B C '2.

Z Z C of IS.A B C is right if and only if A B 2 = A C 2 4‫ ־‬B C 2.

Z C of A A B C is obtuse if and only if A B 2 > A C 2 4‫ ־‬B C 2.

Important: The Triangle Inequality states that for any three points, A, B, and C, we have

Z AB +B C > AC,

where equality holds if and only if B is on A C . Therefore, for nondegenerate triangles


(i.e., those in which the vertices are not collinear), A B 4‫ ־‬B C > A C .

Important: If O O and © P are tangent at point T, then O, P, and T are collinear. We'll often use
this fact in problems with tangent circles by connecting the centers of the circles,
Z knowing this line also goes through the point of tangency.

P roblem Solving S trateg ies

Concepts:
One of the most useful approaches to solving new problems is to try to think of
similar-looking problems you know how to handle.

If you don't use all the given information in a solution, proofread it closely to make
sure you haven’t made a mistake! Sometimes you won't need all the information
you’re given, but often if you haven’t used it all, it's because you made a mistake
somewhere.

Furthermore, when you're stuck on a problem, read it again and see if there is any
information you haven’t used yet!

When facing problems involving lengths of altitudes of a triangle, consider using


area as a tool.

If you don’t see the path to the solution immediately, don't just sit and stare at the
problem! Make some observations - write down statements you can prove that
might be helpful. Perhaps you'll be able to combine these observations to
complete your proof.

When trying to solve geometric inequalities, pay attention to which side of the
inequality you want to prove each length is on. For example, we wouldn’t want to
start Problem 10.16 by observing A M < A B 4‫ ־‬B M , since this inequality has
A M on the smaller side, but what we want to prove has A M on the larger side.
Review Problems

1 0 .1 8 : t V

For each of the groups of three numbers below, state whether the numbers could be the side lengths of a triangle or not. If
they can be, identify whether or not the triangle is acute, obtuse, or right.

(a) 2,3,4

Type your solution, notes and/or work here. Show Solution

(b) 2.1,17,3.9

Type your solution, notes and/or work here. Show Solution

(c) n/5 ,2 ,7 3

Type your solution, notes and/or work here. Show Solution

(d) 199,401,297 (See if you can do it without squaring those numbers!)

Type your solution, notes and/or work here. Show Solution

(e) 1/2,1/3,1

Type your solution, notes and/or work here. Show Solution

(f) 60, 24,48

Type your solution, notes and/or work here. Show Solution

1 0 .1 9 : t V

Ari is solving a problem involving a right triangle with legs 119 and 120. He uses the Pythagorean Theorem and gets 261 as
the hypotenuse. He immediately shakes his head and starts over. How did he know so quickly that he made a mistake?

Type your solution, notes and/or work here. Show Solution

10.20: t V

In A A B C , A B = 5 and B C = 11. For which integer values of A C is A A B C an obtuse triangle?

Type your solution, notes and/or work here. Show Solution

10.21 t V

A i A 2A z • • • j4‫ ״‬is a regular polygon with n > 3. Prove that A\A$ > A] A 2.

Type your solution, notes and/or work here. Show Solution

10.22: t V

Prove that it is impossible for the length of a side of a triangle to be greater than half the triangle's perimeter.

Type your solution, notes and/or work here. Show Solution

1 0 .2 3 : Source: MATHCOUNTS t V

The perimeter of an isosceles triangle is 38 centimeters and two sides of the triangle are whole numbers in the ratio 3 : 8.
What is the number of centimeters in the length of the shortest side?

Type your solution, notes and/or work here. Show Solution

1 0 .2 4 : t V

Find all positive integers x for which it is possible for 2x 4 3 , 3 ‫־‬x 4 8 ‫־‬, and 6x 4- 7 to be the side lengths of a nondegenerate
triangle.

Type your solution, notes and/or work here. Show Solution

1 0 .2 5 : t V

How many different obtuse triangles with integer side lengths and a perimeter of 20 can we draw such that no two of them are
congruent?

Type your solution, notes and/or work here. Show Solution

1 0 .2 6 : t V

The length of each leg of an isosceles triangle is x 4 1 ‫ ־‬and the length of the base is 3.x — 2. Determine all possible values of
x. (Your answer should be an inequality expressing the possible values of x.)

Type your solution, notes and/or work here. Show Solution

1 0 .2 7 : t V

Y Z is the base of isosceles triangle A X Y Z . Given that Y Z > X Y , show that Z X > 60

Hint

Hint

Type your solution, notes and/or work here. Show Solution


Challenge Problems

1 0 .2 8 : t V

I have four stakes in my yard arranged like points W , X, Y, and Z shown. I wish to connect each stake to
Z
the same point in my yard with a string. What point should I choose to minimize the amount of string I
must use? (Make sure to prove that your choice is the best possible!)
w

Type your solution, notes and/or work here. Show Solution

1 0 .2 9 : t V

Orion chooses a positive number and Michelle chooses a positive number. Joshua chooses a positive number that is smaller
than Orion’s number. No matter what number Joshua chooses, we can always form a triangle with the three chosen numbers
as side lengths. Show that Orion’s and Michelle's numbers must be the same.

Hint

Type your solution, notes and/or work here. Show Solution

1 0 .3 0 : t V

Show that it is impossible for any 5‫־‬pointed star like A B C D E F G H I J at right to have b
A B > B C , C D > D E , E F > F G , G H > H I , and I J > J A all be true.

Type your solution, notes and/or work here. Show Solution

10.31 t V

As shown below, Q is on P H and circles © P , ©Q, and © P are drawn such that no two circles intersect and no one circle
contains the other two. Suppose a fourth circle can be constructed that is externally tangent to all three of these circles.

(a) Use the Triangle Inequality to prove that the radius of our fourth circle is greater than the radius of ©Q.

Hint

Type your solution, notes and/or work here. Show Solution

(b) Use the inequalities relating the order of sides in a triangle to the order of angles in a triangle to prove that the radius of
our fourth circle is greater than the radius of ©Q.

Hint

Type your solution, notes and/or work here. Show Solution

1 0 .3 2 : t V

In convex quadrilateral W X Y Z, we have W X > W Y . Show that we must also have X Z > Y Z .

Hint

Hint

Type your solution, notes and/or work here. Show Solution

1 0 .3 3 : t V

Let a, b, and c be three positive real numbers such that a + b~ > c , a + c > b and b 4‫ ־‬c > a . Prove that a, b, and c
can be the lengths of the sides of a triangle.

Hint

Type your solution, notes and/or work here. Show Solution

1 0 .3 4 : t V

Prove that the sum of the lengths of the diagonals of a quadrilateral is greater than half the perimeter of the quadrilateral.

Type your solution, notes and/or work here. Show Solution

1 0 .3 5 ★ : t V

In A X Y Z , we have Z X = 20° and X Y = X Z . Prove that 2Y Z < X Y < 3Y Z .

Hint

Hint

Hint

Hint

Hint

Type your solution, notes and/or work here. Show Solution

1 0 .3 6 ★ : t V

In A A B C and A A 'B 'C ', we have A B = A 'B ', A C = A 'C ', and Z B A C > Z B 'A 'C '. Prove that B C > B 'C '

Hint

Hint

Type your solution, notes and/or work here. Show Solution

1 0 .3 7 ★ : t V

Prove that the distance between any two points inside A A B C is not greater than half the perimeter of A A B C .

Hint

Hint

Hint

Type your solution, notes and/or work here. Show Solution

Extra! Proof That Every Triangle Is Isosceles?

V In triangle A B C below, let D be the midpoint of B C . Let P be the intersection of the angle bisector of
A and the perpendicular bisector of B C . Let M be the foot of the perpendicular from P to A B and N
be the foot of the perpendicular from P to A C , as shown.

Since P lies on the angle bisector of A, Z P A M = Z P A N . Also, Z P M A = Z P N A = 90°, so by


AAS, triangles P A M and P A N are congruent. Therefore, P M = P N and A M = A N .

P lies on the perpendicular bisector of B C , so P B = P C . Also, Z P M B = Z P N C = 90°, so


triangles P M B and P N C are congruent by HL Congruence. Therefore, M B = N C . But
A M = A B + B M , A N = A C + C N , and A M = A N , so A B = A C . Therefore, triangle A B C
is isosceles.

It is obviously not true that every triangle is isosceles, so something is going wrong in the proof. But
what?
Monge s Theorem

Probably no symbol in mathematics has evoked as much mystery, romanticism, misconception and human interest as the number pi. - William L. Schaaf

CHAPTER 11
Circles

11.1 Arc Measure, Arc Length, and Circumference


The portion of a circle connecting two points on the circle's circumference is called an arc. We find the measure of an arc of a
circle by considering what fraction of the circle the arc is. For example, a whole circle is 360°, so an arc that is 1 /6 of the circle
has measure ( l/6 ) ( 3 6 0 ° ) = 60°.

However, not all 60° arcs are the same. Some are much longer than others, as shown in the figure to the
right. Thus, we need more than just angle measure to classify arcs. We need a way to measure of the lengths
of arcs.

We start with the length of an entire circle, which would be the perimeter of a circle. The perimeter of a circle
is called the circle's circumference.

Before reading the rest of this chapter, put yourself in the sandals of the ancient Greeks and try a little experiment. Get a string,
and find numerous circular objects. Measure the distance around each object by wrapping the string around it. Then measure the
diameter of the object. Finally, for each object find the quotient

Distance Around the Object


Diameter of Object
You should find that in each case the quotient is around 3.14. (If you get anything different, try measuring and dividing again!)

In every single circle, the ratio of circumference to diameter is the same. This ratio is called pi, and is given the symbol n. Its
value is approximately 3.14. (If you ask a bunch of your math friends, you're likely to find someone who knows dozens more
digits.)

Sidenote: For some reason, memorizing huge portions of pi is a bit of a sport among math-
lovers. This should help you get started:
n
3 . 141592653589793238462643383279502884197169399
37510582097494459230781640628620899862803482534
21170679821480865132823066470938446095505822317
25359408128481117450284102701938521105559644622
94895493038196442881097566593344612847564823378
67831652712019091456485669234603486104543266482
13393607260249141273724587006606315588174881520
92096282925409171536436789259036001133053054882
04665213841469519415116094330572703657595919530
92186117381932611793105118548074462379962749567
35188575272489122793818301

(Please don’t tell your parents you got this 7r-memorizing idea from us.)

Pi is an irrational number, which means that it cannot be expressed as a ratio of integers. Because pi is irrational, its decimal
expansion does not terminate and does not become periodic. In other words, it does not get to a point where the same set of
numbers is repeated over and over. So, there are no shortcuts to memorizing digits of pi!

P roblem s
Problem 11.1 4 Jump to Solution

The circumference of a circle is 327r. Find the circle’s radius.

Problem 11.2 4 Jump to Solution

The length of a 72° arc of a circle is 15. What is the circumference of the circle?

Problem 11.3 4 Jump to Solution

Chord Y Z of a circle with center O has length 12. The circumference of the circle is 24 tt.

(a) What kind of triangle is A Y O Z l

(b) Find the radius of the circle.

(c) Find the length of Y Z .

Problem 11.4 4 Jump to Solution

Chords A B and C D of © O have the same length. We will prove that A B = C D .

(a) Connect A, B, C, and D to the center of the circle. Find some congruent triangles.

(b) Use your congruent triangles to prove that A B = C D .

Problem 11.5 4 Jump to Solution

(a) Anna and Betty are 180 yards apart, at points A and B in the diagram, respectively. There is a semi­
circular path from Anna to Betty. Chuck is at point C, the midpoint of this path. Anna must walk to
Chuck and then to Betty. How much farther must Anna walk if she follows the path than if she ,cuts
across,‫ ׳‬walking straight to C then straight to B?

(b) What if the path is a 120° arc of a circle instead of a semi-circular arc? (Chuck is on the midpoint of
this new arc, and we wish to see how much farther Anna must walk by staying on the path.)

Hint

Hint

Circumference is a pretty simple concept, once you've finished memorizing all the digits of pi.

Solution for Problem 77.7: Let C be the circumference and d be the diameter. We know that C /d = n and C = 32n. Therefore,
d = 32. The radius is half the diameter, or 3 2 /2 = 16. □

Important: In a circle, let C be the circumference, d be the diameter, and r be the radius.

Z C = lid = 2 n r.

Solution for Problem 11.2: A 72° arc is 72°/360° = 1 /5 of an entire circle. Since this arc has length 15, the entire circumference
of the circle has length 5(15) = 75. □

Problem 11.3 t V

Chord Y Z of a circle with center O has length 12. The circumference of the circle is 24 tt. Find the length Y Z .

Solution for Problem 77.3: To find the length of the arc, we must find Z Y O Z . We don’t have any information
Z
about angles, however, so we start by figuring out some lengths. Since the circumference is 247T, the
diameter is 24 and the radius is 12. Hence, O K = Y Z = ZO , so A Y O Z is equilateral. Therefore, Z Y O Z
is 60°, and our arc is 1 /6 of the circle. So, the length of Y Z is (1 /6 )(2 4 tt) = 47r. □

A central angle of a circle is an angle with the center of the circle as its vertex. Now that we know how to relate arcs to central
angles, we can find a useful relationship between the arcs cut off by congruent chords of a circle.

Problem 11.4 t V

Chords A B and C D of © O have the same length. Prove that A B = C D .

Solution for Problem 11.4: We form two triangles by connecting A, B, C, and D to O. Since A B = C D ,
A O = CO, and B O = D O , we have A A B O = A C D O by SSS Congruence. Therefore,

Z A O B = Z C O D , so arcs A B and C D are congruent. □

As you might suspect (and will prove as an Exercise), this relationship between congruent chords and congruent arcs works in
reverse, too.

Important: If two chords of a circle are congruent, then the arcs they subtend (cut off) are
congruent. Conversely, if two arcs of a circle are congruent, then the chords that
Z subtend them are congruent.

Problem 11.5 t V

(a) Anna and Betty are 180 yards apart, at points A and B in the diagram, respectively. There is a semi­
circular path from Anna to Betty. Chuck is at point C, the midpoint of this path. Anna must walk to
Chuck and then to Betty. How much farther must Anna walk if she follows the path than if she ‫׳‬cuts
across,1walking straight to C then straight to B? A

(b) What if the path is a 120° arc of a circle instead of a semi-circular arc? (Chuck is on the midpoint of
this new arc, and we wish to see how much farther Anna must walk by staying on the path.)

Solution for Problem 11.5:


C
(a) We might immediately see that A A C B is a 45-45-90 triangle, but if we don't, we can start by
connecting C to the center of the circle, O, as shown in the diagram. Since A C = C B = 90°, we ^
have Z A O C = Z C O B = 90°. O A , O B , and O C are all radii of the circle, so A A O C and
A BOC are 45-45-90 triangles. Since A B = 180 yards, we have
A O = O B = O C = 180/2 = 90 yards and A C = B C = C O \/2 = 9 0 \/2 yards. Since arc A B is one-half a circle,
its length is { A B ) i r / 2 = 90tt yards. So, Anna must walk

90 tt - ( A C + B C ) = 90 tt - 180\/2

yards farther if she follows the path.

(b) We start by connecting A, B, and C to the center of the circle of which the path is a part, since we
know more about radii than we do about arcs. Since A B is 120°, and C is its midpoint,
A C = B C = 60°. Therefore, Z A O C = Z C O B = 60°. O A = O C because they are radii of
the same circle. Therefore, A A O C is isosceles with Z A O C = 60° and A O = O C , so A A O C
is equilateral. Similarly, A B O C is also equilateral.

We need to find the radius of this circle to finish the problem. Let A B meet C O at P. Since A A O P = A B O P by SAS,
Z A P O = Z B P O . These angles together make up a straight angle, so Z A P O = Z B P O = 90°. So, A A P O and
A B P O are 30-60-90 triangles. Since A P = A B / 2 = 90 yards, we have A O = A P { 2 /\/3 ) = 6 0 \/3 yards. Since
A C = A O = B O = B C , we have

AC + C B = AO + BO = 120\/3.

The length of A B is (120°/360°)(2.4O 7r) = 407t \ / 3. Therefore, by following the path, Anna walks

A B - (AC + B C ) = 40 ttv/3 - 120%/3

farther.

The key step in the second part of this problem is connecting the points on the path to the center of the circle. This is often an
important step in challenging geometry problems.

Concept: In complicated problems involving a circle, try connecting the center of a circle to
I important points on the circle.

Sidenote: We defined 7r as the ratio of the circumference of a circle to its diameter. But how do
we know what 7r is numerically? In other words, how do we know that
^ 7r = 3.1415926535 ..., and so on?

Since 7r is one of the most important mathematical constants, much time and effort
have been devoted to calculating 7r. At the present time, 7r is known to over one trillion
decimal places. Why bother with so many digits? Certainly not for the sake of
accuracy in everyday use - even 39 decimal places is sufficient to calculate the
circumference of the known universe to within the radius of a hydrogen atom.

The answer, in part, is for the prestige of having set a new record number of places.
The digits of 7r can also be used to check the speed and accuracy of computers.
Finally, mathematicians are interested in seeing if they can find any patterns among
these digits, such as whether the ten digits 0, 1, 2, 3, . . ., 9 appear equally often.
However, despite all this effort, no patterns of note have been found.

So how do we calculate the value of tt? One simple method is as follows: Suppose we
take a circle of radius 1, and circumscribe it, and inscribe it, with a regular n-gon. The
figure below shows the n-gons for n = 6 and n = 8 .

The perimeter of the polygons approximate the circumference of the circle, which is
2 tt. The greater the number of sides in the polygon, the better the approximation. Let
a n denote the perimeter of the circumscribed n-gon (the outer polygon), and let bn
denote the perimeter of the inscribed n-gon (the inner polygon). Then it turns out that

2a A
a 2n — and
a ‫ ״‬4‫ ־‬6 ‫״‬

Thus, if we start with a 6 and be, we can calculate a ! 2 and b\% which can be used to
calculate and 6 2 4 . and so on. This was the observation of the ancient Greek
mathematician Archimedes, after whom this algorithm is named. The following table
shows some results of this algorithm. We begin with the values a,« = 4 \/3 and
be = 6 . (Make sure you see why a,« = 4 \/3 and be = 6; remember, the radius of the
circle is 1 .)

n bn a2/ ‫״‬ b j 2
6 6.92820323 6 .0 0 0 0 0 0 0 0 3.46410162 3.00000000
12 6.43078062 6.21165708 3.21539031 3.10582854
24 6.31931988 6.26525723 3.15965994 3.13262861
48 6.29217243 6.27870041 3.14608622 3.13935020
96 6.28542920 6.28206390 3.14271460 3.14103195
192 6.28374610 6.28290494 3.14187305 3.14145247
384 6.28332549 6.28311522 3.14166275 3.14155761
768 6.28322035 6.28316778 3.14161018 3.14158389

As we can see, the sequences ( an/ 2) and (6n/2 ) both converge to 7r. This is not the
best algorithm for calculating the digits of 7r, but it illustrates how we can use a
simple geometric idea to approximate 7r.

Exercises

1 1 .1 .1 : t V

Find the circumference of a circle with radius 4.

Type your solution, notes and/or work here.

11. 1. 2: t V

Find the radius of a circle with circumference 12tt.

Type your solution, notes and/or work here. •i I!•

1 1 .1 .3 : t V

A C of © O has length 127T, and the circle has radius 18.

(a) Find OC.

Type your solution, notes and/or work here. Show Solution 1

(b) Find Z A O C .

Type your solution, notes and/or work here. Show Solution ₪

(c) ★ Find A C .

Hint

Type your solution, notes and/or work here.

1 1 .1 .4 : t V

Arcs W X and Y Z of © Q are congruent. Prove that W X = Y Z .

Type your solution, notes and/or work here. •. tv• I I •

Extra! Degrees are not the only way to measure angles! Just as there are different units for
measuring length, like inch and centimeter, there are different units for angle
V measures. Besides the degree, the most commonly used unit for angle measure is the
radian. Just as there are 360 degrees in a circle, there are 2n radians in a circle.
Therefore, the measure in radians of a quarter-circle is (27r)/4 = 7r/2 radians, and of
a semicircle is (27r)/2 = 7r radians.

As you might guess, the usage of radians is inspired by circumference. You might
also notice that every arc of a circle with radius 1 has its length in units equal to its
measure in radians. However, radians are not very useful until our study of advanced
trigonometry and calculus, so we'll stick with degrees for now.
TheSim son Line

At a round table there is no dispute about place. - Italian Proverb

CHAPTER 12
i
l Circles and Angles
In t h is c h a p t e r w e w ill e x p lo r e t h e m a n y w a y s in w h ic h a n g le s a n d a r c m e a s u r e s a r e r e la t e d .

12.1 Inscribed Angles

W e s a y t h a t Z P Q R is inscribed i n P R . i n t h e d i a g r a m a b o v e b e c a u s e i t s v e r t e x i s o n t h e c i r c u m f e r e n c e o f t h e c i r c l e a n d i t s
s id e s h it t h e c ir c le a t P a n d R Inscribed angles a r e e n o r m o u s l y u s e f u l t o o l s i n g e o m e t r y ; i n t h i s s e c t i o n w e e x p l o r e h o w
in s c r ib e d a n g le s a r e r e la te d t o t h e a r c s t h e y c u t o f f.

Problems
Problem 12.1 4 J u m p t o S o lu t io n

In S e c tio n 7 .2 h e re , w e le a r n e d th a t th e c ir c u m c e n t e r o f a r ig h t tr ia n g le is t h e m id p o in t o f its
h y p o t e n u s e . T h u s , w h e n w e d r a w t h e c ir c u m c i r c l e o f a r ig h t t r ia n g le , t h e r ig h t a n g le is in s c r ib e d in a
s e m ic ir c le . In t h i s p r o b le m , w e i n v e s t ig a t e i f it is t r u e t h a t a n y a n g le in s c r ib e d in a s e m ic ir c le , s u c h a s
/ A C B in t h e d ia g r a m a t r ig h t , m u s t b e a r ig h t a n g le . L e t O b e t h e c e n t e r o f t h e c ir c le .

(a ) D ra w O C . W h a t d o w e k n o w a b o u t A A O C a n d A ROC?

(b ) L e t / . A — x a n d Z B = y. W h a t o t h e r a n g l e m e a s u r e s x d e g r e e s ? W h a t o t h e r a n g l e m e a s u r e s
y d e g re e s ?

(c ) C o n s id e r t h e s u m o f t h e a n g le s in A A B C t o s h o w t h a t x 4 y = 90°

(d ) M u s t / A C B b e a r ig h t a n g le ?

Problem 12.2 4 J u m p t o S o lu t io n

In t h is p r o b le m w e w ill f in d in s c r ib e d / . A B C s h o w n a t r ig h t , g iv e n t h a t A C — 8 0 ° a n d A C B = 130°

(a ) D r a w O A , O B , a n d O C . F in d / A O C a n d / C O B .

(b ) F in d / O B C a n d / O B A.

(c ) U s e y o u r a n s w e r t o t h e p r e v io u s p a r t t o f in d / A B C .

(d ) R e d o t h e p r o b le m w it h B C = 6 4 ° .

(e ) M a k e a g u e s s a b o u t h o w w e c a n f ig u r e o u t / B f r o m t h e a r c s w it h o u t g o in g t h r o u g h a ll t h e s te p s a b o v e . C a n y o u p r o v e
your guess?

Problem 12.3 4 J u m p t o S o lu t io n

In P r o b le m 1 2 . 1 . w e s h o w e d t h a t i f a n a n g le is in s c r ib e d in a 1 8 0 ° a r c ( a s e m ic ir c le ) , t h e m e a s u r e o f t h e

a n g le is 1 8 0 ° / 2 = 90° In t h i s p r o b le m w e w i l l p r o v e t h a t w h e n / X Y Z is in s c r ib e d in X Z a s s h o w n in

t h e d ia g r a m ( w i t h O , t h e c e n t e r o f t h e c ir c le , in s id e / X Y Z ) , t h e n / Y = X Z / 2.

(a ) D ra w O X , O Y , a n d O Z . L e t / O X Y = x a n d X O Z Y — z. F i n d / X Y Z i n t e r m s o f x a n d 2.

(b ) F in d X X O Y . / Y O Z , a n d X X O Z in t e r m s o f a : a n d 2.

(c ) F in d X Z in t e r m s o f x a n d 2, t h e n s h o w t h a t / X Y Z = X Z ! 2 .

Problem 12.6 4 J u m p t o S o lu t io n

C h o r d s A C a n d B D o f a c ir c le m e e t a t P a s s h o w n . G iv e n A P = P B , s h o w th a t A B i| C D .

In S e c tio n 7 . 2 h e r e , w e le a r n e d t h a t w h e n w e d r a w t h e c ir c u m c ir c le o f a r ig h t t r ia n g le , t h e r ig h t a n g le o f t h e t r ia n g le c u t s o f f a
s e m ic ir c le . W e s t a r t t h i s s e c t io n b y e x p lo r in g w h e t h e r t h e c o n v e r s e is t r u e .

Problem 12.1 t V

W h e n a n in s c r ib e d a n g le c u t s o f f a 1 8 0 ° a r c , w e s a y t h a t it is in s c r ib e d in a s e m ic ir c le . Is s u c h a n a n g le n e c e s s a r ily a r ig h t
a n g le ?

Solution for Problem 12.1: L e t / A C B b e i n s c r i b e d i n a s e m i c i r c l e . S i n c e A B = 1 8 0 ° , A B is a


d ia m e te r o f t h e c ir c le . T h e r e f o r e , t h e m id p o in t o f A B , p o in t O, is t h e c e n te r o f th e c ir c le .
D r a w in g O C f o r m s t w o is o s c e le s t r ia n g le s s in c e A O = B O = CO.

W e suspect /A C B is 9 0 ° , s o w e le t / A C O = x and /B C O = y a n d try to s h o w th a t


x -f y = 9 0 ° S in c e A A O C a n d A B O C a r e is o s c e le s , w e h a v e / O A C = /O CA = x and
/O B C = / O C B = y. F r o m A A B C , w e h a v e / A 4 ‫ ־‬/ A C B 4 ‫ ־‬/ B = 1 8 0 °, so

x 4 {x 4 y) 4 y = 1 8 0 °.

T h e re fo re , x 4 y = 9 0 ° , w h ic h m e a n s / A C B is a r ig h t a n g le . □

Sidenote: Thales
T h a t a n y a n g le in s c r ib e d in a s e m ic ir c le is a r ig h t a n g le is s o m e t i m e s c a lle d
Theorem, a f t e r t h e m a t h e m a t i c i a n Thales o f Miletus, w h o i s r e g a r d e d b y m a n y a s t h e
f a t h e r o f t h e id e a o f ‫׳‬p r o o f . H e is c r e d it e d b y m a n y w it h t h e f ir s t p r o o f s t h a t v e r t ic a l
a n g le s a r e e q u a l, t h a t t h e s u m o f t h e a n g le s in a t r i a n g l e e q u a ls t w o r ig h t a n g le s , a n d
t h a t t h e b a s e a n g le s o f a n is o s c e le s t r ia n g le a r e e q u a l.

T h a le s w as an o u t s t a n d in g p r o b le m - s o lv e r . Legend has it th a t a fa rm e r once


c o m p l a i n e d t o T h a l e s , s a y i n g t h e f a r m e r ’s d o n k e y w o u l d l i g h t e n i t s l o a d o f s a l t o n
t r ip s t o t h e m a r k e t b y r o llin g in a r iv e r t o d is s o lv e t h e s a lt. T h a le s s u g g e s t e d t h e
f a r m e r lo a d t h e d o n k e y w it h s p o n g e s n e x t t im e S ource Journey Through Genius by
William Dunham

Important: A n y a n g le in s c r ib e d in a s e m i c i r c l e is a r ig h t a n g le

Z
H a v in g s h o w n t h a t a n y a n g le in s c r ib e d in a 1 8 0 ° a r c h a s m e a s u r e l 8 0 ° / 2 = 9 0 ° , w e s h o u ld w o n d e r i f t h e r e 's a g e n e r a l r u le
r e la t in g t h e m e a s u r e o f a n in s c r ib e d a n g le t o a r c s in t h e c ir c le .

Concept: W h e n t r y in g t o p r o v e s o m e t h in g o r f in d s o m e g e n e r a l r u le , in v e s t ig a t in g a f e w s a m p le
! c a s e s c a n b e a n e x c e lle n t g u id e .

W e 'l l t r y t h i s s t r a t e g y h e r e .

Problem 12.2 t V

P o in ts .4 , B, a n d C a r e o n 0 O s u c h th a t A C — 8 0 ° a n d .4 C B = 1 3 0 ° . F in d / A B C .

Solution for Problem 12.2: W e k n o w t h a t t h e m e a s u r e o f a n a r c e q u a l s t h e a n g l e f o r m e d b y t h e r a d i i


t h a t c u t o f f t h e a r c ( w e c a ll s u c h a n a n g le a central angle). T h e r e f o r e , w e d r a w r a d i i t o . 4 , B , a n d C ,
t h u s f o r m in g s o m e is o s c e le s t r ia n g le s . S in c e

/B O C = B C = A B -A C = 5 0 °,

w e have

/O B C = /O C B = = 6 5 °.
j
S im ila r ly , / A O B = AB = 1 3 0 °, so

180° - 130°
/O A B = /O B A = ----------- - ----------- = 2 5 °.

T h e re fo re ,

/A B C = /O B C - /O B A = 4 0 °.

W e s e e th a t / A B C = A C / 2 a n d w e w o n d e r i f t h is is a lw a y s t h e c a s e . W e c a n t r y c h a n g in g B C t o s e e i f t h a t m a t t e r s . I f w e le t

BC = 6 4 ° , w e c a n g o t h r o u g h t h e s a m e s e r ie s o f c a lc u la t io n s a s a b o v e t o f in d t h a t , in d e e d , / A B C is s t i l l 4 0 ° □

N o w t h a t w e h a v e a s p e c if ic c a s e a s a g u id e lin e , w e l l t r y t o p r o v e t h a t a n in s c r ib e d a n g le is a lw a y s h a l f t h e a r c it in t e r c e p t s .
U n f o r t u n a t e l y , t o c o m p l e t e l y p r o v e t h i s , w e l l n e e d a n u m b e r o f c a s e s . W e 'l l t r y o n e o f t h e s e c a s e s h e r e , t h e n l e a v e t h e o t h e r s f o r
E x e r c is e s .

Problem 12.3 t V

Prove that if / X Y Z is inscribed in a circle such that the center o f the circle, O , is inside A X Y Z , then / X Y Z — X Z / 2 .

Solution for Problem 12.3: W e u s e o u r s p e c i f i c c a s e a s a g u i d e , a n d w e c o n n e c t t h e r a d i i t o X , Y , a n d Z


to fo rm i s o s c e l e s t r i a n g l e s . W e d o n ’t k n o w a n y a n g l e s t h i s t i m e , s o w e l l h a v e t o u s e v a r i a b l e s . S i n c e w e
know w e w ant to use our is o s c e le s tr ia n g le s , w e le t /O Y X = x and / O Y Z = z, s o th a t
/ X Y Z = x 4 z. I s o s c e l e s t r i a n g l e s A X O Y a n d A Z O Y t e l l u s t h a t / O X Y = x and /O ZY = 2
. T h e re fo re , w e h a v e / X O Y = 1 8 0 ° — 2x fro m A O X Y, and /Z O Y = 1 8 0 ° — 2z fro m A O Z Y .
W e c a n n o w f in d / X O Z in t e r m s o f x a n d 2:

/ X O Z = 360° - / X O Y - /Y O Z = 2x 4 2z = 2 (x 4 z).

T h e re fo re , X Z = 2 (x 4 z) = 2/ X Y Z , s o / X Y Z = XZ/2. □

N o t e t h a t o u r p r o v in g t h a t a n a n g le in s c r ib e d in a s e m i c i r c l e is r ig h t is j u s t a s p e c ia l c a s e o f t h i s r e s u lt . H o w e v e r , w e d r a w s p e c ia l
a t t e n t io n t o t h e r ig h t a n g le c a s e b e c a u s e r ig h t a n g le s a r e s o im p o r t a n t in g e o m e tr y .

L e t ’s t r y u s i n g o u r n e w f o u n d k n o w l e d g e .

Solution for Problem 12.4 Solution 1: S in c e /B is in s c r ib e d in AC, w e have XB = A C /2 = 6 5 °. T h e re fo re ,


XC = 180° — /A — ZB = 45°

Solution 2: S i n c e / A i s i n s c r i b e d i n B C , w e h a v e B C = 2 / A = 1 4 0 ° T h e re fo re , w e h a v e A B = 360° — AC — B C = 9 0 °.

S in c e Z C is in s c r ib e d in A B , w e h a v e Z C = AB/2 = 45° □

Solution for Problem 12.5: S i n c e Z B a n d Z C a r e i n s c r i b e d i n t h e s a m e a r c , t h e y m u s t b e e q u a l ( s i n c e e a c h e q u a l s h a l f t h e a r c ) .

T h e re fo re , Z B = Z C = x . S in c e Z . 4 is in s c r ib e d in B C , w e h a v e Z . 4 = B C /2 = x/2. N o w w e c a n u s e A A P B t o w r ite a n
e q u a t io n f o r x. S in c e

/ A 4 Z A P B + ZB = 1 8 0 °,

w e have

| 4 2 x 4 ! 1 8 0 ° = ‫ ־‬.

3 °
S o lv in g t h is e q u a t io n g iv e s x = 5 1 - . □

W e c o u ld h a v e s o lv e d t h e la s t p r o b le m in m a n y d i f f e r e n t w a y s , b u t o u r s o l u t i o n a b o v e ill u s t r a t e s a p o w e r f u l p r in c ip le t h a t w ill b e
a c r u c ia l s t e p in m a n y p r o b le m s w h e n y o u m o v e o n t o m o r e a d v a n c e d g e o m e tr y .

Important: A n y t w o a n g le s t h a t a r e in s c r ib e d in t h e s a m e a r c a r e e q u a l.

Z
W e ll n o w u s e t h i s i m p o r t a n t f a c t in a p r o o f .

Problem 12.6 t V

Chords A C and B D o f a circle m e e t at P as shown. Given A P — P B , show that A B || C D .

£ — yc

Solution for Problem 12.6: S i n c e A P = P B , w e h a v e X P B A = X P A B . S i n c e / C A B a n d X C D B a r e i n s c r i b e d i n t h e


s a m e a rc , w e h a v e X C D B = / C A B . / C A B is t h e s a m e a n g le a s X P A B , s o w e h a v e

X CD B = XPAB = XPBA = /DBA.

S in c e X C D B = / D B A , w e h a v e A B || C D . □

Exercises
In E x e r c is e s 1 2 . 1 .5 a n d 1 2 . 1 .6 b e lo w , d o n o t a s s u m e t h a t a n in s c r ib e d a n g le e q u a ls h a lf t h e a r c it in t e r c e p t s . Y o u a r e a s k e d t o
p r o v e t h i s f a c t f o r v a r io u s c a s e s in t h e s e t w o p r o b le m s .

P o i n t s P , Q, a n d R a r e o n Q 0 s u c h t h a t P Q = 7 8 ° , Q R = 1 2 3 °, a n d P Q R = 2 0 1 °.

(a ) F in d Z Q P R .

T y p e y o u r s o lu tio n , n o te s a n d / o r w o r k h e re . Show S olution

(b ) F in d / P Q R .

T y p e y o u r s o lu tio n , n o te s a n d / o r w o r k h e re . Show Solution

(c ) F in d X P R Q .

T y p e y o u r s o lu tio n , n o te s a n d / o r w o r k h e re . Show Solution

(d ) F in d Z P O Q .

T y p e y o u r s o lu tio n , n o te s a n d / o r w o r k h e re . Show Solution

(e ) F in d Z P Q O .

T y p e y o u r s o lu tio n , n o te s a n d / o r w o r k h e re . Show S olution

(f) F in d Z P O R . ( B e c a r e f u l o n t h is o n e !)

T y p e y o u r s o lu tio n , n o te s a n d / o r w o r k h e re . Show Solution

P o in ts R a n d S a r e o n 0E s u c h th a t R S = 5 0 ° . P o in t T is a ls o o n 0E. F i n d a ll p o s s ib le v a lu e s o f ZR T S

T y p e y o u r s o lu tio n , n o te s a n d / o r w o r k h e re . Show S olution

P o in ts E , F a n d G a r e o n 0 O s u c h th a t Z E F G = 48° and ZG E F = 7 8 c

(a ) F in d Z E G F .

T y p e y o u r s o lu tio n , n o te s a n d / o r w o r k h e re . Show Solution

(b ) F in d E G .

T y p e y o u r s o lu tio n , n o te s a n d / o r w o r k h e re . Show Solution

(c ) F in d E F G .

T y p e y o u r s o lu tio n , n o te s a n d / o r w o r k h e re . Show Solution

In t h e d ia g r a m , O is t h e c e n t e r o f t h e c ir c le a n d / A C B — 3 6 c

(a ) F in d Z D .

T y p e y o u r s o lu tio n , n o te s a n d / o r w o r k h e re . Show Solution

(b ) F in d A B .

T y p e y o u r s o lu tio n , n o te s a n d / o r w o r k h e re . Show Solution

(c ) F i n d / A B F . ( D o n o t a s s u m e E F || A B a t t h i s p o i n t . )

T y p e y o u r s o lu tio n , n o te s a n d / o r w o r k h e re . Show Solution

(d ) F i n d Z E F B . ( D o n o t a s s u m e E F || A B a t t h i s p o i n t . )

T y p e y o u r s o lu tio n , n o te s a n d / o r w o r k h e re . Show Solution

(e ) F in d C E

T y p e y o u r s o lu tio n , n o te s a n d / o r w o r k h e re . Show Solution

(0 P r o v e t h a t E F || A B

T y p e y o u r s o lu tio n , n o te s a n d / o r w o r k h e re . Show Solution

(g ) P ro v e th a t A D = BC.

T y p e y o u r s o lu tio n , n o te s a n d / o r w o r k h e re . Show Solution

In P r o b le m 1 2 .2 , w e c o m p u t e d / A B C in t h e d ia g r a m a t r ig h t f o r s p e c ific m e a s u r e s o f _ 4 C a n d B C .

W e fo u n d th a t / A B C = A C /2 .

(a ) P ro v e th a t / A B C = A C / 2 f o r a ll in s c r ib e d a n g le s f o r w h ic h t h e c e n t e r o f t h e c ir c le is o u t s id e
t h e a n g le a s s h o w n in t h e f ig u r e a t r ig h t . ( M a k e s u r e y o u s e e h o w t h i s p r o b le m is a lit t le d if fe r e n t
f r o m P r o b le m 1 2 .3 .)

T y p e y o u r s o lu tio n , n o te s a n d / o r w o r k h e re . Show S olution

(b ) P r o v e t h a t / A C B h a s m e a s u r e e q u a l t o o n e - h a lf m a jo r a r c A B .

T y p e y o u r s o lu tio n , n o te s a n d / o r w o r k h e re . Show Solution

In P r o b le m 1 2 .1 , w e s h o w e d t h a t a n a n g le in s c r ib e d in a s e m ic ir c le is a r ig h t a n g le . In t h is p r o b le m ,
w e t a c k le t h e o t h e r a n g le s t h a t a r e f o r m e d w h e n w e c o n n e c t t h e e n d p o in t s o f a d ia m e t e r o f a c ir c le

H in t

T y p e y o u r s o lu tio n , n o te s a n d / o r w o r k h e re . Show Solution

A r c s W X a n d Y Z o f O Q a r e c o n g r u e n t. P r o v e t h a t e ith e r W Y || X Z o r W Z || X Y .

T y p e y o u r s o lu tio n , n o te s a n d / o r w o r k h e re . Show Solution

B a c k in P r o b le m 9.5 o f C h a p t e r 9, w e c o n s i d e r e d t h e r e g u l a r 1 5 ‫־‬g o n A B C D E F G H I J K L M N O , a n d f o u n d X A C D a n d
X A D E . F in d a n o t h e r s o lu t io n w it h y o u r n e w k n o w le d g e a b o u t a n g le s a n d c ir c le s , b y c o n s id e r in g t h e c ir c u m c ir c le o f t h e
p o l y g o n t o f i n d X A C D and X A D E .

Type your solution, notes and/or work here. Show Solution


11.2 Area

P roblem s
Problem 11.6 4 Jump to Solution

In this problem we will develop a formula for the area of a circle with radius r.

(a) The circle in the diagram has radius r and is divided into 16 equal pieces by drawing 8
diameters. These pieces are called sectors. Rearrange the sectors to look very similar to
a quadrilateral whose area we can find.

(b) What is the length of the base of this ‘quadrilateral’ in terms of r?

(c) What is the height of this 'quadrilateral‫ ׳‬in terms of r?

(d) What is the area of this 'quadrilateral‫ ׳‬in terms of r?

Problem 11.7 4 Jump to Solution

Find the area of a circle with diameter 12.

Problem 11.8 4 Jump to Solution

A X Y Z is equilateral with side length 10. A circle is constructed with center X and radius 10, thus passing through Y and Z.
Find the area of sector Y X Z of the circle. (In other words, find the area of the portion of the circle bounded by radii X Y and
X Z and by arc Y Z )

Problem 11.9 4 Jump to Solution

Farmer Tim has 50 feet of fence. He wants to enclose a semicircular area adjacent to his barn, thus using his barn as one side
of the enclosure. What is the area of the space Farmer Tim can enclose?

Having tackled the ‘perimeter’ of a circle, we now turn to its area. While proving the formula for the area of a circle would require
far more advanced tools than we have now, we can develop an intuitive explanation for the formula.

Problem 11.6 t V

Find a formula for the area of a circle by cutting up a circle and rearranging the pieces to look a lot like a rectangle.

Solution for Problem 77.6: We start by cutting the circle into equal slivers, which we call sectors, by drawing equally spaced radii
as shown below. We then rearrange the sectors as shown in the second figure, thus forming a figure that resembles a
parallelogram. Finally, we can take half of one of the end sectors and slide it to the other end as shown in the final diagram. Now
we have a figure that strongly resembles a rectangle.

The 'width‫ ׳‬of this 'rectangle‫ ׳‬is clearly the radius of the circle, which we’ll call r. The ‫׳‬length’ of the 'rectangle' is half the
circumference of the circle, since the circumference of the circle is equally divided among the top and bottom of our ‘rectangle.‫׳‬
Hence, the ‫׳‬length' is ( 2 n r ) / 2 = 7rr. Since the area of a rectangle is its length times its width, the area of our ‫׳‬rectangle’ is
(n r ) ( r ) = n r 2.

This is not a rigorous proof, but it should be clear that if we cut the circle into more and more sectors and do the described
rearrangement, the ‘rectangle’ looks more and more like a real rectangle, z

We now know how to find the area of a circle.

Important: The area of a circle with radius r is n r 2.

Z
Let's use our newfound knowledge of how to find the area of a circle on a few problems.

Solution for Problem 77.7: Since the diameter is 12, the radius is 6. Therefore, the area is (62)7T = 36 tt. □

Problem 11.8 t V

A X Y Z is equilateral with side length 10. A circle is constructed with center X and radius 10, thus passing through Y and Z.
Find the area of sector Y X Z of the circle.

Solution for Problem 11.8: Our sector is shaded in the diagram. To find its area, we must determine what
portion the sector is of the whole circle. Since A X Y Z is equilateral, Z Y X Z — 60°. Since the whole circle
is 360°, this means that our sector is 6 0 °/3 6 0 ° = 1 /6 of the entire circle. The whole circle has area
n r 2 = 7r(102) = 1007T, so the sector has area (1007r)( 1 /6 ) = 507t/3 . □

Important: Following our solution to Problem 11.8, we can find the area of a
sector of a circle of radius r given the central angle of the sector.
Z In the figure to the right, Z A O B is (Z A O B )/3 6 0 ° of the whole
circle. Therefore, we have:

Area o f sector А О В =

You should not have to memorize this formula. Instead, understand that the area of a sector is simply related to the ratio of the
sector's angle to the number of degrees in a whole circle.

Problem 11.9 t V

Farmer Tim has 50 feet of fence. He wants to enclose a semicircular area adjacent to his bam, using his barn as one side of the
enclosure. What is the area of the space Farmer Tim can enclose?

Solution for Problem 11.9: We need to find the radius of the semicircle in order to find its area. We start with
a sketch of the problem. Our fence forms a semicircular arc, so the arc has length 50. Therefore, the
semicircle is half of a circle that has circumference 100. The diameter of such a circle is the circumference
divided byn, or 100/ tt. So, the radius of our semicircle is ( lOO/zr)/ 2 = (50/7r).

The semicircle therefore has an area that is 1 /2 the area of a circle with radius (50/7r), or ( l/2 ) ( 5 0 /7 ‫־‬r ) 2(7r) = 1250/7T square
feet. □

Concept: Start geometric word problems with a sketch.

Exercises

1 1 .2 .1 : t V

Find the area of a circle with diameter 18.

Type your solution, notes and/or work here. Show Solution

11.2.2: t V

Find the area of a circle that has circumference \2n.

Type your solution, notes and/or work here. Show Solution

1 1 .2 .3 : t V

Find the radius of a circle if its circumference is numerically equal to its area.

Type your solution, notes and/or work here. Show Solution

1 1 .2 .4 : t V

If a pizza that is 12 inches in diameter provides four full meals, how many meals are provided by a pizza that is 20 inches in
diameter?

Type your solution, notes and/or work here. Show Solution

1 1 .2 .5 : t V

Points A and В are on the circumference of © O such that Z A O B = 120° and О A = 1 2 .

(a) Find the area of ©O.

Type your solution, notes and/or work here. Show Solution

(b) Find the area of sector A O B .

Type your solution, notes and/or work here. Show Solution

11 .2 .6 : t V

Sector X Q Z of circle Q has area 30tt. Given that the whole circle has area 1007T, find Z X Q Z and Z X Z Q .

Hint

Type your solution, notes and/or work here. Show Solution

Source: HMMT t V

A man standing on a lawn is wearing a circular sombrero of radius 3 feet. Unfortunately, the hat blocks the sunlight so
effectively that the grass directly under it dies instantly. If the man walks in a circle of radius 5 feet, what area of dead grass
will result?

Hint

Type your solution, notes and/or work here. Show Solution

Sidenote: The ancient Greeks didn't have calculus, so how did they find the formula for the area
of a circle? While they didn't have calculus, they did have Archimedes, some of whose
' ' proofs came so close to calculus (which wouldn’t be developed until nearly 2000
years after his death) that some suggest he might be called the father of calculus.
Archimedes used two favorite tactics of the Greeks - contradiction (see Section 2.7
here) and comparing a desired area to that of a figure whose area is easy to find.

Archimedes claimed that a circle has the same area as that of a right triangle with the
circle’s radius as one leg and its circumference as the other leg. He did so by proving
that the area of the circle could be neither more nor less than that of the triangle. We’ll
show you his proof that the circle's area could not be greater than that of the triangle,
and leave you to walk in his footsteps and supply the proof for the other half.

Let the radius of the circle be r, the area of the circle be A, and the area of the triangle
be T. Then, we assume that A > T. As we saw in our discussion of calculating n,
we can use regular polygons with more and more sides to approximate a circle.
Specifically, at some point, the difference between the area of such a polygon, which
we’ll call P, and the area of the circle is less than A — T. Therefore, we have
A - P < A - T, s o T < P.

However, we showed in Section 9.3 here that the area of such a regular polygon is
one-half the product of its perimeter, p, and its apothem, a. Since the perimeter of the
polygon is smaller than the perimeter of the circle, we have p < 2 n r. Moreover, the
apothem of the polygon is clearly less than the radius of the circle. Therefore, we can
use p < 2 n r and a < r to write P = a p /2 < ( r ) ( 2 n r ) / 2 = T. So, we have both
T < P and P < T. This is impossible, so we have reached a contradiction. We can
thus conclude that our assumption A > T was false. See if you can complete the
proof by showing that it is impossible to have A < T . As you might guess, you’ll
have to consider a polygon circumscribed about the circle! Source: Journey Through
Genius by William Dunham

Extra! GggXyZ the Martian has been told by the Martian King to make a rope to lasso the
Earth. GggXyZ makes a rope that will fit snugly around the equator of the Earth. The
V Martian King decides he wants a little slack so he can put the rope around the Earth
more easily. He wants the rope to fit around the equator such that there is one inch
between the Earth and the rope all the way around the equator.

GggXyZ says this is no problem. She pulls out a short piece of rope and says she can
just add this little piece to her rope to make the rope long enough. The Martian King
laughs at her, saying she'll need miles of rope because the diameter of the Earth is
almost 8,000 miles. Who is right? How much rope does she need?
Funky Areas 11.3

Z Z S S ‫” ״״‬ ‫“ ' ׳״ ־‬ ‫“־‬ ‫« ״«« • ״ ■ * " ־ ״ ״ " ' ״‬ » « *« ‫ ־ » ־״ » ■ ־ * ״‬I‫ ־‬together ,0 find ,he ‫ ״‬, ,‫״‬

P roblem s
Problem 11.10
4 Jump to Solution
() is the center of the shown circle and O A
- 8. The shaded region between chord A B and the circle is
called a circular segment.

(a) Find the area of sector A O B .

(b) Find the area of A A O B.

(c) Find the area of the shaded region.

Problem 11.11

4 Jump to Solution
I have a barn that is a regular hexagon, as shown. Each side of the barn is 100 feet lono I tether m‫״‬

burro c ^ n g ia r e ! “ ' ' 5° » '‫“ ״‬ ‫ «״ ■״ “ " ״‬- « ‫ ׳״‬- ■ ‫“ ׳־ » ׳‬ ^ t o ZS, Z


(a) Imagine you are the burro. Sketch out the area in which you can graze. Pay close attention to
what happens when you try to go past point B towards point C.

<6> , « ‫ ׳‬w ‫ ״ ״ ״‬1 ‫״ ״ ״‬ ‫ ״‬,o ‫״‬ ‫״‬ ‫״ ״’ ״‬ •‫ ״ ״ **״‬- « . ‫» ״‬ « ‫״‬, . ‫ ־ ״‬. . . . , ‫« ״‬

Problem 11.12
Source: AMC 10 4 Jump to Solution

The shaded portion of the figure is called a lune. Given that A B = 1 C D = V 2 and that ‫־‬A B * ‫ ״‬rf O n
are diameters of the respective semicircles shown, we wish to find the area of The lune

(a) Let () be the center of the larger semicircle. Draw O A


and O B . How long are these segments?
What kind of triangle is A A O B ?
V2
(b) Find the area of circular segment A B (i.e., the unshaded part of the small
semicircle).
(c) Find the area of the lune.

Problem 11.13
4 Jump to Solution

‫ “־‬: i s K s s s s s a r ,h,o‫״־‬
‫ ״‬A“ ‫ ״‬- ‫״‬ -*- - ‫ « ״‬- -
P0W* " " * * “ * » r « r . C onnut A aro, « to each other and to 1 A

(b) What kind of triangles are A X A B and A Y A B ?

<c) ‫־‬T p S ‫ ^־‬r d S r ; r ‫־‬tr ;V S S : “ ‫■״״״< “ ־״״*־‬FM*‫״‬ ‫ ״׳״ “■״״״״‬- . toto‫״‬


(d) Find the shaded area.

‫׳״‬ >‫« ■ * * “׳‬ ‫ «■«׳‬- » » to ....... . . ‫ ־ ־‬.» ‫ ״ „•־» ״‬, . ,‫ ״‬.

Problem 11.10
t V

IT ‫־ ״ • ־ ״‬ « ‫ ״‬The otrodtar ‫״‬


areas are easy to find, so we have
Area o f c irc u la r segment A B
= A rea o f sector A O B - A rea o f A A O B

- ( £ ) *m - »
= 16tt - 32.

Concept:
and^ h / V 3' 6f problems are solved by expressing the funky area as sums
a n d M differences of pieces whose areas we can easily find. The first step should be

when weXwrote'n9 V ‫ ״ ' “ ״״‬terms ° f Simple areas' as we did in Problem 11.10

A rea o f c irc u la r segment A B


= A rea o f sector A O B - A rea o f A A O B .

Sometimes our region isn't already drawn for us.

Problem 11.11
t V

walks until part of the rope coincides with A B . The burro then is at point G on A § Since A B - 1(in

are * is S8me S',Uati° n ° n the ° ther side‫ ׳‬leadi" 9 s e c t o r H F I . Thus, our desled

A rea o f m a jo r sector H A G
4-A rea o f sector J B G
4-Area o f sector H F I .

^ G f s 6 ? /3 6 0 2‫ ״‬- ° i / 6 CafUSe A f iS 9 re9Ular heXa9° n)‫ ־‬We haVe Z J B ° = 180° 60° = 120° ‫ ־‬Therefore sector
/3 6 0 - 1 /6 of a circle with radius 50 feet. Hence, its area is ( l/6 ) ,r ( 5 0 2) = 1250tr/3 square feet

On the other end of ,he burros range is sector H F I , which is exactly the same as sector J B G , so it contributes 1250rr/3 ft‫ ־‬to

( s T ? - 1 2 0 ‫ )׳‬S ' y- e2hA V? e T ; betWeen maj° r arC ° H and radii^ and M . Since / . F A B = 120“ the sector is
(360 120 )/3 6 0 - 2 /3 of a circle. The radius of this sector is 150 feet, so its area is (2/3)«-(150a) = 15000*■
Combining these three parts gives us a grazing area of

i,n n p T , n / 125Û7T \ 47500‫״‬


ioUUUtr + 2 ;— j = — ------ square feet.

Once again, notice that one of our key steps is to


tackle. express our desired region in terms of not-so-funky pieces we know how to

Problem 11.12
Source: AMC 70 t V
The shaded portion of the figure is called a lune. Given that A B = 1 C D
- V 2 . and that A B and C D
are diameters of the respective semicircles shown, find the area of the lune

“ ‫ ״ >" ״‬- « ‫״ «״ ״ ״‬ “ to« • ‫ ׳« *«׳‬to‫־־« ׳־'״«׳‬to‫ ״ ־‬.4 « ,‫״־״‬

A rea o f lune

= A rea o f sm all sem icircle - A rea o f c irc u la r segment A B . .ÿ O ^


To get the area of the circular segment, we start by connecting its endpoints to the center, O. of its circle.

A rea o f circ u la r segment A B


= A rea o f sector A O B - A rea o f A A O B ,
we have

A rea o f lune —A re a o f sm a ll sem icircle


- A rea o f circ u la r segment A B
—A re a o f sm a ll sem icircle
- (A re a o f sector A O B - A rea o f A A O B )
= A re a of sm all sem icircle
- A rea o f sector A O B 4- A rea o f A A O B .

To find the area of sector A O B , we need/ A O B . Since C D = s/2, we have

O C = O A = O B = O D = y /2 /2 .

Seeing ,hose v/2s makes us think of 45-45-90 triangles. We check and see that indeed

A O 2 + B O '2 = 1 /2 + 1 /2 = 1 = A B 2,

s o / A O B is a right angle since the sides of A A O B satisfy ,he Pythagorean Theorem.

AOB 7 90“ /360°e= U 4 ° (r ' " 6I The ° f the T " SemiCirC'e iS 1 /2 ' 50 its area - - d / 2 ) 7 2 = tr/8 . Sector
[A O B ] = ( A O ) ( O B ) / 2 = 1/4 , so^e^a ve ^ rad'US S° itS ^ i$ ( 1/ 4) 7r ( V ^ / 2 ) 2 = tt/ 8. Finally,

A rea o f lune = — - — + 1 = 1
8 8 4 4*

Again, look at our key step -


expressing our funky area (the lune) in terms of not-so-funky areas (semicircle, sector, triangle).

Sidenote:
You might have noticed that the area of the lune in Problem 1112
n equals the area of A A O B . This isn't a coincidence! Moreover, the
discovery that the area of the lune described in Problem 11.12
equals the area of A A O B led to one of the earliest notable proofs
in math history.

tAheTldediSmeaS‫״‬ tded *ai‫ ״‬to square ,he circle (to 'square


he circle means construe, a square with the same area as a given circle). However
the Greek mathematician Hippocrates succeeded in squaring the lune usinq the
insight that the area of the lune of Problem 11.12 equals the area of ,he triang^w Jh
the lune s d im e ter and the center of the larger semicircle as vertices. See if you can
walk ,‫ ״‬his footsteps by both re-creating ,he proof, and by performing “ e
construction. Source: Journey Through Genius by William Dunham

Problem 11.13
t V
Circle A passes through B and circle B passes through A. Given that A B
shaded region common to both circles. — 6, find the area of the

So,‫ ״ ״‬on for Problem 11.13: We have a funky circular area, so we start by drawing some segments to cut

Specifically, * ^ * 3 radiUS ° f ° ne (° r bo,h) of the circles■ * them are equal in length.

A X = X B = AB = AY = YB = 6

Shaded Area

= A rea o f sector X B Y 4‫ ־‬A re a o f sector X A Y - A rea o f X A Y B .

We subtract [ X A Y B \ because when we add ,he sectors, we include the overlap, [ X A K B ] twice. We could also have written

Shaded A re a = [ X A Y B ] 44 ‫(־‬A re a o f c irc u la r segment X B ) <

5 ™ ‫ * ־' ? ״‬. “ '‫ ’ “ ״‬m m ‫ ״ "' « ״ ״ ״ ״' “ ״ ” * ״‬k« ‫ »׳ » ״ > ■«'* « ״‬.« ‫ ״ ״‬. ‫ « ״'« ״‬. . . . i „ w i t o

/ X B Y = / X B A + / Y B A = 120°,
to . area of ^ c t w X B Y is ( 120“ /3 6 0 * )„ ‫ ( ״‬$2) , 12 . 5« „ * , „ , ‫ „ ״‬, „ „ ) , ‫״‬
‫ ״‬, ‫ « ״‬. „ . t o , . , to .„ 6 . ‫ ־‬,‫ ־‬. » ,‫־ ״‬,‫ ״‬. Each 6 , , ‫ «״‬V 5 / 9 ‫ ־ ־‬V3. , ‫ ־‬, X A Y B ] 18 ‫ ־‬2 ( 9 ^ / ‫ ־‬J lT j,

Shaded A rea = 247T - 18^3.

Concept:
i E S T 7 ° ‫ ־‬IVin9 mU',iple CirCleS' connectin9 the centers can be helpful In
problems mvolving intersecting circles, connecting the intersection points to the
centers (and to each other) is often useful.

Exercises

I ' T A 6 a' ea ° ! the shaded re9'on 9 iven that O is the center of the circle, / A O B = 120°, and the radius
of the circle is 6.

Hint

Type your solution, notes and/or work here.


Show Solution

» ‫ ״‬, ™ t o ' , “ 1‫ * ״‬,<l ‫ » " ״׳‬9 ™ » • ‫ ״ ־‬a o, » . ‫׳‬,gion t o , „ , » , ‫ ־‬, ‫« ־‬ , o, to , M ‫ ״ ״‬,‫ ״ ״‬,

Type your solution, notes and/or work here.


Show Solution

Type your solution, notes and/or work here.


Show Solution

‫־ ״‬ ‫“ ־‬ - - ‫“ ״‬ -1 ‫״‬ ° °< - . ‫• ״‬to x B C D . B

Lj B

Type your solution, notes and/or work here.


Show Solution

Hint

Hint &
Type your solution, notes and/or work here.
Show Solution

a s ? side l e n 9 t h 4 ‫ ׳‬T w o q u a r , e r ‫־‬c i r c , e s a n d a d i a 9 0 n a l a r e d r a w n

Hint

Hint '‫־‬-‫־‬J B

Hint

Type your solution, notes and/or work here.


Show Solution

Extra!
?92U iStaheCT eltn aPP; ° Ximati0n ° f 355/113 is an even better approximation. In
V approximation ™,hematician Ramanujan provided the uncanny

192
y 92 +
22‫״‬
How close is this to 7r? (And how in the world did he find this?)
12.2 Angles Inside and Outside Circles
A line that intersects a circle in two points is called a secant. In this section we'll explore relationships between the angles
formed by intersecting secants and the arcs of circles these angles cut off.

P roblem s
Problem 12.7 4 Jump to Solution

Our goal in this problem is to find Z A E B given that A B = 40" and D C = 80:

(a) Draw A D to create inscribed angles. Find Z D A E and Z A D E .

(b) Find Z A E D and Z A E B .

(c) How is Z A E B related to A B and C D ?

Problem 12.8 4 Jump to Solution

In this problem we will find a relationship between an angle inside a circle and the arcs the angle

intercepts. In the diagram, let Q R = x and P S = y.

(a) Draw P Q and find Z P Q T and Z Q P T in terms of x and/or y.

(b) Find, with proof, Z Q T R in terms of x and y.

Problem 12.9 4 Jump to Solution

In this problem we will find / . D A E given D E = 110° and B C = 20°.

(a) Draw B E to create inscribed angles. Find Z B E C and Z D B E .

(b) Find Z D A E .

(c) Can you find a general relationship that must hold among / A , B C , and D E ? (In other
words, what if we replace 110° and 20° with x and y?)

Problem 12.10 4 Jump to Solution

Given that X V = b and Y W = a (these are arc measures, not lengths), prove that
Z Z = { b - a ) / 2.

Problem 12.11 4 Jump to Solution

Given that A D = 40°, A B = 120°, and D C = 100°, find Z B P C and Z B E C .

Problem 12.12 4 Jump to Solution

In the figure, Z F = 58°, Z W = 20°, and X Z 1 W X


Y
(a) Find ZY V O V .

(b) FindZZXV.

Problem 12.13 Source: Mandelbrot 4 Jump to Solution

w ^ r ^ 1 ^
Z P meets the circle at four points as shown. The arcs W X , W Z , X Y , and Y Z are in order
from smallest to largest, and each of the last three is 42° larger than the arc listed before it.
Find Z P .

Hint

Having found a general formula for inscribed angles, we wonder how angles formed by secants are related to the arcs they
intercept. We start, of course, with a specific example.

Solution for Problem 12.7: We try to put what we just learned about inscribed angles to work by

drawing A D to create some inscribed angles. We have Z D A C = C D / 2 = 40° and

Z A D B = A B / 2 = 20° Therefore,

Z A E D = 180° - Z E A D - Z E D A = 120°,

so, Z A E B = 180° — Z A E D = 60°. (Note that we could have seen that Z A E B = 2 0 °+ 40°
since it is an exterior angle of /Z A E D .) □

With that basic case as a guide, we re ready to tackle finding a general formula.

Problem 12.8 t V

Chords Q S and P R intersect at T . Given that Q R = x and P S = y, find a formula for Z Q T R in terms of x and y.

Solution for Problem 12.8: We form inscribed angles by adding P Q to our diagram. Since Z Q P R is

inscribed in QR, Z Q P R = x /2 . Similarly, Z P Q S = y/ 2. Since Z Q T R is an exterior angle of


A P Q T , we have

ZQ TR = ZP Q T + ZQ P T =
2

Important: The measure of the angle formed by two intersecting chords


is the average of the measures of the arcs intersected by the
Z chords. For example, in the diagram at right, we have

P S + QR a + t/
ZPTS = ZQTR =
2 2 ‘

We’ve tackled angles on a circle and angles inside a circle. You know where we're headed now: outside the circle.

Solution for Problem 12.9: We succeeded with inscribed angles before, so we try using them
again here by drawing BE. We have Z B E C = 2 0 °/2 = 10° and
Z D B E = 1 1 0 7 2 = 55° Thus,

Z A B E = 180° - Z D B E = 125°,

and we have

Z D A E = 180° - Z A B E - Z B E C = 180° - 125° - 10° = 45°.

Note that we could have seen that Z D B E is an exterior angle of A A B E , so Z D B E = Z B A E 4‫ ־‬Z B E C . As before, this
gives us

Z B A E = Z D B E - Z B E C = 45°.

Now we have a clear path to prove the general formula.

Solution for Problem 12.10: We draw Y V and have Z Y V W = a /2 and


Z X Y V = b/2. Since Z X Y V is an exterior angle of A V Y Z , we have
z x y V = Z Z + Z K K Z . Therefore,

ZZ = ZXYV - ZYVZ =
2
i

Important: Two secants that meet at a point outside a


circle form an angle equal to half the difference
Z of the arcs they intercept. For example, in the
diagram we have

b —a
ZZ =

Let's solve a few problems with our new relationships.

Solution for Problem 12.11: Since all the arcs together in a circle must sum to 360°, we have

B C = 360c 120° - 40° - 100° = 100°.

Therefore, we have

B C 4- A D
ZBEC

BC - AD
ZBPC

Not all problems can be solved in one or two steps. Here's an example in which we have to do a little more work.

Solution for Problem 12.12: We can’t find the angles we want right away, but we
have a circle with our angles, so we go hunting for measures of arcs. Since

Z X W Z = 20°, we know that X Z = 40°. Since Z K = ( W V - X Z ) / 2, we


have

W V - 40°
58° =

Solving for W V gives W V = 156°. Now we can find ZYVO V:

ZW O V = v = 1560 + 400 = 98 ‫־‬ ‫ ׳‬+.

We can use VZ to find ZZXV, so we focus on VZ. Since W V Z = 2 Z Z X W = 180° we have

V Z = 180° - W V = 24°. Therefore, Z Z X K = 12°. □

Concept: Exploration can help unravel problems whose solutions require several steps. In
I geometry problems, effective exploration often requires drawing an accurate diagram
I and finding any angle measures or segment lengths you can. When you find any new
information, you should label your diagram with your new information, as we did in
the solution to Problem 12.12. This helps you see when you have the information you
need to solve the problem.

Problem 12.13 Source: Mandelbrot t V

Z P meets the circle at four points as shown. The arcs W X , W Z , X Y , and Y Z are in order from
smallest to largest, and each of the last three is 42° larger than the arc listed before it. Find Z P .

‫ ־ ־‬Ö ,

Solution for Problem 12.13: We start by labeling our diagram with the given information.
X X
We let our smallest arc have measure x, and we are given that the other three arcs are
x 4 4 2 ° ‫־‬, x 4 8 4 ° ‫־‬, and x 4 1 2 6 ° ‫־‬. We could find x if we wanted to, but we don't have to,
because

XY - WZ (x 4 84°) - (x 4 42°)
ZP = = 21c

Just because we use a variable in a problem doesn't mean we have to find its value. However, the variable did make finding the
solution easier.

Concept: When you can't find any more lengths or angles in a problem but you still haven’t
solved it, try assigning a variable to one of the lengths or angles. Then, find other
lengths or angles in terms of that variable. Finally, label your diagram with everything
you find - this will make it easier to see when you have enough information to solve
the problem.

Exercises

1 2 .2 .1 : t V

In the diagram at right, A B = 40°, B C = 103°, and C D = 83°. Find Z A E D .

Type your solution, notes and/or work here. Show Solution

12.2 .2 : t V

Find the following in the diagram at right if X Y = 78°, X V = 32°, and Z Y W Z = 30.5°.

(a) YZ

Type your solution, notes and/or work here. ■ Show Solution I

(b) KZ

Type your solution, notes and/or work here. Show Solution

1 2 .2 .3 : t V

Points P, Q, R, and S are in that order on © O . P R and Q S meet at Z. Given that P Q = 2Q R = 3R S = ASP, find
ZQZR.

Hint

Type your solution, notes and/or work here. Show Solution

1 2 .2 .4 : t V

Given that X Y = 75°, X Y Z = 117° and Y Z W = 173° in the diagram at right, find
the following:

(a) ZYPZ

Type your solution, notes and/or work here. Show Solution

(b) ZYOZ

Type your solution, notes and/or work here. Show Solution

(c) ZXYYV

Type your solution, notes and/or work here. Show Solution

(d) ZO YVZ

Type your solution, notes and/or work here. Show Solution

1 2 .2 .5 : t V

OR 4 OTS
Prove that Z P O R = -------- ---------in the diagram at right.

Hint

Type your solution, notes and/or work here. Show Solution


11.4 Summary

Definitions:
■ The set of all points that are the same distance from a given point is a circle. The
given point is the center of the circle, and the fixed distance is the radius.

■ We often refer to a circle by its center using the symbol 0 : © O refers to a circle
with center O.

■ A line that touches a circle in a single point is tangent to the circle, while a line that
hits a circle in two points is a secant. A segment connecting two points on a circle
is a chord, and a chord that passes through the center of its circle is a diameter.

■ The portion of a circle that connects two points on the circle is an arc, which we

denote with the endpoints of the arc: M N is the shorter arc that connects M and
N.

■ The perimeter of a circle is called the circle's circumference.

Important: If a circle has diameter d and radius r, then:

Z ■ d = 2r.

■ The circumference of the circle is nd, or 2nr.

■ The area of the circle is n r2.

Definitions: A portion of a circle cut out by drawing two radii of the circle is called a sector of the
circle. A portion of a circle between a chord and the arc of the circle connecting the
endpoints of the chord is a circular segment of the circle.

Important: We can find the area of a sector of a circle of radius r given the
central angle of the sector. In the figure to the right, since /.AOD
Z is (Z .A O B )/3 6 0 of the whole circle, we have:

( ZAOB\
Area of sector A O B r 2n.
V 360° )

We can find the area of a circular segment by first finding the area of the sector
formed by drawing radii to the endpoints of the segment's chord, then subtracting the
area of the triangle with these radii and the chord as sides.

P roblem Solving S trateg ies

Concepts:
■ In complicated problems involving a circle, try connecting the center of the circle
to important points on the circle.

■ Start geometric word problems with a sketch.

■ Nearly all funky area problems are solved by expressing the funky area as sums
and/or differences of pieces whose areas we can easily find. The first step should
be clearly expressing the funky area in terms of simple areas.

■ In problems involving multiple circles, connecting the centers can be helpful. In


problems involving intersecting circles, connecting the intersection points to the
centers (and to each other) is often useful.

Extra! .2
7T _ J_ J_ J_ J_ J_ J_ J_ J_
V i ‫ ־‬F + 2i + 3ï + ? + 5ï + f f + 7ï + 8ï +
Review Problems

1 1 .1 4 : t V

Circle O has radius 3 \/3 . Points A and Z are on the circumference of the circle such that Z A O Z = 90:

(a) Find the area of ©O.

Type your solution, notes and/or work here. Show Solution

(b) Find the circumference of ©O.

Type your solution, notes and/or work here. Show Solution

(c) Find the length of A Z .

Type your solution, notes and/or work here. Show Solution

(d) Find the area of sector A O Z .

Type your solution, notes and/or work here. Show Solution

(e) Find the area of circular segment A Z .

Type your solution, notes and/or work here. Show Solution

1 1 .1 5 : Source: MATHCOUNTS t V

A giant earth-mover has rubber circular tires 11.5 feet in diameter. Given that there are 5280 feet in a mile, how many
revolutions does each tire make during a 6‫־‬mile trip? (Answer to the nearest full revolution.)

Type your solution, notes and/or work here. Show Solution

1 1 .1 6 : t V

Find the length of a 78° arc of a circle that has radius 14.

Type your solution, notes and/or work here. Show Solution

1 1 .1 7 : t V

Find a formula that expresses the area, A, of a circle in terms of its circumference, C.

Type your solution, notes and/or work here. Show Solution

1 1 .1 8 : t V

Radius O A of © O is a diameter of ©7i. Radius O B of © / i is a diameter of © C .

(a) Find the ratio of the area of © C ‫ ׳‬to the area ©O.

Type your solution, notes and/or work here. Show Solution

(b) Find C A / O A .

Type your solution, notes and/or work here. Show Solution

1 1 .1 9 : t V

A 36° arc of © C is 24?r units long. Find the circumference and the area of the circle.

Type your solution, notes and/or work here. Show Solution

11.20: t V

Regular hexagon A B C D E F is inscribed in © O with radius 6. What is the ratio of the circumference of the circle to the
perimeter of the hexagon?

Flint

Type your solution, notes and/or work here. Show Solution

11.21 Source: MATHCOUNTS t V

On each side of a right triangle a semicircle is constructed using that side as a diameter. Flow many square centimeters are in
the area of the semicircle on the hypotenuse of a right triangle if the areas of the semicircles on the legs of the triangle are 36
and 64 square centimeters?

Type your solution, notes and/or work here. Show Solution

1 1 .2 2 : Source: MATHCOUNTS t V

What is the number of square centimeters in the area that is not shaded in the diagram at right? The
radius of the large semicircle is 1 centimeter, the radius of the small circle is 0.5 centimeters, and the
length of the longer leg on the right triangle is 3 centimeters.

Type your solution, notes and/or work here. Show Solution

1 1 .2 3 : Source: ARML t V

The larger circle at right has radius 1.5 times the smaller circle. Compute the ratio of the partial ring
A B C D to the area of sector B O C .

Type your solution, notes and/or work here. Show Solution

1 1 .2 4 : Source: MATHCOUNTS t V

A rectangle that is 12 inches long and 5 inches wide is inscribed in a circle. What is the area of the region that is inside the
circle but outside the rectangle?

Type your solution, notes and/or work here. Show Solution

1 1 .2 5 : Source: AMC 10 t V

Charlyn walks completely around the boundary of a square whose sides are exactly 5 km long. From any point on her path she
can see exactly 1 km horizontally in all directions. What is the area of the region consisting of all points Charlyn can see during
her walk?

Type your solution, notes and/or work here. Show Solution


12.3 Tangents

Figure 12.2: A Tangent Line

A line is tangent to a circle when it touches the circle in only one point. For example, line m is tangent to the circle at point A in
the diagram above. In this section we explore how a tangent line is related to the circle it touches.

Line m passes through point B on a circle with center O. In this problem, we will first
show that if m is tangent to the circle, then m A O B . We do so by showing that if
Z O B C is not 90°, then m must hit the circle a second time. Then, we will show that
if m -L O B, then m must be tangent to the circle.

(a) Suppose Z O B C < 90° as shown at right. If so, there must be some other line
through O besides t> è that is perpendicular to m. Let this line hit m at point D.
What kind of triangle is A O B D ?
(b) Show that O D < O B. Why does this mean that m must hit the circle a second time?

(c) Show that if Z O B C > 90°, then m must hit the circle a second time. Conclude that if m is
tangent to the circle at B, then m _L O B.
(d) Our first three parts show that a tangent line must be perpendicular to the radius drawn to
the point of tangency. Now, we show that if a line m passes through a point B o n a QO
such that m _L O B, then m must be tangent to the circle.

Suppose line m meets QO at B such that m A O B . Let X be any point on m besides B.


Consider A O B X and show that O X > O B Does this prove that m can't hit the circle a second time if m A OB?

4 Jump to Solution
Problem 12.15

Our goal in this problem is to find Z Z A B given that line m is tangent to the circle at A and

AB = 80°.

(a) Draw diameter A C and find BC.

(b) Find Z B AC , Z Z A C , and Z Z A B .

(c) What relationship holds between Z Z A B and AB?

Problem 12.16

In this problem we will explore how an angle between a secant and a tangent is related to the arcs

intercepted by the angle. In the diagram, let B D = 110° and B C = 40°. Ä B is tangent to the
circle at B.

(a) Draw B D and find Z B D C and Z D B A .

(b) Find ZA.

4 Jump to Solution
Problem 12.17

In this problem we investigate the relationship between the lengths of two tangent
segments to a circle from the same point, as well as how to find the angle between
them. P% and P i l are tangent to the circle, which has center O.

(a) Prove that A P O B = A POA.

A X B -A Y B
(b) Prove Z P =

4 Jump to Solution
Problem 12.18

Point P is 25 centimeters from the center, O, of a circle with radius 10 centimeters. Point Q is on the circle such that P Q is
tangent to the circle. Find PQ.

We start this section discovering a highly useful fact about tangents

t V
Problem 12.14

Given that line m is tangent to © O at B, prove that radius O B is perpendicular to m. Conversely, prove
that if a line m passes through point B on © O such that O B J_ m, then m is tangent to © O at B.

Solution for Problem 12.14: This is one of those ‫׳‬obvious‫ ׳‬facts in geometry that isn't so easy to prove rigorously. Here’s one
'obvious' solution that doesn't quite work:

Bogus Solution: Since the diagram is the same on both sides of O B , the angles on both sides of B
must be the same. These angles must also add to 180°, so they must be right angles.

The problem with this solution is that the statement ‫׳‬the diagram is the same on both sides of O B ’ essentially aS®U™e®
are trying to prove. Moreover, the solution is not clearly written - what do we mean by the angles on both sides of B . We
to find another way.
We let C be a point on m besides B. After trying and failing to directly show that
Z O B C = 90°, we decide to try proving it can’t be anything else. First, we consider
Z O B C < 90°. Since Z O B C is not 90°, there must be some other point D on line
m such that O D A m. Hypotenuse O B of right triangle A O B D must be the largest
side of A O B D so O D < OB. Since O B is a radius, this means that line m must
go inside the circle if Z O B C < 90°. Therefore, the line will hit the circle a second
time. Specifically, consider the point F on m such that B D = D F (i.e., D is the
midpoint of B F ). By SAS Congruence we have A B D O = A FD O , so O B - O F . . . . .
Therefore, point F is on QO. so if Z O B C < 90°, then line m cannot be tangent to the circle since it will hit the circle twice.
Similarly, if Z O B C > 90°, then Z O B E < 90°, and we can use exactly the same logic to show that m cannot be a tangent line
in this case either. Therefore, if m does not form a 90° angle with O B , it cannot be a tangent line.
Unfortunately, we are not finished yet! We have shown that a line that makes^any angle besides
B X 90° can't be a tangent line, but we haven't shown that a line that makes a 90° angle is always a
tangent. It might even be true that it's impossible for there to ever be a tangent line. Fortunately, its
not, and we'll prove it by considering line m at left, which hits the circle at B such that m A O B .
q I jo show that m is a tangent line, we must show that it does not hit the circle again.

We consider some point on line m besides point B, and call this point X . Since O X is the
hypotenuse of A O B X , we must have O X > OB. Hence, point X must be outside the circle,
since its distance from O is greater than the circle's radius. This is true for any point on line m besides point B, so all points on
m besides B are outside the circle. Thus, line m is indeed tangent to the circle at point B. □

We therefore have a very non-obvious proof for our 'obvious' fact that:

Important: A line tangent to a circle is perpendicular to the


radius drawn to the point of tangency. Conversely, a
z line drawn through a point on a circle that is
perpendicular to the radius drawn to that point must
be tangent to the circle. For example, for the
diagram at right, we can write:

B (1' js perpendicular to radius O B iff f / 7‫ '׳‬is tangent to circle O.

(Recall that iff is an abbreviation for "if and only if".)

You’ll be seeing this 'radius perpendicular to a tangent at the point of tangency again and again.

t V
Problem 12.15

Line rn is tangent to a circle at point ,4. Given point B on the circle such that AB = 80°, find the acute angle formed by chord
A B and line m „

Solution for Problem 12.75: We start by drawing diameter AC^since we knowrthis line is

perpendicular to m at point A. Since A C is a semicircle, B C = 180° - A B = 100°

Thus, Z B A C = B C / 2 = 50°. Finally,

Z Z A B = Z Z A C - Z B A C = 90° - 50° = 40°.

Note that Z Z A B = A B /2 . □

As an Exercise, you can follow these same steps to show:

cut off.
We now turn to angles formed by a tangent and a secant.

t V
Problem 1 2 . 1 6 ______________________________________________________

In the diagram, S D = 110° and B C = 40°. P P is tangent to the circle at B. Find Z B AD.

Solution for Problem 12.76: Since this looks so much like problems involving an angle
between two secants (such as Problem 12.9), we try the same approach by drawing B D .

We have Z B D C = B C / 2 = 20° and Z X B D = B D /2 = 55°.


exterior angle of AABD, we have A
Finally, ZXBD
since is an
Z X B D = Z B A D + Z B D C , so
Z B A D = Z X B D - Z B D C = 55° - 20° = 35°.


Unsurprisingly, the angle formed by a tangent and a secant has the same relationship to the arcs the angle intercepts as an angle
formed by two secants does.

Important: The angle formed by a tangent and a secant is half the


difference of the intercepted arcs. For example,
z
B D -B C
ZB AD =

Next we consider two tangents from the same point to the same circle.

t V
Problem 1 2 . 1 7 __________________________________ __________________________

Segments ‫־‬PA and ‫־‬P B are tangent to the same circle r t A and B, respectively. Prove that PA PB, and that / . A P B

equals half the difference of major arc AB and minor arc AB.

Solution for Problem 12.77: We start by drawing the radii to the points of tangency, thus
forming right angles. We also draw P O to complete a pair of right triangles. Since
A O = OB, we have A O A P = A O B P by HL Congruence. Therefore, P A = P B .

We can find Z A P B in a variety of ways. One is to use quadrilateral P A O B to note that

Z A P B = 360° - Z P A O - Z A O B - Z O B P

= 180° - Z A O B

= 180° - A Y B .

Since A X B = 360° - A Y B , we also have

A X B -A Y B 360° - 2 A Y B = 18Qo _ = ZAPR

The information about the angle between the tangents is unsurprising, as it's essentially the same as the angle between two
secants. However, the information about the two tangents from the same point is new, and is one of the most useful tange
facts:

Important: As shown in the diagram at right, we can draw two


tangent segments to a circle from a point outside the
z circle. These tangents are always equal to each other in
length.

We'll finish with a very common problem involving tangents.

t V
Problem 1 2 . 1 8 __________________________________________________________ ________ ____________________

Point P is 25 centimeters from the center, O, of a circle with radius 10 centimeters. Point Q is on the circle such that PQ is
tangent to the circle. Find PQ.

Solution for Problem 72.78: We start by drawing a radius to the point of tangency as shown. Since Q
‫־‬P Q is tangent to the circle, A P O Q is a right triangle. Thus, O P 2 = O Q 2 + P Q 2, so

P Q = yJOP2 - O Q 2
= V252 - 102

= >/ 5* ( 5‫ ־‬- 2‫)־‬


= 5v/52 - 2'2

= 5 n/ 21 centimeters.

Notice how a little factoring makes our arithmetic easier, a

E x e rc is e s

Point T is outside QO. X is on QO such that T X is tangent to QO. The radius of QO is 6 and T X - 12. Find 7 O.

Show Solution
Type your solution, notes and/or work here.

Generalize Problem 12.15 by proving that the angle formed by a tangent and a chord that has the point of tangency as an
endpoint is half the arc intercepted by the angle.

Show Solution
Type your solution, notes and/or work here.

Y and Z are on QG and X is outside QG such that X Y and X Z are tangent to QG. Given that Z Y X Z - 51°, find the

measure of minor arc Y Z .


Show Solution
Type your solution, notes and/or work here.

Given a circle and a point outside the circle, it is intuitively clear that there are exactly two lines through the point that are
tangent to the circle. However, intuitively isn't good enough - we’re going to prove it. Let the point be P and the circle be QO.

(a) Consider the circle with diameter ‫־‬O P. Call this circle C. Why must C hit QO in at least two different points?

Show Solution
Type your solution, notes and/or work here.

(b) Why is it impossible for C to hit QO in three different points?

Hint
Show Solution
Type your solution, notes and/or work here.

(c) Let the points where C hits QO be A and B. Prove that ZPAO — ZPBO 90

Show Solution
Type your solution, notes and/or work here.

(d) Prove that P A and P B are tangent to QO.

Show Solution
Type your solution, notes and/or work here.

(e) ★ Now for the tricky part - proving that these are the only two tangents. Suppose D is on QO such that PD is tangent to
QO. Why must D be on C?

Show Solution
Type your solution, notes and/or work here.

(f) Why does the previous part tell us that P I and P ê are the only lines through P tangent to QO?

Show Solution
Type your solution, notes and/or work here.

QO is tangent to all four sides of rhombus A B C D , A C — 24, and A B — 15.


(a) Prove that ‫־‬A C and B D meet at O (i.e., prove that the intersection of the diagonals of A B C D is the center of the
circle.)

Hint
Show Solution
Type your solution, notes and/or work here.

(b) What is the area of QO?

Hint

Show Solution
Type your solution, notes and/or work here.

Extra! President James A. Garfield used the diagram at right to


prove the Pythagorean Theorem. How did he do it?
V
Hint

Extra! The feet of the perpendiculars from any point on the


circumcircle of a triangle to the sides of the triangle are
V collinear, as shown at right. The line through these points is
called the Simson line. See if you can prove that these three
points are always collinear!

Hint
Challenge Problems

11.26: t v

Three circles of radius 12 lie in a plane such that each passes through the center of the other two.
Find the area common to all three circles.

Hint

Type your solution‫ ״‬notes and/or work here. Show Solution

11.27: t V

When my car has wheels with a diameter of 24 inches, its speedometer reports the correct speed of my car. I recently replaced
my 24-inch wheels with 28-inch diameter wheels. I didn't change my speedometer, however. When the speedometer tells me
the car is going 40 miles per hour (and I'm driving with my 28-inch wheels), how fast is my car really going?

Hint

Type your solution, notes and/or work here. Show Solution

11.28: Source: MATHCOUNTS t V

Two congruent circular coins, A and Z are touching at point P. A is held stationary while Z is rolled around it one time in
such a way that the two coins remain tangent at all times. How many times will Z revolve around its center?

Hint

Type your solution, notes and/or work here. Show Solution

11.29: Source: MATHCOUNTS t V

The number of centimeters in the perimeter of a semicircle is numerically the same as the number of square centimeters in its
area. What is the number of centimeters in the radius of the semicircle?

Hint

Type your solution, notes and/or work here. Show Solution

11.30: t V

A cross-section of a river is a trapezoid with bases 10 ft and 16 ft and slanted sides of length 5 ft. At this section the water is
flowing at 7r mph. A little ways downstream is a dam where the water flows through 4 identical circular holes at 16 mph. What
is the radius of the holes?

Hint

Type your solution, notes and/or work here. Show Solution

11.31 t V

Given that A A B D in the diagram at right is a right triangle with B D = 8 and A B = 8 \/3 , find the
total area of the shaded regions.

Hint

Type your solution, notes and/or work here. Show Solution

11.32: Source: HMMT t V

A circle of radius 3 passes through the center of a square with side length 2. Find the positive difference between the areas of
the nonoverlapping portions of the figures.

Hint

Type your solution, notes and/or work here. Show Solution

11.33: t V

Six 120° arcs are drawn centered at the vertices of regular hexagon A B C D E F . Each arc
connects two vertices of the hexagon as shown in the diagram at right. Given that A F = 6, find the
total area of the shaded regions.

Hint

Type your solution, notes and/or work here. Show Solution

11.34: Source: AMC 10 t V

The number of inches in the perimeter of an equilateral triangle equals the number of square inches in the area of its
circumscribed circle. What is the radius of the circle?

Type your solution, notes and/or work here. Show Solution

11.35: Source: HMMT t V

A circle has two parallel chords of length x that are x units apart. If the part of the circle included between the chords has area
2 + 7r, find x.

Type your solution, notes and/or work here. Show Solution

11.36★ : t V

Quarter circles are drawn centered at each vertex of square A B C D as shown at right. Given that
A B = 12, find the area of the shaded region.

Hint

Hint
B
Hint

Type your solution, notes and/or work here. Show Solution

11.37★ : t V

The figure at right shows a quarter-circle of radius 1, with .4 on B D such that Z A O D = 30°. What
must the distance O X be such that the region bounded by A X , B X , and A B occupies half the area of
the quarter circle?

Hint

Hint

Hint

Type your solution, notes and/or work here. Show Solution

Extra! In the third century B.C., the Greek scholar Eratosthenes (the same man who invented the prime number
sieve) was the director of the great library at Alexandria. At the time, other Greek scholars knew that the
V Earth was round, but had no idea how large it was. Eratosthenes learned of a deep well in Syene, a city
south of Alexandria, whose bottom was sunlit at noon every year on the summer solstice. This meant
that the sun was directly overhead. This gave Eratosthenes an ingenious idea: He could use this
observation to measure the size of the Earth.

He assumed that since the sun was so far away, the rays coming from the sun were virtually parallel.
Eratosthenes then measured a shadow (some say of a stick, some say of a tower) in Alexandria on the
summer solstice, and found that the rays made an angle of approximately 7.2° with the object.
Eratosthenes then required one more piece of data: the distance between Alexandria and Syene, which
he estimated at 800 km.

Since there are 360° in a circle, Eratosthenes estimated the circumference of the Earth to be

360°
800 km x -------= 40,000 km.
7.2°

This is remarkably close to the actual figure, which is approximately 40,075.16 km.

Eratosthenes used an ancient measure of distance called a "stadium," whose actual length in today’s
units is uncertain. Also, Syene is 729 km, not 800 km, from Alexandria and slightly southwest instead of
due south. Furthermore, Syene is not exactly on the Tropic of Cancer (where the sun is directly overhead
on the summer solstice), but 55 km north. So a number of errors seem to have fortuitously cancelled out
to give Eratosthenes such an accurate figure.

What is remarkable about this experiment is not the accuracy of Eratosthenes’ estimate, but rather the
conception of the experiment itself. As Dave Hanes, professor of astronomy at Queen’s University, puts it
on his website,

The critical point is that Eratosthenes recognized the nature of the problem, found a method,
and was able to derive an answer that was correct in spirit in the sense that he correctly
deduced that the Earth was an immense body that was very much larger in extent than the
then-known lands of the Mediterranean basin, the home of Greek civilization at the time. The
sense of the discovery is the wonderful thing, not the mere accident that the numerical value
was also correct.

Even if Eratosthenes had been off by a factor of two or more in his calculations, it would not have taken
away from his brilliant insight into taking a simple observation of the sunlit well and deducing the
circumference of the Earth, demonstrating the power of a simple idea.
12.4 Problems

P ro b le m s
4 Jump to Solution
Problem 12.19

A quadrilateral is said to be a cyclic quadrilateral if a circle can be drawn that passes through all four of its vertices. Prove that
if A B C D is a cyclic quadrilateral, then / . A A Z C = 180°.

4 Jump to Solution
Problem 12.20

In the figure, P C is tangent to the circle and P I ) bisects /.C P E . Furthermore,

C D = 70°, D E = 50°, and / D Q E = 40°. In this problem we determine the p


measure of the arc from A to E that does not include point C .

(a) Find B C .

(b) Find / C P B .

(c) Find A B .

(d) Finish the problem.

4 Jump to Solution
Problem 12.21

Three congruent circles with radius 1 are drawn inside equilateral A A B C such that each circle is
tangent to the other two and to two sides of the triangle. Find the length of a side of A A B C .

Hint

4 Jump to Solution
Problem 12.22

3c fr is tangent to both circle O and circle P. Given that O P — 40, and the radii of circles O
and P are 31 and 7, respectively, find X Y.

Hint

Hint

4 Jump to Solution
Problem 12.23

Median A M of A A B C has length 8. Given that B C = 16 and A B = 9, find the area of A A B C .

Hint

Hint

4 Jump to Solution
Problem 12.24

The diagram shows the incircle of A A B C . X , Y , and Z are the points of tangency where the
incircle touches the triangle. In this problem we will find an expression for A Z in terms of the
sides of the triangle.

(a) Find equal segments in the diagram and assign them variables.

(b) Let A B = c, A C = b, and B C = a. Use your variables from the first part to write
equations that include these lengths.

(c) Solve your resulting equations for A Z .

Hint

We'll now apply our knowledge of circles, angles, and tangents to more challenging problems and develop some useful
geometric concepts.

t V
Problem 12.19

A quadrilateral is said to be a cyclic quadrilateral if a circle can be drawn that passes through all four of its vertices. Prove that
if A B C D is a cyclic quadrilateral, then / A 4‫ ־‬Z C = 180°.

Solution for Problem 72.79: Since/ A and Z C are inscribed angles, we have

BCD BAD BCD A BAD 360°


/ A + / C = —— A — r— = ---------- o-----------= ‫ ־‬T 18 0 = ‫־‬

Important: A quadrilateral is a cyclic quadrilateral if a circle can be drawn that passes through all
four of its vertices. Such a quadrilateral is said to be inscribed in the circle. The
Z opposite angles of any cyclic quadrilateral sum to 180°.

You'll be seeing a lot more of cyclic quadrilaterals when you move into more advanced geometry.

t V
Problem 12.20

___ A
In the figure, ‫־‬P C is tangent to the circle and P D bisects / C P E . If C D = 70°.

'D E = 50°, and / D Q E = 40°, then determine the measure of the arc from A to h /?- Q D
4d _
that does not include point C.

Solution for Problem 72.20: We can't directly find the desired arc, so we try finding whatever we can. First, we note that since

Z D Q E is the average of D E and C D , we have C D = 30°. Now that we have C D , we can find Z C P D .

Z C P D = ( C D - C B ) / 2 = 20°.

Since D P bisects Z C P E , we know Z D P E = Z C P D = 20°. Because Z D P E = ( D E - A B ) /2 , we find A B = 10°.


Now that we have all the other arcs of the circle, we can find our desired arc by subtracting from 3bU .

360° - 50° - 70° - 30° - 10° = 200°.

Concept: Label all the given information in the problem, and all the information you find as you
find it. This will help you discover new facts about the problem. For example, your
I final diagram in the last problem might look like the figure below. Notice that our
various arc measures and angles are marked, and that the equal angles at P are
marked equal.

3(У/
P 20е в \Q
2...-‘ - ' ‫ ־‬a \4a
\D
ia|

Problem 1 2 . 2 1 ____________________________ _________________________________________________ ______________ _____

Three congruent circles with radius 1 are drawn inside equilateral A A B C such that each circle is tangent to the other two and
to two sides of the triangle. Find the length of a side of l \A BO .

Solution for Problem 12.21: We need to create simple figures to work with, so we start by A
connecting the centers of our circles and drawing radii to tangent points. (Note that
connecting the centers of the circles is the same as drawing radii to where the circles are
tangent.) Since W X Y Z is a rectangle (because X W = Y Z , X W || Y Z , and
X X W Z = 90°), we have W Z = X Y = 2 . Hence, we need only find C IV to finish,
since B Z is the same as C W .

We draw ‫־‬C X to build a right triangle and note that this segment bisects / A C B because
circle X is tangent to both A C and B C (and hence its center is equidistant from them).
Since X X C W = ( / A C B ) / 2 = 30°, A C X W is a 30-60-90 triangle. Thus,
C W = X W V 3 = v/3. Finally, we have

B C = B Z 4‫ ־‬Z W A W C = 2 43 /\ 2 ‫־‬.

Concept: When you have tangents in a problem, it's often very helpful to draw radii to points ot
tangency to build right triangles. When you have tangent circles, connect the centers.
I (In fact, if you have multiple circles in a problem, connecting the centers will
sometimes help even when the circles aren't tangent.)

t V
Problem 1 2.2 2

X is on circle O and Y on circle P such that A 1' is tangent to both circles. Given that O P = 40, and the radii of circles О and
P are 31 and 7, respectively, find X Y .

Solution for Problem 12.22: We start with the usual segments to draw: X Y , the radii
to points of tangency, and the segment connecting the centers. We still don't have a
right triangle to work with, but we do know that radii O X and P Y are both
perpendicular to tangent XY as shown. Since both O X and P Y are perpendicular
to the same line, they are parallel. We make a right triangle and a rectangle by drawing
a line through P parallel to X Y . Z X = P Y = 7, so O Z = O X — Z X == 24.
Since O P = 40, we have P Z = 32 from right triangle A O Z P . Since X Y P Z is a
rectangle, we have X Y = Z P = 32, so the length of the common tangent is 32.

X Y is called a common external tangent of the two circles. As an Exercise, you'll find the length of the common internal tangent,
too. □

t V
Problem 12.23

Median A M of A A B C has length 8. Given that B C = 16 and A B = 9, find the area of A A B C .

Solution for Problem 72.23: When we draw the figure and label all our lengths, we see that
B M = A M = C M = 8. Therefore, a circle centered at M with radius 8 goes through all three
vertices of A A B C . Since ‫־‬B C is a diameter of this circle, Z B A C is inscribed in a semicircle and
therefore must be a right angle. So, A C = y / B C * - A B * = 5 ^ 7 . A A B C is a right triangle, so
its area is half the product of its legs:

(A B )(A C ) (9 )(5 v 45 _ (7‫׳‬y/7


[A B C \ =
2 2 2

Using this same reasoning, we can also prove this important fact:

Important: If the length of a median of a triangle is half the length of the side to which it is drawn,
the triangle must be a right triangle. Moreover, the side to which this median is drawn
Z is the hypotenuse of the right triangle.

also look to this problem for some important problem solving techniques:

Concept: When stuck on a problem, always ask yourself ‫׳‬Where have I seen something like this
before?‫ ׳‬In Problem 12.23, we have a median that is half the side to which it is drawn.
I This should make us think of right triangles, since the median to the hypotenuse of a
right triangle is half the hypotenuse. Then we go looking for right triangles.

Concept: Always be on the lookout for right triangles.

I
t V
Problem 1 2 . 2 4 ______________________________________________________ ______________________ _

Let Z be the point where the incircle of A A B C meets A B . Find A Z in terms of the sides of A A B C .

Solution for Problem 12.24: We start by labeling the equal tangents from the vertices as
shown in the diagram. We want to relate x to the sides of the triangle, so we write the
sides of the triangle in terms of x, y, and 2 . We let the sides of the triangle be A B = c,
A C = 6, and B C = a and we have:
AB = c = xA y
AC = b = x A z
BC = a = y A z

We want x in terms of a, b, and c. Adding the three equations will give us x A y A z,


which we can use with y A z = a to find x:
aA bA c
a A b A c = 2 (x A y A z) so x A y A z = ‫־‬ •

We can then subtract the equation y A z = a from x A y A z = (a A b A c ) / 2 to find

a A bA c
x = ----------------- a = s — «,
2

where s is the semiperimeter (half the perimeter) of the triangle. Similarly, y = s - b and z = s - c. z

Concept: Symmetric systems of equations can often be easily solved by either multiplying all
the equations or adding them.
I

Important: The lengths from the vertices of A A B C to


the points of tangency of its incircle are given
Z as follows:

= AY = s —a
BZ = BX = s —b
CX = CY = s —c
where A B = c, A C = b, and B C — a, and
the semiperimeter of A A B C is s. s-b X 5‫ ־‬C

E x e rc is e s
Problems 12.4.3, 12.4.4, 12.4.5, and 12.4.8 are very important relationships that you'll be seeing again in your study of more
advanced geometry. Be sure to pay special attention to them.

Is every quadrilateral cyclic?

Show Solution
Type your solution, notes and/or work here.

Prove the following about cyclic quadrilaterals:

(a) A cyclic parallelogram must be a rectangle.

Show Solution
Type your solution, notes and/or work here.

(b) A cyclic rhombus must be a square.

Show Solution
Type your solution, notes and/or work here.

(c) A cyclic trapezoid must be isosceles.

Show Solution
Type your solution, notes and/or work here.

Prove that if median ‫־‬X M o f A X Y Z has half the length of Y Z , then A X Y Z i s a right triangle with hypotenuse Y Z .

Show Solution
Type your solution, notes and/or work here.

A B C D is a cyclic quadrilateral. Prove that /A C B — / A D B .

Show Solution
Type your solution, notes and/or work here.

Quadrilateral A B C D in the diagram at left is a circumscribed quadrilateral, meaning that it is


circumscribed about a circle, so the circle is tangent to all four sides of A B C D . Show that
AB a C D = B C A AD.

Hint

Show Solution
Type your solution, notes and/or work here.

Does every quadrilateral have an inscribed circle (a circle tangent to all four sides), as A B C D does in the previous problem?

1. III([•,

Type your solution, notes and/or work here.

In the figure at right, A B is tangent to both © O and © P . The radius of © O is 8, the radius of © P
is 4, and O P = 36. find A B . (A common tangent like A B is sometimes called the common
internal tangent of two circles.)

Hint

Show Solution
Type your solution, notes and/or work here.

Prove that the inradius of a right triangle with legs of length a and b and hypotenuse c is (a + b - c )/2 .

Solution
Type your solution, notes and/or work here.

The five circles in the diagram are congruent and A B C D is a square with side length 4. The four
outer circles are each tangent to the middle circle and to the square on two sides as shown. Find the
radius of each of the circles.

Hint

Hint

Show Solution
Type your solution, notes and/or work here.
12.5 Construction: Tangents
Now you'll use your newfound tangent knowledge to construct tangents to circles.

Problems
Problem 12.25 4 Jump to Solution

Given a circle with center 0 and point A on the circle, construct a line through A that is tangent to the circle.

Problem 12.26 4 Jump to Solution

Given a circle with center 0 and point P outside the circle, construct a line through P that is tangent to the circle.

Problem 1 2.25 t V

Given a circle with center 0 and point A on the circle, construct a line through A that is tangent to the circle.

Solution for Problem 12.25: Make sure you see why this is not a correct solution:

Bogus Solution: We place our straightedge on the paper so that it goes through the point A, then we
slowly turn it, keeping it through A, until it only touches the circle in one point.

This Bogus Solution is not a Euclidean construction - it's just a sketch. We need to be able to prove our construction works.
Here, the proof would essentially be 'Our line touches the circle once because we say so.' That's not good enough. We need to
use geometric principles to prove our line is tangent, not just eyeball judgment.

We know how to construct perpendiculars, and we know that a tangent is perpendicular to the
radius drawn to the point of tangency. Putting these together, we have a pretty straightforward
construction. We draw b À , which includes the radius to point A, then construct the line through A
perpendicular to O A . This line is our tangent line.

Our construction of this tangent line is indicated in the diagram at right. We draw a circle centered
at A, which hits OA at B and C. We then draw two pairs of intersecting arcs with the same radius
centered at B and C to find X and Y. 3^ Y is the perpendicular bisector of BC, and is therefore
our tangent line. □

So we can handle a tangent through a point on a circle; how about one through a point outside the circle?

Problem 1 2 .2 6 t V

Given a circle with center O and point P outside the circle, construct a line through P that is tangent to the circle.

Solution for Problem 12.26: Unfortunately, we can't simply construct a radius, then draw a
perpendicular line, because we don’t know which radius to draw! So, feeling a little stuck, we
look for a simpler problem to solve. But that doesn't get us anywhere, either - there's not any
obvious simpler problem that will help us with this one. Therefore, we draw the completed
diagram and look for clues how we might possibly construct it given only the circle and point
P. We go ahead and include the radius to the point of tangency because right angles are very
useful, and we know how to construct perpendicular lines. If we can construct point A, where
the tangent meets the circle, we’ll be set. The only seemingly useful information we have is that Z.OAP is a right angle, i.e.,
A O A P is a right triangle. So, we consider what we know about right triangles.

The circumcenter of any right triangle is the midpoint of its hypotenuse. Since A must be on the
circumcircle of A O A P , and A O A P must be a right triangle with hypotenuse OP, we know >k
that A is on the circle that has O P as a diameter. Therefore, to find A, we construct M, the
midpoint of OP. We then draw a circle with center M and radius MO. Where this circle hits © 0
is the point A where our tangent touches the circle. n is the tangent. (And the other point
where the new circle hits © 0 gives us the other tangent to © 0 through P ) □

Concept: When tackling challenging construction problems, try drawing a completed diagram
I (without straightedge & compass construction). Then use observations about your
' completed diagram to figure out how to construct it with straightedge & compass
alone.

Now that we've mastered tangent constructions, and have an intriguing new construction-finding strategy, let’s try an even more
challenging problem.

Problem 1 2.27 t V

Given two nonintersecting circles with different radii, construct a line that is tangent to both.

Solution for Problem 12.27: This construction looks pretty complicated, so we start
from the completed diagram. We should probably add radii to the tangent points to
give us perpendicular lines to work with. As we draw these, we realize, ‘Hey, we’ve done
a problem very much like this!‫ ׳‬We think back to Problem 12.22, which we solved by
creating the diagram shown at right.

Maybe this diagram will give us a clue. If we can find a way to construct any of X, Y,
or even Z given only circles © 0 and © P , we'll be able to construct a line tangent to
both circles.

At this point, we might think:

Bogus Solution:
Pick a point Z, draw Ô Ê until it hits © 0 . That
gives us point X. Then construct a line through X
that is tangent to © P using the construction from
Problem 12.26.

Unfortunately, this badly fails. If we just take any old Z, we might get a diagram like the one we want, but we probably won't,
is indeed tangent to © P , since we constructed it to be tangent to © P . However, this line is not necessarily tangent to © 0 , as
our diagram clearly points out.

So, we can’t just pick any point to be Z (or X, or Y). And we can’t just keep trying different
possibilities for Z until we get one that works! We must find a specific one that we can prove
works. Looking again at our ,working backwards’ diagram we drew first, we see that Z is the vertex
of the right angle of right triangle A O Z P . As we saw in Problem 12.26, this means that Z is on
the circle that has O P as a diameter.

So, starting from just our circles and their centers, we at least have a
first step: we construct a circle with O P as a diameter. Point Z must
be on this circle. If we can find another circle or line with Z on it, we'll be
finished. Thinking one more time about Problem 12.26 and our ‘working
backwards’ diagram, we note that O Z is the difference between the
radii of the two circles. Call this difference d. We can construct a
segment with length d since we know the radii of the circles! After
constructing a segment with length d, we can draw a circle with center
0 and d as the radius. Since Z must be d away from 0 , it must be on
this little circle. Therefore, the intersection of this circle and the circle
with O P as diameter is Z\ Extending 0 ^ to hit © 0 gives us X, and
drawing the line perpendicular to 0 2 through X gives us the common tangent.

Of course, you’ll get to prove that this final line is tangent to © P as an Exercise. □

E x e rc is e s

12.5.1: t V

Given two intersecting lines, construct a circle that is tangent to both of them.

Type your solution, notes and/or work here. Show Solution

12.5.2: t V

Prove that our construction in Problem 12.27 does produce a line that is tangent to both circles.

Type your solution, notes and/or work here. Show Solution

12.5.3★ : t V

In Problem 12.27, we constructed the common external tangent of two circles. How can we construct a common internal
tangent of two nonintersecting circles (i.e., a line tangent to both circles that intersects the line connecting the centers of the
circles)?

Hint

Hint

Hint

Type your solution, notes and/or work here. Show Solution


12.6 Summary

Definition: An angle formed by two chords of a circle is inscribed in the angle it cuts off.

Important: ■ Any angle inscribed in a semicircle is a right angle.

Z ■ An inscribed angle equals 1 /2 the measure of the arc it


intercepts. For example,

PR
ZPQ R =

Any two angles that are inscribed in the same arc are equal.

Important: The measure of the angle formed by two intersecting chords


is the average of the measures of the arcs intersected by the
Z chords. For example, in the diagram at right, we have

P S 4‫ ־‬Q R x 4‫ ־‬y
ZPTS = ZQTR =

Important: Two secants that meet at a point outside a


circle form an angle equal to half the difference
Z of the arcs they intercept. For example, in the
diagram we have

b —a
ZZ =

Important: A line tangent to a circle is perpendicular to the


radius drawn to the point of tangency. Conversely, a
Z line drawn through a point on a circle that is
perpendicular to the radius drawn to that point must
be tangent to the circle. For example, for the
diagram at right, we can write:

liC ' is perpendicular to radius O B iff B O is tangent to circle O.

(Recall that iff is an abbreviation for ‫״‬if and only if‫״‬.)

Important: An angle formed by a tangent and a chord with


the point of tangency as an endpoint equals one-
Z half the arc intercepted by the angle. For
example, in the figure at right, line m is tangent
to the circle at A, so

AB
Z.Z A B =

The angle formed by a tangent and a secant is half the


difference of the intercepted arcs. For example,

B D -B C
ZB AD =

Important: As shown in the diagram at right, we can draw two


tangent segments to a circle from a point outside the
Z circle. These tangents are always equal to each other in
length.

Important: A cyclic quadrilateral is a quadrilateral that can be inscribed in a circle. The opposite
angles of any cyclic quadrilateral sum to 180°.
Z
Important: If the length of a median of a triangle is half the length of the side to which it is drawn,
the triangle must be a right triangle. Moreover, the side to which this median is drawn
Z is the hypotenuse of the right triangle.

Important: The lengths from the vertices of A A B C to


/A1
the points of tangency of its incircle are given
Z as follows: / \ 5‫ ־‬3
s‫־‬
P - \ y
AZ = AY = s —a zr \
BZ = BX = s -b
CX = CY = s -c
1
1

where A B = c, A C = 6, and B C = a, and


the semiperimeter of A A B C is s. B s -b X S -c c

Important: The length of the inradius of a right triangle with hypotenuse of length c and legs of
lengths a and b is
Z
a + b —c

P ro b le m S o lv in g S tra te g ie s

Concepts:
When trying to prove something or find some general rule, investigating a few
sample cases can be an excellent guide.

When you can’t find any more lengths or angles in a problem but you still haven’t
solved it, try assigning a variable to one of the lengths or angles. Then, find other
lengths or angles in terms of that variable. Finally, label your diagram with
everything you find - this will make it easier to see when you have enough
information to solve the problem.

When you have tangents in a problem, it's often very helpful to draw radii to points
of tangency to build right triangles. When you have tangent circles, connect the
centers. (In fact, if you have multiple circles in a problem, connecting the centers
will sometimes help even when the circles aren't tangent.)

When stuck on a problem, always ask yourself ,Where have I seen something like
this before?’

Always be on the lookout for right triangles.

Symmetric systems of equations can often be easily solved by either multiplying


all the equations or adding them.

When tackling challenging construction problems, try drawing a completed


diagram (without straightedge &: compass construction), then using observations
about your completed diagram to figure out how to construct it with straightedge
&: compass alone.
Review Problems

12.28: t V

Find x in each of the following

(a)

Type your solution, notes and/or work here. Show Solution

(b)

Type your solution, notes and/or work here. Show Solution

(c)

Type your solution, notes and/or work here. Show Solution

(d) In the diagram, Z A C B = 10°.

Type your solution, notes and/or work here. Show Solution

(e )

Type your solution, notes and/or work here. Show Solution

(0
Q

Type your solution, notes and/or work here. Show Solution

12.29: t V

Find x in each of the following. (Assume that all lines that look like tangent lines are tangent lines.)

(a)

Type your solution, notes and/or work here. Show Solution

(b)

Type your solution, notes and/or work here. Show Solution

(c)
Q

Type your solution, notes and/or work here. Show Solution

(d)

Type your solution, notes and/or work here. Show Solution

(e) In the diagram, Z J L I = 43°.

Type your solution, notes and/or work here. Show Solution

12.30: t V

A C is a diameter of © O in the diagram at right. Given Z A C B = 58° and B D = C D , find the


following:

(a) AB.

Type your solution, notes and/or work here. Show Solution

(b) Z B AC.

Type your solution, notes and/or work here. Show Solution

(c) ZDBC.

Type your solution, notes and/or work here. Show Solution

12.31 t V

Points A, B, C, and D are on a circle in that order, and A C and B D meet at E. Given that A B = 34°, C D = 102°,

D A = 109°, find Z B E C .

Type your solution, notes and/or work here. Show Solution

12.32: t V

Prove that )OIrit B on circle O such tl perpendicular to O B , then m must be tangent to the
circle. (Try

Hint

Type your solution, notes and/or work here. Show Solution

12.33: t V

Use your knowledge about inscribed angles to explain why the circumcenter of an obtuse triangle is outside the triangle.

Type your solution, notes and/or work here. Show Solution

12.34: t V

Point T is on minor arc R S of ©O . Given that Z R O S = 53°, find Z R T S .

Type your solution, notes and/or work here. Show Solution

12.35: t V

K T meets © 7 1at Y. Given that Y K = 6 and Y T = 2, find the length of the tangent segment from K to Q)T

Type your solution, notes and/or work here. Show Solution

12.36: t V

Point Y is on © F and X outside © F such that X Y is tangent to F . * x t ' intersects © F at A and B such that X A < X B .

Given that Y B = 134°, compute the following:

(a) AY.

Type your solution, notes and/or work here. Show Solution

(b) ZYXF.

Type your solution, notes and/or work here. Show Solution

(c) ZXYA.

Type your solution, notes and/or work here. Show Solution

12.37: t V

A T and B T are tangent to © O as shown at right. Given A T — 6 and A B — 120 , find


O T , Z A T B , and [O A T B \

Type your solution, notes and/or work here. Show Solution

12.38: Source: MATHCOUNTS t V

12.39: t V

A B C D E F G H I J K L is a regular dodecagon. Use the circumcircle of the dodecagon to find the following:

(a) ZABC.

Type your solution, notes and/or work here. Show Solution

(b) ZACD.

Type your solution, notes and/or work here. Show Solution

(c) ZA D J.

Type your solution, notes and/or work here. Show Solution

(d) the acute angle between A F and B I.

Type your solution, notes and/or work here. Show Solution

12.40: t V

Given Z V W Z = 81° and Z W Z X = 38° in the diagram, find the following:

(a) ZKZZ.

Type your solution, notes and/or work here. Show Solution

(b) ZW YZ.

Type your solution, notes and/or work here. Show Solution

(c) ZZZK.

Type your solution, notes and/or work here. Show Solution

(d) ZXTZ.

Type your solution, notes and/or work here. Show Solution

(e) ZW XY.

Type your solution, notes and/or work here. Show Solution

12.41 t V

A B C D is a square with area 100. © O is tangent to all four sides of the square. Diagonal B D meets the circle at X and Y ,
with X closer to D than to B. Find D X .

Type your solution, notes and/or work here. Show Solution

12.42: t V

Right triangle A X Y Z and its incircle are shown at right. As shown, Z is 6 units from the point Y
where the incircle touches X Z , and Y is 3 units from the point where the incircle touches X Y .
Find Y Z .

Type your solution, notes and/or work here. Show Solution

12.43: t V

Prove that the incircle of a given triangle is unique. (In other words, prove that there is only one circle tangent to all three sides
of a given triangle.)

Type your solution, notes and/or work here. Show Solution

12.44: t V

Circles ©^4 and Q B are tangent at point C. P is on (•)A and Q is on Q B such that P Q is Q
tangent to both circles. Given A C = 3 and B C = 8, find PQ.

Type your solution, notes and/or work here. Show Solution

12.45: t V

A circle is inscribed in a regular hexagon and another circle is circumscribed about the hexagon (i.e. it passes through all six
vertices of the hexagon). Find the ratio of the area of the smaller circle to the area of the larger circle.

Hint

Type your solution, notes and/or work here. Show Solution


Challenge Problems

12.46: t v

In the diagram at right, lines t and m are tangent to the circle. Prove that £ II m.

Type your solution, notes and/or work here. Show Solution

12.47: t V

A B C D is a parallelogram. B C extended past B meets the circumcircle of A A B D at


point E as shown at right. Prove that E D = C D .

Type your solution, notes and/or work here. Show Solution

Let A, B, C, and D be points on a circle w, in that order. Let P and Q be points on cj such that P A bisects / . D A B and Q C
bisects / B C D . Prove that P Q is a diameter of a;.

Type your solution, notes and/or work here. Show Solution

12.50: t V

Let the incircle of triangle A B C be tangent to sides B C , A C , and A B at points D, E, and F , respectively. Given that
/ A = 32°, find / E D E .

Hint

Type your solution, notes and/or work here. Show Solution

12.51 t V

A B is a diameter of © O and A X and A Y are chords of the circle such that A X = A Y . Prove that A B is the perpendicular
bisector of X Y .

Hint

Hint

Type your solution, notes and/or work here. Show Solution

12.52: t V

Find X Z in the right triangle described in Problem 12.42.

Hint

Type your solution, notes and/or work here. Show Solution

12.53: Source: MATHCOUNTS t V

The areas of two adjacent squares are 256 square inches and 16 square inches, respectively, and
their bases lie on the same line. What is the number of inches in the length of the segment that joins
the centers of the two inscribed circles?

Hint

Hint

Type your solution, notes and/or work here. Show Solution

12.54: Source: AMC 12 t V

Given a circle of radius 2, there are infinitely many line segments of length 2 that are tangent to the circle at their midpoints.
Find the area of the region consisting of all such line segments.

Hint

Type your solution, notes and/or work here. Show Solution

12.55: t V

Let C\ and C-2 be two circles that intersect at two points, .4 and B. Let P be the point diametrically opposite A on C\, and let Q
be the point diametrically opposite A on C2. Prove that P, B, and Q are collinear.

Hint

Type your solution, notes and/or work here. Show Solution

12.56: t V

Chord C E is a segment of the perpendicular bisector of radius O A of © O . A X is a diameter of ©O. Find / O C X .

Hint

Type your solution, notes and/or work here. Show Solution

12.57: Source: Mandelbrot t V

Three of the sides of A B C D are tangent to the circle shown at right. Given that A B = 6,
B C = 7, C D = 8, and A D = 9, find C Y + D X .

Hint

Type your solution, notes and/or work here. Show Solution

12.58: Source: HMMT t V

A room is built in the shape of the region between two semicircles with the same center and
parallel diameters such that the smaller semicircle is contained within the larger. The room is
shown at right. The farthest distance between two points with a clear line of sight is 12m. What is
the area in m2 of the room?

Hint

Type your solution, notes and/or work here. Show Solution

12.59: t V

The incircle of A A B C is tangent to A B at Z . Show that if the inradius of A A B C equals A Z , then the angle / B A C is a
right angle.

Hint

Type your solution, notes and/or work here. Show Solution

12.60: t V

The sides of right triangle A A B C all have integer lengths. Prove that the inradius of A A B C also has integer length.

Hint

Type your solution, notes and/or work here. Show Solution

12.61: t V

Let lj be the circumcircle of triangle A B C . Let A h B !, and C ! be the midpoints of arcs B C , C A , and A B , respectively, of
circle cj . In this problem we prove that the incenter of triangle A B C is the orthocenter of triangle A \ B \ C!.

(a) Let / be the incenter of A A B C . Why must aJ passthrough A\?

Type your solution, notes and/or work here. Show Solution

(b) Why must A A \ be perpendicular to B [C \ ?

Hint

Type your solution, notes and/or work here. Show Solution

(c) Combine the previous parts to conclude that the incenter of triangle A B C is the orthocenter of triangle A \ B \C \.

Type your solution, notes and/or work here. Show Solution

12.62★ : t V

Prove that quadrilateral A B C D is cyclic if / A 4‫ ־‬/ C = 180°. (Note that this is the converse of what we discussed in
Problem 12.19.)

Hint

Hint

Type your solution, notes and/or work here. Show Solution

12.63★ : Source: Mandelbrot t V

We are given points A, B, C, and D in the plane such that A D = 13 while A B = B C = A C = C D = 10. Find / A D B .

Hint

Hint

Type your solution, notes and/or work here. Show Solution

12.64★ : t V

Let P Q be a diameter of a circle and T be a point on the circle besides P and Q. The tangent to the circle through point Q
intersects ) yi ' at R, and the tangent through T intersects Q R at M . Prove that M is the midpoint of QR.

Hint

Hint

Type your solution, notes and/or work here. Show Solution

Extra! Most construction problems involve the two classical tools of geometry, the straightedge and compass.
These tools can solve an array of construction problems, many of which are discussed in this book, but
v perhaps even more interesting are construction problems that are impossible to solve with these tools.

Three famous classical construction problems that the ancient Greeks couldn't solve are listed below.

1. Squaring the Circle: Given a circle, construct a square of equal area.


2. Doubling the Cube: Given a cube, construct another cube with twice the volume of the first cube.
(According to Greek legend, the Athenians appealed to the Oracle for advice about a plague that was
ravaging their city. The Oracle's answer was to have their cube-shaped altar to the god Apollo doubled
in size. The Athenians then attempted to follow this advice by building a new altar that had twice the
side length of the original altar, but this had the effect of increasing the volume of the altar by a factor
of 28 = {‫־‬, not 2, and as a result, the plague worsened.)
3. Trisecting the Angle: Given an arbitrary angle, divide it into three equal angles.
We now know that the ancient Greeks couldn’t solve these problems because all three of these
constructions are impossible to perform with straightedge and compass.

Modern mathematicians proved these constructions are impossible by determining what lengths could
be constructed if we are given segments of lengths a, b, and 1. With these segments as a starting point,
the lengths a + b, a — b, nb, and a /b can be constructed. See if you can figure out how to construct ab
and a/b. Even \/a. can be constructed, as the diagram below suggests. (See if you can figure out how
this construction works!)

This means that any of the following lengths can be constructed:

!/3 + 8 / 7
-v/5 +

In other words, any length involving the integers and the arithmetic functions + , —, x, +, and \ T can be
constructed. Furthermore, these are the only lengths that can be so constructed. So how can this fact be
used to solve the three problems?

The first problem, squaring the circle, is equivalent to constructing \fF . However, it was shown by
Lindemann in 1882 that n is transcendental, meaning that n is not the root of any polynomial with integer
coefficients, which also implies the same for y/n. Since every constructible number is the root of some
polynomial with integer coefficients, we know that neither tt nor \ [ F can be constructed.

The second problem, doubling the cube, is equivalent to constructing \/2 . A branch of mathematics,
known as Galois theory, tells us that \f2 cannot be expressed using arithmetic functions and square
roots of integers. Hence, \J 2 cannot be constructed.

For the third problem, trisecting the angle, certain angles can be trisected. For example, given a straight
angle of 180 , a third of it would be 60°, which can be easily constructed. But can a 60° angle itself be
trisected? In other words, can a 20° angle be constructed? It can be shown that if x = cos 20°, then
SxA — 6x — 1 = 0. Again, Galois theory tells us that such an x cannot be expressed using arithmetic
functions and square roots. This counterexample of trisecting the 60° degree angle shows that trisection
of angles cannot be done in general.

Despite the fact that these problems have been conclusively solved for years, many amateur
mathematicians still attempt to find constructions for these problems and submit them to mathematical
journals, hoping for fame. All these attempts are completely futile.

The same Galois theory that rules out these constructions also tells us which regular polygons can be
constructed. The construction of an equilateral triangle and a regular hexagon are easy, but beyond that,
which other regular n-gons are constructible?

To answer this question, we introduce the Fermat numbers. The k lh Fermat number is F *. = 22* + 1, so
Fo = 3, F ! = 5, F -2 = 17, and so on. A Fermat prime is a Fermat number that is also prime. At the time
of writing, there are only five known Fermat primes, namely F 0, F h F2, F 3, and F 4.

A regular n-gon is constructible if and only if n is a power of 2 or is of the form

n = 2J 1
F ш /F m2 • F-* »ni

where j is an arbitrary non-negative integer and F mv F ma‫ ׳‬..., F m, are distinct Fermat primes. So the
regular polygon with 3 - 1 7 = 51 sides is constructible, but the regular polygons with 7 sides and 9 sides
are not. Gauss first proved that the regular 17-gon is constructible, and was said to have been so proud
of this fact that he requested that a regular 17-gon be etched into his tombstone.

It is surprising that either one of the straightedge or compass by itself can essentially do the same work
as both of them put together. The Mohr-Mascheroni Theorem states that any construction that can be
performed by straightedge and compass can be performed by compass alone. If the target of a
construction is a line, the compass alone cannot produce the line; however, it can produce two points on
the line, which defines the line correctly. Analogously, the Poncelet-Steiner Theorem states that any
construction that can be performed by straightedge and compass can be performed by straightedge
alone, as long as a single circle and its center are given.
Poncelet's Porism

Man's mind, stretched by a new idea, never goes back to its original dimensions. - Oliver Wendell Holmes

CHAPTER

I
I Three-Dimensional Geometry
So far in this book we’ve confined ourselves to zero dimensions (points), one dimension (lines and line segments), or two
dimensions (basically everything else in the book). However, we all know the world around us seems three-dimensional - that in
addition to left and right, and forwards and backwards, there's up and down.

In this chapter we step off the page and into the ,real world’ by considering three-dimensional objects. As we'll see, this isn’t
much of a step; much of our work in three-dimensional geometry will consist of reducing problems to two-dimensional situations
we already know how to handle.

14.1 Planes
As mentioned in Section 1.2, a plane is a ,flat’ two-dimensional surface that extends forever. For example, when your book is
completely flat, this page is part of a plane. In this section, we'll explore planes, as well as how lines and planes can intersect in
space.

P ro b le m s
Problem 14.1 4 Jump to Solution

We know that given any two points, we can draw exactly one line through the two points.

(a) Given two points, how many different planes pass through the two points?

(b) Given three points, is it always possible to find a plane that passes through all three points?

(c) Given any four points, is it always possible to find a plane that passes through all four points?

Problem 14.2 4 Jump to Solution

(a) When two lines intersect, their intersection is a point. When two planes intersect, what kind of figure is their intersection?

(b) Is it possible for two planes to never intersect?

(c) Can there be a line and a plane that never intersect?

(d) Remember that two lines are parallel if they are in the same plane but do not intersect. Can two lines in space be such that
they are not parallel but still do not intersect?

(e) Given any two intersecting lines, is there always a plane that contains both lines?

Problem 14.3 4 Jump to Solution

(a) How can we tell if a line is perpendicular to a plane? Give an example in the 'real world' of a line and a plane that are
perpendicular.

(b) How can we tell if two planes are perpendicular? Give an example in the ‘real world' of two planes that are perpendicular.

We start with a fundamental question about planes.

Problem 14.1 t V

How many points are needed to determine a unique plane?

Solution for Problem 14. V. If we only have two points, we can find a line through them, but there are
infinitely many planes that can pass through this line (and therefore pass through our initial two points).
An example is shown in the figure at right, in which three planes are shown passing through the same
line.

Given three or more collinear points (meaning they are all on the same line), we can still find
infinitely many planes that pass through all three points, but if we have three points that are
not collinear, then we have a triangle. The plane of this triangle is the unique plane that passes
through the three points, as shown at left.

Sidenote: Why do most telescopes have three legs? Why do photographers use a tripod, which
has three legs, to stabilize their cameras instead of something with four legs? The
D answer: three points determine a plane, so wherever you place a stand with three legs,
it will be stable.

While any three points are coplanar, meaning there is a plane through all three of them, it is easy to find
a set of four points that are noncoplanar. Three vertices of a triangle and a point not in the plane of the
triangle give us an example in which no plane passes through four given points. Such a situation is
depicted at right, where B C is dashed because it is in the ‘back’ of the figure. (We wouldn't be able to
see B C if this figure were rendered in three dimensions.) □

Important: Three noncollinear points determine a plane.

Z________________________
Now we consider how lines and planes in space can be related.

Problem 14.2 t V

(a) When two lines intersect, their intersection is a point. When two planes intersect, what kind of figure is their intersection?

(b) Is it possible for two planes to never intersect?

(c) Can there be a line and a plane that never intersect?

(d) Remember that two lines are parallel if they are in the same plane but do not intersect. Can two lines in space be such that
they are not parallel but still do not intersect?

(e) Given any two intersecting lines, is there always a plane that contains both lines?

Solution for Problem 14.2:

(a) Consider a standard rectangular room as shown at right, where E F G H is the ‫׳‬floor’ and
A B C D is the ‫׳‬ceiling'. The walls are portions of planes. Look at where a wall meets the floor,
and you'll see an example of two planes intersecting to form a line. Similarly, any two planes
that intersect form a line.

(b) In a typical room, the ceiling is a constant distance from the floor. This means that if the ceiling
and the floor extended forever in all directions, they would be planes that never intersect. Such planes are called parallel,
just as lines in the same plane that never intersect are called parallel. For example, in our figure above, A B C D and
E F G H are parallel planes.

(c) The intersection of a wall and the ceiling (line A B , for example) will never meet the floor (plane E F G I I ) . This is an
example of a line that is parallel to a plane.

(d ) Lines A $ and CT» do not intersect and are not in the same plane. Since they are not coplanar, they are not parallel. But they
still don't intersect! We call noncoplanar lines that do not intersect skew lines.

(e) Yes. Let X be the intersection point of the two lines, Y be a different point on one of the lines, and Z a different point on the
other line. These three points determine a plane. Clearly, both \ Y' and *X '1are in the plane because X Y and X Z are.

After dealing with parallels, you know what's next.

Problem 14.3 t V

(a) How can we tell if a line is perpendicular to a plane? Give an example in the 'real world' of a line and a plane that are
perpendicular.

(b) How can we tell if two planes are perpendicular? Give an example in the 'real world' of two planes that are perpendicular.

Solution for Problem 14.3: We’ll use our trusty room for our ‫׳‬real world’ examples.

(a) Suppose line k intersects plane V at point X . If k is perpendicular to every line in V that passes
through X , then we say that line k is perpendicular to the plane. The line of intersection
between two adjacent walls in a typical room is perpendicular to the ceiling and the floor of the
room. For example, A^J is perpendicular to A B C D .

(b) Suppose planes V and Q intersect in line m, and let O be a point on line m.
Consider lines j and k in V and Q, respectively, such that each of j and k is
perpendicular to m at point O. The angle between j and k, which is marked with
a 6 in the diagram, is the angle between the planes. We call such an angle
between planes a dihedral angle. If this angle is a right angle, the planes are
perpendicular.

In a typical room, a wall is perpendicular to the ceiling and to the floor. For
example, plane A B C D is perpendicular to plane B C G F in our ,room' above.

E x e rc is e s

14.1.1: t V

Line m is perpendicular to plane V at point X . Line k is in plane V and passes through X . Are lines m and k necessarily
perpendicular?

Type your solution, notes and/or work here. Show Solution

14.1.2: Source: AMC12 t V

Triangle P A B and square A B C D are in perpendicular planes at right. Given that P A = 3,


P B = 4, and A B = 5, what is P D ?

Type your solution, notes and/or work here. Show Solution

14. 1.5: t V

Two planes, M and A r are each perpendicular to a third plane, V. A ‫׳‬f and A f intersect in line m. Is line m perpendicular to V ?
(No proof necessary.)

Type your solution, notes and/or work here. Show Solution


Radical Axis Theorem

Power corrupts. Absolute power is kind o f neat. - John Lehman, Secretary of the Navy, 1981-1987

CHAPTER 13
1
i Power of a Point
In the previous chapter we explored the angles formed by lines that meet inside, outside, or on a circle. In this chapter, we explore
the lengths of segments that meet inside or outside a circle. Together, the relationships we will prove and use in this chapter are
given the lofty name ,Power of a Point.‫׳‬

13.1 What is Power of a Point?

P ro b le m s
Problem 13.1 4 Jump to Solution

Chords A B and C I ) meet at point X as shown. In this problem we will prove that

(.X A ) ( X B ) = ( X C ) ( X D ) .

(a) Draw A D and H C . Why is Z X D A = Z X B C ?

(b) Show that the two triangles in the diagram are similar. Use these similar triangles to prove that
X A /X C = X D /X B .

(c) Complete the proof that ( X A ) ( X B ) = ( X C ) ( X D).

(d) Is every chord of the circle that passes through X split into two pieces that have a product equal to (X A ) ( X B )? (In
other words, is there anything special about chord C D or will this proof work for every chord through X?)

Problem 13.2 4 Jump to Solution

Point P is outside a given circle and A is on the circle such that P A is tangent to the circle. C and
E are on the circle such that secants P C and P E intersect the circle again at B and D,
respectively. In this problem we prove that

P A 2 = (P B ) ( P C ) = (P D ) ( P E ).

(a) Draw A B and A C . Why does Z A C P = Z B A P I

(b) Use the angle equality from the first part to identify two triangles that are similar.

(c) Use the triangle similarity to prove P A 2 = ( P B ) ( P C ) .

(d) How can we show that ( P D ) ( P E ) = ( P B ) ( P C )?

Problem 13.3 4 Jump to Solution

Chords T Y and O P meet at point K such that T K = 2, K Y = 16, and K P = 2(K O ). Find O P .

We begin this section by proving the Power of a Point Theorem in two parts. First, we consider points inside a circle.

Problem 13.1 t V

G iven c h o rd s AB and C I) th a t m e e t a t p o in t X, p ro ve th a t

(X A )(X B ) = (X C )(X D ).

Solution for Problem 13.1: When we rearrange what we want to prove, we have
X A / X C = X D / X B . The ratios suggest we look for similar triangles, so we draw A D and B C to
form triangles. Since Z B and Z D are inscribed in the same arc, we have Z B = Z D . We also have
Z A X D = Z C X B , so we have A A X D ~ A C X B by AA Similarity. Therefore, we have
X A / X C = X D / X B , so ( X A ) ( X B ) = ( X C ) ( X D ) .

Note that there's nothing special about chords A B and C D . We can use the same exact approach as
above to show that each of this circle’s chords through X is divided into two pieces such that the
product of the lengths of these pieces equals ( X A ) ( X B). □

Having finished with points inside the circle, we turn to points outside the circle.

Problem 13.2 t V

P o in t /} is o u ts id e a g ive n c irc le a n d A is o n th e c irc le s u c h th a t PA is ta n g e n t to th e c irc le . C and E a re o n th e c irc le s u c h


th a t s e c a n ts PC ’ a n d P E in te rs e c t th e c irc le a g a in a t B and D, re s p e c tiv e ly . P ro ve th a t

P A 2 = (P B ) ( P C ) = (P D )(P E ).

Solution for Problem 13.2: We start as we did in the previous problem, drawing A B and A C to

make triangles. Since Z A C B = A B / 2 and Z P A B = A B / 2, we have A P A B ~ A P C A


by AA Similarity (since Z P is the same in both triangles as well). Therefore,
P A / P B = P C /P A , so P A 2 = ( P B ) ( P C ) .

Just as there was nothing special about the chords we looked at in the previous problem, there's
nothing special about secant P C here. By exactly the same reasoning, we have
{ P D ) ( P E ) = P A 2, so ( P D ) ( P E ) = ( P B ) ( P C ) . Notice that once again there's nothing special about the tangent and the
secants we have chosen except that they have point P in common. □

We can put the facts we've proved in the last two problems together in a single statement.

Important: Suppose a line through a point P intersects a circle in two points,


U and V. The Power of a Point Theorem states that for all such
Z lines, the product ( P U ) ( P V ) is constant. We call this product
the power of point P. For example, in the figure at right, applying
Power of a Point to X with respect to the circle shown gives

(X A )(X B ) = (X C )(X D ).

In the figure at left, the power of point P with respect to


the circle gives us

P A 2 = (P B )(P C ) = (P D )(P E ).

We think of Power of a Point whenever we have a problem


involving intersecting segments and a circle.

Let's try using Power of a Point on a few problems.

Problem 13.3 t V

C h o rd s TY and OP m e e t a t p o in t K s u c h th a t TK = 2, KY = 16, a n d K P = 2(K O ). F ind OP.

Solution for Problem 13.3: A quick sketch suggests how to apply Power of a Point. From the power of point
K , we have

( I< P ) ( K O ) = ( K T ) ( K Y ) .

Substituting the given information in this equation yields

2 ( K O ) ( K O ) = 2(16),

from which we find K O = 4. Therefore, K P = 2K O = 8, so O P = K O 4■ K P = 12. □

Problem 13.4 t V

In th e d ia g ra m , CB = 9, BA = 11, a n d CE = 18. F ind DE.

aZ Z Z A b c

\
D
'

Solution for Problem 13.4: We have two intersecting secants, so we apply Power of a Point, which gives

(C D )(C E ) = (C B )(C A ).

Therefore, ( C D ) ( IS ) = 9(20), so C D = 10 and D E = C E - C D = 8. □

WARNING!! A very common mistake in applying Power of a Point is


to write ( B C ) ( B A ) = ( D C ) ( D E ) when faced with
O the figure at right . This alleged equality is not what
Power of a Point tells us! Note that whenever we write
a Power of a Point relationship, the same point must
appear in all of the segments in the equation, as point
C does when we write the correct relationship for the secants in the figure above:

(C D )(C E ) = (C B )(C A ).

Solution for Problem 13.5: Circles, chord lengths, and secant lengths. This is a job for Power of a Point. The power of point M
gives us

(M R )(M S ) = (M T )(M Q ).

We know that R M = M S , so substitution gives M R 2 = (2)(8), i.e., M R = 4. Therefore, P R = M R = 4 and


P S = 3 ( M R ) = 12. Since U is the midpoint of P Q , we have P Q = 2P U . Now we can apply the power of point P to find:

( P U ) ( P Q ) = (P R )(P S ).

Substitution gives ( P U ) ( 2 P U ) = 4(12), so P U = 2 \/6 . z

E x e rc is e s

13. 1.1: t v

(a) Find B C .

Type your solution, notes and/or work here. Show Solution

(b) Q S = 9 and T Q < T S ; find TQ .

Type your solution, notes and/or work here. Show Solution

(c) Find Z Y .

Type your solution, notes and/or work here. Show Solution

13.1.2: t V

(a) Find D E .

Type your solution, notes and/or work here. Solution

(b) Given X Z = 12, find W Z .

Type your solution, notes and/or work here. Show Solution

13. 1.3: t V

Chords X Y and A B intersect at Q. Given A Q = 4, B Q = 6, and X Q = 8 ,find X Y .

Type your solution, notes and/or work here. Show Solution

13. 1.4: t V

Chord U V bisects chord S T at point M . Given S T = 12 and U V = 15, find all possible values of U M .

Type your solution, notes and/or work here. Show Solution

13.1.5: t V

In this problem we use Power of a Point to prove the Pythagorean Theorem. C


(a) Chord A B is perpendicular at point X to diameter C D of circle O. Let the radius of the circle
be c, the length of A B be 2a, and O X = b. Find C X and X D in terms of b and c.

Type your solution, notes and/or work here. Solution

(b) Use Power of a Point to show that c2 = a 2 4‫ ־‬b2.

Type your solution, notes and/or work here. Show Solution

(c) How can you use this argument to prove the Pythagorean Theorem?

Hint

Hint

Type your solution, notes and/or work here. Show Solution

13.1.6★ : t V

In the figure at right, we have W X = Y Z . Prove that A W = A Y .

Hint

Type your solution, notes and/or work here. Show Solution


13.2 Power of a Point Problems
We now apply Power of a Point to some more challenging problems. As well see, Power of a Point can be particularly useful for
proofs involving segment lengths when circles are part of the problem.

P ro b le m s
Problem 13.6

Given that X Y = 6, Y Z = 5, and that point X is 9 units from the center, O, of the circle, we
wish to find the area of the circle.

(a) Extend X O to meet the circle again at V.

(b) Let the radius of the circle be r; use the power of point X to find r 2, and hence, the area of
the circle.

4 Jump to Solution
Problem 13.7

(a) Find BF.

(b) Find ED.

Source: AHSME 4 Jump to Solution


Problem 13.8

In the diagram, we have B F = 8, A B = 10, C D = 7, and Z A P C = 60c A

(a) Find CP.

(b) Show that A A C P is a 30-6090‫ ־‬triangle.

Hint

(c) Find the area of the circle.

4 Jump to Solution
Problem 13.9

In this problem we wish to show that X A = B Z in the diagram at right, given that D Y bisects
zxrz.
(a) Use Power of a Point to find expressions for X A and B Z in terms of other segments in the
diagram.
(b) Use the Angle Bisector Theorem to prove that your expressions from the first part are equal.

Source: AHSME 4 Jump to Solution


Problem 13.10

Point O is the center of the circle, A B _L BC, A P = AD, and AB has length twice the C
radius of the circle. Prove that

A P 2 = (P B ) ( A B ).

We start where we left off last section by finding lengths with Power of a Point in slightly more complicated problems.

t V
Problem 13.6 ______________________

, Z is on © O and secant X Z hits the circle at Y such that XY\ = 6 and Y Z = 5. Given that X is 9 units from the center of
this circle, find the area of the circle.

Solution for Problem 13.6: We start with a diagram. We continue segment X O to hit the
circle again at V, so that we can use the power of point X: ( X W ) ( X V ) = ( X Y ) ( X Z ) .
If we let the radius of the circle be r, substitution gives
( X O —r ) ( X O + r) = 6(6 4 5 ‫)־‬. Since X O = 9, we have (9 — r ) ( 9 4‫ ־‬r ) = 66, from
which we find r 2 = 15. Therefore, the area of the circle is n r 2 = 157T. z

Concept: If a problem includes a segment that abruptly stops in the middle of a circle, consider
. continuing the segment until it hits the circle. Then see what Power of a Point gives
I you.

Solution for Problem 13.7:

(a) We can find B F with Power of a Point: ( F E ) ( F B ) = (F A)(FD), so

F B = (F A ) ( F D ) / ( F E ) = 20/3.

(b) While Power of a Point won't get us anywhere with ED, our proof of Power of a Point guides us to the answer. Specifically,
since Z F D E = Z F B A and Z A F B = Z E F D , we have A A F B ~ A EFD, so E D / B A = E F / A F . Hence.
E D = ( E F ) ( B A ) / A F = 18.


We used Power of a Point in combination with similar triangles in the previous problem. Now let's try using it with some of our
other geometric tools.

Source: AHSME t V
Problem 13.8

In the diagram, we have 13P = 8. A B = 10, C D = 7, and Z A P C 60° = ‫־‬. Find the area of the
circle.

C %
D

Solution for Problem 13.8: It's not immediately obvious how we will find the radius, so we start by finding
what we can. The power of a point P gives us

(P C )(P D ) = ( P B ) ( P A ),

so ( P C ) ( P C + 7) = 144. Therefore, P C 2 4- 7 P C - 144 = 0, so ( P C 4- 1 6 )(P C - 9) = 0.


PC must be positive, so P C = 9.
Seeing that Z A P C = 60° makes us wonder if there are any equilateral or 30-60-90 triangles lurking
about. Since C P = A P /2 and the angle between these sides is 60°, the sides adjacent to the 60° angle in A A C P are in the
same ratio as the sides adjacent to the 60° angle in a 30-60-90 triangle. Therefore, A A PC is similar to a 30-60-90 triangle by
SAS Similarity, so A A PC must be a 30-60-90 triangle with right angle at Z A C P\

Since Z A C D is right and inscribed in AD, we know AD is a semicircle. Therefore, AD is a diameter of the circle. Since
AC = C P \ ! 3 = 9 / 3 from our 30-60-90 triangle, we have

A D = y/AC 2 4‫ ־‬C D 2 = /2 4 3 + 49 = 2 /7 3 .

Finally, the radius of the circle is A D /2 — / 7 3 , so the area is ( / 7 3 ) tt — 737r. □

Concept: If you see a 60° or 30° angle (or even a 120° angle) in a problem, you should be on
the lookout for 30-60-90 triangles.
I
Now we will use Power of a Point in a couple proofs.

t V
Problem 13.9

Given that Y D bisects Z X Y Z in the diagram at right, prove that X A — ZB.

Solution for Problem 13.9: This problem involves lengths, an angle bisector, chords, secants, and circles. Therefore, we should
consider the Angle Bisector Theorem and Power of a Point. The Angle Bisector Theorem applied to angle bisector Y D of
A X Y Z gives us
X D _ ZD
XY ‫ ־‬ZY‫־‬
However, we need something with X .4 and ZB. Therefore, we turn to the powers of points X and Z, which respectively give us

(XA)(XY) = (XD )(XZ)


(ZD)(ZX) = (ZB)(ZY).

Seeing some common terms in these two, along with terms from our Angle Bisector Theorem equation, we know we're on the
right track. We solve these two equations for X ,4 and Z B , and we have
(XD )(XZ)
XA =
XY
(ZD)(XZ)
ZB =
ZY
We need only show these two expressions are equal. We have X Z in common, and we already have X D / X Y — Z D / Z Y
from our Angle Bisector Theorem. We can put these together to show

(X D ) ( X Z)
^ ^ X y

■ ‫״‬ ( i )

‫־‬ x z ( I f )
(ZD)(XZ)
ZY
= ZB.

Source: AHSME t V
Problem 13.10

Point O is the center of the circle, A B _L BC, A P = AD, and A B has length twice the |
radius of t he circle. Prove that A P 2 = (PB)(AB).

Solution for Problem 13.10: We have a lot of information here. What we want to prove looks a lot like Power of a Point when we
have a tangent. A В sure looks tangent, but we have to prove it. Fortunately, since diameter BC is perpendicular to A В at point
В on the circle, A В must be tangent to the circle. Therefore, A B 2 = (AD)(AE).
This problem has a lot of different segments. We could get lost in a blizzard of A P s and AD's and DE's and so on. So, we
have to stay organized. One way to do this is making a list of what we know and what we want to show. Letting the radius of the
circle be r, such a table might look like this:

What We Know What We Want


AP = AD A P 2 = ( P B )(A B )
A B 2 = (A D )(A E )
A B = 2r

This has the advantage of letting us work in two directions. We can work forwards from what we know to try to reach what we
want, or we can work backwards from what we want. Let's try a little of both here. First, going forwards, we note that
BC = D E = 2r because they are diameters of the circle. Combining this with A B = 2r gives A B = B C = DE.
Going backwards, we write our desired relationship in terms of fewer lengths. Specifically, since P B = A B —AP, we can
write A P 2 = ( P B ) ( A B ) in terms of just A B and AP. We can also note that A B = 2r, and write the equation we want in
terms of r and AP. Now our table looks like:

What We Know What We Want

AP = AD A P 2 = (P B ) ( A B )
A B 2 = (A D ) ( A E ) A P 2 = (A B - A P ) ( A B )
A B = 2 r = D E = BC A P 2 = (2 r - AP)(2r)

Going forwards with our new information, we don’t see anything we can do with BC , but D E is part of A E , so we have

A B 2 = (A D )(A E ) = ( A D ) ( A D + D E ) = ( A D ) ( A D + AB).

All the stuff on our ‫׳‬What We Want’ side has A P s in it, so we use A D = A P to write

A B 2 = ( A D ) ( A D + AB)
= (AP)(AP + AB)
= A P 2 + (AP)(AB).

Now our table looks like this:

What We Know What We Want

AP = AD A P 2 = (P B ) ( A B )
A B 2 = (A D ) { A E ) A P 2 = (A B - A P ) ( A B )
A B = 2 r = D E = BC A P 2 = (2r - AP)(2r)
A B 2 = A P 2 + (A P )(A B )

Now we have an equation in our ‫׳‬What We Know’ that is very close to an equation in our ,What We Want.‫ ׳‬A little rearranging turns
one into the other. Subtracting ( A P )(A B ) from both sides of

A B 2 = A P 2 + ( A P )(A B )

gives A B 2 — ( A P )(A B ) = A P 2. Factoring then gives

(A B - A P ) ( A B ) = A P 2,

which is the second equation in our ‫׳‬What We Want’ column.


Ah-ha! We have a path from something We Know to something We Want! However, we can’t just say 'Done!‫ ׳‬We have to write a
nice solution. We do so by retracing our steps. We have to examine our work above to figure out three things:

1. How we get from the given information to the equation A B 2 = A P 2 + (AP)(AB).

2. How we get from this equation to the one in our list of ‫׳‬What We Want’: A P 2 = ( A B — AP)(AB).

3. How we get from A P 2 = (A B — A P ) ( A B ) to what we want to prove: A P 2 = (PB)(AB).

After reviewing our work above, we write our solution:

Since diameter BC is perpendicular to A B at point B on the circle, A B must be tangent to the circle. Therefore, the power of
point A gives us A B 2 = (AD)(AE). We are given A B equals twice the radius of the circle of which D E is a diameter, so
A B = DE. Since we are also given A D = AP, we have
A B 2 = (A D ) ( A E )
= (AD)(AD + DE)
= (AP)(AP + DE)
= ( A P ) ( A P + AB).

Expanding and A B 2 = ( A P ) ( A P + A B ) gives A B 2 — (A P )(A B ) = A P 2.


rearranging Factoring yields
(A B —A P ) ( A B ) = A P 2. Since A B - A P = PB, we have the desired ( P B ) ( A B ) = A P 2.
Notice that we have some items on our ,What We Know' and ,What We Want’ lists that we don’t use. This will usually be the case
for challenging geometry proofs, z

Important: When writing a solution, you should write a clean solution like what we have after
'After reviewing our work above, we write our solution.' All the work we did before that
Z point was just our investigation to find the solution. This often involves working both
forwards and backwards, as we saw. However, once we find our way, we write a nice
clean solution forwards, starting from what we are given and ending at what we want
to prove.

Concept: Organize your work on challenging geometry proofs. Keep track of what you know
and what you can derive from what you know. Also keep track of what you need to
I prove, and work backwards from there to list more statements that, if proven, would
mean you are finished. When you know how to get from a statement on your ,What
We Know’ list to a statement on your ‫׳‬What We Want’ list, then you can construct your
proof.

E x e rc is e s

In the figure at right, A B is tangent to ©O . Given that A B — 4 and B D = 12, find the
area of the circle.

Show Solution
Type your solution, notes and/or work here.

As shown in the diagram at right, X .4 is tangent to the circle at A and a secant


through X meets the circle at B and C, with X B < XC. Prove that
X B < X A < XC.
Hint

Show Solution
Type your solution, notes and/or work here.

A circle is tangent to side BC of equilateral triangle A A B C at point Q as shown at


right. The circle intersects sides A B and AC in two points each, as shown. Given that
A W = AY, prove that Q is the midpoint of BC.

Hint

Show Solution
Type your solution, notes and/or work here.
Challenge Problems

14.31 t V

Base E F G H of right prism E F G H I J K L is a parallelogram with E F = 8, F G = 6, and Z E F G = 60°. Given that the
height of the prism is 9, find the following:

(a) [E F G H \

Hint

Type your solution, notes and/or work here. Show Solution

(b) the surface area of E F G I I I J K L .

Type your solution, notes and/or work here. Show Solution

(c) the volume of E F G H I J K L .

Type your solution, notes and/or work here. Show Solution

14.32: t V

The altitude from vertex V of tetrahedron V W X Y meets face W X Y at the circumcenter of A W X Y . Prove that
V W = V X = VY.

Hint

Type your solution, notes and/or work here. Show Solution

14.33: t V

Each side of the cube shown has length 10. What is the volume of the solid whose edges are formed
by connecting all the midpoints of the edges of the cube as shown?

Hint

Type your solution, notes and/or work here. Show Solution

14.34: Source: MATHCOUNTS t V

A soccer ball is constructed using 32 regular polygons with equal side lengths. Twelve of the polygons are pentagons, and the
rest are hexagons. A seam is sewn wherever two edges meet. What is the number of seams in the soccer ball?

Hint

Type your solution, notes and/or work here. Show Solution

14.35: Source: MATHCOUNTS t V

The figure at right with 5 square faces and 10 equilateral triangular faces is folded into a 15-
faced polyhedron. How many edges does the polyhedron have? How many vertices does the
polyhedron have?

Type your solution, notes and/or work here. Show Solution

14.36: t V

An octahedron is formed by connecting the centers of the faces of a cube. What is the ratio of the volume of the cube to the
volume of the octahedron?

Hint

Type your solution, notes and/or work here. Show Solution

14.37: Source: AMC 12 t V

An insect lives on the surface of a regular tetrahedron with edges of length 1. It wishes to travel on the surface of the
tetrahedron from the midpoint of one edge to the midpoint of the opposite edge. What is the length of the shortest such trip?
(Note: Two edges of a tetrahedron are opposite if they have no common endpoint.)

Hint

Type your solution, notes and/or work here. Show Solution

14.38: t V

S T U V W X Y Z is a right rectangular prism with X Y = 12, W X = 3, and T X = 4.

(a) Find W U . \ \ u
.-

Type your solution, notes and/or work here. Show Solution

(b) Find the distance from Z to W U .

Hint

Hint

Type your solution, notes and/or work here. Show Solution

(c) Find the volume of T W X Y .

Type your solution, notes and/or work here. Show Solution

(d) ★ Find the volume of pyramid V S U Y W . (V is the vertex.)

Type your solution, notes and/or work here. Show Solution

(e) ★ Find the volume of W X Z U .

Hint

Hint

Type your solution, notes and/or work here. Show Solution

14.39: Source: AHSME t V

A cube of cheese C = {(x ,;t/, 2:)|0 < x , y ,z < 1} is cut along the planes x = y ,y = z, and z = x. How many pieces are
produced this way?

Hint

Hint

Type your solution, notes and/or work here. Show Solution

E F G H I J K L is a cube. How many different planes pass through at least three vertices of E F G H I J K L?

Hint

Type your solution, notes and/or work here. Show Solution

14.41 t V

P In cube A B C D E F G H shown, A B = 12. Find the distance from B to plane A C F .


B C Hint
H \\
\ Hint

Type your solution, notes and/or work here. Show Solution

14.42: Source: AMC 12 t V

The sum of the lengths of the twelve edges of a rectangular box is 140, and the distance from one corner of the box to the
farthest corner is 21. What is the total surface area of the box?

Hint

Type your solution, notes and/or work here. Show Solution

14.43: Source: MATHCOUNTS t V

Congruent wedges A and B are sliced as shown. Each wedge has a pair of
parallel triangular faces. Wedge A is cut vertically so that slices are made
perpendicular to the base; wedge B is cut horizontally so that slices are made
parallel to the base. Six slices 1/4 inch thick are cut from each wedge. What is
the ratio of the remaining volume of wedge A to the remaining volume of wedge
B?

Hint

Type your solution, notes and/or work here. Show Solution

A B C D E F shown at right is a regular octahedron with edge length 1. The midpoints of edges
A B , A C , A D , and A E are M , N , O, and P, respectively. The midpoints of edges B F , C F , D F ,
and E F are Q, B, S, and T , respectively. Find the volume of M N O P Q R S T .
B
Hint

Type your solution, notes and/or work here. Show Solution

14.45★ : t V

Parallelogram E F G I I is a base of right prism E F G H 1 .1 K L at right. Given E F = 8, F G = (j, K


E l = 9, and Z E F G = 60°, find F L and E K .

Hint H

Type your solution, notes and/or work here. Show Solution

14.46★ : t V

A right square frustum is formed by cutting a right square pyramid with a plane parallel to its base. Suppose the original
pyramid has base length 6 and height 9, and that the plane cutting the pyramid to form the frustum is 3 units from the base of
the pyramid.

(a) Find the volume of the frustum.

Hint

Hint

Type your solution, notes and/or work here. Show Solution

(b) Find the total surface area of the frustum.

Type your solution, notes and/or work here. Show Solution

14.47★ : Source: HMMT t V

Let A B C D be a regular tetrahedron with side length 2. The plane parallel to edges A B and C D and lying halfway between
them cuts A B C D into two pieces. Find the surface area of one of these pieces.

Hint

Type your solution, notes and/or work here. Show Solution


13.3 Summary

Important: Suppose a line through a point P intersects a circle in two points,


U and V. The Power of a Point Theorem states that for all such
Z lines, the product ( P U ) ( P V ) is constant. We call this the power
of point P. For example, in the figure at right, applying Power of a
Point to X with respect to the circle shown gives

(X A )(X B ) = (X C ){X D ).

In the figure at left, the power of point P with respect to


the circle gives us

P A 2 = (P B ){P C ) = (P D )(P E ).

We think of these whenever we have a problem involving


intersecting segments and a circle.

Important: While finding a solution often involves working both forwards and backwards, when
we write our solution, we write a nice clean solution forv/ards, starting from what we
Z are given and ending at what we want to prove.

P ro b le m S o lv in g S tra te g ie s

Concepts:
If a problem includes a segment that abruptly stops in the middle of a circle,
consider continuing the segment until it hits the circle. Then see what Power of a
Point gives you.

If you see a 60° or 30° angle (or even a 120° angle) in a problem, you should be on
the lookout for 3 0 9 0 ‫־‬60‫ ־‬triangles.

Organize your work on challenging geometry proofs. Keep track of what you know
and what you can derive from what you know. Also keep track of what you need to
prove, and work backwards from there to list more statements that, if proven,
would mean you are finished. When you know how to get from a statement on your
'What We Know' list to a statement on your ,What We Want' list, then you can
construct your proof.

T h in g s To W a tc h O u t For!

WARNING!! A very common mistake in applying Power of a Point to


a point outside a circle as in the figure at right is to
O write ( B C ) ( B A ) = ( D C ) ( D E ). This alleged
equality is not what Power of a Point tells us! Note that
whenever we write a Power of a Point relationship, the
same point must appear in all of the segments in the
equation, as point C does when we write the correct relationship for the secants in
the figure above:

( C D ) ( C E ) = (C B ) ( C A ).

Extra! The set of all points that have the same power with respect
to two given circles is a line called the radical axis of the
V two circles. One particularly important use of radical axes is
the Radical Axis Theorem, which states that given three
circles, the three radical axes of the three pairs of circles are
concurrent. The diagram at right is an example of the
Radical Axis Theorem for three intersecting circles. Do you
see why the radical axis of two intersecting circles must
include the chord connecting the points where the circles meet?

As an extra challenge, try proving first that the set of all points that have the same
power with respect to two given circles is a line, then use this to prove the Radical
Axis Theorem. (The first part is tougher than the second!)
Review Problems

13.11 t V

(a) Find E C .

Type your solution, notes and/or work here. Show Solution

(b) Find P R .

Type your solution, notes and/or work here. Show Solution

13. 12: t V

(a) Find V Z .

Type your solution, notes and/or work here. Show Solution

(b) Find B C and C D .

A 6 D

Type your solution, notes and/or work here. Show Solution

(c) Find PS.

Type your solution, notes and/or work here. Show Solution

13. 13: t V

Chords G I I and / . / of © 0 are perpendicular at M . Given G M = 2M H = 6 / M = 12, find G.J.

Flint

Type your solution, notes and/or work here. Show Solution

13. 14: t V

Lines m and n meet at P. © 0 meets m at points A and B, and meets n at C and D, such that P A = 3, A B = 9, P C = 4,
and C l ) = 5. Why must P be outside © 0 ?

Type your solution, notes and/or work here. Show Solution

13. 15: t V

Earth is roughly 8000 miles in diameter.

(a) l‫׳‬m riding in a hot air balloon 1 mile above the surface of Earth. Approximately how far away is the horizon? (In other
words, how far away is the farthest point on the surface of Earth that I can see.)

Type your solution, notes and/or work here. Show Solution

(b) What if l‫׳‬m in a plane 6 miles above the surface of the Earth?

Type your solution, notes and/or work here. Show Solution

(c) What if l‫׳‬m in a spaceship 100 miles above the surface of Earth?

Type your solution, notes and/or work here. Show Solution

13. 16: t V

Given Z Y < Z W in the diagram at right, show that Z V < Z X .

Type your solution, notes and/or work here. Show Solution

13. 17: t V

The figure at right shows two circles that intersect at A and B. P is on A B , and P Q and P R are tangents
as shown. Prove that P Q = P R .

Type your solution, notes and/or work here. Show Solution

13. 18: t V

Is it possible for chords A B and C I ) of a circle to intersect at X such that X is the midpoint of A B , but is closer to C than
to IP.

Type your solution, notes and/or work here. Show Solution

13. 19: t V

Is it possible for two chords A B and C D of a circle to meet at a point X such that A X = B X , C X = 2 D X , and
A B = CD?

Type your solution, notes and/or work here. Show Solution

13.20: t V

Find the ratio A C / A D in the figure at left.

Type your solution, notes and/or work here. Show Solution

13.21 t V

Point A is on © 0 such that P A is tangent to circle © 0 . Point B is on circle © 0 such that P B = P A . Must P B be tangent
to © 0?

Type your solution, notes and/or work here. Show Solution

13.22: t V

Let P be a point in the same plane as a circle centered at 0 with radius r. A secant through P intersects the circle at points A
and B. Use to prove that if P lies outside the circle, then P A • P B = P O 2 — r 2. What is the formula if P lies inside
the circle?

Type your solution, notes and/or work here. Show Solution

13.23: t V

Use the diagram in the figure at right to find a proof of the Pythagorean Theorem.

Type your solution, notes and/or work here. Show Solution


14.2 Prisms
We move now from considering one- and two-dimensional figures in space to thinking about solid figures. Our ,room‫ ׳‬was so
useful in the previous section that well start with it here.

Mathematically speaking, A B C D E F G H at the right is a rig h t re c ta n g u la r p ris m . Since all the


boundaries of A B C D E F G H are polygons, A B C D E F G H can also be called a p o ly h e d ro n .
Each of these boundary polygons is a fa c e of the polyhedron. The ,rectangular‫ ׳‬part of ,right
rectangular prism’ refers to the fact that the top and bottom faces (A B C D and E F G H ) are
rectangles. We typically call the ,top' and ,bottom' the b a se s of the prism. The 'right‫ ׳‬part refers to the
fact that all the edges connecting the two bases meet the bases at right angles.

Which leaves us with the ,prism' part. A p ris m is a three-dimensional solid figure with two congruent parallel faces, and with
parallelograms as the other faces. The faces that are not bases are sometimes called the s id e s of the prism. In a r ig h t p ris m all
of these sides are rectangles. As already noted, the congruent parallel faces are the bases of the prism. Hence, any pair of
opposite sides of A B C D E F G H above could be considered the bases of the prism.

Figure 14.1 shows some more prisms. The first is a right regular pentagonal prism because the base is a regular pentagon and
the segments connecting the bases are perpendicular to the bases. The second is a hexagonal prism because the bases are
hexagons (note that we don't call this one a ,right hexagonal prism‫)׳‬. The third has parallelograms as its bases, as well as all its
sides. Such a prism is given the special name p a ra lle le p ip e d . As one last bit of vocabulary, if there's a ,regular‫ ׳‬in the description
of a prism, it means the base is a regular polygon.

Make sure you understand what a prism is, but you don't have to worry about all those other names. One reason you don't have to
worry too much is that they aren't used consistently. For example, the 'right' is often left out of the description, and you’re meant
to infer from the problem that the prism is indeed a right prism. Sometimes in this book we'll do this, such as by calling our
original prism A B C D E F G H a ,rectangular prism’. Unfortunately, the vocabulary lesson isn't over yet. Just as we measure the
region contained in a two-dimensional figure with area, we can measure the space inside a three-dimensional figure with v o lu m e .
We do so in much the same way as we measure area. Instead of measuring how many l x l squares fit, we measure how many
1 x 1 x 1 ,blocks'(or portions of blocks) fit inside the solid.

Finally, we can still use area to measure three-dimensional figures. The to ta l s u rfa c e a re a of a figure is the area of all of the
surfaces that form the borders of the solid. The la te ra l s u rfa c e a re a is the total area of all the faces that are not considered
,bases.' We often just ignore lateral surface area and say 'surface area,' which means the same as ,total surface area.‫׳‬

P ro b le m s
P ro b le m 1 4 .4 4 Jump to Solution

A B C D E F G H shown is a rectangular prism with A B = 3, B C = 4, and A E = 5.

(a) Find /? F a n d E H .

(b) Find the volume of A B C D E F G H .

(c) Find the total surface area of A B C D E F G H .

(d) Find A C .

(e) What kind of triangle is A A C G ?

(f) Find AG .

P ro b le m 1 4 .5 4 Jump to Solution

A c u b e is a special rectangular prism in which all edges have the same length (i.e., its base is a square
and its height has the same length as a side of the base).

(a) Find the formula for the volume of a cube with side length s.

(b) Find the formula for the total surface area of a cube with side length s.

(c) Find a formula for the length of S Y in the diagram given that the figure is a cube with side length s.

P ro b le m 1 4 .6 4 Jump to Solution

Once again, we consider cube S T U V W X Y Z .

(a) What geometric shape is S X Y V? V \


(b) What is the area of S X Y V if S V = 4? j
V\
\ U

P ro b le m 1 4 .7 4 Jump to Solution

The right regular hexagonal prism shown has sides of length 4 on base A B C D E F and a height
of 8 units.

(a) Find the lateral surface area of the prism.

(b) Find the total surface area of the prism.

(c) Find the volume of the prism.

(d) Find A B ', A C ', and A D ' by building the appropriate right triangles.

We start off by finding volumes, areas, and lengths of a (right) rectangular prism.

Problem 1 4.4 t V

ABCDEFGH s h o w n is a re c ta n g u la r p ris m w ith AB = 3, B C = 4, a n d A E = 5.


D
(a ) F ind BF and EH.
B C
(b ) F ind th e v o lu m e o f ABCDEFGH.

(c ) F ind th e to ta l s u rfa c e a re a o f ABCDEFGH.

(d ) F ind AC. H \

\
V
N

(e) W h a t k in d o f tria n g le is AACGl

(f) F ind AG .

Solution for Problem 74.4:

(a) All of the faces are rectangles. Since A E F B is a rectangle, B F = A E = 5. Similarly,


rectangle E F G I I tells us that E H = FG , and rectangle B F G C gives us F G = B C = 4.
Therefore, E H = 4. We could also have seen that E H = B C by noting that both represent the
distance between parallel faces A E F B and D H G C .

(b) We approach this much as we did finding the area of a rectangle with l x l squares in Section
4.2 here. We can organize a layer of (A B ) ( B C ) = 1 2 1 x 1 x 1 cubes on face A B C D . This
plus four more such layers completely fills our prism, so the volume is 3 x 4 x 5 = 60.

(c) To find the total surface area, we need to add the areas of all the faces. Each face is a rectangle,
but rather than finding the areas of six different rectangles, we can note that the rectangles come
in pairs of congruent rectangles. For example, A B C D = E F G H . Therefore, our total surface
area is:

2 { [A B C D ] + [B C G F ] + [A B F E } ) = 2[(3 )(4 ) + (4 )(5 ) 45 )(3 ) ‫])־‬


= 94.

(d) A C is a diagonal of rectangle A B C D (or hypotenuse of triangle A B C ), so it has length


v/32 44. .5 = 42 ‫־‬C‫ ׳‬is sometimes called a fa c e d ia g o n a l of the prism.

(e) Since C G is perpendicular to plane A B C D , it is perpendicular to A C . Therefore, A A C G is a right triangle.

(f) From right A A C G , we have

AG = V G C 2 + A C 2 = V25 + 25 = 5 v 2 ‫׳‬.
Note that

V A B 2 + B C 2 + GC 2 = 5 v 2 ‫ = ׳‬AG.
Is this a coincidence?

Segment A G is called a s p a c e d ia g o n a l of the prism. The lengths A B , A D , and A E are sometimes referred to as the prism's
d im e n s io n s . We might use these to describe the prism as a 3 x 4 x 5 rectangular prism. We can follow the exact same logic we
followed in Problem 14.4 to find formulas for the volume, total surface area, and the length of a space diagonal of a rectangular
prism.

Im p o rta n t: The three dimensions of a rectangular prism are commonly called


f
the length, /, the width, w, and the height, h. For such a prism, we
have:

Volume = Iwh
Surface area = 2 (Iw 4‫ ־‬wh 4- Ih)
Space Diagonal = V l 2 4‫ ־‬w 2 4‫ ־‬h 2

Notice that our volume is simply the area of a base of the prism times the height of the prism. This is true for any prism.

Im p o rta n t: The volume of a prism equals the area of a base times the distance between the
bases (i.e. the height).

Notice that by ,distance between the bases', we do not necessarily mean the length of the edges
connecting corresponding points on the bases. If the prism is not a right prism, then the height is the
length of a segment from one base to the other that is perpendicular to both bases. For example, the
height between bases A B C D and E F G H of the prism at left is X Y , not F B .

Problem 14.5 t V

A cube is a s p e c ia l re c ta n g u la r p ris m th a t h a s a ll its d im e n s io n s th e s a m e (i.e., its b a s e is a s q u a re and


Wk
its h e ig h t h a s th e s a m e le n g th a s a s id e o f th e b a se ).

(a ) F ind th e fo rm u la fo r th e v o lu m e o f a c u b e w ith s id e le n g th s.
Ä ‫ ״ ־‬iV \
(b ) F ind th e fo rm u la fo r th e to ta l s u rfa c e a re a o f a c u b e w ith s id e le n g th s.
U
(c ) F ind a fo rm u la fo r th e le n g th o f SY in th e d ia g ra m g ive n th a t th e fig u re is a c u b e w ith s id e le n g th s.

Solution for Problem 14.5:

(a) Since a cube is a rectangular prism, its volume is the product of all three dimensions. Since all three dimensions are the
same, s, the volume is simply s‫\־‬

(b) The faces of a cube are all squares. Therefore, each of the 6 faces has area s2, so the total
surface area is 6s2.

(c) Once again, we can simply use the formulas we already found for a rectangular prism. But to be a
little more sporting, let's prove the formula directly for a cube. To find SY, wed like to create a
right triangle that has SY as a side, so we can use the Pythagorean Theorem. Hence, we draw
SY and SU, creating right triangle A SUY. We have UY = s, so all we have to do is find SU.
SU is the hypotenuse of A STU, so SU = s\/2. Therefore,

S Y = VSU 2 + U Y 2 = v '2 s2 + s2 = s V 3 .

Im p o rta n t: A cube with side length s has:

Z Volume = s’*
Surface area = 6s2
Space Diagonal = s \/3

Problem 1 4 .6 t V

O nce a g a in , w e c o n s id e r c u b e STUVW XYZ. W h a t is th e a re a o f SXYV if SV = 4?

Solution for Problem 14.6: We start by drawing S X Y V so we can figure out what sort of shape it is.
Since S V and X Y are perpendicular to faces S T X W and V U Y Z, they are both perpendicular to
both S X and Y V . So, S X Y V is a rectangle. We already have one side of the rectangle, S V = 4.
Therefore, we only need to find S X . Since S X is a diagonal of square S W X T , S X = 4 \/2 , so
[ s x y V ] = ( S V ) ( S X ) = 1 6 v 2 ‫׳‬. o

One way to view rectangle S X Y V of the previous problem is as the intersection of a plane and the cube, where the plane
passes through vertices S, X , Y, and V of the cube. We call such an intersection of a plane and a solid a c r o s s - s e c tio n of the
solid. Many three-dimensional geometry problems are solved by choosing the right cross-section of the problem to consider.
Sometimes we even have to consider multiple cross-sections!

Lest you start to think that all prisms are rectangular ones, we'll try a problem involving a regular hexagonal prism.

Problem 1 4.7 t V

T h e rig h t re g u la r h e x a g o n a l p ris m s h o w n h a s s id e s o f le n g th 4 o n b a s e ABCDEF a n d a h e ig h t o f


8 u n its .

(a ) F ind th e la te ra l s u rfa c e a re a o f th e p ris m .

(b ) F ind th e to ta l s u rfa c e area o f th e p ris m .

(c ) F ind th e v o lu m e o f th e p ris m .

(d ) F ind A B ', A C ', and A D '.

Solution for Problem 14.7:

(a) Each lateral face of the prism is a 4 x 8 rectangle. There are 6 lateral faces, so the lateral surface area is 6(4 x 8) = 192.

(b) To find the total surface area, we must add the area of the two bases to the lateral surface area. Each base is a regular
hexagon. As we saw back in Problem 9.7, we can find the area of a regular hexagon of side length 4 by breaking it into six
equilateral triangles with side length 4. Therefore, the area of each base is

So, the total surface area of the prism is 192 43/<48 ‫ ־‬2 ( 24 \ / 3 ) = 192 4‫־‬.

(c) We've already done the hard part! The volume is just the area of a base times the height, or
8(24v/3) = 192 \/3 .

(d) Since A B ' is a diagonal of rectangle A B B 'A ', A B ' = \ / 4 2 45 /\ 4 = 82 ‫־‬.

The other two are a little trickier. For A C ', we consider the cross-section A C C A' and notice that
A C ' is a diagonal of rectangle A C C ' A'. We already know C C ' = 8, so we only have to find A C .
Since A A C D is a 30-60-90 triangle (make sure you see why), we have A C = C D \ fZ = 4 \/3 .
Hence,

A C ' = V A C 2 + C O 2 = v/48 + 64 = 4 v 7 ‫ ׳‬.

Similarly, AD' is a diagonal of rectangle ADD'A'. To find AD, we recall the dissection of a regular
hexagon by its long diagonals into 6 equilateral triangles as shown, so AD = 2 ( B C ) = 8.
Therefore,

AD' = V A D 2 + D'D2 = VM + 64 = 8 v 2 ‫ ׳‬.

C o n c e p t: Building right triangles works in three dimensions every bit as well as it does in two
dimensions.

E x e rc is e s

14.2.1: t V

Describe as fully as possible a cross-section of a prism that is parallel to the bases of the prism.

Type your solution, notes and/or work here. Show Solution

14.2.2: t V

A right rectangular prism has dimensions 2,5, and 3 \/2 .

(a) Find the volume of the prism.

Type your solution, notes and/or work here. Show Solution

(b) Find the total surface area of the prism.

Type your solution, notes and/or work here. Show Solution

(c) Find the length of a space diagonal of the prism.

Type your solution, notes and/or work here. Show Solution

(d) Find the length of the longest face diagonals.

Type your solution, notes and/or work here. Show Solution

14.2.3: t V

Find the volume of a rectangular prism that has a space diagonal of length 10 and two sides of length 3 and 8.

Type your solution, notes and/or work here. Show Solution

14.2.4 : t V

Find the volume of a cube given that its space diagonal has length 6.

Type your solution, notes and/or work here. Show Solution

14.2.5: t V

How many different space diagonals does a cube have?

Type your solution, notes and/or work here. Show Solution

14.2.6: t V

If all the dimensions of a right rectangular prism are doubled, what happens to the surface area? What happens to the volume?

Type your solution, notes and/or work here. Show Solution

14.2.7: t V

Shown is right rectangular prism A B C D E F G H . Given that A B = 4, B C = 3, and A B G H is a A D


square, find the following: B
(a) BD. F
H

Type your solution, notes and/or work here. Show Solution

(b) AG .

Type your solution, notes and/or work here. Show Solution

(c) FD.

Type your solution, notes and/or work here. Show Solution

(d) the volume of A B C D E F G H .

Type your solution, notes and/or work here. Show Solution

14.2.8: Source: ARML t V

Bridget places a box that is 4 x 6 x 8 on the floor. She then places a 2 x 3 x 5 box on top of the first box, forming a two-box
tower. She will then paint all the surfaces she can paint without moving either of the boxes. She wants to paint as little as
possible, so she places the boxes in a way that minimizes the amount she'll have to paint. What is the total area she has to
paint?

Type your solution, notes and/or work here. Show Solution


Challenge Problems

13.24: Source: ARML t V

In circle O, P O J_ O B and P O equals the length of the diameter of ©O. Compute P A /A B .

Hint

Hint

Type your solution, notes and/or work here. Show Solution

13.25: t V

Chords P Q and R S of a circle meet at point X . Given that P Q = R S, show that P X = R X or Q X = R X (or both).

Hint

Type your solution, notes and/or work here. Show Solution

13.26: t V

Jake is working on a problem in which chords A B and C D , when extended past B and D, respectively, meet at P. He is
given P B , A B , and C D . He mistakenly uses Power of a Point improperly by using ( P B ) ( B A ) = ( P D ) ( D C ) . However, he
still gets the right answer for P D . Prove that the question Jake was answering must have had A B = C D .

Hint

Type your solution, notes and/or work here. Show Solution

13.27: t V

A and B are two points on a circle with center O, and C lies outside the circle, on ray .4 A Given that A B = 24, B C = 28,
and O A = 15, find O C .

Hint

Type your solution, notes and/or work here. Show Solution

13.28: t V

Circles C! and C2 have the same center, O. The radius of C! is r! and the radius of C-2 is r where r ! > r 2.

(a) Prove that it is impossible for a point P to have the same power with respect to both circles if P is outside both circles.

Hint

Type your solution, notes and/or work here. Show Solution

(b) Prove that it is impossible for a point P to have the same power with respect to both circles if P is inside both circles.

Type your solution, notes and/or work here. Show Solution

(c) Show that it is possible for a point inside C! but outside C2 to have the same power with respect to both circles, and find
all such points that have equal power with respect to both circles.

Type your solution, notes and/or work here. Show Solution

13.29: t V

W X and Y Z meet at P such that ( W P ) ( P X ) = ( Y P ) ( P Z ) . Prove that the circumcircle of A X Y Z goes through point
W.

Hint

Type your solution, notes and/or work here. Show Solution

13.30: t V

Two circles C! and C2 intersect at two points, A and B. Let P Q be a chord of C! and R S a chord of C2 such that they
intersect on A B . Prove that points P, Q, R, and S all lie on a circle.

Hint

Hint

Type your solution, notes and/or work here. Show Solution

13.31 t V

Given two segments with lengths a and b, construct a segment with length \fab.

Hint

Type your solution, notes and/or work here. Show Solution

13.32: t V

Equilateral triangle A B C is inscribed in a circle. Let P be a point on arc B C , and let A P intersect B C
at Q. Prove that

1 1 1
+
PQ PB PC

Hint

Hint

Type your solution, notes and/or work here. Show Solution

13.33: Source: ARML t V

In the diagram, A B = 8, A P = 2, and P C = 4.

(a) Prove that [ A B E \ / [ C D E ] = (A B / C D )2.

Type your solution, notes and/or work here. Show Solution

(b) Prove that [P B C \ /[P A D ] = ( P C / P A ) 2.

Type your solution, notes and/or work here. Show Solution

(c) ★ Find [ P A E C ] / [ B A E \

Hint

Hint

Type your solution, notes and/or work here. Show Solution

13.34★ : Source: ARML t V

Equilateral triangle A B C is inscribed in a circle. Points D and E are midpoints of A B and B C , respectively, and F is the
point where D È meets the circle. Find D E / E F .

Hint

Hint

Type your solution, notes and/or work here. Show Solution

13.35★ : Source: Mandelbrot t V

Segment I F is tangent to the circle at point / as shown in the diagram at right. We are given that
I F = 21 v 2 ‫׳‬. IO = 20, O N = 12, R F = 18, and O R > GO. Find O R .

Hint

Type your solution, notes and/or work here. Show Solution

13.36★ : t V

Chords A B and C D of © O are perpendicular at point P. Given C P = 2, A P = 3, and P D = 6, find the following:

(a) O P.

Hint

Hint

Type your solution, notes and/or work here. Show Solution

(b) The radius of ©O.

Hint

Type your solution, notes and/or work here. Show Solution


14.3 Pyramids
If we connect all the vertices of a polygon to a point that is not in the same plane as the polygon, we form a
pyramid. This point is called the apex of the pyramid and the polygon is the pyramid’s base. As we can see at right,
the non-base faces of a pyramid are all triangles. The lateral surface area of a pyramid is the sum of the areas of
these triangles.

/I As with prisms, a pyramid is 'regular‫ ׳‬if its base is a regular polygon. A regular pyramid is 'right‫ ׳‬if the
center of the base is the foot of the altitude from the apex to the base (i.e., the apex is directly 'over‫ ׳‬the
center of the base). Usually when we speak of a pyramid, we mean a right pyramid. The height of a
pyramid is the distance from the apex to the base. For right regular pyramids, we also define a slant
height, which is the distance from the apex to a side of the base. For example, at left, A G is the height
and A F is the slant height of the pyramid.

Important: The volume of a pyramid is one-third the product of the pyramid’s height and the area
of the pyramid's base.
Z
Deriving this formula of a pyramid's volume is very challenging; however, the formula is not hard to apply, so it is occasionally
used in introductory three-dimensional geometry.

P ro b le m s
Problem 14.8 4 Jump to Solution

A B O D E is a right square pyramid such that B C = (i and A C = 5 as shown at right.

(a) Let O be the center of the square. Why is A O perpendicular to the base of the pyramid? Show
that A C = A D .

(b) Find [A C D \ by first finding the length of an altitude from A to C D .

(c) Find the total surface area of A B O D E .

(d) Let M be the midpoint of C D . What kind of triangle is A A O M ?

(e) Find AO, then find the volume of the pyramid.

Problem 14.9 4 Jump to Solution

Find a formula for the lateral surface area of a right pyramid whose base is a regular n -gon with side length s, and whose slant
height is /.

Problem 14.10 4 Jump to Solution

A triangular pyramid is more commonly called a tetrahedron. A regular tetrahedron is a


tetrahedron whose edges all have the same length. In the diagram, regular tetrahedron
A B C D has side length 6.

(a) Let G be the foot of the altitude from A to face A B C D . What kind of triangles are
A A G B , A A G C and A A G D ?

(b) Use part (a) to show that G is the circumcenter of A B C D . Why must it also be the
centroid of A B C D ?

(c) Let M be the midpoint of C D . Find B M and BG .

(d) Find AG .

(e) Find the volume of A B C D .

We'll start by investigating a square right pyramid.

Problem 1 4.8 t V

A B O D E is a square right pyramid such that B C = 6 and A C = 5. Find the total surface area and the volume of the
pyramid.

Solution for Problem 14.8: We can find the area of the base quickly: B C 2 = 36. But to find the
A
area of the triangular faces, we'll need an altitude of one of the faces. We start by drawing AO ,
the altitude of the pyramid. Because the pyramid is right, О is the center of base B O D E . We
suspect that each triangular face is isosceles because the pyramid is right. We can prove this
by noting that since A O is perpendicular to the base, it is perpendicular to both O C and O D .
We have O C = O D because О is the center of the square, so A A O C = A A O D by SAS
Congruence. This triangle congruence gives us A C = A D .

Therefore, A A C D is isosceles, so the median from A to C D is also an altitude. Since


A C = 5 and C M = C D /2 = 3, we have A M = 4 from right triangle A A M C. Therefore,
[A C D ] = ( A M ) ( C D ) / 2 = 12. Our total surface area consists of four of these triangles plus the square, or
4(12) - f 36 = 84.

Turning to the volume, we need the height of the pyramid, AO . We can build a right triangle by connecting О to either M or to
one of the base vertices. In the diagram, we have right triangle A A O M . Since O M = В С / 2 = 3 and A M = 4, we have
A O = у /A M 2 — O M 2 = y fl. Since the area of the base is 36, the volume of the pyramid is (3 6 )(\/7 )/3 = \ 2 \ f l . □

Once again, we built right triangles to find the lengths we needed.

As we saw in that last problem, finding the lateral surface area of a regular right pyramid is easy once we know the slant height
and the side length. We can even build a formula for it.

Problem 1 4.9 t V

Find a formula for the lateral surface area of a right pyramid whose base is a regular n-gon with side length s, and whose slant
height is /.

Solution for Problem 14.9: Each of the lateral faces of the pyramid is a triangle with height / and base s.
Hence, each of these triangles has area s i/2 . There is one triangle for each side of the base, so there are
n triangles, which have a total area of n ( s l / 2) = n s l j 2. Note that n s equals the perimeter of the base,
so the lateral surface area of a regular pyramid is half the product of the slant height and the perimeter
of the base. □

That formula is so much easier to derive than to memorize that we won’t even put it in an ‘Important’ box.

Problem 1 4.1 0 t V

A triangular pyramid is more commonly called a tetrahedron. A regular tetrahedron is a tetrahedron whose edges all have the
same length. Find the volume of a regular tetrahedron that has sides of length 6.

Solution for Problem 14.10: To find the volume, we need an altitude, so we draw altitude A G from A
to A B C D . Since A G is perpendicular to plane B C D , triangles A A G B , A A G C , and A A G D are
all right triangles. Because A B = A C = A D and A G is obviously the same in all three triangles,
we have A A G B = A A G C = A A G D by HL Congruence. Therefore, B G = C G = D G , which
means that G is the circumcenter of A B C D because it is equidistant from the vertices of A B C D .
Since A B C D is an equilateral triangle, G is also the centroid of A B C D .

We can build more right triangles by continuing B G to M . Since A B C D is equilateral, B M is a


median and an altitude. Therefore, D M = D C /2 = 3and B M = 3 \/3 (from 30-60-90 A B M D ) .
Since the centroid of a triangle divides its medians in a 2 : 1 ratio, we have
B G = (2 /3 ) B M = 2 \/3 . Finally, we can find A G from right triangle A A G B .

A G = y /A B 2 - B G 2 = \/3 6 - 12 = 2 \/6 .

Since the area of A B C D is ( D C ) ( B M ) / 2 = 9 \/3 , our volume is ( [B C D ]) ( A G ) /S = 18y/2.

Similarly, we can show that the volume of a regular tetrahedron with edge length s is sA\/2 /1 2 . □

Important: Problem 14.10 is a typical challenging three-dimensional geometry problem in that


our general tactic is to reduce it to a series of two-dimensional problems. When you
Z can work through this problem on your own, you're ready for some serious three-
dimensional geometry.

E x e rc is e s

14.3.1: t V

A B O D E is a right square pyramid with base A B C D . Given that A B = 4 and A E = 8, find the following:

(a) The height of the pyramid.

Type your solution, notes and/or work here. Show Solution

(b) The slant height of the pyramid.

Type your solution, notes and/or work here. Show Solution

(c) The volume of the pyramid.

Type your solution, notes and/or work here. Show Solution

(d) The total surface area of the pyramid.

Type your solution, notes and/or work here. Show Solution

14.3.2: t V

When we first created Problem 14.8, we let B C = 8 instead of B C = 6 in the problem statement. Why did we have to
change it?

Hint

Type your solution, notes and/or work here. Show Solution

14.3.3: Source: MATHCOUNTS t V

A regular pyramid with a square base has base edges of length 6 inches and height 4 inches. What is the ratio of the number
of cubic inches in the volume of the pyramid to the number of square inches in its surface area?

Type your solution, notes and/or work here. Show Solution

14.3.4: Source: (b) MATHCOUNTS t V

A 4 " x 6 " x 8 " rectangular solid is cut by slicing through the midpoints of three adjacent sides as shown,

(a) Find the volume of the shaded piece that is cut off.

Type your solution, notes and/or work here. Show Solution

(b) Find the sum of the lengths of the edges of the shaded piece that is cut off.

Type your solution, notes and/or work here. Show Solution

14.3.5: t V

S T U V W X Y Z is a cube as shown with ST = 4.


\
(a) What are the volume and surface area of pyramid STUXV?

s>

Type your solution, notes and/or work here. Show Solution

(b) What are the volume and surface area of pyramid S T U X ?

Hint

Type your solution, notes and/or work here. Show Solution

(c) What are the volume and surface area of pyramid STU Z ?

Type your solution, notes and/or work here. Show Solution

Extra! I have two fish tanks that are both rectangular prisms like the
one shown to the right. I have a fish that is very picky, and will
V only be happy in a tank that has exactly 50 gallons of water.
Unfortunately, my fish tanks are both 60-gallon tanks. Worse
yet, I don't have anything I can use to measure the sides of
the tank and mark off a 'fill line.’ All I have is a hose to pour
the water. How can I fill one of my tanks with exactly 50 gallons?

Extra! Back in Section 9.3 here, we noted that any polygon can be dissected and rearranged
to form any other polygon that has the same area. You might wonder if it is possible
V to dissect any polyhedron and rearrange the pieces to form any other polyhedron with
the same volume. You wouldn't be the first to wonder this! In fact, this question was
one of Hilbert’s famous problems (see Section 3.3 here). It was also the first to be
solved, by Max Wilhelm Dehn. Dehn used the powerful problem solving technique of
invariants to solve the problem. He created a function of the edge lengths and the
dihedral angles of a polyhedron and showed that this function must be equal for any
two polyhedra that can be dissected into the same pieces. The function is different
for a cube and a tetrahedron, so these two cannot be dissected into the same pieces.
The Seven Circles Theorem

Everything in nature adheres to the cone, the cylinder and the cube. - Paul Cezanne

CHAPTER
I
Curved Surfaces

15.1 Cylinders
Instead of using congruent polygons for bases as we do with a prism, suppose we use congruent circles. The resulting solid is
called a cylinder, an example of which is shown below. The line connecting the centers of the bases is called the axis of the
cylinder (()O ' in the diagram) and the radius of a base is also considered the radius of the cylinder.

Figure 15.1: A Right Circular Cylinder

You won't be surprised to learn that if the axis is perpendicular to the bases, then the cylinder is a right cylinder as shown above.
Furthermore, we can have cylinders in which the bases are any sort of wacky curves. Therefore, our simple cylinder above is
more precisely called a right circular cylinder, where the ,circular‫ ׳‬tells us that the bases are circles. When we speak of a cylinder,
we nearly always mean a 'right circular cylinder,' instead of one of the wacky cylinders shown in Figure 15.2 below.

Figure 15.2: More Cylinders

P ro b le m s
Problem 15.1 4 Jump to Solution

(a) Suppose we take a cross-section of a cylinder that is perpendicular to the axis of the cylinder. What shape is this cross-
section?

(b) What is the shape of a cross-section of a cylinder that contains the axis of the cylinder?

(c) What is the shape of a cross-section of a cylinder that is parallel to the axis of the cylinder?

Problem 15.2 4 Jump to Solution

The figure at right shows a right circular cylinder (a.k.a. a cylinder) with radius 3 and height 5.

(a) Find the volume of the cylinder. ° j— H


(b) Suppose we cut the curved surface of the cylinder along the dotted line shown (assume the
dotted line is perpendicular to the bases) and ,unroll’ the surface into a plane. What type of ! 1
1
figure results?
/,> I
1 " N
(c) What is the lateral surface area of the cylinder? ‫׳‬: 0 * 1— ‫ ״‬3

(d) What is the total surface area of the cylinder?

(e) Find formulas for the volume, lateral surface area, and total surface area of a cylinder with radius r and height h.

Problem 15.3 4 Jump to Solution

The radius of a cylinder is 2 /3 its height. Find the total surface area of the cylinder if its volume is 96 tt.

We’ll start our investigation of cylinders by investigating a few cross-sections of a cylinder.

Problem 15.1 t V

(a) Suppose we take a cross-section of a cylinder that is perpendicular to the axis of the cylinder. What shape is this cross-
section?

(b) What is the shape of a cross-section of a cylinder that contains the axis of the cylinder?

(c) What is the shape of a cross-section of a cylinder that is parallel to the axis of the cylinder?

Solution for Problem 15.1:

(a) Intuitively, it seems clear that the cross-section is a circle. To prove this, we
consider A and A ‫ ׳‬on the bases such that .4,4‫ ׳‬is parallel to the axis ( ) ( ) ‫׳‬. We let
X and Y be the points where 0 0 ' and .4,4', respectively, meet our cross-section
plane. Since this plane is perpendicular to the axis, we have X Y ± ( ) O '. So,
X Y A O is a rectangle, and X Y = OA. Hence, X Y equals the radius of the
cylinder.

But there’s nothing special about the points A and A ' we chose on the
circumferences of the bases! In exactly the same way, we can show that any point on the intersection of our plane and the
curved surface of the cylinder is exactly the cylinder’s radius away from point X on the axis. Therefore, our cross-section is
indeed a circle.

(b) Since .4.4‫ ׳‬and B B ' are both parallel to the axis of the cylinder, they are perpendicular to both
bases. So, A A f B 'B is a rectangle.

(c) Since our cross-section is parallel to the axis of the cylinder, it must be perpendicular to both bases.
Therefore, as in the previous part, our cross-section is a rectangle.

One result of part (a) above is that the set of all points that are a fixed distance from a given line in space is a cylindrical surface,
as shown below. The surface is basically a cylinder without bases that continues forever along the line that is its axis.

Figure 15.3: A Cylindrical Surface and Its Axis

Now that we have some sense for what a cylinder is like, let's investigate the area and volume of a cylinder.

Problem 15.2 t V

The figure shows a right circular cylinder (a.k.a. a cylinder) with radius 3 and height 5.

(a) Find the volume of the cylinder.

(b) What is the lateral surface area of our cylinder?

(c) What is the total surface area of our cylinder?

(d) Find formulas for the volume, lateral surface area, and total surface area of a cylinder with radius r and
height h.

Solution for Problem 15.2:

(a) Our cylinder is just like a prism - its cross-sections have the same area from bottom to top. Therefore, its volume is just the
area of a base times the height. The area of the base is тг(32) = 9тг, so the volume is (97г)(5 ) = 457!‫־‬.

(b) We don't have any tools to deal with finding the area of a curved surface, so we’ll have to do something
clever. Suppose our cylindrical surface is made of paper, so we can cut along a segment like X X ' that
is perpendicular to both bases (and hence the same length as a height). Then we ‘unroll' the surface.
The top and the bottom are always 5 units apart (the height of the cylinder), so the unrolled surface is a
rectangle.

The unrolled surface is shown at right. One dimension of this rectangle is the circumference ot top
height of the cylinder; the other is the circumference of the cylinder. For our Xn X
4-‫׳‬
given cylinder, the height is 5 and the circumference is 27rr = (>7r. Therefore, our
‫־‬5 )
lateral surface area is (5)(67r) = 30 tt. <n_
<-»• Л
circumference of bottom r \^>
(c) For the total surface area, we add the bases to our lateral surface area. Each
base has area 97r, s o the total surface area is

ЗОтг 4 2(97г) = 4871‫־‬.

(d) We simply follow the steps we used to solve the preceding parts. The area of each base is n r 2, so the volume is 7Tr 2h. The
curved surface unrolls to form a rectangle with dimensions h and 2!r r (the circumference of each base), so it has area
2irvh. Therefore, our lateral surface area is 2 irrh and the total surface area is 2 ttr h 4 2 ‫־‬n r 2.

Important: A cylinder with height h and radius r has:

Z Volum e = n r 2h
L a te ra l Surface A rea = 2 n rh
T o ta l Surface A rea = 2 n rh 4 2‫׳‬K r2
Don’t memorize these formulas! If you take the time to understand them, they’ll
always be obvious to you.

Concept: Problems involving the curved surface of a cylinder can often be untangled by
I ,unrolling’ the curved surface into a rectangle.

Let’s try applying these formulas.

Problem 15.3 t V

The radius of a cylinder is 2 /3 its height. Find the total surface area of the cylinder if its volume is 9Ö7T.

Solution for Problem 15.3: We let the height be h, so the radius is r = 2 /i/3 . Since the volume is 96 tt, we have 7rr 2h = 96 tt.
Substituting for r gives

fiTT = 96 tt,

and solving this equation for h gives h = 6. Therefore, r = 2h/3 = 4. Now we can find our total surface area:

2 n rh + 27r r 2 = 487T 4 327T = 807!‫־‬.


!

Concept: These basic area/volume word problems are often no different from other word
problems. The key to solving them is to assign variables and use the area and volume
I information to set up equations with the given information.

E x e rc is e s

15.1.1: t V

A cylinder has radius 8 and height 4.

(a) Find the volume of the cylinder.

Type your solution, notes and/or work here. Show Solution

(b) Find the lateral surface area of the cylinder.

Type your solution, notes and/or work here. Show Solution

(c) Find the total surface area of the cylinder.

Type your solution, notes and/or work here. Show Solution

15. 1.2: t V

The diameter of a cylinder equals its height. The total surface area of the cylinder is 150tt. What is the volume of the cylinder?

Type your solution, notes and/or work here. Show Solution

15.1.3: t V

Shown at right is a circular cylinder that is not right. The bases have centers A and B as shown,
and point C is on the circumference of a base. Given that A B = 8, B C = 4, and Z.ACB = 90°,
find the following:

(a) The height of the cylinder. (Note that A B is not an altitude of the cylinder because it is not
perpendicular to the bases! Remember, the height of a cylinder or prism is the distance
between its bases.)

Type your solution, notes and/or work here. Show Solution

(b) The volume of the cylinder.

Type your solution, notes and/or work here. Show Solution

15. 1.4: t V

The lateral surface area of cylinder C equals the sum of the areas of its bases. What is the ratio of the radius of C to the height
of C?

Type your solution, notes and/or work here. Show Solution

15. 1.5: t V

A cylinder is inscribed in a square prism. As shown at left, each of the bases of the cylinder is inscribed in
a face of the prism, and the cylinder is tangent to the other four faces of the prism. Given that A B = 4
and A E = 8, find the volume of the cylinder.

Type your solution, notes and/or work here. Show Solution

15. 1.6★ : t V

Does there exist a group of seven cylinders such that it is possible to arrange them so that each cylinder is tangent (i.e.
touches at only one point) to the other six?

Type your solution, notes and/or work here. Show Solution


14.4 Regular Polyhedra
A

Figure 14.2: A Regular Tetrahedron and a Cube

We have already mentioned that a polyhedron is a solid figure that is bounded on all sides by polygons. A regular polyhedron is a
convex polyhedron in which all the faces are congruent regular polygons, and there are the same number of edges at each
vertex. We've already seen examples of two such regular polyhedra - a cube and a regular tetrahedron, examples of which are
shown in Figure 14.2. The faces of the cube are congruent squares and those of the tetrahedron are congruent equilateral
triangles.

These aren’t the only types of regular polyhedra, however. In this section we discover the other regular polyhedra, and we tackle a
problem involving one of the types of regular polyhedra.

P ro b le m s
Problem 14.11 4 Jump to Solution

To make a regular tetrahedron, we might start with four equilateral triangles in a


plane as shown at right. We then fold the triangles until A', D, and D ' coincide as
shown in the regular tetrahedron. As well see in this problem, thinking about A'<
,folding up' regular polygons like this allows us to limit the possibilities of regular
polyhedra that can be made.

(a) Consider three squares, A B C D , A B E F , and A F G H , arranged around a point as shown. If we £ B


C
'fold' this arrangement so that D and / / coincide, we'll be on our way to forming what type of regular
polyhedron? p D
/1
(b) Can we fit three regular pentagons around a point in a plane? Can we then fold the resulting figure to
G H
start to form a polyhedron?

(c) Can we fit three regular hexagons around a point in a plane? Can we then fold the resulting figure to start to form a
polyhedron?

(d) Can we fit four equilateral triangles around a point in a plane? Can we then fold the resulting figure to start to form a
polyhedron? How about four squares?

(e) Can we fit five equilateral triangles around a point in a plane? Can we then fold the resulting figure to start to form a
polyhedron? How about five squares?

(f) At most how many different types of regular polyhedra are there?

Problem 14.12 4 Jump to Solution

A B C D E F shown is a regular octahedron, which has eight congruent equilateral triangles as


✓AI
faces. We can also think of a regular octahedron as a pair of square pyramids glued together at
their bases. In this problem, let A B = 4. / \ \
(a) Find B D .
B-::— - v ___
(b) Find A F .

(c) Find [B C D E ].

(d) Find the volume of A B C D E F . F

There aren’t many types of regular polyhedra. With a little physical intuition, we can discover which types of polyhedra might
possibly exist.

Problem 14.11 t V

To make a regular tetrahedron, we might start with four equilateral triangles in a


plane as shown at right. We then fold the triangles until A, D, and D ' coincide as
shown in the regular tetrahedron. As we'll see in this problem, thinking about •
'folding up’ regular polygons like this allows us to limit the possibilities of regular
polyhedra that can be made.

(a) Consider three squares, A B C D , A B E F , and A F G H , arranged around a point as shown. If we


'fold' this arrangement so that D and / / coincide, we'll be on our way to forming what type of regular
polyhedron?

(b) Can we fit three regular pentagons around a point in a plane? Can we then fold the resulting figure to
start to form a polyhedron?

(c) Can we fit three regular hexagons around a point in a plane? Can we then fold the resulting figure to start to form a
polyhedron?

(d) Can we fit four equilateral triangles around a point in a plane? Can we then fold the resulting figure to start to form a
polyhedron? How about four squares?

(e) Can we fit five equilateral triangles around a point in a plane? Can we then fold the resulting figure to start to form a
polyhedron? How about five squares?

(f) At most how many different types of regular polyhedra are there?

Solution for Problem 14.11:


B C
(a) Folding up our squares gives the figure shown, which is clearly well on its way to being a cube.
D=H
(b) We can fit three pentagons around a point with room to spare. Therefore, we can fold the pentagons to
possibly form part of a regular polyhedron. Such a polyhedron does indeed exist, as shown below. This
polyhedron is called a regular dodecahedron. It has 12 regular pentagons as its faces.

We can fit three regular hexagons around a point, but since each angle of a regular hexagon is 120°, the
three fit snugly. In other words, we can’t bend the hexagons together as we’ve done earlier with the
pentagons, squares and triangles. Therefore, there are no more regular polyhedra that have three of
each polygon meeting at a point.

Four equilateral triangles fit around a point with room to spare, so we can fold them together. The result
is a square pyramid without its base. Gluing another square pyramid to this one gives us a polyhedron with eight congruent
equilateral triangles as faces. This polyhedron, shown in the diagram below, is a regular octahedron.

Four squares, of course, fit snugly around a point, since 4(90 ') = 360°. Hence, we can’t fold the squares to make a
polyhedron.

(e) As with three and four equilateral triangles, we can fit five triangles around a point, then fold them together. This gives us
the start of a regular icosahedron, which has twenty congruent equilateral triangles as its faces.

Obviously, we can’t fit five of any other regular polygon around a point.

(f) We can fit six equilateral triangles snugly around a point, so we can't ,fold’ them into the start of a
regular polyhedron. Obviously, any regular polyhedron must have at least three faces meet at each
vertex, so we’ve exhausted all the possibilities for regular polyhedra.

Important: There are five regular polyhedra, which are described below.

Z Name Face Type # Faces # Edges # Vertices


Tetrahedron Triangle 4 6 4
Cube Square 6 12 8
Octahedron Triangle 8 12 6
Dodecahedron Pentagon 12 30 20
Icosahedron Triangle 20 30 12

These five regular polyhedra are collectively named the Platonic solids, after the great
Greek philosopher Plato.

Icosahedra and dodecahedra are pretty hard to work with, but since regular octahedra are just two square pyramids stuck
together, they're much more manageable.

Problem 1 4.1 2 t V

A B C D E F shown is a regular octahedron with A B = 4.

(a) Find B D .

(b) Find A F .

(c) Find [B C D E ).

(d) Find the volume of A B C D E F .

Solution for Problem 14.12:

(a) Since B C D F is a square, B D = B C \ / 2. All the sides of a regular polyhedron have the same length, so
B C = A B = 4 and B D = 4 \/2 .

(b) Instead of thinking of the octahedron as square pyramids A B C D E and F B C D E glued together, we can think of it as
B A E F C and D A E F C glued together. Therefore, A C F E is also a square, so A F = B D = 4 \/2 . (We can also just
note that since the polyhedron is regular, the distance between directly opposite vertices must be the same no matter which
pair of opposite vertices we choose.)

(c) B C D F is a square with side length 4, so its area is 42 = 16.

(d ) To find the volume of the regular octahedron, we think of it as two congruent square pyramids, A B C D E and F B C D E .
Since the altitudes from A and from F , respectively, of these two pyramids together make up A F , the height of each
pyramid is A F /2 = ( 4 \/2 ) /2 = 2 \P l. The area of a base is 42 = 16, so the volume of each pyramid is
( l6 ) ( 2 \ / 2 ) / 3 = 3 2 \/2 /3 . Therefore, our octahedron has volume 2 (3 2 \/2 /3 ) = 6 4 \/2 /3 .

E x e rc is e s

14.4.1: t V

Compute the quantity

(Number of vertices) — (Number of edges) + (Number of faces)

for each regular polyhedron (cube, tetrahedron, octahedron, dodecahedron, icosahedron). Notice anything interesting? Try it on
other polyhedra, such as various types of prisms and pyramids and see if your interesting observation still holds.

Type your solution, notes and/or work here. Show Solution

14.4.2: t V

The six vertices of a regular octahedron are snipped off, leaving a square face in place of each corner and a hexagonal face in
place of each original face of the octahedron. How many vertices, faces, and edges will the new polyhedron have?

Type your solution, notes and/or work here. Show Solution

14.4.3★ : t V

A B C D E F shown at right is a regular octahedron with side length 1. Let O be the center of face
A B C , P be the center of face A C D , and Q be the center of face D F F .

(a) Find O P .
B
Hint

Type your solution, notes and/or work here. Show Solution

(b) ★ Find OQ.

Hint

Type your solution, notes and/or work here. Show Solution

(c) Find [O PQ ].

Hint

Type your solution, notes and/or work here. Show Solution


14.5 Summary

Definitions: The volume of a three dimensional figure is a measure of the space inside the figure.
The total surface area of a figure is the total area of all the surfaces that form a
boundaries of the solid. The lateral surface area is the total area of all the surfaces
that are not considered 'bases'.

Definitions: A polyhedron is a solid figure with polygons as its boundaries. A prism has two
congruent parallel faces as bases and all remaining faces (called sides) are
parallelograms. In a right prism all of these side faces are rectangles. The bases are
used to describe the prism, as in 'right rectangular prism’ (shown below) or ,hexagonal
prism’.

Important: The three dimensions of a right rectangular prism are commonly


/
called the length, /, the width, w, and the height, h. For such a
Z prism, we have: \ w
,\ N|
Volum e = Iw h
Surface area = 2 (Iw 4‫ ־‬wh 4‫ ־‬Ih)
\ \\j t.
\\
Space D iagonal = \ / / 2 4‫ ־‬w 2 4‫ ־‬h? \ \

The volume of a prism equals the area of the base times the distance between the
bases (i.e. the height).

Definition: A cube is a special right rectangular prism in which all the edge
lengths are the same (i.e., its base is a square and its height has
the same length as a side of the base).

Important: A cube with side length s has:

Z Volum e = s"
Surface area = 6s 2
Space D iagonal = s \/3

Definitions: If we connect all the vertices of a polygon to a point that is not in the
same plane as the polygon, we form a pyramid. This point is called the
apex of the pyramid and the polygon is the pyramid's base. As we can
see at right, the non-base faces of a pyramid are all triangles. The lateral
surface area of a pyramid is the sum of the areas of these triangles. A
tetrahedron is a pyramid with a triangular base.

The height of a pyramid is the distance from the apex to the base. If the base is a
regular polygon, the pyramid is a regular pyramid. For regular pyramids, we also
define a slant height, which is the distance from the apex to a side of the base.

Important: The volume of a pyramid is one-third the product of the pyramid's height and the
area of the pyramid's base.
Z
The lateral surface area of a regular pyramid equals one-half the product of the
slant height and the perimeter of the pyramid's base.

Definitions: A regular polyhedron is a polyhedron whose faces are all congruent regular polygons.

Important: There are five regular polyhedra, which are described below.

Name Face Type # Faces ffEdges # Vertices


Tetrahedron Triangle 4 6 4
Cube Square 6 12 8
Octahedron Triangle 8 12 6
Dodecahedron Pentagon 12 30 20
Icosahedron Triangle 20 30 12

P ro b le m S o lv in g S tra te g ie s

Concept:
Building right triangles works in three dimensions every bit as well as it does in two
dimensions.
15.4 Problems
The following problems are challenging extensions of the material in this chapter and the previous chapter. Each problem
illustrates useful problem solving techniques for three-dimensional geometry problems. Rather than write the terms ,two-
dimensional' and ,three-dimensional' repeatedly, we'll frequently use '2-D1and ,3-D' instead.

P ro b le m s
Problem 15.13 4 Jump to Solution

My company produces 6 inch tall cans of paint. The cans have a radius of 15 inches. (Yes, these are very oddly shaped cans!)
We want to offer a super-size can of paint. Toby wants to leave the can just as wide as it is, but increase the height by x inches.
Maryanna thinks the fat cans sell better, so she wants to make them even wider. She wants to increase the radius by x inches.
After arguing for a while, they decide to use the design that produces the greatest increase in paint volume. They reach for their
scratch paper and figure out how much more paint is needed to fill their new can designs. They both use the same value of x,
and they find that both designs require the same increase in paint volume! What value of x did they use?

Problem 15.14 4 Jump to Solution

The areas of three of the faces of a rectangular prism are 24,32, and 36. In this problem we will find the volume of the prism.

(a) Let the dimensions of the prism be x, y, and 2 . Use the information given to find three equations.

(b) In terms of x, y, and 2 , what is the volume of the prism?

(c) How can you combine your equations from the first part by adding, subtracting, and/or multiplying them to find the volume
of the prism?

Problem 15.15 4 Jump to Solution

Cube S T U V W X Y Z is shown. Find / W Y T .

Problem 15.16 4 Jump to Solution

A ball with radius 6 inches is tangent to the wall and the floor in my house. Given that the wall is perpendicular to floor, what is
the radius of the largest ball I can slide into the space between the ball and the corner where the wall meets the floor?

Hint

Hint

Problem 15.17 4 Jump to Solution

Three congruent spheres are inside a cylinder such that each sphere is tangent to the other two spheres, to both bases of the
cylinder, and to the curved surface of the cylinder. In this problem we will find the volume of the cylinder given that the radius of
each sphere is 12.

(a) Find the height of the cylinder.

(b) Draw a cross-section of the figures in the problem that contains the centers of the spheres. What kind of triangle is formed
by connecting the centers of the spheres?

(c) Let X be the point where your cross-section intersects the axis of the cylinder. How is X related to the triangle mentioned
in part (b)?

(d) Find the radius of the cylinder, then the volume of the cylinder.

Problem 15.18 4 Jump to Solution

Adam the Ant is at point A of cube A B C D E F G H with edge length 1. He wishes to walk along the surface of the cube to
point G, where G is the vertex of the cube such that A G is a space diagonal of the cube. What is the length of the shortest
path Adam can take?

Problem 15.19 4 Jump to Solution

Adam the Ant is now on point A of the cylinder shown. He wishes to walk along the curved surface of
the cylinder to point B, where B C is a height of the cylinder and C is on the top base such that A C is
a diameter as shown. Given that the cylinder has height 4 and radius 3, what is the length of the
shortest path Adam can take?

We start off with a couple problems in which we apply algebra to geometry problems. Then we II move on to challenging spacial
geometry problems.

Problem 15.13 t V

My company produces 6 inch tall cans of paint. The cans have a radius of 15 inches. (Yes, these are very oddly shaped cans!)
We want to offer a super-size can of paint. Toby wants to leave the can just as wide as it is, but increase the height by x inches.
Maryanna thinks the fat cans sell better, so she wants to make them even wider. She wants to increase the radius by x inches.
After arguing for a while, they decide to go with the design that produces the greatest increase in paint volume. They reach for
their scratch paper and figure out how much more paint is needed to fill their new can designs. They both use the same value of
x, and they find that both designs require the same increase in paint volume! What value of x did they use?

Solution for Problem 15.13: We are given

Volume (Toby’s design) = Volume(Maryanna’s design),

so we find each volume in terms of x:

7t(152)(6 4‫ ־‬x ) = 7t( 15 4‫ ־‬x ) 2(6).

After a few lines of algebra, we have x (2 x — 15) = 0. Since x must be nonzero (otherwise we wouldn’t be making much of a
change to the can design!), we have x = 15/2.

We can quickly check our work by considering the dimensions of the new cylinders. Toby’s has radius 15 and height
64 2 7 / 2 = 15/2 ‫־‬, so it has volume 7t( 152)(2 7 /2 ) = 7r(35 • 52)/2 . Maryanna’s has height 6 and radius 15 4 4 5 /
so it has volume 7t(4 5 /2 )2(6) = 7r(35 • 52)/2 . (Notice we don't have to bother multiplying all those numbers out. Looking at
the prime factorizations allows us to quickly verify that the volumes are the same!)

So, they must have both been using a value of 15/2 = 7.5 for x. □

Concept: For straightforward problems involving the volume and/or surface area of basic
I solids, usually we just turn the words of the problem into equations using volume and
' area formulas. Then we solve the equations, check our work, and make sure we
answer the question that is asked.

Problem 15.14 t V

The areas of three of the faces of a rectangular prism are 24,32, and 36. Find the volume of the prism.

Solution for Problem 15.14: We start off by assigning variables and writing the information in the problem as equations. We let
the dimensions of the prism be x, y, and 2 , so the given information about areas of faces becomes:

xy = 24
xz = 32
yz = 36

We seek the volume, which in terms of x, y, and 2 is just xyz. We notice from the nicely symmetric form of the left-hand sides of
our equations {xy, yz, zx) that if we multiply all three, we'll get x 2y 2z2 Specifically, we find that multiplying all three equations
gives x 2y 2z2 = (24)(32)(36). Taking the square root of this yields x y z = y j (2 4 )(32)(36) = 9(>>/3, which is the volume of
the prism.

Notice that we didn't even find x, y and z\ But now that we have xyz, we can find them quickly. You'll get a chance to do so as an
Exercise, z

Concept: Sometimes turning a geometry problem into algebra results in a nice symmetric
I system of equations, as we saw in Problem 15.14. We often solve such symmetric
systems by adding or multiplying all the given equations.

Now we turn to using our two-dimensional problem solving strategies on three-dimensional problems.

Problem 15.15 t V

Cube S T U V W X Y Z is shown. Find/ W Y T . ...

‫״‬K 4 4 a y
"ïÀ
i b

Solution for Problem 15.15: We don't really know how to handle angles in three dimensions, so we
consider a two-dimensional figure that includes our angle. In other words, we consider the cross-section
that contains / W Y T . We form this cross-section by cutting through vertices W , Y , and T of the cube,
as shown. Thus, our cross-section is a triangle. Moreover, we note that since each of the sides of
A W Y T is a diagonal of a face of the cube, the sides of A W Y T are all equal. Hence, A W Y T is
equilateral, so each of its angles, including /.W Y T , is 60° □

Concept: Most 3-D problems are 2-D problems in disguise. Considering cross-sections that
• include important pieces of the problem often allows us to use our two-dimensional
strategies to find the solution.

We'll try a couple more 3-D problems that can be solved with well-chosen cross-sections.

Problem 15.16 t V

A ball with radius 6 inches is tangent to the wall and the floor in my house. Given that the wall is perpendicular to floor, what is
the radius of the largest ball I can slide into the space between the ball and the corner where the wall meets the floor?

Solution for Problem 15.16: We have a 3-D problem involving spheres that are tangent to
each other and to some planes, so we consider a cross-section including the centers of
the spheres and points of tangency. Hopefully this allows us to build helpful right
triangles. (Tangents mean right angles!)

O and P are the centers of our spheres, which are tangent at X . These spheres are
tangent to the floor at M and N as shown. Sphere O is tangent to the wall at L. Since
the spheres are tangent to the wall and the floor, O and P are equidistant from both.
Therefore, both O and P are on the angle bisector of X L C M as shown. Since
/ L C M = 90°, we have / O C M = / P C N = 45°. So, A O C M is an isosceles
right triangle. Similarly, A P C N is an isosceles right triangle, so
P C = P N V 2 = 6 v 2 ‫׳‬. and X C = P C - P X = 6 \/2 - 6. If we let the radius of
our small sphere be r, we now know that

O C = X C - r = 6 \/2 - 6 - r.

However, from A O C M , we also have

O C = O M s / 2 = r v 2 ‫׳‬.

Therefore, we have two expressions for O C , which we can set equal:

r V 2 = 6 \/2 — 6 — r.

Therefore, r 4‫ ־‬r V 2 = 6 \/2 - 6, so

6 \/2-6
r =
1+ ^ 2 ‫־‬

We can rationalize the denominator by multiplying the top and bottom of the fraction by 1 — \[2 \

_ W 2 -6 1 - ^ _ —18 + ■2V2 =
l+ v /2 l- v /2 -1

Concept: A great many geometry problems are solved by assigning a variable to a length, then
I finding two different expressions for some other length in terms of that variable. We
can then set these expressions equal to solve for the variable.

Problem 15.17 t V

Three congruent spheres are inside a cylinder such that each sphere is tangent to the other two spheres, to both bases of the
cylinder, and to the curved surface of the cylinder. Find the volume of the cylinder given that the radius of each sphere is 12.

Solution for Problem 15.17: Since each sphere is tangent to the top and bottom base of the cylinder (and these bases are
parallel), the height of the cylinder must equal the diameter of the sphere. Thus, the height of the cylinder is 2(12) = 24.

To get the radius of the cylinder, we start with a well-chosen cross-section. We have
spheres and tangency, so we take the cross-section that includes the centers of the
three spheres, the points where the spheres touch each other, and the points where
spheres touch the cylinders. Our cross-section is shown at right. A A B C connects the
centers of our spheres, and O is where the cross-section meets the axis of the cylinder.

O is equidistant from P, Q and R because the cross-section of the cylinder is a circle


with center O. Since 0^4, © B, and © C ‫ ׳‬are tangent to ©O, rays ~rX, Q i , and P $ all
pass through O. O is therefore equidistant from A, B, and C because
O R = O Q = O P and A R = B Q = C P . So, O is the circumcenter of A A B C .
Since O P = O C + C P , we only need to find O C to find the radius of the cylinder.

A A B C is equilateral because each side equals twice the radius of a sphere. Therefore,
O is also the incenter and the centroid of A A B C . Moreover, the points where the
spheres’ cross-sections touch each other are midpoints of the sides of A A B C . Hence,
A O X C is a 30-60-90 right triangle because O C bisects / A C B and O X ± A C (make sure you see why). Therefore,

2X C 24
OC = 20 X = = 8 n/3 .
Vs Vs

The radius of the cylinder is then O C 4‫ ־‬C P = 8 \/3 4-12. Thus, our volume is:

7r r 2h = 247t(8 \/3 42(12 ‫־‬


= 24 tt(42) ( 2 v/3 42(3‫־‬

= (24 • 16)tt(21 4- 12s/S)

= 80647T 446087 ‫־‬t \/3 .

Concept: To solve 3-D problems involving a sphere, consider cross-sections including the
I center of the sphere. If you have multiple spheres in the problem, try finding a cross-
' section that includes multiple centers. If the sphere is tangent to anything, try
including the point of tangency.

Sometimes the 2-D problem is more subtly hidden in our 3-D problem. In these cases we need more than a simple cross-section
to coax it out.

Problem 15.18 t V

Adam the Ant is at point A of cube A B C D E F G H with edge length 1. He wishes to walk along the surface of the cube to
point G, where G is the vertex of the cube such that A G is a space diagonal of the cube. What is the length of the shortest
path Adam can take?

Solution for Problem 15.18: We might think that Adam should walk along edge A B and then along face
diagonal B G . However, if we put a big glob of jelly at G and put ants at A, the ants will quickly find a
shorter path. They might go diagonally across face A B F E to a point X on B E , then on to point G. We
suspect point X will be the midpoint of B E , but how can we be sure? (And worse yet, what if
A B C D E F G H isn't a cube? You'll get a shot at dealing with that complication in the Exercises, of
course!)

A B C We make our 3-D problem into a 2-D problem by unfolding the cube and placing the two faces
Adam crawls across in the same plane as shown. This doesn’t change the distance Adam must
crawl at all, but it does make our answer clear. Adam's shortest path is the straight segment
from A to G. Since A B = B C = C G = 1, we have A G = V C G 2 4‫ ־‬A C 2 = V s .
Therefore, the shortest path Adam can take has length Vs. Notice that since A B = F G and
A C || E G , we can use congruent triangles A A B X = A G F X (by ASA) to show that
B X = E X , i.e., that Adam's shortest path indeed goes through the midpoint of one of the
edges of the cube. □

Concept: The 2-D problem lurking in a 3-D problem isn't always just a cross-section. Sometimes
I unfolding or unrolling a 3-D figure reveals a simpler 2-D problem.

Problem 15.19 t V

Adam the Ant is now on point A of the cylinder shown. He wishes to walk along the curved surface of the
cylinder to point B, where B C is a height of the cylinder and C is on the top base such that A C is a
diameter as shown. Given that the cylinder has height 4 and radius 3, what is the length of the shortest
path Adam can take?

Solution for Problem 15.19: When we found the lateral surface area of a cylinder, we
discovered how to deal with problems involving the curved surface of a cylinder - we unroll
the cylinder. We try that here and we get the rectangle shown at right.

B C still equals the height of the cylinder, so B C = 4. However, here A C is not a diameter
of the original cylinder, but rather the distance from A to C along the circumference of the
base. Therefore, A C is half the circumference of the base of our cylinder, or 3 tt. From right triangle A A B C we can now find

A D = s / A C 2 + D C 2 = v‫ ׳‬lC + !hr2.

(Note that this is not the distance from A to B in space in our original cylinder. This is the length from A to B along the curved
surface of the cylinder. Also, note that if Adam is willing to walk across a base of the cylinder, there is a shorter path he can take!)

E x e rc is e s

15.4.1: t V

The areas of three of the faces of a rectangular prism are 24,32, and 36. Find the dimensions of the prism.

Hint

Type your solution, notes and/or work here. Show Solution

15.4.2: t V

A B C D is a regular tetrahedron. G is the centroid of A A B C and / / is the centroid of A B C D . Given A B = 8, find G H .

Hint

Type your solution, notes and/or work here. Show Solution

15.4.3: t V

W X Y Z is a regular tetrahedron such that W X = 4. M is the midpoint of W X and N is the midpoint of Y Z .

(a) Prove that M N J_ W X .

Type your solution, notes and/or work here. Show Solution

(b) ★ Find M N .

Type your solution, notes and/or work here. Show Solution

15.4.4: t V

Three spherical balls are snugly in a row inside a cylindrical can such that the first is tangent to the lid of the can and the
middle ball, the second is tangent to the other two balls, and the third is tangent to the bottom of the can and the middle ball.
Each ball is fits snugly inside the can so that it is tangent to the curved surface of the can and cannot move side-to-side. Given
that the radius of each ball is 2 inches, what is the volume of the can?

Type your solution, notes and/or work here. Show Solution

t V

Annie the ant is on vertex A of right rectangular prism A B C D E F G H shown. Given that A B = 4, A D
B C = 6, and A E = 8, what is the shortest distance Annie can walk to reach vertex G? \ B

\ H

Type your solution, notes and/or work here. Show Solution

15.4.6: t V

Spheres S and T have radii 6 and 8, respectively, and centers O and P, respectively. O P = 10.

(a) Why must the spheres intersect?

Type your solution, notes and/or work here. Show Solution

(b) Let X be a point that is on both spheres. What is X 0 7 What is X P i

Type your solution, notes and/or work here. Show Solution

(c) What kind of triangle is A X O P l

Type your solution, notes and/or work here. Show Solution

(d) Let the altitude from X to O P meet O P at Y . What is X Y ?

Type your solution, notes and/or work here. Show Solution

(e) What is O K?

Type your solution, notes and/or work here. Show Solution

(f) Suppose we pick another point Z that is on both spheres. Must X Y = Z Y ?

Type your solution, notes and/or work here. Show Solution

(g) Use the previous part to show that the intersection of the two spheres is a circle.

Type your solution, notes and/or work here. Show Solution

(h) Find the area of this circle.

Type your solution, notes and/or work here. Show Solution

15.4.7: t V

B
&
4 77'

A B C D E F G H is a cube as shown. M is the midpoint of F G and N is the midpoint of G H . Given that A B = 6, find the
following:

(a) MN.

Type your solution, notes and/or work here. Show Solution

(b) AM.

Type your solution, notes and/or work here. Show Solution

(c) ■ k[A M N \

Hint

Type your solution, notes and/or work here. Show Solution

(d) ★ the volume of A E M N.

Hint

Type your solution, notes and/or work here. Show Solution

15.4.8: Source: MATHCOUNTS t V

The height of a cylindrical pole is 12 feet and its circumference is 2 feet. A rope is attached to a point on the circumference at
the bottom of the pole. The rope is then wrapped tightly around the pole four times before it reaches a point on the top directly
above the starting point at the bottom. What is the minimum number of feet in the length of the rope?

Type your solution, notes and/or work here. Show Solution

15.4.9★ : Source: MATHCOUNTS t V

Two regular square pyramids have all edges 12 cm in length. The pyramids have parallel bases and parallel edges, and each
has a vertex at the center of the other pyramid's base. What is the total number of cubic centimeters in the volume of the solid
of intersection of the two pyramids?

Hint

Type your solution, notes and/or work here. Show Solution

Extra! We saw in here that squares, hexagons, and triangles are the
only regular polygons that will tile the plane. Pentagons, with
V their quirky 108° angles, simply can't add up to 360°, no matter
how many of them get together at a vertex. But, on a sphere,
pentagons get their due! We can view the dodecahedron we
discovered in Section 14.4 here (and shown at right) as a tiling
of a sphere with regular pentagons.

Each of the other types of polyhedra can be considered a method of tiling a sphere
with regular polygons. Notice that while there is only one way to tile a plane with
equilateral triangles, there are three ways to tile a sphere with them!

There's also one well-known example of a tiling that uses both hexagons and
pentagons - it’s commonly known as a soccer ball. This fabulous structure has been
around since before the invention of soccer, too, in the form of
\Def{buckminsterfullerene}, C &a a recently discovered form of carbon. Drs. Richard
Smalley and Robert Curl received the Nobel Prize in 1996 for that discovery.

Extra! The mathematician Johannes Kepler wondered how densely spheres can fill space.
The typical stacking you see for oranges at the grocery store fills just 74% of space.
V Is there a different arrangement that gets more oranges in the same space? It took
nearly 400 years before mathematicians Thomas Hales and Samuel Ferguson were
able to answer Kepler’s question and prove in 1998 what grocers have known all
along: there isn't a better way to pack oranges.
Review Problems

14. 13: t V

A diagonal of one of the faces of a given cube has length 4.

(a) Find the length of an edge of the cube.

Type your solution, notes and/or work here. Show Solution

(b) Find the length of a space diagonal of the cube.

Type your solution, notes and/or work here. Show Solution

(c) Find the total surface area of the cube.

Type your solution, notes and/or work here. Show Solution

(d) Find the volume of the cube.

Type your solution, notes and/or work here. Show Solution

14.14: t V

why?
\r U
W
l
A
\ \

Type your solution, notes and/or work here. Show Solution

14. 15: t V

A right rectangular prism has space diagonal 3 \ / l 3 and two sides of length 3 and 7, respectively.

(a) Find the third dimension of the prism.

Type your solution, notes and/or work here. Show Solution

(b) Find the volume of the prism.

Type your solution, notes and/or work here. Show Solution

(c) Find the total surface area of the prism.

Type your solution, notes and/or work here. Show Solution

14.16: t V

A space diagonal of cube A is an edge of cube B.

(a) Find the ratio of the surface area of A to the surface area of B.

Type your solution, notes and/or work here. Show Solution

(b) Find the ratio of the volume of A to the volume of B.

Type your solution, notes and/or work here. Show Solution

14.17: t V

V W X Y Z is a right square pyramid with square base W X Y Z . Given Y Z = 10 and Y V = 13 n/ 2, find the following:

(a) the height of the pyramid.

Type your solution, notes and/or work here. Show Solution

(b) the slant height of the pyramid.

Type your solution, notes and/or work here. Show Solution

(c) the total surface area of the pyramid.

Type your solution, notes and/or work here. Show Solution

(d) the volume of the pyramid.

Type your solution, notes and/or work here. Show Solution

14.18: t V

A/, N , O, and P are midpoints of edges A B , B C , C D , and D A , respectively, of right rectangular prism
A B C D E F G H shown. Q, /?, S, and T are the midpoints of edges E F , E G , G H , and B E , va
respectively. Given A B = 8, B C = 10, and B E = 5, find the following:
pi
_1__
1

(a) MN.
\
Type your solution, notes and/or work here. Show Solution

(b) MO.

Type your solution, notes and/or work here. Show Solution

(c) ★ the volume of M N O P Q R S T .

Type your solution, notes and/or work here. Show Solution

14. 19: t V

What figure is formed by connecting the centers of the faces of a:

(a) cube?

Type your solution, notes and/or work here. Show Solution

(b) regular octahedron?

Type your solution, notes and/or work here. Show Solution

(c) regular tetrahedron?

Type your solution, notes and/or work here. Show Solution

(d) regular dodecahedron?

Type your solution, notes and/or work here. Show Solution

(e) regular icosahedron?

Type your solution, notes and/or work here. Show Solution

14.20: t V

Shown is cube A B C D E F G H with side length 8.

(a) Find [A C F \ V a C

Hint
\ \

Type your solution, notes and/or work here, Show Solution

(b) Find [A C G \

Type your solution, notes and/or work here. Show Solution

14.21 t V

How many edges does a regular hexagonal prism have?

Type your solution, notes and/or work here. Show Solution

14.22: Source: MATHCOUNTS t V

This shape at right is folded into a regular octahedron with the equilateral triangles shown as faces.
What is the sum of the numbers on the faces sharing an edge with the face with a "1 “ on it?

Type your solution, notes and/or work here. Show Solution

14.23: t V

A B C D E F G is a right pyramid with regular hexagonal base A B C D E F and apex G with A B = 6 and A G = 6 \/3 .

(a) Find the volume of A B C D E F G .

Type your solution, notes and/or work here. Show Solution

(b) Find the total surface area of A B C D E F G .

Type your solution, notes and/or work here. Show Solution

14.24: t V

Find the volume and the surface area of a regular tetrahedron with side length 9.

Type your solution, notes and/or work here. Show Solution

14.25: t V

On the center of each face of a cube with side length 4 inches, we glue a cube of side length 1 inch (so that an entire face of
each small cube is glued to the big cube). We then paint the entire resulting figure red. How many square inches of red paint
will we use?

Type your solution, notes and/or work here. Show Solution

14.26: t V

W X Y Z is a triangular pyramid with X Y = Y Z = Z X = 9 and W X = W Y = W Z = 18. Let G be the centroid of


A X Y Z and M the midpoint of X Y.

(a) Prove that W G is perpendicular to plane X Y Z. (If you're stuck, check out Problem 14.10.)

Type your solution, notes and/or work here. Show Solution

(b) Find W M .

Type your solution, notes and/or work here. Show Solution

(c) Find Z M .

Type your solution, notes and/or work here. Show Solution

(d) Find the volume of W X Y Z .

Type your solution, notes and/or work here. Show Solution

14.27: Source: HMMT t V

In a cube with side length 6, what is the volume of the tetrahedron formed by any vertex and the three vertices connected to
that vertex by edges of the cube?

Type your solution, notes and/or work here. Show Solution

14.28: t V

Each of the three lines k, m, and n is perpendicular to the other two lines. All three lines pass through point P. K is on line k,
M is on line m, and N is on line n such that K P = 7, K M = 9, and K N = 11.

(a) Find the length of M N .

Type your solution, notes and/or work here. Show Solution

(b) Find the volume of K N M P.

Type your solution, notes and/or work here. Show Solution

(c) ★ Find the length of the segment from K to the midpoint of M N .

Hint

Hint

Type your solution, notes and/or work here. Show Solution

14.29: t V

In tetrahedron A /N O P , we have M N = OP, M O = N P, and M P = NO. Prove that Z A I N O = Z M P O

Type your solution, notes and/or work here. Show Solution

14.30: t V

Find a formula for the volume of a regular octahedron with edge length s.

Type your solution, notes and/or work here. Show Solution


15.2 Cones
Just as turning the bases of a prism into circles makes a cylinder, we can turn the base of a pyramid
into a circle to get a cone. The figure on the right shows a right circular cone. The point A at the tip of
the cone is the vertex of the cone and the distance from the vertex to the base is the height The line
connecting the vertex to the center of the base is the axis of the cone. The radius of the base is
considered the radius of the cone, and for right circular cones, the distance from the vertex to a point
on the circumference of the base is the slant height.

As with cylinders, cones don't have to be 'right' or 'circular.' On the left are a couple of these
less common cones. Despite the existence of these weird cones, when we simply say
,cone,‫ ׳‬we almost always mean ,right circular cone.‫׳‬

Having already learned that the volume of a pyramid equals one-third its height times the
area of its base, you won’t be surprised to learn that the same holds for a cone.

Important: The volume of a circular cone with height // and radius r is

1 21
Volum e = - 7r r rl.

P ro b le m s
Problem 15.4

(a) What is the shape of a cross-section of a cone that contains the axis of the cone?

(b) What is the shape of a cross-section of a cone that is perpendicular to the axis of the cone?

In this problem we will find a formula for the lateral surface area of a cone with base radius r and
slant height /.
(a) Use the result of Problem 14.9 to take a guess at the formula.

(b) Let A be the vertex of the cone and B a point on the circumference of the cone. Suppose we
cut the curved surface of the cone along A B , then 'unroll' it into a flat figure. What type of
figure do we thus form?
(c) How long is the curved portion of the figure you make in (b)? Use this information to find a
formula for the lateral surface area of the cone.

Problem 15.6

Find the volume of a right circular cone that has radius 6 and slant height 8.

Problem 15.7

A semicircle is rolled up to form a cone as shown. If the radius of the semicircle is 8, what is the volume of the cone’

B=B‘

Problem 15.8

A cone with vertex A. height A B = 9, and radius B C = 12 is given. The cone is cut in two pieces by a
plane perpendicular to A B at point X , where A X = 6.
(a) The top piece, of course, is a cone. What is the height of this cone? What is the volume of this cone?

(b) What is the volume of the other piece?


(c) What is the ratio of the volume of the little cone to the volume of the large cone? Is this surprising?

We start by getting a feel for cones by looking at some cross-sections.

t V
Problem 1 5.4 __________________________________________________

(a) What is the shape of a cross-section of a cone that contains the axis of the cone?

(b) What is the shape of a cross-section of a cone that is perpendicular to the axis of the cone?

Solution for Problem 75.4:


(a) A cross-section of a cone that contains the cone's axis consists of two segments of equal length
connecting the vertex to two points on the circumference of the base of the cone, as well as the
segment connecting these two points along the base of the cone. So, our cross-section is an isosceles
triangle, such as A A B C in the diagram at right.

(b)
Intuitively, it seems clear that a cross-section of a cone perpendicular to the axis of the cone is a
circle. Suppose the plane of our cross-section meets the axis at O'. To prove our cross-section is a
circle, we must show that every point where our plane hits the curved surface of the cone is
equidistant from O'. Consider point B ‫׳‬, the intersection of A B and our cross-section plane, as
--------- -^TT- .. . T 7 T __ I____ n / f M II /?/■) T h ^ ro fn rp
shown. Since /?7O 7 and B O are each perpendicular to AO , we have B ’ O ‫ || ׳‬B O . Therefore,
A A O 'B ' ~ A A O B . Since A A O 'B ' ~ A A O B , we have B 'O '/ B O = A O /A O . Therefore,
we find B 'O ' = ( A O '/A O ) ( B O ) . Since B O is just the radius of the cone and A O is the cone's
height we have B 'O ' = ( r /h ) ( A O ') . Similarly, we can show that all points of the cross-section are
( r / h ) ( A O ') away from O'. Since A O ' is fixed, all the points of our cross-section are the same
distance from O'. Therefore, the cross-section must be a circle. (Make sure you see why every point on this circle must be in
the cross-section.)

While the proof of the volume formula for cones requires some very advanced tools, we can find a formula for the area of the
curved surface of a cone with techniques we already have.

t V
Problem 1 5 . 5 ___________________________________________________________________

Find a formula for the lateral surface area of a right circular cone with base radius r and slant height l.

Solution for Problem 15.5: Since cutting and unrolling the curved surface
was so successful in finding the lateral surface area of a cylinder, we try
it with a cone as well. We cut the curved surface of the cone along AB,
where A is the vertex and B is a point on the circumference of the base.

Since every point on the circumference of the cone's base is the same
distance from the cone's vertex (the slant height), when we unroll the
curved surface, these points will still all be the same distance from the
vertex. Hence, our ,unrolled' surface is a sector of a circle as shown at
right above. (B and B ' coincide when the sector is rolled up to form a
cone.)
The radius of this sector is AB, the slant height of the cone. To find the area of the sector, we must determine what portion of a

whole circle the sector is. We know that the length of B B ' is just equalto the circumference of the cone's base. The base of the

cone has radius r, so its circumference is 2 * r. Thus, the length of B B ' is 2 * r . Since a whole circle with radius A B = l has
circumference 2 7 r/, our sector is {2 irr) j (2 irl) = r / l of a whole circle.

A full circle with radius l has area trl2, so the area of a sector that is r / l of this circle is ( r / l ) ( i r l 2) = n rl. Recall from Problem
14.9 that we showed that the lateral surface area of a regular pyramid is half the product of the slam height and the per“ °
the base The proof we used there wouldn't work for cones, since we dont have triangular faces as the sides of a cone. However
since cones are essentially just pyramids with circular bases, we expect the formula to work for cones, too. Trying rt, we note that
the perimeter of the base of a cone is 2 * r , so our formula gives us (1 /2 )(2 ttr ) ( l) - * r l for the lateral surface area.
Unsurprisingly, this matches the formula we already proved. □

Concept: As with cylinders, problems involving the curved surface of a cone can often be
solved by ,unrolling‫ ׳‬the curved surface. As we‫׳‬ve seen, the result is a sector of a
circle.

t V
Problem 1 5.6 ________________________________

Find the volume of a right circular cone that has radius 6 and slant height 8.

Solution for Problem 15.6: We need the height in order tofind the volume. As we did with a pyramid,
we can build a
YVW V U M
right triangle to find the height.
w " 3 ‫י 'י‬
v u i i m W
We draw A______
O , the axis of the cone,
_______
and connect A and
o to B, a point on the circumference of the base. Since O B is a radius and A B is a slant height, we
have A B = 8 and O B = (>. Since the cone is ,right,‫ ׳‬/. A O B is a right angle, so
A O = V A B 2 - O B 2 = 2V 7. Therefore, we have:

= ^ = ! № A ) = 2 a
Volum e

As we've just seen, a radius, an altitude, and a slant height make a right triangle in a right circular cone. We could immortalize this
in yet another formula, l 2 = r 2 4- h 2. but there's no need to - this is just the Pythagorean Theorem!

Back in Problem 15.5, we unrolled the curved surface of a cone to get a sector. What happens if we go the other way?

t V
Problem 1 5.7 ____________________________________________________________________

A semicircle is rolled up to form a cone as shown. If the radius of the semicircle is 8. what is the volume of the cone‫׳‬

‫״‬-» d• A
B\— 1— ‫ ג‬B

'J>B =B ‘

Solution for Problem 75.7: Here we need to f.nd both the radius and the height of the cone. At first it seems that all we are given
is the slant height; however, the information about the ,unrolled‫ ׳‬curved surface can be used to find the radius couple
different ways.

Method One: Ro‫ ״‬it up! When we roll up the semicircle into a cone, the arc B O B ' becomes the circumfemnce‫־‬of the base of the
cone. Since this arc of the semicircle is half the circumference of a circle with radius 8, it has length 8 tt. Therefore, the
circumference of the base of the cone is 8 7 r , so its radius must be 4.

Method Two: Use the iaterai surface area‫ ׳‬The semicircle has area tr(8 2) / 2 = 3 2 * When rolled up, this is the curv«l surface of
the cone, which we know has area trrl, where r is the radius of the cone and / is its slant height. Since the radius of
semicircle is the slant height of the cone, we have l = 8. Hence, we can solve * r l = 3 2 * to find r - 6 2 / 1 - 4.

Once we have the radius, we can find the height by using right triangle A A O B in the cone as before. A
Specifically, h = \ / 8 2 - 42 = 4-^3. Thus, our volume is

* r 2/i 6 4 * \/3
Og -

Now that we have a good understanding of cone volumes and basic cross-sections of a cone, let's try putting them together.

t V
Problem 1 5 . 8 ____________________________________________________________ ________________________________

A cone with vertex .4, height A B = 9, and radius B C = 12 is given. The cone is cut in two pieces by a plane perpendicular to
A B at point X , where A X = 6. Find the volume of the two smaller pieces thus formed.

Solution for Problem 15.8: We showed in Problem 15.4 that a cross-section of a cone perpendicular to its
axis is a circle. So, one of our pieces is itself a cone. The other piece is called a right circular frustum. We
don't have any tools to deal with a frustum, but we do know how to find the volume of a cone. The original
cone has volume trr2h / 3 = * ( 1 2 2 ) ( 9 ) / 3 = 4 3 2 *. We have the height of the smaller cone, A X = 6, so
all we have to do is find the radius.
Since ~XY and ‫־‬BC are each perpendicular to AB, we have Z A X Y = / A B C and
/ X A Y = / B A C , so A A X Y A A B C . (Notice that we are essentially considering the cross-
section of the cone that contains A A B C here - three-dimensional problems are often just two-
dimensional problems in disguise!) Therefore, X Y / A X = BC/AB, so

X Y = ( B C / A B ) ( A X ) = ( 1 2 /9 ) ( 6 ) = 8 .

Hence, our little cone has volume i r ( X Y 2) ( A X ) / 3 = 128*.


To get the volume of the other piece, we merely subtract the little cone from the big one, which yields 4 3 2 * - 1 2 8 * = 3 0 4 *.

Notice that

Volum e o f sm a ll cone 128 7T


Volum e o f large cone 432 tt

This shouldn't be a surprise, because our cones are similar figures. □

Important: Just as the ratio of the areas of similar two-dimensional figures is the square of the
ratio of their corresponding sides, the ratio of the surface areas of similar three-
Z dimensional figures is the square of the ratio of their corresponding side lengths.
Moreover, the ratio of the volumes of similar three-dimensional figures is the cube of
the ratio of their corresponding side lengths.

E x e rc is e s

A cone has height 5 and radius 2. Find the volume and total surface area of the cone.

Show Solution
Type your solution, notes and/or work here.

A cone has a lateral surface area 5 4 * and radius 6. Find the slant height, height, and volume of the cone.

Show Solution
Type your solution, notes and/or work here.

A quarter-circle with radius 4 is rolled up to form a cone.

(a) Find the lateral surface area of the cone.


Show Solution
Type your solution, notes and/or work here.

(b) Find the volume of the cone.


Show Solution
Type your solution, notes and/or work here.

Cone A has twice the radius, but half the height, of cone B. What is the ratio of the volume of A to the volume of B?

Show Solution
Type your solution, notes and/or work here.

Prove that the vertex of a right circular cone is equidistant from every point on the circumference of the cones base

Hint
Show Solution
Type your solution, notes and/or work here.

Is it possible for the slant height of a cone to be smaller than the cone's radius?

Show Solution
Type your solution, notes and/or work here.

How could we have used the fact that the two cones in Problem 15.8 are similar in order to find the volume of the small
quickly?
Show Solution
Type your solution, notes and/or work here.

In this problem we will develop a formula for the volume of a right circular frustum, which was
described in Problem 15.8. Call the bases 5 ! and B2 and let their radii be r! and rL respectively, with
r! < r 2. Furthermore, let the distance between B\ and B2 be h. In Problem 15.8, we formed a
frustum by cutting a cone with a plane parallel to the base. Let's try running this backwards
recreating a cone by extending the curved surface of the frustum up to a point. Call this point V, and
let ‫־‬A B be a diameter of the small base and Y Z be a diameter of a the large base as shown.

(a) What do we know about A V A B and A V Z Y ?

Show Solution
Type your solution, notes and/or work here.

(b) Use part (a) to find the distance from V to B\ in terms of h, r!, and r2.

Type your solution, notes and/or work here.

(c) What is the volume of the cone with vertex V and base B\ in terms of h, r!, and r2?

Type your solution, notes and/or work here.

(d) What is the volume of the cone with vertex V and base B2 in terms of h, r!, and r2?

Type your solution, notes and/or work here.

(e) What is the volume of the frustum in terms of h, r !, and r 2?

Type your solution, notes and/or work here.


15.3 Spheres

i f “ af C‫״‬C'e iS the set of a" poin,s in a plane that are the same distance from a given point a sphere is
Ipheres * " SP8Ce ,h8t afe equidis,ant from a 9iven Poi" ‘ • balls and Jobes are examples of

Fortunate|y, we can take a break from the vocabulary bombardment in this chapter The only significant

Important: A sphere of radius r has:

Z 4 n r:i
Volum e =

Surface A rea = 4 n r2

P ro b le m s
Problem 15.9
4 Jump to Solution
The volume of a sphere is numerically equal to twice its surface area. What is the radius of the sphere?

Problem 15.10
4 Jump to Solution

l r' , ^ W? that 9 ‫ ״‬plane passes throu9h the interior of a sphere then the
intersection of the plane and the sphere is a circle. Let O be the center of the sphere, X be the foot
TterT P.erpenfd^ ula.r segment O to a plane cutting the sphere, and A be a point on the
intersection of the plane and the surface of the sphere (as shown).
(a) What kind of triangle is A O A X ?

(b) Find X A in terms of O A and O X .

(C) S T y° Ur anSWer t0 part (b) te‫ ״‬y0U that the in,ersec,ion of the p|ane and the sphere is a

Problem 15.11
4 Jump to Solution

â t ï S X Ï Ï ‫ ' ?־‬d“ ™ " ‫׳‬ M " ‫ ״‬Cl0' - * ‫״ ״‬ ‫ » « >״ • ״ < ״ ״ ״‬- » ■I» ‫ ״‬. . O, me ‫ ־ ״‬,‫־‬

Problem 15.12
4 Jump to Solution

‫״‬ - - — ‫—■ ״ ״‬ - —

(b) Use your cross-section to find the length of an edge of the cube. Find the volume of the cube

“ Ä S S L - S Ä T - ‫ ׳״ ״ ׳‬-
(d) Use the radius of the sphere to find the length of an edge of the small cube, and then find the cube's volume.

We II start our work with spheres by using the formulas for volume and surface area.

Problem 15.9
t V
The volume of a sphere is numerically equal to twice its surface area. What is the radius of the sphere?

v tT .Ï 2 r» P ‫ ״‬ai‫־ ״‬ ‫“ * •־ * ״ ״ * ״ * ״‬ - ‫״‬ '‫ ״ * ״ ■ » « ״ • ״ ־ ״ ־ ״‬. .‫ ״‬. ‫״‬ we ‫ ־‬. ‫״‬

Therefore, we have

47rr3
— = 2 (47r r 2).

Solving this equation gives r = (j. □

We'll continue our exploration of spheres by considering a cross-section of a sphere.

Problem 15.10
t V
Prove that if a plane passes through the interior of a sphere, then the intersection of the plane
and the sphere is a circle.

tion for Problem 75.70: Intuitively, the cross-section seems to be a circle. We'll prove it is a circle the
radTusTTTp K ‫׳‬T ar cro®s‫־‬sections are circles earlier. Let O be the center of our sphere r be iis
Te h I ° Ur PT ne‫׳‬ * be the f° 0t ° f the PerPendicular segment from O X o V X is the point
we think is the center of our circle. Let A be a point where the surface of the sphere meets V.

Since A O X A is a right triangle, we have * . 4 = y /O A * - O X 2. O A is just the radius of the sphere


so we have X A - V r ~ O X 2. Similarly, every point where the surface of the sphere meets V is the
, ~ ... same dls‘ ance- V r - O X 2, from X . Hence, the intersection of the plane and the sphere is a circle in

is o n T h e T p h ^ e T w a k e T u r^ y o u T e e ^ h y th ls h s tru e ^ a n T ^ T it^ ^ p o rta rtl^ o * ° " ‫^ " ״‬ “ * and radiUS


We have now proved two very useful facts about cross-sections of a sphere.

Important:
Every cross-section of a sphere is a circle (or a point, when the cross-section plane is
Z tangent to the sphere). The segment connecting the center of the sphere to the center
of this circle is perpendicular to the plane of the cross-section.

A cross-section of a sphere that has the center of the sphere as its center is sometimes called a great circle of the sphere
Lets try using our new cross-section knowledge on a problem.

Problem 15.11

: f t n.X V .hS ‫ „־‬l“ r • ‫ “ ״ ״ ״ ■*' “ ״‬Cl° ‫« ״ ״‬ “ »« ‫» » ״ ״ ״ ״‬ < ‫« ״״״‬ . « « ‫ ״‬I . Intersection

Solution for Problem 75.7 7: In this problem we are simply considering a specific cross-section of a

Z : T aS’- *X T T
A Y - /Tq2‫ ־‬K2 2,6/^ 3 ‫ ־־‬a
pr mr ‫ ״ “ ״ ־ ״ ״ ״ ״‬i ‫״ ״‬
X = ° ‫ ׳‬Fr0m right trian9le A A X O we f'nd
/!A - v u - 5 = 12 (maybe you recognized the 5-12-13 Pythagorean triple). We know that this
cross-section is a circle, so its area is 122tr = 144Trcm2 □

Now we'll use cross-sections to examine a problem that includes more than just spheres.

Problem 15.12

include•! th em , K the ed9e len9,h 0f the lar9er cube‫׳‬ in s id e r a cross-section that


includes the center of the sphere and four of the points of tangency where the sphere touches the r
cube at the centers of the faces of the cube. The cross-section of the sphere is a circle Since the ^ U" uB
! n ‫ ' !׳‬° T es bu CeT er 0f the Sphere' the radiUS ° f the circle equals the radius of the sphere The
r
cross-section of the cube is a square whose side lengths equal the edge lengths of the cube O

Drawing radii to the points of tangency at E and F, we see that A E F D is a rectangle and TSF is a

T -1 7 2 S 1 ? I HenC6' A D t “ E F = 2 (° £ ) = 12' « L lhe « * ‫ • ״‬of ‘ he large cube is


™ i w ” * * 'ob,io‫ ״‬s' ‫ ״ * ' ־ ״‬E F ‫ ״ ־ ״ ״ « **־ ■*״‬o . t o . ‫״‬

We have several ways to find the volume of the small cube. We'll start with a judiciously chosen cross-
sect'on. We want to include the center of the sphere since we know the radius of the sphere. Therefore
we include the center of the sphere and four vertices of the cube as shown at left. X Y and Wz are
diagonals of faces of the cube, while W X j r t Y Z are edges of the cube. Therefore W and Y are
opposite vertices of the cube, so that W Y is a space diagonal of the cube. Since WY is also a
diameter of the sphere, we know that IV Y = 12. Since W Y is a space diagonal of the small cube the
side length of the small cube is W Y ^ 3 = 4 ^ 3 . So, the volume of ,he small cube is
(4 V 3 ‫)׳‬:I = 192\/3. □

Concept:
Cross-sections including the center of the sphere are often useful in problems
involving a sphere. Typically, we want to include other important points in the cross-
section as well, such as points where the sphere is tangent to other figures in the

E x e rc is e s

The volume of a sphere is 36 tt. What is the surface area of the sphere?

Type your solution, notes and/or work here.


Show Solution

I t sphere
the : : : 5isinS,Crib6T
along the^ curved
3 Cy‫״‬nder; meanin9
surface of the that il iS tan9em t0 b0,h bases- and ‘ ba‘ one great circle of
cylinder.

(a) Find the ratio of the volume of the sphere to the volume of the cylinder.
Hint

Type your solution, notes and/or work here.


Show Solution

(b) Find the ratio of the surface area of the sphere to the lateral surface area of the cylinder.

Type your solution, notes and/or work here.


Show Solution

15.3.3
Source: MATHCOUNTS t V

An ice cream cone has radius 1 inch and height 4 inches. What is the number of inches
that has the same volume as the cone? in the radius of a sphere of ice cream

Type your solution, notes and/or work here.


Show Solution

Type your solution, notes and/or work here.


Show Solution

ÎÏÏ £ S ' ' 1’ ■ " ” “ « ‫־ ־ » ״‬ ‫>*« ׳*■״־״ “ ■ ״ ׳ ״ » ״ ״ ׳ ׳‬

Type your solution, notes and/or work here.


Show Solution

In this problem we explore facts about tangency and spheres.

(a) ConsirdefthV an,9! nt T T ef


at POim T • and the C" SP ^ 5 is °
Consider the solution to Problem 12.14, then prove that O T J_ T X
Let X be some point in plane V besides T

Type your solution, notes and/or work here.


Show Solution

(b) Spheres 5 , and S2 are externallytangen, at point T . The centers of the spheres are O , and O , Consider the solution to
Problem 10.17, then prove that 0 ! 0 2 passes through T.

Type your solution, notes and/or work here.


Show Solution

(‫) ־‬ Suppose 5 , 1, „„g ,„, „ * ‫״‬ ■. m.epipg & » w * M OotP pass ,‫ ״‬mug‫ ״‬T . Let ‫ ־‬,‫ ״ ״‬,, Ä . ‫״‬.
o 2 be O ! and 0 %respectively. Must 0 , 0 2 pass through T?

Type your solution, notes and/or work here.


Show Solution

without
iS 3 di3meter ° f the Sphere• (Write a comP|ete Pro°f W
Hint

Hint

Hint

Type your solution, notes and/or work here.


Show Solution

Extra!
7T I 1_1 1 1
V 4 = 1 3 5 7 + 9 ÎT + " ‫־‬
15.5 Summary

Definitions: If a prism has congruent curved figures as bases instead of polygons, we call it a
cylinder. When we talk about a cylinder, we almost always mean a right circular
cylinder, in which the bases are circles. The line connecting the centers of the bases
is the axis of the cylinder.

Important: A cylinder with height h and radius r has:

Z Volum e = 7Tr 2h
L a te ra l Surface A rea = 2 n rh
T o ta l Surface A rea = 2 n r h -l- 2 n r 2

Don't memorize these formulas! If you take the time to understand them, they’ll
always be obvious to you.

Definitions: A pyramid with a curved base is a cone. At the right is a right


A
circular cone, which is usually what we mean when we write
,cone.‫ ׳‬The point at the tip of the cone is the vertex. The distance
from the vertex to the base is the height and the line connecting
the vertex to the center of the base is the axis of the cone. In a
right circular cone, the distance from the vertex to a point on the
circumference of the base is the slant height.

Important: In a cone with radius r, slant height /, and height h, we have:

Z Volum e = - n r 2h

L a te ra l Surface A rea = n rl
T o ta l Surface A rea = n r l 4‫ ־‬n r 2

Definition: The set of all points in space that are the same distance from a given point is a
sphere.

Important: A sphere of radius r has:

Z A n r3
Volume = —- —
O

Every cross-section of a sphere is a circle (or a point, when the plane is tangent to the
sphere). The segment connecting the center of the sphere to the center of this circle
is perpendicular to the plane of the cross-section.

Important: Just as the ratio of the areas of similar two-dimensional figures is the square of the
ratio of their corresponding sides, the ratio of the surface areas of similar three-
Z dimensional figures is the square of the ratio of their corresponding side lengths.
Moreover, the ratio of the volumes of similar three-dimensional figures is the cube of
the ratio of their corresponding side lengths.

P ro b le m S o lv in g S tra te g ie s

Concepts:
Problems involving the curved surface of a cylinder can often be solved by
,unrolling‫ ׳‬the curved surface into a rectangle.

Similarly, problems involving the curved surface of a cone can often be solved by
,unrolling'the curved surface into a sector.

Basic area/volume word problems are often no different from other word
problems. The key to solving them is to assign variables and use the area and
volume information to set up equations with the given information.

To solve 3-D problems involving a sphere, consider cross-sections including the


center of the sphere. If you have multiple spheres in the problem, try finding a
cross-section that includes multiple centers. If the sphere is tangent to anything,
try including the point of tangency.

Most 3-D problems are 2-D problems in disguise. Usually considering cross-
sections including important pieces of the problem allow us to use our two-
dimensional strategies to find the solution.

The most useful of these 2-D strategies is the Pythagorean Theorem. Building right
triangles is just as powerful in three dimensions as in two.

The 2-D problem lurking in a 3-D problem isn’t always just a cross-section.
Sometimes we have to manipulate our 3-D figures a little through unfolding or
unrolling to discover our 2-D problem.

A great many geometry problems are solved by assigning a variable to a length,


then finding two different expressions for some other length in terms of that
variable. We can then set these expressions equal to solve for the variable.

Extra!
7T 2 2 4 4 6 6 8 8
v 2 “ 1 3 3 5 5 7 7 9
Review Problems

15.20: t V

The number of cubic inches in a sphere’s volume equals the number of square inches in its surface area. What is the radius of
the sphere?

Type your solution, notes and/or work here. Show Solution

15.21 t V

A cone has slant height 7 and height 5.

(a) Find the volume of the cone.

Type your solution, notes and/or work here. Show Solution

(b) Find the total surface area of the cone.

Type your solution, notes and/or work here. Show Solution

15.22: t V

A sphere has diameter 8. Find the surface area and the volume of the sphere.

Type your solution, notes and/or work here. Show Solution

15.23: t V

A right circular cylinder has radius 3 and height 6.

(a) Find the volume of the cylinder.

Type your solution, notes and/or work here. Show Solution

(b) Find the lateral surface area of the cylinder.

Type your solution, notes and/or work here. Show Solution

(c) Find the total surface area of the cylinder.

Type your solution, notes and/or work here. Show Solution

15.24: t V

A space diagonal of cube C is a diameter of sphere S. The edges of the cube have length 8. Find the volume of the sphere.

Type your solution, notes and/or work here. Show Solution

15.25: t V

The radius of cylinder A is three times the radius of cone B, but the height of cylinder A is half the height of cone B. Find the
ratio of the volume of A to the volume of B.

Type your solution, notes and/or work here. Show Solution

15.26: t V

A B C D is a square inscribed in one base of a cylinder, and E F G H is a square inscribed in the other base of the cylinder.
Given that A B C D E F G H is a cube with side length 9, find the volume of the cylinder.

Type your solution, notes and/or work here. Show Solution

15.27: t V

A 40° sector of a circle with radius 9 is rolled up to form a cone. Find the volume of the cone.

Type your solution, notes and/or work here. Show Solution

15.28: t V

The intersection of a plane with sphere Q is a circle with area 3(>7r. Find the volume of the sphere if the center of this circle is 8
units from the center of Q.

Type your solution, notes and/or work here. Show Solution

15.29: t V

Two spheres are inside a rectangular box such that each sphere is tangent to five faces of the box and to the other sphere.
Each sphere has radius 4. Find the volume of the box.

Type your solution, notes and/or work here. Show Solution

15.30: t V

A cross-section of cylinder C is a square in a plane parallel to the axis of C. Given that the area of the square is 36 square units
and that the plane of the cross-section is 4 units from the axis of the cylinder, find the volume of the cylinder.

Type your solution, notes and/or work here. Show Solution

15.31 t V

A cone with radius 9 and height 12 is cut in two pieces by a plane parallel to the base of the cone such that the plane is 8 units
from the base of the cone. Find the total surface area of each piece thus formed.

Type your solution, notes and/or work here. Show Solution

15.32: t V

Sphere S is tangent to all 12 edges of a cube with edge length 8. Find the volume of the sphere.

Type your solution, notes and/or work here. Show Solution

15.33: t V

A sphere is inscribed in a cone as shown at right. The cone has radius 9 and height 12. Find the radius of the
sphere.

Type your solution, notes and/or work here. Show Solution

15.34: t V

In this problem we use volume to find the radius of a sphere inscribed in regular tetrahedron A B C D with edge length 6. You
may want to review Problem 7.14, since we use a similar tactic on this problem.

(a) Find the length of the altitude from D to face A B C of the tetrahedron.

Type your solution, notes and/or work here. Show Solution

(b) Find the volume of tetrahedron A B C D .

Type your solution, notes and/or work here. Show Solution

(c) Let the radius of the sphere inscribed in the tetrahedron be r and the center of this sphere be (). Why is O a distance of r
away from each face of the tetrahedron?

Type your solution, notes and/or work here. Show Solution

(d) In terms of r, what is the volume of tetrahedron O A B C ?

Hint

Type your solution, notes and/or work here. Show Solution

(e) In terms of r, what are the volumes of tetrahedra O A B D , O A C D , and O B C D ?

Type your solution, notes and/or work here. Show Solution

(f) Find r.

Type your solution, notes and/or work here. Show Solution

15.35: t V

The height of right circular cone C equals the cone's radius in length. Given that A is the vertex of the cone and X Y is a
diameter of the cone’s base, prove that A X A Y is a right angle.

Type your solution, notes and/or work here. Show Solution


Challenge Problems

15.36: t v

Right circular cone C has vertex V , radius 6, and height 6. Sphere S has center V and is tangent to the base of cone C.

(a) What is the radius of the sphere?

Type your solution, notes and/or work here. Show Solution

(b) Let X be a point where S meets the curved surface of C. Find the distance from X to the axis of the cone.

Hint

Type your solution, notes and/or work here. Show Solution

(c) Show that the intersection of S and the curved surface of C is a circle.

Type your solution, notes and/or work here. Show Solution

(d) Find the area of the circle from the previous part.

Type your solution, notes and/or work here. Show Solution

15.37: t V

A cylinder is inscribed in a right regular hexagonal prism such that each base of the cylinder is a circle that is tangent to all six
sides of a base of the prism. Find the ratio of the volume of the cylinder to the volume of the prism.

Type your solution, notes and/or work here. Show Solution

15.38: Source: Mandelbrot t V

Dennis's scoop of ice cream has a radius of 2 cm. It rests in a cone that has a radius of v^3 cm at the
widest part, and the scoop is tangent to each line containing a slant height of the cone. He eats some of the
ice cream and then finds that the remainder of the ice cream can be pushed down to fill the cone exactly.
How many cubic centimeters of ice cream did he eat?

Hint

Type your solution, notes and/or work here. Show Solution

15.39: Source: MATHCOUNTS t V

An edgy ant walks along the edges from A to B of the dodecahedron formed by the folding up
the figure shown at right. What is the number of edges in the shortest path that ant could take?

Type your solution, notes and/or work here. Show Solution

Two parallel planes intersect sphere S, forming two congruent circles. Show that the center of the sphere is the midpoint of
the segment connecting the centers of the two circles.

Hint

Hint

Type your solution, notes and/or work here. Show Solution

15.41 t V

Planes V \ and V± are perpendicular. Their intersection is line m. Plane V \ is 4 units from point O and plane V 2 is 6 units from
O. A sphere with radius 9 centered at O meets line m at points A and B. Find A B .

Hint

Hint

Type your solution, notes and/or work here. Show Solution

15.42: t V

X is on the circumference of one base of a cylinder and Y is on the other base such that X Y is bisected by the axis of the
cylinder. Given that the diameter of the cylinder is 8 and X Y = 12, find the volume of the cylinder.

Type your solution, notes and/or work here. Show Solution

15.43: t V

Arnav the ant is on the outside of a cylindrical glass, halfway up the glass. The glass is 8 inches tall and has a radius of 2
inches. Arnav wants to walk to a point on the inside of the glass that is diametrically opposite the point he's now at (i.e.,
halfway up the glass, on the inside surface of the glass, exactly opposite where he is now). What is the shortest distance he
can walk?

Hint

Hint

Type your solution, notes and/or work here. Show Solution

15.44: Source: MATHCOUNTS t V

A spiral staircase with radius 3 feet turns 270° as it rises 10 feet. What is the number of feet in the length of the outer
handrail? (The outer handrail is a curved rail along the entire staircase. It is everywhere 3 feet from the central axis of the
staircase.)

Hint

Type your solution, notes and/or work here. Show Solution

15.45: Source: A M C 10 t V

A right circular cylinder with its diameter equal to its height is inscribed in a right circular cone. The cone has diameter 10 and
altitude 12, and the axes of the cylinder and cone coincide. Find the radius of the cylinder.

Hint

Type your solution, notes and/or work here. Show Solution

15.46: t V

P Q R S is a regular tetrahedron. The distance from the midpoint of P Q to the midpoint of R S is 6 units. Find the volume of
PQRS.

Hint

Type your solution, notes and/or work here. Show Solution

15.47: Source: Mandelbrot t V

The wire frame at right consists of three mutually perpendicular segments A I) , B D , and C D , each
3 cm in length. A quarter-circle of radius 3 is attached to each pair of segments as shown. The
curved portions of the wire frame will fit snugly against a sphere of a certain size, so that the entire
lengths of all three quarter circles make contact with the sphere. For what radius is this possible?

Hint

Hint

Type your solution, notes and/or work here. Show Solution

15.48★ : t V

One cone of radius 4 and height 12 and another cone of radius (>and height 12 intersect as shown at righ t,
so that the vertex of one coincides with the center of the base of the other. Find the volume of the
intersection of the two cones.

Hint

Hint

Type your solution, notes and/or work here. Show Solution

15.49★ : t V

(a) In tetrahedron A B C D , let V be the volume of the tetrahedron and r the radius of the sphere that is tangent to all four
faces of the tetrahedron. Let K \ K 2, K & and K 4 denote the areas of faces B C D , A C D , A B D , and A B C ,
respectively. Prove that

W
r =
K x+ K 2 + K 3 + K 4

Hint

Type your solution, notes and/or work here. Show Solution

(b) X V X Y Z is a tetrahedron with X Y = Y Z = Z X = 9 and XVX = W Y = XVZ = 18. Find the radius of the sphere
that is inscribed in tetrahedron X V X Y Z . (You might remember working with this pyramid in Problem 14.26.)

Type your solution, notes and/or work here. Show Solution

15.50★ : t V

Find a formula for the total surface area of a right circular frustum with base radii r ! and r 2 and
slant height /. (The slant height of a frustum is the distance from a point on the circumference
of one base to the nearest point on the circumference of the other base, as shown at right.)

Hint

Hint

Type your solution, notes and/or work here. Show Solution

15.51★ : t V

A ball of radius 1 is in the corner of a room, tangent to two walls and the floor. A smaller ball is also in the corner, also tangent
to both walls, the floor, and the larger ball. The walls are perpendicular to each other, and they are perpendicular to the floor.
Find the radius of the smaller ball.

Hint

Type your solution, notes and/or work here. Show Solution

Extra! We’ve already had a glimpse of the brilliance of Archimedes in Section 11.2 here in his proof of the
formula for the area of a circle. Although the mathematical and scientific accomplishments of
V Archimedes could fill several books, the feat he allegedly treasured most was his determination of the
volume and surface area of a sphere. Legend has it that he even asked that his tombstone be inscribed
with a sphere inscribed in a cylinder to commemorate his feat.

Archimedes used the tactics we saw in his area of a circle proof, although with considerably more
complicated figures. He compared a sphere to figures he already knew how to handle: cones and
cylinders. To get a feel for the volume formula, consider the figures below. Shown is a cone that shares a
base with a cylinder such that both the cone and the cylinder have height equal to the radius of the base.
Next to these is a hemisphere, or half a sphere, with the same radius as the base of the cylinder and
cone.

Consider the cross-sections of each taken at the same height, h, from the base. Letting the radius of the
hemisphere be r, find the areas of the cross-section of each figure in terms of r and h. Next, find the area
inside the cross-section of the cylinder, but outside the cross-section of the cone. Notice anything
interesting?
/1 /4 ‫׳‬

Fagnanos Problem

The mathematical sciences particularly exhibit order, symmetry, and limitation; and these are the greatest forms o f the beautiful. - Aristotle

CHAPTER

I _____________________________________________ The More Things Change...


A geometric transformation is a rule we apply to a geometric figure that usually results in the figure being moved or changed in
some way. In this chapter we will explore a few basic types of transformations by sliding, spinning, and flipping figures. As we'll
see, these transformations change the locations of figures to which they are applied, as opposed to turning them into entirely
different figures.

16.1 Translations
When we apply a translation to a figure, we simply slide it a specified distance in a given direction. For example .4 'and B ' below
are the result of translating A and B, respectively, in the direction and by the distance suggested by the arrows.

Figure 16.1 : A Translation

We use some special terms when performing transformations. We say that A ' is the image of A under the translation shown
above. We can also say that the translation maps A to A'.

Of course, we can translate more than just points. We can translate any figure - a segment, a line, a triangle, etc. - and the result
will be congruent to our original figure. This is one of those 'obvious‫ ׳‬things that we won't worry about proving right now.

P ro b le m s
Problem 16.1 4 Jump to Solution

A fixed point of a transformation is a point that is its own image. The identity transformation is the transformation that maps
every point to itself. In other words, the identity is the ,do nothing’ transformation. Can a translation that is not the identity have
a fixed point?

Problem 16.2 4 Jump to Solution

(a) Locate point U on the grid shown such that R S T U is a parallelogram.

(b) Is the U you found the only point that can be combined with R, S, and T to form the
vertices of a parallelogram? If not, find the others.

Translations are the simplest transformations, so there aren't many interesting problems involving translations. We’ll use these
simple transformations to introduce a couple more terms.

Problem 16.1 t V

A fixed point of a transformation is a point that is its own image. The identity transformation is the transformation that maps
every point to itself. In other words, the identity is the ‫׳‬do nothing‫ ׳‬transformation. Can a translation that is not the identity have
a fixed point?

Solution for Problem 76.7: In a translation, every point is moved the same distance in the same direction. If there is a fixed point,
then that point is not moved at all. Since all points must move the same distance in the same direction, this means none of the
points is moved by the translation. Hence, the only translation that has a fixed point is the identity. □

Concept: Understanding transformations is often more about learning what stays the same
I rather than what changes. In the three basic transformations we’ll study in this
I chapter, everything will stay the same for each transformed figure except its location
and orientation. That may sound like a lot to change, but it really isn’t much. Line
segments remain line segments, circles remain circles, points that are one unit apart
remain one unit apart, and so on.

Here's one example of using translations to solve a problem:

Problem 1 6.2 t V

(a) Locate point U on the grid shown such that R S T U is a parallelogram.

(b) Is the U you found the only point that can be combined with R, S, and T to form the
vertices of a parallelogram? If not, find the others.

Solution for Problem 16.2:

(a) If R S T U is a parallelogram, then S R || T U and S R = T U . In other words, the same translation that maps S to R must
map T to U. Since we must go down 5 and left 4 to get from S to R, we must go down 5 and left 4 to get from T to U.
Therefore, U is at the point shown in the diagram below.

(b) In the first part, we ,completed’ the parallelogram that has Z R S T as an angle. We could
instead have chosen Z S R T or Z S T R ! as an angle of the parallelogram. Let points X and
Y, ‫׳‬complete‫ ׳‬parallelograms S R T X and S T R Y , respectively.

To complete parallelogram S R T X , we note that we must go right 3 and up 2 to get from R


to T. Therefore, we go right 3 and up 2 from S to get to X . Similarly, to complete S T R Y , we
see that we go left 3 and down 2 to get from T t o R, so we must go left 3 and down 2 from S
to find Y. Our other two parallelograms are shown in the diagram.

E x e rc is e s

16.1.1: t V

A B C D E F is a regular hexagon. Why must the translation that maps E to C also map F t o B?

Hint

Hint

Type your solution, notes and/or work here. Show Solution ||


16.2 Rotations
When we rotate a figure, we spin it by some angle about some point. In other words, the image of point A A
under a rotation of angle 9 counterclockwise about point O is the point A ' such that O A = O A ' and
Z A O A ' = 9, as shown at right. There are two points that satisfy this definition of A', which is why we must
specify the direction of the rotation as clockwise or counterclockwise. Point O is the center of the rotation,
and 9 is the angle of rotation.

As with translations, the image of any figure upon rotation is congruent to the original figure. This is one of
those 'obvious‫ ׳‬facts that you might try proving on your own for specific shapes like line segments or circles.

Sidenote: The contributions of the ancient Greeks to mathematics have been immortalized in
many ways. One of these is shown in our introduction of rotation. It's a common
convention to use Greek letters as variables for angle measures. Above, we use the
letter 9, or 'theta.‫ ׳‬The letter 0 is also often used for angles, and sometimes the Greek
letters a, fi, and 7 are used to stand for the angles of a generic triangle A A B C .
Greek letters are also fun to write!

P ro b le m s
Problem 16.3 4 Jump to Solution

Let O be the center of equilateral A X Y Z .


X
(a) Through what angles between 0° and 360° can we rotate Y about Z such that the result is point
X?

(b) Through what angles between 0° and 360° can we rotate X about O such that the result is point
V?

(c) Is there a rotation about Y that maps O to Z?

Problem 16.4 4 Jump to Solution

Through how many different positive angles 9 less than 360° is it possible to rotate a regular dodecagon clockwise about its
center such that its image coincides with the original dodecagon?

Problem 16.5 4 Jump to Solution

After I draw a figure, I rotate it 48° about some point P. If the image of this rotation coincides exactly with the original figure, is
it true that the image of a rotation of the figure 72° about P must also coincide with the figure?

We’ll explore rotations with a few basic problems. As you’ll see, rotations are all about angles.

Problem 16.3 t V

Let O be the center of equilateral A X Y Z .

(a) Through what angle(s) between 0° and 360° can we rotate Y about Z such that the result is point
X?

(b) Through what angle(s) between 0° and 360° can we rotate X about O such that the result is point
Y?

(c) Is there a rotation about Y that maps O to Z?

Solution for Problem 76.3:

(a) Since Y Z = X Z and Z Y Z X = 60°, a 60° rotation clockwise about Z takes Y to X . Of course, a 300° rotation the
other way will also do the job!

(b) Since O is the center of equilateral A X Y Z , X O and Y O bisect Z Z X Y and Z Z Y X , respectively. Therefore,
Z Y X O = Z O Y X = 30°, so

Z X O Y = 180° - Z Y X O - Z O Y X = 120°.

Since O X = O Y and Z X O Y = 120°, if we rotate X 120° counterclockwise about O, it is mapped to point Y . Note that
we can also rotate X 240° clockwise around O to get to Y.

(c) By the definition of rotation, every rotation about Y maps O to a point that is as far from Y as O is. Since Y O and Y Z are
not equal in length (make sure you see why!), it is impossible for a rotation about Y to map O to Z.

Problem 16.4 t V

Through how many different positive angles 9 less than 360° is it possible to rotate a regular dodecagon clockwise about its
center such that its image coincides with the original dodecagon?

Solution for Problem 16.4: Let A B C D E F G H I J K L be our regular dodecagon and O its center.
Each of the interior angles has measure 180° — 3 60°/12 = 150°, and each segment from a
vertex to the center bisects an interior angle, so Z O A B = Z O B A = 75°. Therefore,
Z A O B = 30°. So, if we rotate the dodecagon 30°, every vertex will be 'shifted over‫ ׳‬by one vertex,
and the dodecagon will map to itself.

Because a rotation of 30° works, we know a rotation of 30° - 1 6 0 ° = (30°)2 = 30° ‫ ־‬will work too.
A 60° rotation is just the result of two 30° rotations, and each of these maps the dodecagon to
itself. We could also think of this as shifting each vertex over two vertices. Similarly, a rotation of
each multiple of 30° will map the dodecagon to itself. This gives us 11 positive angle rotations less
than 360° about the center that will take the dodecagon to itself.

It is impossible for two different clockwise rotations between 0° and 360° about O to map ;4 to the same vertex, and we can’t
map A to itself with such a rotation. Therefore, we know that the 11 rotations we’ve found are the only clockwise rotations
between 0° and 360° that map the dodecagon to itself. □

Problem 16.5 t V

After I draw a figure, I rotate it 48° about some point P. If the image of this rotation coincides exactly with the original figure, is
it true that the image of a rotation of the figure 72° about P must also coincide with the figure?

Solution for Problem 16.5: We know that rotating the figure 48° leaves the figure unchanged, so to think about what happens
when we rotate by 72°, we start by finding what other rotations leave the figure unchanged. Spinning the figure by 48° twice will
leave it unchanged, so a 96° rotation leaves the figure unchanged. Similarly, other angles of rotation about P that don’t change
the figure are

3(48°) = 144°, 4(48°)


= 192°?240°, 288°. 336°, 384°, 432°, 4 8 0 ° ,...

Taking a look at those 'over 360°‫ ׳‬rotations, we see that one of them is 432°. A 432° rotation is equivalent to a
432° — 360° = 72° rotation. Therefore, a 72° rotation will also coincide with the original figure. □

E x e rc is e s

16.2.1: t V

A B C D E F G H is a regular octagon, with the vertices labeled counterclockwise.

(a) A clockwise rotation with center B and angle 9 maps A to C. Find 9 such that 9 < 360°.

Type your solution, notes and/or work here. Show Solution

(b) A clockwise rotation with center B and angle 0 maps I I to I). Find 0 such that 0 < 360c

Type your solution, notes and/or work here. Show Solution

16.2.2: t V

B ' is the image of B rotated 90° about A. Given A B = 5, find B B '.

Type your solution, notes and/or work here. Show Solution

16.2.3: t V

O is the intersection of the diagonals of quadrilateral A B C D . A 90° rotation about O maps A B C D to itself, meaning each
vertex of the image is also a vertex of the original quadrilateral. Must A B C D be a square?

Hint

Type your solution, notes and/or work here. Show Solution

16.2.4★ : t V

Regular tetrahedron A B C D is rotated 60° about its altitude from A to face B C D .

(a) What kind of shape is the intersection of the original tetrahedron and its image?

Hint

Type your solution, notes and/or work here. Show Solution

(b) ★ Find the volume of intersection of the original tetrahedron and its image given that A B = 6 .

Hint

Type your solution, notes and/or work here. Show Solution


16.3 Reflections
We ve tried sliding and we've tried spinning; now, we’ll try flipping. If you’ve looked in a mirror, you have experienced reflection.

Mathematically speaking, the image when we reflect point A over line m is the point A ' such that m is the
perpendicular bisector of A A '. In other words, if we folded our paper along line m, A and A ' would coincide.

Of course, we can flip more than just points. In the diagram below, A A 'B 'C ' is the image of A A B C upon
reflection over line k.

/1

If a figure maps to itself under a reflection over a certain line, that line is called a line of symmetry of the figure. For example,
every diameter of a circle is part of a line of symmetry of a circle, and each diagonal of a square is part of a line of symmetry of
the square.

P ro b le m s
Problem 16.6 4 Jump to Solution

What are the fixed points of a reflection over a given line m l In other words, what points are their own images when reflected
over line m l

Problem 16.7 4 Jump to Solution

Given two different points, X and Y, describe all lines m such that the image of X upon reflection overm is point Y .

Problem 16.8 4 Jump to Solution

How many different lines of symmetry does a regular hexagon have?

Problem 16.9 4 Jump to Solution

I own a cabin in the woods that is 4 miles directly north of a river that runs east-west. I am currently out exploring the woods. I
am 2 miles north of the river, and 3 miles west and 2 miles south of my cabin. I want to walk to the river, then walk to the cabin.
What is the length of the shortest possible route I can take?

Problem 16.10 4 Jump to Solution

My miniature golf course has only two holes, which are shown below.

(a) On Hole 1, the ball starts at Y , and we want to hit it to X . Find the point on ‫־‬A B at which we should aim to hit the ball to
X.

(b) On Hole 2, the ball starts at W , and we want to hit it to Z. There is a barrier between W and Z, as shown. Where should
we aim on U T to hit the ball to Z ?

A U

X•

Y•
3 r
Hole 1 Hde2

One of the best ways to understand any geometric transformation is to analyze what stays the same when everything else is
changing.

Problem 16.6

What are the fixed points of a reflection over a given line m l

Solution for Problem 16.6: Clearly every point on m maps to itself, since m is the ‫׳‬fold.' To see that there are no other fixed points,
we note that m divides the plane into two pieces. Any point on one side of m is mapped to a point on the other side. Since a
point can’t be on both sides of m , a point not on m can’t be mapped to itself! z

Suppose we are given a point and its image under a reflection. How do we find the line over which the point was reflected?

Problem 16.7

Given two different points, X and Y, describe all lines m such that the image of X upon reflection over m is point Y.

Solution for Problem 16.7: The image of a point A upon reflection over a line is the point A ' such that the line is the perpendicular
bisector of A A '. Since the image of X in our problem is Y, the line over which we are reflecting must be the perpendicular
bisector of X Y . □

Now that we've mastered reflecting points, let’s reflect a more complicated figure.

Problem 16.8
t V

How many different lines of symmetry does a regular hexagon have?

Solution for Problem 16.8: Intuitively, it seems like the only lines of symmetry are the six dashed
lines shown that cut the regular hexagon in half, but how can we be sure these are the only ones?

To make sure there aren't any others, we note that if the reflection maps the hexagon to itself, then
A must be mapped to some vertex of the hexagon. Furthermore, for any two different vertices,
there is only one reflection that will map the vertices to each other - the reflection over the
perpendicular bisector of the segment connecting the two vertices. Therefore, there's only one
reflection that could possibly map A to B, and one that maps A to C, etc. Each of these reflections
gives us a line of symmetry. This doesn’t include lines of symmetry through A. A D is the only
possible line of symmetry through A, since it’s the only line through A that leaves two vertices on
each side of the line, z

Now we'll use reflections to solve a couple of problems.

Problem 16.9

I own a cabin in the woods that is 4 miles directly north of a river that runs east-west. I am currently out exploring the woods. I
am 2 miles north of the river, and 3 miles west and 2 miles south of my cabin. I want to walk to the river, then walk to the cabin.
What is the length of the shortest possible route I can take?

Solution for Problem 16.9: Point M is where we start, the cabin is at C, and \ rZ is the river. One
approach we can take is to suppose we hit the river at point D, x miles east of where we start. We
can then use the Pythagorean Theorem to make an expression in terms of x for how far we have to
walk. Since Y D = x, we have D Z = 3 — x, so from right triangles A M Y D and A C Z D , we
have

M D + C D = %/4 + x 1 + ^ 1 6 + (3 - x ) 2.

Figuring out the smallest possible value for this expression is going to be very hard. We need to find
a more clever approach.

Since we’re stuck, we try thinking of similar, simpler problems that we know how to solve. For example, if we didn't have to go to
the river, the answer would be easy - we just go from M to C and we're done. Another simplified problem would be if we were
on the other side of the river. Then we could still just go straight from our starting point to the cabin, and we’d still hit the river.
Aha! We can cross the river.

Suppose we reflect ourselves over the river, to point M ' as shown at right. Since M X is the mirror
image of M 'X , we have M X ‫־‬+‫ ־‬X C = M ' X + X C . Therefore, finding the shortest total distance
from M to the river to C is the same as finding the shortest total distance from M ' to the river to C.

Any path from M ' to C hits the river, so clearly our desired shortest distance is M 'C . Since M is 2
miles north of the river, M ' is 2 miles south of it. So, M ' is 2 -P 4 = 6 miles south of the cabin. Since
M ' is also 3 miles west of the cabin, M 'C = \/3 6 + 9 = 3 \/5 .

Therefore, the shortest path I can take is 3 \/5 miles long, z

Concept: If you can't solve a problem, try thinking of a similar, simpler problem and solve that.
Then try to use your solution to the simpler problem to solve the harder problem.

Next time you play miniature golf or shoot pool, you’ll likely think of reflections. Here’s why:

Problem 16.10
t V

My miniature golf course has only two holes, as shown below.

(a) On Hole 1, the ball starts at ) , and we want to hit it to X . Find the point on A B at which we should aim to hit the ball to
X.

(b) On Hole 2, the^ball starts at W , and we want to hit it to Z . There is a barrier between W and Z, as shown. Where should
we aim on T U to hit the ball to Z?

\ \

Hole1 H ole2

Solution for Problem 16.10:

(a) For our first hole, we need to bounce the ball off A B so that it goes directly to X . To figure out where to aim the ball, we
note that the path after the ball bounces is the reflection of what the path would be if there were no wall ‫־‬A B and the ball
just kept going straight. Therefore, if we aim at the image of X upon reflection over ‫־‬A B , the ball will bounce off ‫־‬A B and go
straight to X .

So, to find the point on A B to aim at, we reflect X over A B to get its image X '. The intersection of Y X ' and A B is the
point we should target.

(b) On Hole 2, the barrier not only prevents us from hitting the ball straight from W to Z, but it also prevents us from just
bouncing the ball off T U and having it go straight to Z . We’ll have to go for a double-bank shot. Therefore, we need it to
bounce off T U , then hit the appropriate point on the top wall so that it continues to point Z.

We still don't know where to aim along T U , but we can figure out where we want the ball heading
after it banks off of T U . After coming off T U , the ball should be heading towards the image of Z
upon reflection over the top wall.

We now know where we want the ball heading once it comes off T V , but we still don't know where to
aim along T U . Since we need the ball to go from W and bounce off T V so that it is heading straight
towards Z ', we think to try our reflection tactic on W as well.

Point W ' is the image of W upon reflection over T U . If we were at W ' and there were no wall
T U , we would aim directly at Z '. The path W 'Z ' meets T V at P. Since W ’ P is the image of
W P , if we aim the ball at P, it will bounce off T V , and head towards Z '. Since Z ' is the image
of Z upon reflection over the top wall, a ball heading from P towards Z ' will bounce off the top
wall and head straight to Z as desired. Have fun beating your friends next time you play
miniature golf!
Hole2

E x e rc is e s

16.3.1: t V

How many lines of symmetry does each of the following figures have?

(a) A rectangle that is not a square.

Type your solution, notes and/or work here. Show Solution

(b) A rhombus that is not a rectangle.

Type your solution, notes and/or work here. Solution

(c) A parallelogram that is not a rectangle or a rhombus.

Type your solution, notes and/or work here. Show Solution

(d) A regular pentagon.

Type your solution, notes and/or work here. Solution

(e) A regular dodecagon.

Type your solution, notes and/or work here. Show Solution

(f) A regular polygon with n sides.

Type your solution, notes and/or work here. Solution

16.3.2: t V

Square A B C D is reflected over & D . A ' and B ' are the images of A and B, respectively. Given C D = 4, find A 'A and
A 'B .

Type your solution, notes and/or work here. Solution

16.3.3: t V

Lines k and £ are not parallel, and line m is the image of line k upon reflection over L Show that £ bisects a pair of the angles
formed by line m and k.

Type your solution, notes and/or work here. Solution

16.3.4: t V

Given intersecting lines £ and m and point X not on either line, let Y be the image of X upon reflection over £ and let Z be the
image of X upon reflection over m.

(a) Is it possible for the reflection of Y over m and the reflection of Z over £ to be the same point?

Type your solution, notes and/or work here. Solution

(b) Must the reflection of Y over m and the reflection of Z over £ be the same point?

Type your solution, notes and/or work here. Show Solution

16.3.5★ : t V

We can define reflections in space through a plane similar to the way we define reflections over a line. Specifically, given a
point K and a plane V , then the image of K upon reflection through V is the point K ' such that V is perpendicular to ‫־‬K K 1
and bisects K K '. A plane of symmetry of a figure is a plane such that the image upon reflecting the figure through the plane
is the figure itself. For example, any plane through the center of a sphere is a plane of symmetry of the sphere.

(a) How many planes of symmetry does a right square pyramid have?

Type your solution, notes and/or work here. Show Solution

(b) How many planes of symmetry does a right rectangular prism have if the dimensions of the prism are all different?

Type your solution, notes and/or work here. Show Solution

(c) How many planes of symmetry does a regular tetrahedron have?

Hint

Type your solution, notes and/or work here. Show Solution

(d) How many planes of symmetry does a cube have?

Hint

Type your solution, notes and/or work here. Show Solution

(e) Show that any plane of symmetry of a right circular cone must include the axis of the cone.

Hint

Type your solution, notes and/or work here. Show Solution


16.6 Construction: Transformations
As a thorough test of your spinning and flipping abilities, you'll now have the chance to spin and flip some points given only a
ruler and compass.

P ro b le m s
Problem 16.19 4 Jump to Solution

Given are points C and T. Construct point U such that U is the image of point T under a 30° rotation about
C.

Problem 16.19 t V

Given are points C and T . Construct point U such that U is the image of point T under a 30° rotation about C.

Solution for Problem 76.79: Since U is the image of T rotated 30° about C, we must have
C T = C U and Z .T C U = 30° Since C T = C U , U must be just as far from C as T is.
Therefore, U must be on the circle with center C and radius C T . So, we draw that circle. Since
Z T C U = 30°, U must be on a ray through C that forms a 30° angle with C T . We construct
this ray by first constructing an equilateral triangle, A C T X , then bisecting Z C of this triangle.

The intersection of this angle bisector and our circle is the desired point U such that C T = C U
and Z T C U = 30°. Therefore, U is the image of T under a 30° rotation about C. □

Concept: In construction problems in which we must locate a single point, our general strategy
I is to find two figures, usually circles or lines, which must contain the point. The
intersection of these two figures is the point we want.

You’ll flip over this one...

Problem 1 6.2 0 t V

Given is point A and line m. Construct point B such that B is the image of point A upon reflection over m.

Solution for Problem 16.20: Since B is the image of point A upon reflection over m, we must find the
A
point B such that m is the perpendicular bisector of A B . Since A B _L m, B must be on the line
through ;4 perpendicular to m (see Problem 6.24 for this construction). Let this perpendicular line meet
m at M . Since m bisects A B , we must have A M = B M . Therefore, B is on the circle with center M
and radius A M . Where this circle meets our earlier line through A is the desired point B such that m is
the perpendicular bisector of A B . □

Solution for Problem 16.21: We seek point M on m and point N on n such that
A P M N is equilateral. If we can find either M or N , we can easily construct our
equilateral triangle, since once we have a side of the triangle, we can make the whole
thing. Therefore, well focus on finding N .

We already have one figure that contains iV - line n. Therefore, we only need to find one
more line or circle that contains N . Point P and line n are not enough to find N \ we need
more. The only other piece of information we have is that M is on m, so we need to use
m to find N .

Since A P N M must be equilateral, we know that a 60° rotation about P maps M to N .


But we don’t know where M is! However, we do know that M is on m, so the image of
M upon rotating 60° about P must be on the image of m when m is rotated 60° about P
. Since this image of point M is point N , all we have to do is rotate m 60° about point P
to have another line that contains 7V!

To rotate m 60° about P, we simply pick two points on m and rotate them both. Connecting the images of these points upon
rotation gives us in ', the image of m upon 60° rotation about P. Since N is on both in and n, where these lines meet gives us N
! Now we can easily use P N to construct the equilateral triangle. □

One key step in finding our solution was thinking about what information we hadn’t used yet, thereby focusing on using line ra to
find N . When we focused on how to use m to get N , we had to think about how to relate M to N given point P. This led us to a
new problem solving tactic:

Concept: When stuck on a problem involving equilateral triangles, think about 60° rotations!

E x e rc is e s

1 6 .6 .1 : t V

Given points A, A ', and Q, construct point Q ' such that Q ' is the image of Q under the same translation that maps A to A '

Type your solution, notes and/or work here. Solution

1 6 .6 .2 : t V

Given a point P and two intersecting lines, £ and m, construct the image of P upon reflection over £. Let P ’ be the image of P
upon reflection over £. Construct the image of P ’ upon reflection over m. Try this for a variety of points (reflecting the point
over £, then reflecting its image over m). Notice anything interesting?

Type your solution, notes and/or work here. Show Solution


16.4 Dilation
The last transformation we will explore is dilation. In plain English, a dilation results from
stretching or shrinking a figure.

At right is an example of dilation. Point O is the center of dilation. The image of point A is
the point A ' on ( Â Î such that O A '/ O A = 2. Similarly, the image of point B is the point B ' o<
on o è such that O B '/ O B = 2, and the image of point C is the point C on ü Ô such that
O C / O C = 2. The image of the center of dilation is itself. We can think of the dilation as
'stretching‫ ׳‬A A B C away from O.

In general, the image of a point P upon dilation with scale factor k and center O is the point P ' on o f t such that
O P ' = k (O P ) (when k is positive). In our example above, the scale factor is 2.

The scale factor need not be positive. At right we have an example in which the scale factor
is —2. Here, the image of point X upon dilation about O is the point X ' on the ray starting
from O but going in the opposite direction from X such that O X '/ O X = 2. In other
words, point X ' is the point such that O X '/ O X = 2 and O is on X X ' .

P ro b le m s
Problem 16.11 4 Jump to Solution

Triangle A ' B 'C is the image of a dilation of triangle A B C about the centroid of triangle A B C . The dilation has a positive
scale factor.

(a) Suppose A A ‫ ׳‬B ‫ ׳‬C ‫ ׳‬is entirely outside A A B C . Is the scale factor greater than 1 or less than 1?

(b) Suppose A A 'B 'C ' is entirely inside A A B C . Is the scale factor greater than 1 or less than 1?

(c) What happens if the scale factor equals 1?

Problem 16.12 4 Jump to Solution

Square A 'B 'C 'D ' is the image of a dilation of square A B C D about A with scale factor 3. Suppose A B = 2.

(a) Find A A \ A B ', and B B '.

(b) Find B C and C D '.

Problem 16.13 4 Jump to Solution

Let A ' and B ' be the images of points A and B, respectively, under a dilation with center O and positive scale factor k.

(a) Show that A A O B ^ A A 'O 'B '.

(b) Show that A B || A 'B ' and A 'B '/ A B = k.

Problem 16.11 t V

Triangle A ' B 'C is the image of a dilation of A A B C about the centroid of triangle A A B C .

(a) Suppose A A 'B 'C ' is entirely outside A A B C . Is the scale factor greater than 1 or less than 1?

(b) Suppose A A 'B 'C ' is entirely inside A A B C . Is the scale factor greater than 1 or less than 1?

(c) What happens if the scale factor equals 1?

Solution for Problem 16.11: Let G be the centroid of A A B C . The centroid of a triangle is always inside the triangle, so G is
inside A A B C .

(a) Because A ' is the image of A upon dilation about G, point A ' is on (3%. If A ' is outside the triangle on this ray, then it is
beyond A on g X. So, we must have G A > G A, which means G A '/G A > 1. Therefore, the scale factor is greater than
1. An example of this case is shown at left below.

A'

(b) As before, we know that A ' is on GA. However, this time, A ' is inside A A B C , so we have G A ' < GA. Therefore, we
have G A '/G A < 1, so the scale factor is less than 1. An example of this case is shown at right above.

(c) If the scale factor equals 1, then the image of A A B C is itself.

Important: If the scale factor of a dilation is greater than 1, then the dilation of a figure 'stretches’
the figure. The resulting image is larger than the original figure. If the scale factor is
Z positive but less than 1, then the dilation is a ‫׳‬shrinking‫ ׳‬of the original figure, which
gives an image that is smaller than the original figure. A dilation of a figure with scale
factor equal to 1 leaves the figure unchanged.

Problem 16.12 t V

Square A 'B 'C D ' is the image of a dilation of square A B C D about A with scale factor 3. Suppose A B = 2.

(a) Find AA', A B ', and B B '.

(b) Find B C and C D '.

Solution for Problem 16.72:


A =A' b
(a) At right, we have the original square A B C D and its image under the dilation with scale
factor 3 and center A The image of A upon a dilation about A is just itself. (The image of D
the center of a dilation is always the center itself.) Therefore, we have A A ' = 0. (Yeah, that
was a bit of a trick question!)

Because B ' is the image of B upon a dilation about A with scale factor 3, we have
A B '/A B = 3. In other words, B ' is 3 times as far from A as B is. Since A B = 2, we have D'
A B ' = 6. Points A, B, and B ' are collinear because B ' is the image of B upon the dilation
about A. So, we have A B 4‫ ־‬B B ' = A B '. From this, we find that B B ' = A B ' — A B = 4.

(b) We seek two lengths and we have lots of right angles, so we look for right triangles on which
we can use the Pythagorean Theorem. In the previous part, we found that A B ' = 6, so
square A B 'C D ' has side length 6. We also found that B B ' = 4, so right triangle B B 'C '
gives us

B C ‫ = ׳‬/ ( B B ‫ ׳‬y + ( B 'C ') 2 = v 3 + 16‫ ׳‬G = 2 / 1 3 .

In the same way we found B B ' = 4 in part (a), we can find that D D ' = 4. Right triangle
C D D ' then gives us

C D 1 = V C D 2 + D 'D 2 = n/ I T T G = 2 v 5 ‫׳‬.

This problem raises an interesting question about dilations. Points B ‫ ׳‬and C ‫ ׳‬are the images of B and C under the dilation, and
we see that B 'C 1 || BC, and B 'C / B C equals the scale factor. Note that we didn't prove these facts; we were told to assume
that the image of A B C D upon dilation is a square. Let's see if we can prove that these observations are not a coincidence.

Problem 16.13 t V

Let A ' and B ' be the images of points A and B, respectively, under a dilation with center O and positive scale factor k. Show
that AZ? || Â 7 F and A 'B '/A B = k.

Solution for Problem 16.13: We want to prove something about a ratio, and we want to show that two
lines are parallel. The dilation also gives us information about ratios. This looks like a job for similar
triangles. Our diagram at right shows the set-up where k > 1. (The case k < 1 can be solved with
the same steps we will use for k > 1.)

We're looking for similar triangles, and we need to find some way to use the fact that A and B ' are
the images of A and B under the same dilation about O. The dilation information tells us that ( X l
passes through A and OI*$ passes through B', as shown. But that's not enough! The dilation also
gives us information about ratios. Specifically, it tells us that O A '/O A and O B '/O B both equal the scale factor, so

O A OB'
= k.
OA OB
Since we also have Z A O B = Z A OB', we have A A O B ~ A A 'O B ' by SAS Similarity. From this triangle similarity, we have
A B '/A B = O A ‫ ׳‬/O A = k. We also have Z O A B = Z O A 'B ‫׳‬, which tells us that A B || A 'B '. □

Notice that we did not prove in Problem 16.13 that the image of A B is A 'B '. We only proved that A B '/ A B = k and
A B || A 'B '. You'll have a chance to prove that the image of a segment under dilation is a segment as a challenging Exercise.
However, the parallel lines suggest something very useful about dilations.

Important: A figure and its image upon dilation are similar. The ratio of corresponding sides of
the figure and its image equals the scale factor of the dilation.
Z
You’ll have a chance to prove this for triangles as an Exercise. This is just the beginning of interesting dilation properties. In Art of
Problem Solving’s Intermediate Geometry, we will build on these basics to find powerful and intriguing applications of dilation.

E x e rc is e s

16.4.1: t V

In A X Y Z , we have X Y = 3 , Y Z = 4, and X Z = 5. Suppose A X Y Z is the image of A X Y Z under a dilation with


scale factor 4.

(a) What is X Y ?

Type your solution, notes and/or work here. Show Solution

(b) What is the area of A X ' Y ' Z'?

Type your solution, notes and/or work here. Show Solution

(c) Is it possible to determine X X ' with the information given in the problem?

Type your solution, notes and/or work here. Show Solution

16.4.2: t V

Point Q is the image of point P under a dilation with center O and scale factor 5. If P Q = 20, what is O P ?

Type your solution, notes and/or work here. Show Solution

16.4.3: t V

Show that if A , B ', and C ' are the images of A, B, and C, respectively, under a dilation with center O and scale factor k, then
A A B C ^ A A 'B ’ C'. (You can assume k is positive.)

Hint

Type your solution, notes and/or work here. Show Solution

16.4.4★ : t V

In this problem, we show that the image of a segment upon a dilation is itself a segment. Suppose that A and B ' are the
images of A and B, respectively, upon dilation about O with scale factor k. (You can assume k > 0; the proof for k < 0 is
very similar.)

(a) Let X be a point on A B , and let X ' be the image of X under the dilation. Show that Z O X A = Z O X 'A ' and
Z O X B = Z O X 'B '.

Type your solution, notes and/or work here. Show Solution

(b) Use part (a) to show that Z A X B = 180° Why does this mean that the image of each point on A B is a point on
Â7F?

Type your solution, notes and/or work here. Show Solution

(c) Show that each point on A 'B ' is the image of some point on A B under the dilation. Why is this step necessary to
complete the proof?

Type your solution, notes and/or work here. Show Solution


16.5 Changing the Question
Sometimes problems can be quickly solved by rearranging the problem or by considering special cases of the problem. Typically
this requires a little creativity and flexibility.

P ro b le m s
Problem 16.14 Source: MATHCOUNTS 4 Jump to Solution

The side length of the largest square at right is 10 inches. The midpoints of the sides of the largest
square are connected to form a smaller square. Opposite vertices and the midpoints of opposite sides of
the smaller square are then connected as shown. What is the number of square inches in the area of the
shaded region?

Problem 16.15 Source: MATHCOUNTS 4 Jump to Solution

A circle is inscribed in a large square and circumscribed about a smaller square. The area of the larger
square is 8 square meters. What is the number of square meters in the area of the smaller square?

Problem 16.16 Source: Mandelbrot 4 Jump to Solution

In the figure, A B C D is a square with side length 1, and A / and N are the midpoints of A B and C D ,
respectively. Find the area of the shaded region.

Problem 16.17 4 Jump to Solution

In the diagram we have two concentric circles, and chord A B of the large circle is tangent to the
smaller circle. Given that A B = 8, find the area of the region between the two circles.

Problem 16.18 4 Jump to Solution

In the diagram, Z I N T = 80° The angle bisectors of two of the exterior angles of A T I N meet at point
K , as shown. Compute Z 1 K T .

Since rotations, reflections, and translations don’t alter lengths or areas, we can often use them to simplify problems. Here are
several examples.

Problem 16.14 Source: MATHCOUNTS t V

The side length of the largest square at right is 10 inches. The midpoints of the sides of the largest square
are connected to form a smaller square. Opposite vertices and the midpoints of opposite sides of the
smaller square are then connected as shown. What is the number of square inches in the area of the shaded
region?

Solution for Problem 16.14: Solution 1: Each of the shaded regions is a 4 5 9 0 ‫־‬45‫ ־‬triangle,so we can find theareaof each by
finding one of the legs. One leg of each triangle is half a diagonal of one of the 5 x 5 quarters of the largestsquare.Therefore,
each triangle has a leg of length 5 \/2 /2 , so each triangle has an area of (5 > /2 /2 )2/2 = 25/4. There are four such triangles, so
the total shaded area is 25.

Solution 2: Instead of finding the leg lengths of each little triangle to find the area, we could have instead noted that each of these
little triangles is 1/4 of one of the 5 x 5 squares. Hence, each has area 25/4, so the total shaded area is 4 (2 5 /4 ) = 25.

Solution 3: Since the area of a triangle doesn't change if we slide, spin, or flip it, we can
manipulate the shaded pieces to make a region whose area we know how to find easily.
We first slide the bottom triangles up to complete a large shaded isosceles right triangle.
Then, we can slide half of this triangle to the right to complete one of the 5 x 5 squares
as shown. Clearly, then, the area of the shaded region is 52 = 25. □

Concept: In addition to our strategy of chopping areas into easy‫־‬to‫־‬handle pieces, we can also
I sometimes rearrange them to form an easy‫־‬to‫־‬handle whole.

Problem 16.15 t V

A circle is inscribed in a large square and circumscribed about a smaller square. The area of the larger
B
square is 8 square meters. What is the number of square meters in the area of the smaller square?

K
D

Solution for Problem 16.15: As with the last problem, we can work through several steps to the answer, or we can make a little
transformation to simplify the problem.

Solution 1: Since our large square has area 8, each side has length VS = 2 \/2 . Because the circle is tangent to all four sides of
the large square, the diameter of the circle equals the side length of the large square. But the diagonal of the small square is also
a diameter of the circle, so the diagonal of the small square has length 2 \/2 . Therefore, its sides are each ( 2 \/2 ) / \/2 = 2 and
its area is 4.

Solution 2: We can spin the small square without changing its area, so we spin it so that its vertices are the
M B
points where the circle is tangent to the large square. When we draw the diagonals of the small square
M N O P , we divide the large square into four equal pieces. Half of each of these pieces is inside M N O P ,
so [A /N O P ] = [ A B C D ] / 2 = 4 □

Problem 16.16 Source: Mandelbrot t V

In the figure, A B C D is a square with side length 1, and A / and N are the midpoints of AB and CD,
M B
respectively. Find the area of the shaded region.

D N

Solution for Problem 16.16: M B Y X is a trapezoid (why?), but it's not clear how we’ll find the height or either of the bases. It's
also not clear how we could chop M B Y X into easy‫־‬to‫־‬handle pieces. However, it looks like M B Y X is congruent to N D X Y .
We don't have any tools for proving quadrilaterals are congruent like we do for triangles. But if we can transform one
quadrilateral into the other with flips, spins, or slides, then we’ll know they are congruent.

We might think to reflect A A B C over A C , but that won’t map A I to N . However, if we rotate A A B C 180° about the center of
the square, we’ll have the transformation we want. (You’ll prove this as an Exercise.) This rotation maps A / B Y X to N D X Y ,
showing that these quadrilaterals are indeed congruent. Therefore, the area of A / B Y X is half the area of A I B N D . M B N D
is a parallelogram with base N D = 1 / 2 and height B C = 1, so its area is 1/2. Therefore, the shaded region has area
(1 /2 J /2 = 1/4. □

Transformations aren’t the only ways we can manipulate diagrams to help us with problems.

Problem 16.17 t V

In the diagram we have two concentric circles, and chord AB of the large circle is tangent to the smaller
circle. Given that AB = 8, find the area of the region between the two circles.

Solution for Problem 16.17: The problem doesn’t tell us what the radius of either the larger circle or the
smaller circle is. We only know the length of that one tangent chord. Therefore, the problem suggests
that the shaded region between the circles has the same area no matter what these two radii are, as
long as that tangent chord has length 8. We can get a quick answer for what that area is by picking a
special case that’s easy to work with. The simplest is to shrink the smaller circle down to a point.
Then, our tangent chord becomes a diameter of the larger circle, and our shaded region is this whole
circle. The area of this region, then, is just ( 8 /2 ) “ tt = 167T.

Notice that we have not proved that the shaded region is always 167T when our tangent chord is 8 units long. We’ve only found it
for one case. However, if the area is always the same, we know it’s 16tt. To prove that it’s always the same, we have to find some
way to get information about the radii of the two circles. We have a tangent line, so naturally we build a right triangle by drawing
radii as shown below.

Since O C ± A B , O C bisects chord A B . Therefore, B C = 4. From right triangle A O B C , we have


O B 2 — O C 2 = B C 2 = 16. Since O B and O C are the radii of our circles, the area we seek is

( O B>22) 7T - (O C 2)n = ( O B 2 - O C 2) t v = 16;r.

Concept: When working on a problem with a variable set-up, consider the extreme possibilities.

WARNING!! Before you start examining those extremes, make sure you really can vary the set-up.
If you vary the set-up in a way that violates information given in the problem, then your
O conclusions don’t necessarily apply to the original problem! For example, if we are
asked to find the area of a triangle with sides of length 13 ,14, and 15, we can’t simply
pretend two of the sides are perpendicular to get the answer!

Let's try this technique on one more problem.

Problem 16.18 t V

In the diagram, Z I N T = 80°. The angle bisectors of two of the exterior angles of A T I N meet at point
K , as shown. Compute Z I K T .

/ [ ___ _ K
tc /
1\ /
N[ v ‫׳‬- —
T B

Solution for Problem 16.18: Since Z I T N can vary, we’ll call it x. From A I N T , we find that

Z T I N = 180° - 80° - x• = 100° - x.

Therefore,

Z A I T = 180° - Z T I N = 80° + x

and Z I T B = 180° — x. Since I K and T K bisect these angles, we have Z K I T = 40° 4‫ ־‬x / 2 and Z K T I = 90° — x /2 .
Finally, we can use A K I T :

Z K 4- Z K I T A Z K T I = 180°.

Substitution gives

Z K + (40° 4- x /2 ) + (90° - x /2 ) = 180°,

from which we find Z K = 50°. Notice that in this problem, we can place points / and T anywhere on the sides of Z A N B
without violating the problem. We only need Z I N T = 80°. Therefore, to quickly check the answer we found above, we can
consider the extreme case of putting / at N .

When / is at N , Z A I T is Z N and I K is the bisector of Z N . Therefore, Z K I T = 8 0 °/2 = 40°.


Furthermore, Z I T B is a straight angle, so its bisector is perpendicular to I T . Since A K I T is a right
triangle, we then have Z I K T = 90° — 40° = 50°. Notice that this is not a proof that Z l K T is
always 50° - it’s just a quick way to check the answer we found earlier. □

Concept: Considering extremes in problems with varying set-ups is a great way to check your
answer. But remember this warning:
1

WARNING!! Considering extremes does not constitute a proof! You still have to show your answer
holds in all cases.
0

E x e rc is e s

16.5.1 Source: MATHCOUNTS t V

The rectangle at right is inscribed in the circle, and a rhombus is inscribed in the rectangle by connecting
the midpoints of the sides of the rectangle as shown. If the radius of the circle is 4 ft, how many feet are in
the perimeter of the rhombus?

Type your solution, notes and/or work here. Show Solution

16.5.2: Source: MATHCOUNTS t V

I, l y io uic 11 iiufjuii il I x_/Vv‫ ׳‬, ai iu nie aica /.\ 1 j i s 1 10 •


‫־‬r D
mber of square centimeters in the area of rectangle A B C D ?

Type your solution, notes and/or work here. Show Solution

In Problem 16.16, we asserted that D N Y X is the image of B M X Y upon rotation about the center of the square. Complete
our solution by proving this fact.

Hint

Type your solution, notes and/or work here. Solution

16.5.5★ : Source: AMC 10 t V

In trapezoid A B C D , A B and C D are perpendicular to A D , with A B 4‫ ־‬C D = B C and A D = 7. What is A B • C D ?

Hint

Type your solution, notes and/or work here. Solution

Extra! Fagnano’s Problem asks what is the triangle of smallest perimeter that can be formed
by connecting a point on each side of an acute triangle. In an acute triangle, the
v triangle formed by connecting the feet of the altitudes is the triangle with smallest
perimeter that can be formed by connecting a point on each side of the triangle. This
was originally proved by Fagnano using calculus, but mathematician HA Schwarz
came up with the amazing proof without words offered below, using the property of
reflection repeatedly. See if you can figure out how this proof works!
16.7 Summary

Definitions: In this chapter we explored four geometric transformations. A figure's image under a
transformation is the result of applying the transformation to that figure. We also say
that a transformation maps a figure to its image. A fixed point of a transformation is a
point that is its own image. Four important transformations are described below.

■ When we slide a figure a given amount in a specified direction, we perform a


translation.

■ When we apply a rotation to a figure, we spin it by some angle about some point.
The point is the center of rotation.

■ The image of a figure upon reflection over a line is what results when the figure is
flipped over the line. If a figure is its own image upon reflection over a line, the line
is a line of sym m etry of the figure.

■ Loosely speaking, when we apply a dilation about a point O to a figure, we stretch


the figure away from O or shrink it towards O.

Translations, rotations, and reflections preserve length, area, and angles, which
means that the lengths, areas, and angles in a figure's image equal the corresponding
quantities in the figure.

P roblem Solving S trateg ies

Concepts:
■ Understanding transformations is often more about learning what stays the same
rather than what changes. Whenever you are learning about a new transformation,
focus on what stays the same.

■ If you can't solve a problem, try thinking of a similar, simpler problem and solve
that. Then try to use your solution to the simpler problem to solve the harder
problem.

■ In addition to our strategy of chopping areas into easy‫־‬to‫־‬handle pieces, we can


also sometimes rearrange them to form an easy‫־‬to‫־‬handle whole.

■ When working on a problem with a variable set-up, consider the extreme


possibilities. Considering extremes is also a great way to check your answer.

■ In construction problems in which we must locate a single point, our general


strategy is to find two figures, usually circles or lines, which must contain the point.
The intersection of these two figures is the point we want.

■ When stuck on a problem involving equilateral triangles, think about 60° rotations!

T h in g s To W atch Out For!

WARNING!!
■ Before you start examining extremes in a problem, make sure you really can vary
O the set-up. If you vary the set-up in a way that violates information given in the
problem, then your conclusions don't necessarily apply to the original problem!

■ Considering extremes does not constitute a proof! You still have to show your
answer holds in all cases.
Review Problems

1 6 .2 2 : t V

A B C D E F G H is a regular octagon. A translation maps G to D. To what point does this same translation map I I I

Type your solution, notes and/or work here. Show Solution

1 6 .2 3 : t V

O P Q R S T U V W X Y Z is a regular dodecagon.

(a) A rotation of 0! degrees about U maps V to T. Given 0! < 180°, find 0!.

Type your solution, notes and/or work here. Show Solution

(b) A rotation of 02 degrees about U maps X to R. Given 02 < 180°, find 02.

Type your solution, notes and/or work here. Show Solution

(c) A rotation of 0;{ degrees about U maps W to S. Given 0;! < 180°, find 0;*.

Type your solution, notes and/or work here. Show Solution

(d) A rotation of 0,! degrees about U maps Y to Q. Given 0.! < 180°, find 0,!.

Type your solution, notes and/or work here. Show Solution

(e) A rotation of 05 degrees about U maps Z to P. Given 05 < 180°, find 05.

Type your solution, notes and/or work here. Show Solution

1 6 .2 4 : t V

All of the angles of octagon G I I I J K L M N are 135°. If

GH = IJ = K L = M N = 1

and

H I = J K = L U = G N = 2,

then how many lines of symmetry does G I I I J K L M N have?

Type your solution, notes and/or work here. Show Solution

1 6 .2 5 : t V

(a) G / I and G I are line segments such that G I I = G I. Must there exist a reflection that maps G / I to G i l

Type your solution, notes and/or work here. Show Solution

(b) X Y and A R are line segments such that X Y = A B . Must there exist a reflection that maps X Y to A B ?

Type your solution, notes and/or work here. Show Solution

1 6 .2 6 : t V

A B C D is a square. M is the midpoint of C D .

(a) Is there a rotation about M that maps A to B ?

Type your solution, notes and/or work here. Show Solution

(b) Is there a rotation about M that maps A to C l

Type your solution, notes and/or work here. Show Solution

(c) Is there a reflection over some line that maps M to A?

Type your solution, notes and/or work here. Show Solution

1 6 .2 7 : Source: MATHCOUNTS t V

In the figure at right, the shaded region is formed by drawing two parallel segments that connect the
midpoints of congruent squares. Each square has side length 1 centimeter. What is the area of the
shaded region?

Type your solution, notes and/or work here. Show Solution

1 6 .2 8 : t V

The image upon reflection of regular hexagon Q W E R T Y over \ v È is regular hexagon Q 'W 'E 'R 'T 'Y '

(a) Why must W ' and E ' be W and E, respectively?

Type your solution, notes and/or work here. Show Solution

(b) Find Z Q W Q '.

Type your solution, notes and/or work here. Show Solution

(c) ★ Find Y T ' if Q W = 6.

Hint

Type your solution, notes and/or work here. Show Solution

1 6 .2 9 : t V

Line £ is a line of symmetry of A A B C .

(a) Must line £ pass through a vertex of A A B C ? Why or why not?

Type your solution, notes and/or work here. Show Solution

(b) Is it possible for A A B C to be scalene (i.e., all sides have different lengths)? Why or why not?

Type your solution, notes and/or work here. Show Solution

1 6 .3 0 : t V

(a) Point R is the image of point S under a dilation with center T and scale factor 3. If 77? = 18, what is T S l

Type your solution, notes and/or work here. Show Solution

(b) Point E is the image of point I ) under a dilation with center O and scale factor —3. If E D = 9, what is O D l

Type your solution, notes and/or work here. Show Solution

(c) Point X is the image of point Y under a dilation with center A and scale factor 0.25. If X Y = 20, what is A Y ?

Type your solution, notes and/or work here. Show Solution

1 6 .3 1 t V

The image of circle © O under a dilation about P with scale factor 4 is © O '. The area of © O ' is 48 tt. What is the area of
©O?

Type your solution, notes and/or work here. Show Solution

1 6 .3 2 : t V

The rules for a race require that all runners start at A, touch any part of the 1200-meter b
wall, and stop at B. What is the number of meters in the minimum distance a
500 m
participant must run? 300m
il
1200 m

Type your solution, notes and/or work here.

1 6 .3 3 : t V

Lines £ and m are perpendicular at point X . Point A is not on either line. B is the image of A upon reflection over £, and C is
the image of B upon reflection over m. Show that Z A B C = 90° and X A = X B = X C . Must A C pass through X I

Type your solution, notes and/or work here. • III •

1 6 .3 4 : Source: MATHCOUNTS t V

A square sheet of paper has area 6 cm2. The front is white and the back is shaded. When the sheet is
folded so that point A rests on the diagonal as shown, the visible shaded area is equal to the visible white
area. How many centimeters is A ' from its original position, ;4?

Hint

Type your solution, notes and/or work here. Show Solution

1 6 .3 5 : t V

A rotation about A maps X to Y . A different rotation about X maps A to Y. Show that A A X Y is equilateral.

Type your solution, notes and/or work here. Show Solution

1 6 .3 6 : t V

Point O is inside quadrilateral A B C D . A rotation of 180° about O maps the quadrilateral A B C D to itself, meaning each
vertex of the image is also a vertex of the original quadrilateral.

(a) Must A B C D be a parallelogram?

Type your solution, notes and/or work here. Show Solution

(b) Must A B C D be a rectangle?

Type your solution, notes and/or work here. Show Solution

(c) Must A B C D be a square?

Type your solution, notes and/or work here. Show Solution

1 6 .3 7 : t V

The image of point A upon a 90 degree clockwise rotation about point B is point C. The image of C upon a 90 degree
rotation clockwise about point D is point A. Prove that A B C D is a square.

Type your solution, notes and/or work here. Show Solution

1 6 .3 8 : t V

Let A B C D be a rhombus with Z B A D = 60° Let P and Q be points on A D and C D ,


respectively, such that Z P B Q = 60°. Find the other two angles of triangle P B Q .

Type your solution, notes and/or work here. Show Solution

1 6 .3 9 : t V

A A B C h a s A B = 3, B C = 4, and A C = 5.

(a) A cone is formed by rotating the triangle around A B . What is the volume of this cone?

Type your solution, notes and/or work here. Show Solution

(b) A cone is formed by rotating the triangle around B C . What is the volume of this cone?

Type your solution, notes and/or work here. Show Solution

(c) ★ A solid is formed by rotating the triangle around A C . What is the volume of this solid?

Type your solution, notes and/or work here. Show Solution

1 6 .4 0 : t V

Show that if X ' and Y ' are the images of X and Y under iter O and seal vhere k is negative, then
we have X W 7 || ‫־‬X Y and X ' Y ' / X Y = |fc|.

Type your solution, notes and/or work here. • III •


Challenge Problems

16.41 t V

C D is the image of A B under some rotation that is not the identity. A B and C D have the same midpoint, M . Must M be
the center of the aforementioned rotation that maps A B to C D ?

Hint

Type your solution, notes and/or work here. Show Solution

1 6 .4 2 : t V

You might recognize the diagram at right from Problem 8.22 in Section 8.6 here. It is indeed the
same problem. A B C D is a square. The circle with center ;4 and radius A B intersects the
perpendicular bisector of A D in two points, of which O is the one inside the square. Use the
principles we have explored in this chapter to quickly find Z A O C .

Hint

Hint

Type your solution, notes and/or work here. Show Solution

1 6 .4 3 : t V

Consider the octagon of Problem 16.24.

(a) Show that there is a point inside the octagon that is equidistant from all 8 vertices of the octagon.

Hint

Type your solution, notes and/or work here. Show Solution

(b) Show that a 90° rotation about the point found in part (a) maps the octagon to itself.

Hint

Type your solution, notes and/or work here. Show Solution

Source: MATHCOUNTS t V

A A B C with vertices A ( 2, 4), B (6 , 4) and C 4 , 10)‫ )׳‬is graphed in a coordinate plane. What will the sum of the abscissas (x-
coordinates) of the vertices be when A A B C is reflected over the line x = 8?

Type your solution, notes and/or work here. Show Solution

1 6 .4 5 : t V

The image of the incircle of equilateral triangle A A B C upon a dilation about the center of the circle with scale factor k is the
circumcircle of A A B C . Find k.

Type your solution, notes and/or work here. Show Solution

1 6 .4 6 : t V

Use a clever dissection of the section of regular octagon A B C D E F G H that is traced in bold at B
right to prove that

[A B C D E F G H ] = (v 2 ‫[) ׳‬A C E G ].

( 0 is the center of the octagon.)

Hint

Hint

Type your solution, notes and/or work here. Show Solution

1 6 .4 7 : Source: Mandelbrot t V

A laser beam is fired from point A in the U-shaped room at right. It reflects from each wall as from a
perfect mirror. What is the minimum distance the beam travels before hitting a target at point B? (A and
B are the midpoints of their respective sides.)

Type your solution, notes and/or work here. Show Solution

1 6 .4 8 : t V

f n d ß t o ß 11 *‫ ׳‬A B and A ' B ' / A B = k' Where k * 1 ln this Problem• we P ™ e that there is a dilation that maps .4 to >1‫׳‬

A---- V
(a) Why must .4,4' and B B ' intersect?

Type your solution, notes and/or work here. Show Solution

i---- V
(b) Let P be the intersection of A A ' and B B '. Show that A P A B ^ A P A 'B '

Type your solution, notes and/or work here. Show Solution

(c) Show that there is a dilation with center P that maps ;4 to A ' and B to B '.

Type your solution, notes and/or work here. Show Solution

1 6 .4 9 : t V

Let A B C D be a square, and let M and N be the midpoints of A B and B C , respectively. Prove that C M _L D N .

Hint

Type your solution, notes and/or work here. Show Solution

1 6 .5 0 : t V

The center of the cue ball on my rectangular pool table is directly above point A on the table. I wish to bounce the cue ball off
a rail such that after it bounces off the rail, the center of the ball will pass directly over point B on the table. The radius of the
cue ball is 1 in. A is 6 inches from the rail, and B is 9 inches from the rail. A is 5 inches from B. How far from the nearest
point on the rail to point B do I want the cue ball to hit the rail?

Hint

Hint

Type your solution, notes and/or work here. Show Solution

1 6 .5 1 ★ : » V

What is the total length of the shortest path that goes from (0 .4 ) to a point on the x-axis, then to a point on the line y = 6,
then to (18,4)?

Hint

Hint

Type your solution, notes and/or work here. Show Solution

1 6 .5 2 ★ : t V

I am standing at point A in the diagram at right. I wish to walk to a point on B(5, then to a point on
Rz3, then back to my starting point. Construct with straightedge and compass the shortest such
path I can take.

Hint

Hint

Type your solution, notes and/or work here. Show Solution

1 6 .5 3 ★ : t V

Regular octahedron A B C D E F has volume 360 cubic units. What is the volume of that portion of the octahedron that
consists of all points that are closer to A than to any other vertex?

Hint

Type your solution, notes and/or work here. Show Solution

1 6 .5 4 ★ : Source: Mandelbrot t V

In the diagram at right, A A B C , A P B R , A A Q R , and A P N Q are equilateral. Prove that N c


B Q = R N = AP.

Hint

Type your solution, notes and/or work here. Show Solution

1 6 .5 5 ★ : t V

Prove that the image of a circle under rotation about any point is also a circle.

Hint

Type your solution, notes and/or work here. Show Solution

Extra! Suppose we have a circle © 0 with radius r. The geometric transformation called inversion maps each

V point P other than O to the point P ' on o f t such that O P ‫ ־‬O P ' = r 2. To make inversion work, we
define the point at infinity to be the image of the center of inversion, O. Experiment with inversion
yourself by finding out what happens to various points upon inversion. What happens to points inside the
circle? Points outside the circle? Points on the circle?

Here are some more questions to guide further exploration of inversion:

■ What is the image of © 0 upon inversion with respect to © 0 ?

■ What is the image of a line that passes through 0 ? Of a line that does not pass through 0?

■ What is the image of a circle that passes through 0 ? Of a circle that does not pass through 0?

Well explore these properties and more in later books in the Art of Problem Solving series.
New facts often trigger new ideas. - Alex F. Osborn

Much of the mathematics developed by the ancient Greeks was geometry. In fact, even much of the number theory and algebra
that they developed was expressed in geometric terms. However, by the 16‫ ־״‬or 17‫ ' ״‬century A.D., algebra was recognized as an
extremely important field of mathematics in its own right, possibly equaling, or even exceeding, geometry.

In 1637, the great mathematician and philosopher Reno Descartes brought these two great fields of mathematics together when
he described a general method to represent geometric figures with algebraic equations. This combination of algebra and
geometry is often referred to as analytic geometry In this chapter, we review how to describe lines and circles with equations
then we apply these methods to both algebra and qeometrv Droblems

WARNING!! If you have not studied graphing lines and circles in the Cartesian plane, you may find
the material in this chapter extremely difficult. If so, we recommend studying the
fundamentals of graphing lines and circles in our Introduction to Algebra textbook.

Moreover, the algebraic manipulation required for parts of this chapter is considerably
more complicated than in the rest of this text. Specifically, if you are not comfortable
with completing the square, you may wish to study it in Introduction to Algebra (or
another text) before returning to this chapter.

Descartes s great insight that unified geometry and algebra has become so important that we
still use Descartes's name to describe the result. We use the Cartesian plane to describe
geometric figures with algebraic equations.

At right is the Cartesian plane. The center of the plane, where the bold lines meet, is called the
origin. On the Cartesian plane each point is represented by an ordered pair of numbers. These
numbers denote the position of the point relative to the origin.

For example, we denote the point that is 3 steps to the right and 2 steps up from the origin with
the ordered pair (3,2). The "ordered" part is very important! The first number always denotes
how far the point is to the right (or left) of the origin, and the second number denotes how far the
point is above (or below) the origin. We call the two numbers in an ordered pair the coordinates
of the point. By convention, we call the horizontal (left-right) coordinate the *-coordinate and we
call the vertical (up-down) coordinate the ;!/-coordinate.

As you might have guessed, the *-coordinate of a point is negative when the point is to the left
of the origin, and the !/-coordinate is negative when the point is below the origin. For example,
the point ( - 5 , - 1 ) shown in the diagram is 5 steps to the left and one step below the origin!
The point (0 ,0 ) is 0 steps to the right and 0 steps up from the origin. In other words (0 .0 ) is the
origin itself.

У When the point is neither above nor below the origin, its !/-coordinate is 0. Such a point is directly
to the left or right (or on) the origin. Therefore, the point must be on the bold horizontal line in the

diagram at left. We call this line the *-axis. Similarly, the vertical line consisting of points directly
ra
above or below (or on) the origin is called the ;!/-axis. We usually label the *-axis and ;!/-axis with
/r-axib (4 0) an * and a y, respectively, as shown in each of our diagrams.

( 0, ‫ ־‬3)

Finally, we aren't restricted to using integers for our coordinates. We can plot any point У
represented by an ordered pair of two real numbers on the Cartesian plane. For example, the
diagram at right depicts the point (3.5, —2.7), which is 3.5 steps to the right and 2.7 steps
below the origin. Points that have integers for both coordinates are called lattice points.

17.1 Lines
In this section, we review how equations can be used to represent lines. We cover this material (3.5,*-2 7 )
in much more detail in Introduction to Algebra. If you are unfamiliar with using equations to
represent lines, you may wish to study the fundamentals covered in Introduction to Algebra
before continuing.

The graph of an equation that has * and y as variables consists of ail points ( * , y) on the
Cartesian plane that satisfy the equation. For example, the graph of the equation 2 * - f 3 „ = 6 is У
shown at right As we see, the graph is a line. Indeed, the graph of any equation of the form
A x + B y - C. where A, B. and C are constants and A and B are not both 0, is a line. As a
result, such an equation is often called a linear equation.

The points where a graph intersects the *-axis are called the x-intercepts of the graph, and the
points where a graph intersects the !/-axis are called the y-intercepts of the graph For example
the only *-intercept of the graph of 2 * + 3y = 6 at right is (3 ,0 ) and the only ;!,-intercept of the
graph is (0,2). One way we describe graphed lines is by indicating the 'direction' of the line, which
we call the slope of the line. We define the slope, m, of the line that passes through the points
)and ( * 2,t/ 2 )to be

У2 - 2/1
m -
* 2 - # 1 ‫׳‬

A line with positive slope goes upward as it goes from left to right, and a line with negative slope goes downward as it goes from
left to right.

Figure 17.1: A Line With Positive Slope Figure 17.2: A Line With Negative Slope

The slope of a horizontal line is equal to 0, and the slope of a vertical line is undefined. Moreover, the greater the magnitude of
the slope of a line, the more 'steep' the line is.

У
. 1f
!/
.... 1!
......,1
t
Slope = 1/4 Slope = 1 Slope = 4

P roblem s
Problem 17.1
4 Jump to Solution
Find an equation whose graph is the line passing through the points ( - 1 , 7 ) and ( - 5 , - 3 ) .

Problem 17.2
4 Jump to Solution
Find an equation whose graph is the line with slope 4 and ‫ ״‬-intercept (0, - 3 ) .

Problem 17.3
4 Jump to Solution
(a) Find the length of the segment with endpoints ( - 4 , - 3 ) and (6,1).

(b) Find a formula for the distance between the points ( * ! , ‫ )! ״‬and (x 2 , y 2).

Problem 17.4
4 Jump to Solution
Let point ,4 be ( * 1 , ‫)! ״‬and point /3 be ( * 2 . 2 ‫ ■) ״‬Let M be the point with coordinates

/ * ! + * 2 ‫ ״‬, + ‫ ״‬2\
V 2 ’ 2 J '

In this problem we show that M is the midpoint of A B .

(a) Show that A M = M B .

(b) Show that M is on A B .

Problem 17.5
4 Jump to Solution
Explain why two different lines that have the same slope are parallel.

Problem 17.6
4 Jump to Solution

In this problem we show that if two lines are perpendicular, and neither line is vertical, then the product of their slopes is - 1.

Shown at right are perpendicular lines fc and t Let P be the point where these lines intersect. We
construct two right triangles such that their hypotenuses are along lines fc and t , respectively, as
shown. Moreover, each right triangle has a horizontal leg and a vertical leg.
(a) Show that A A D P ~ A B E P .

(b) Use the similar triangles from part (a) to show that

DA EP
D P ‫ ׳‬EB ~ l‫־‬

Why does this tell us that the products of the slopes of k and t is -1 ?

We start with a review of some different forms in which we can write equations whose graphs are lines.

Problem 17.1
-------------------------------------------------------------------------------------------------------------------------------------------- ----------------------- J_ V
Find an equation whose graph is the line passing through the points ( - 1 , 7 ) and ( - 5 , - 3 ) .

Solution for Problem 17.1: The slope of the line through these two points is

7 - (-3 ) _ 10 _ 5
—1 — ( —5 ) ‫־‬ 4 ‫ ־‬2‫־‬

Therefore, if ( * , y) is any point on this line other than ( - 1 ,7 ) , then the slope between ( * , y) and ( - 1 , 7) must be 5/2. So, we

У -7 _ 5
x - ( - 1) ~ 2
r
Multiplying both sides of this equation by * ( —!)gives us у — 7 : - [ * - ( - 1 ) ]. This is an example of the point-slope form
of a linear equation.

Important: The graph of the equation

z y - y ! = m ( x - x ])

is a line through ( * i, 2 / i) with slope m. This is called a point-slope form of the


equation.

While we often use point-slope form to find an equation of a line given the slope of the line and a point on the line, we often then
convert the equation to standard form

Important: The standard form of a linear equation is A x + B y = C, where, if possible, A. B,


z and C are integers, A is positive, and A, B, and C have no common factors besides
1 .

5
We start converting y 7 - - [ * - ( - 1 ) ] to standard form by first multiplying both sides by 2 to get 2 ( „ - 7) = 5 ( * + 1).
Expanding both sides and rearranging gives 5 * — 2 „ = —19.

We can check our answer by making sure that the two points ( * , y) = ( - 1 , 7) and ( * , y) = ( - 5 , - 3 ) satisfy our equation.
Both do, so we know the equation 5 * - 2y = - 1 9 represents the line through ( - 1 , 7 ) and (—5, - 3 ) . □

One more form that is occasionally useful is slope-intercept form, which we'll investigate in the next problem.

Problem 17.2
t V
Find an equation whose graph is the line with slope 4 and ;(/‫־‬intercept (0, - 3 ) .

Solution ^ P r o b le m 172: Because the line passes through (0, - 3 ) and has slope 4, a point-slope form of the line is
y' ( ™ lsolatln9 y on the left side then gives us y = 4 * - 3. Notice that the coefficient of * equals the slope
of our line and the constant term on the right equals the ;!/-coordinate of the ;!/-intercept. This is the slope-intercept form of the
equation.

Important: The slope-intercept form of a linear equation is

z y = m x + b,

where m is the slope of the line and b is the ;„-coordinate of the ;„-intercept.

As an Exercise, you'll prove that the graph of an equation of the form y = m x + b has slope m .

We'll now review some particularly useful relationships regarding segments and lines on the Cartesian plane.

Problem 17.3
t V
(a) Find the length of the segment with endpoints ( - 4 , 3‫ ) ־‬and ( 6 . 1).

(b) Find a formula for the distance between the points ( x i , y x) and ( x 2, y2).

Solution for Problem 77.3:


У
(a) We start by labeling our points A and B such that A is ( - 4 , - 3 ) and B is (6,1).

One of our favorite tactics for finding the length of a segment in a geometry problem
is to build a right triangle with that segment as a side. Here, an easy way to build a
right triangle with A B as a side is to extend segments horizontally from A and
vertically from B to meet at point C, as shown. Since a horizontal line and a vertical _..**
line meet at a right angle, Z A C B is a right angle. So, if we can find A C and B C , we
г
can use the Pythagorean Theorem to find A B . c

The *-coordinate of C is the same as the *-coordinate of B and the ;„-coordinate of


C is the same as the !/-coordinate of A . So, point C is (6, - 3 ) . Therefore,
A C = 6 - ( - 4 ) = 10 and B C = 1 - ( - 3 ) = 4, and the Pythagorean Theorem
gives us

.413 = sjA C * + BC"2 = V100 + 16 = v/ITe = 2^29.

(b) Building a right triangle worked well when finding the distance between two specific points, so let's try using that tactic to
find a formula for the distance b e tw e e n ^ *„ „ , ) and ( x 2 , y 2). Using part (a) as a guide, we let A be ( * „ „ ! ) and /3 be
(# 2 , 2/2 ) and build a right triangle with A B as a side.

In the diagram at right, we assume that /3 is above and to the right of A . As before, we
draw a vertical line down from /3 and a horizontal line to the right from A. We label the
point where these lines meet C. Since C is directly below B, its *-coordinate is the
same as /3's, or * 2 (in other words, it is just as far horizontally from the ‫„׳‬-axis as B is).
Similarly, since C is directly to the right of A, its „-coordinate is the same as A's.

Now we're ready to use the Pythagorean Theorem. We can use our coordinates to see
that C is * 2 - * , to the right of A and y2 - ‫ ״‬, below B . So, we can use the
Pythagorean Theorem to find

A B 2 = A C * + B C 2 = ( * 2 - * , ) 2 + (y2 - ‫ ״‬l } 2.

We can take the square root of both sides of this equation to find a formula for the distance between the points ( * , ‫ ״‬,)
and ( * 2, 2 ‫) ״‬: v 1>yl'

AD = \ J { x 2 - x ! ) 2 + (2( ! ‫ ״‬2 - ‫ ״‬.

This is often called the distance formula. Notice that i f * , = x %then the distance between ( * , , ‫ ״‬,) and ( * 2, 2 ‫ ) ״‬is just the
nonnegative difference between the ‫ ״‬-coordinates. Let's see if that's what the distance formula gives us when * , = * 2 :

\Д Х 2 X !) 2 -j- ( 2 ‫ — ״‬y l ) * = y j ( * 2 - * 2)2 + (2( , ‫ ״‬2 - ‫״‬

= \ J(2/2 2( 2/1 ‫־‬


= I2/2 -2/1I■
Indeed, the distance formula gives the expected result when * , = * 2. (We use absolute value signs when writing

y (2/2 - 2/i) — I2/2 —2/ 1 1because square roots, like distances, must be nonnegative.) Similarly, the distance formula tells
us that the distance between (as,, ‫ ״‬,) and (x 2, 2 ‫ ) ״‬when ‫ ״‬, = y2 is the nonnegative difference between the *-coordinates.

Important: The distance in the plane between the points ( * ! , ‫) ! ״‬and ( * 2 , 2/2 ) is

\A * 2 ‫ ־‬x ! ) 2 + ( 2 ( 2/ 1 - 2‫״‬ .

This is often referred to as the distance formula.

The distance formula is essentially the same thing as the Pythagorean Theorem. In fact, because you know the Pythagorean
Theorem, you basically know the distance formula already, so you shouldn't have to memorize the distance formula.

Problem 17.4
t V
point A be { x \ , y i ) and point B be (x 2, y2). Let M be the point with coordinates

xi +*2 Уi + y2
2 ’ 2
Show that M is the midpoint of A B .

Solution for Problem 77.4: In order to show that M is the midpoint of A B . we must show that A M = M B and that M is on
A B . To show that A M = M B , we turn to the distance formula. We have

x ! -p * 2 2 /1 + 2/2
AM = — X! I + ‫־‬ 2/ 1

* 2 - * i \ 2+ / 2/2 ‫ ־‬I/!' 2

2
MB = * 2 - * !+ * ^ / ‫ ׳ ! ־‬, + Î ‫׳‬2

*2 - x! ■ \2 ^ (y 2 - 2 '1 ‫״‬

So, we have A M = M B .

To show that M is on â Ê , we compare the slopes of and il I A We have

VI +И2 _

Slope o f A M _
= 2 _2/1 2/2 Vi-
£1 ±2 Î 2 _ *t 1, ~ _
*2 — x!
_ У1 -Н/2
2 2 /2 2/1
Slope o f \ f È = — ‫־‬

* ;,2 -------
_ £1 ±2 2 2* *2 - *!

When * 2 ^ * 1, the slopes of S a I and 1\ I & are the same, so A, M , and /3 are collinear. If * 2 — * 1 , then A. M and /3 are on
the same vertical line.

Since A, M , and B are collinear and A M = M B , we know that M is the midpoint of A B . □

Important: The midpoint of the segment with endpoints ( * ! , ‫ )! ״‬and (x 2, y2) is

* x2+ 1 ‫״‬1 + ‫״‬2

Problem 17.5
t V
Explain why two different lines with the same slope are parallel.

Solution for Problem 77.5. Slope is a measure of the direction of a line. Therefore, we expect
two lines with the same slope to be parallel. We can prove this quickly with geometry. If our two У
lines both have slope 0, then they are both horizontal, and clearly are parallel. Otherwise, we D к
can draw a horizontal line segment with one endpoint on each line, as shown in the diagram at
right. We can build two right triangles by drawing two vertical line segments, as shown. If we
/
show that Z A C B = Z C B D , then we can deduce that k || £. The only given information we В г ...e
j 7 ‫“־‬
have is that the lines have the same slope, so we need to find some way to express the slope of c
each line to allow us to use this information. Fortunately, our diagram gives us the answer. The
slope of k is D C / B C (change in ‫ ״‬-coordinate divided by change in *-coordinate), and the
slope of line / is A B / B C . Because the slopes of k and t are equal, we have X
D C / B C = A B / B C , so D C = A B . We also have A A B C = Z B C D , so we have
ADCn SAS Con9 ^ n c e . Therefore, we have Z A C B = Z C B D , so k || i, as desired. You'll have the chance
to prove the converse, that two parallel lines have the same slope (if they are not vertical) as an Exercise. □

Important: If two lines have the same slope, then they are parallel. Conversely, if two non-vertical
lines are parallel, then they have the same slope.
z
Weve seen that the relationship between the slopes of two parallel lines is pretty straightforward. What about perpendicular

Problem 17.6
t V
Show that if two lines are perpendicular, and neither line is vertical, then the product of their slopes is - 1.

Solution for Problem 77.6. Because the lines are perpendicular and neither is vertical, one line
has positive slope and the other has negative slope. To see why, suppose the lines intersect at P
, and that their slopes have the same sign. Then, the smaller angle formed by the lines is smaller
than the right angle formed by the vertical line and horizontal line through P . So, if the slopes of
the lines have the same sign, the lines cannot be perpendicular. Let the line with negative slope
be k and the line with positive slope be t Building right triangles worked so well in the last
problem that we should try it again for this problem. We’re also guided to build right triangles
since these will give us a way to express the slopes of k and L We can use essentially this same
proof for any configuration of k and £ in which the lines are perpendicular and neither line is
vertical. We make right triangles A A D P and A P B E such that each has a horizontal leg and
a vertical leg. From A P B E , we see that the slope of £ is E P / E B . From A A D P, we see that
the slope of fc is - D A / D P . Make sure you see why we need the negative sign; the slope of k is clearly negative. Now that we
have expressions for the slopes of A: and £, we can write an equation we wish to prove. We must show that

EP DA
E B ) \ DP

The ratiosjn this expression make us think of similar triangles. The right triangles are a pretty big clue, too. We already have
‫ " ־‬A ‫׳״‬ ‫ ־ ״‬e ° nly need t0 flnd one more Pair of eq‫ ״‬al angles to prove the right triangles are similar. We probably
need to use the fact that k and £ are perpendicular, so we focus on that. Since Z A P B = Z D P E = 90°, we have

Z A P D = 90° - Z D P B = Z B P E .

Combining Z A P D = Z B P E with Z A D P = Z B E P gives us A A D P ~ A B E P by AA Similarity. This similarity gives us

DA _ EB
DP ~ E P '

Multiplying both sides by - 1 and by E P / E B gives us the desired ( E P / E B ) ( - D A / D P ) = - 1 . Therefore, the product of
the slopes of fc and / ,s - 1 . As an Exercise, you'll have the chance to prove the converse: if the product of the slopes of two lines
is —1, then the two lines are perpendicular, d

Important: If two non-vertical lines are perpendicular, then the product of their slopes is - 1 .
z Conversely, if the product of the slopes of two lines is - 1 , then the two lines are
perpendicular.

Exercises

Find an equation, in standard form, whose graph is a line through ( - 2 , 5 ) and ( - 5 ,9 ) .

Type your solution, notes and/or work here.

Find the *-intercept and ‫ ״‬-intercept of the line with slope 4 that passes through ( - 7 ,2 ) .

Type your solution, notes and/or work here.

The distance between ( 4 ,3 ) and (a, 0) is 5. Find all possible values of a.

Type your solution, notes and/or work here.

When finding the distance between two points using the distance formula, does it matter which point we call ( * ‫ ״ ״‬,) and
which we call ( * 2 , 2/2 )? Why or why not? '

Type your solution, notes and/or work here.

Point P is ( —5 ,2 ) and point Q is ( —8,8).

(a) Find PQ .

Type your solution, notes and/or work here.

(b) Find the midpoint of P Q .

Type your solution, notes and/or work here.

(c) Find T such that Q is the midpoint of P T .

Type your solution, notes and/or work here.

Line fc is perpendicular to the graph of 4 * - 3 0 = 14 + ‫ ״‬and passes through ( - 3 , 2 ) . Find an equation in standard form
whose graph is line k.

Type your solution, notes and/or work here.

Show that the graph of ‫ = ״‬m x + b, where m and b are constants, has slope

Type your solution, notes and/or work here.

Show that if the product of the slopes of lines fc and £ is —1, then fc J_ /

Type your solution, notes and/or work here.

S u p p le A, B, and C are constants and /3 ф 0. Show that the slope of the graph of the equation A x + B y + C = 0 i

Type your solution, notes and/or work here.


17.2 Circles
In this section, we use the distance formula to find equations whose graphs are circles.

P roblem s
Problem 17.7 4 Jump to Solution

(a) Find an equation whose graph is a circle with center (4, —5) and radius 3 \/2 .

(b) Explain why the graph of the equation

( * ‫ ־‬h )2 + (y - k )2 = r 2,

where h, k, and r are constants with r > 0, is a circle with center ( h , k) and radius r.

Problem 1 7.7 t V

(a) Find an equation whose graph is a circle with center (4 , —5) and radius 3 \/2 .

(b) Explain why the graph of the equation

(* ‫־‬ h)2 {y
+ - k ) 2 = r 2,

where /?, k, and r are constants with r > 0, is a circle with center (/i, k) and radius r.

Solution for Problem 77.7:

(a) If a point (x , y) is on the circle, then it must be 3 \/2 from the center of the circle, which is (4, —5). So, the distance
between (x, y) and (4, —5) is 3 \/2 . The distance formula then gives us

\J { x - 4 )2 + (y + 5)2 = 3 n/2 .

Squaring both sides of this equation gives

(x - 4 f + (y + 5)2 = 18.

We could multiply out both (x — 4 )2 and (y 42(5 ‫־‬, but leaving the equation in this form makes it particularly easy to see
that the graph of this equation is a circle with center (4, —5) and radius 3 \/2 . In our next part, well see why.

(b) Taking the square root of both sides of (x — h )2 4‫( ־‬y — k )2 = r 2 gives us

\ j ( x - h )2 + ( y - k) 2 = r.

Since h and k are constants and r is a positive constant, this equation tells us that the point (x, y) is a distance of r from
the point (h, k). Therefore, every point (x , y) that satisfies this equation is on the circle with center (h, k) and radius r.

Conversely, if a point (x , y) is a distance of r from the point (h, k), then the distance formula tells us that

y j( x - h )2 + ( y - k )2 = r.

Squaring both sides of this equation gives us (x — h )2 + (y — k )2 = r 2, so we see that every point on the circle with
center (h, k) and radius r satisfies the given equation. Therefore, the graph of the equation (x — h )2 4‫( ־‬y — k )2 = r 2 is
a circle with center ( h , k) and radius r.

Important: The standard form of an equation whose graph is a circle is

Z ( x - h )2 + ( y - k )2 = r 2,

where /?., k, and r are constants with r > 0. The center of the circle is ( h , k) and the
radius of the circle is r.

Sometimes we have to do a little work to put an equation in standard form.

Solution for Problem 17.8: The 2 x 2 and 2y2 terms make us think of the standard form of a circle, because expanding the squares
on the left side of

(x - h)2 + (y - k )2 = r 2

will give us an x 2 and y 2. So, we try to write the given equation in this form. Our first step is to divide both sides by 2, to make the
coefficients of x 2 and y 2 both equal to 1. This gives us

x 2 - Ax 4‫ ־‬y 2 + Sy = 2.

We then complete the square in both x and y by adding 4 and 16 to both sides:

x 2 — Ax + 4 4‫ ־‬y 1 + Sy + 16 = 2 4 1 6 ‫ ־‬4 4 ‫־‬.

Since x 2 - 4x 4 4 ‫( = ־‬x — 2)2 and y 2 4‫ ־‬Sy - f 16 = (y 42(4 ‫ ־‬we have

(x - 2)2 + (y + 4 )2 = 22.

This equation is in the desired standard form of a circle. Therefore, we know that the graph of this equation is a circle with center
(2, —4) and radius y/2 2 .

WARNING!! Make sure you see why the center is not ( —2, 4) or (2,4). The standard form of a
_ circle is
o
(X - h )2 + (y - k )2 = r 2.

Comparing this to

(x - 2 f + (y + 4 )2 = 22

gives us h = 2 and k — —4.

Because the graph is a circle with radius \/2 2, its area is (\/2 2 )27T = 22 tt. □

Exercises

17.2.1: t V

Find the center and the radius of the circle that is the graph of the equation 3x*2 — \2 x 4 3 ‫־‬y 2 4 6 ‫;־‬r/ = 15.

Type your solution, notes and/or work here. Show Solution

17.2.2: t V

Find the standard form of the equation whose graph is a circle with center ( —2, ?)that passes through ( —5, 9)

Type your solution, notes and/or work here. Show Solution

17.2.3: t V

What is the length of the longest chord of the graph of

x 2 + 2x 4‫ ־‬y 2 — 0>y = Q>

that passes through the point ( —2, 4.5)?

Type your solution, notes and/or work here. Show Solution

17.2.4★ : t V

Find the two points at which the graphs of 2x — y = 7 and x — lOx* 4 y + 4y = —4 intersect

Type your solution, notes and/or work here. Show Solution


17.3 Basic Analytic Geometry Problems
In this section, we begin to explore the power of analytic geometry. Because analytic geometry gives us a tool to relate algebra
prob^em ™ ^' * 9'V6S ^ 3 ^ *° aPP'y 9eometric tools t0 al9ebraic Problems, and to use algebraic tools to solve geometry

Problems
Problem 17.9

The area of the shaded region between line k and the axes in the diagram at right is 36. If line k
passes through (12, 0), then what is the slope of A*? k

4 Jump to Solution
Let A be ( - 5 , 6), B be ( - 7 , 9), and C be (1,10).

(a) Find the length of the median of A A B C from B to A C .

(b) Find the slope of each side of A A B C . Notice anything interesting?


(c) Find the area of A A B C .

(d) Find the length of the altitude from A to 73C.

Source: MATHCOUNTS 4 Jump to Solution

In this problem, we determine the number of points (a, 6) such that a 2 + 62 = 25 and the area of the triangle with vertices
( - 5 ,0 ) , (5,0), and (a, b) is 10.

(a) What does the fact that (a, b) satisfies a 2 4‫ ־‬b2 = 25 tell us about where the point (a, b) is located on the coordinate
plane?

(b) What does the fact that the area of the triangle with vertices ( - 5 ,0 ) , (5 ,0 ) and (a, b) is 10 tell us about where the point
(a, b) is located on the coordinate plane?

(c) How many points satisfy the conditions you found in parts (a) and (b)?

4 Jump to Solution

A B C D is a rectangle with A B = Sand A D = 12. Points A and Y are on A B and C D , respectively, such that A X = 7 /3
and D Y = 1. X Y meets diagonal A C at point P. In this problem, we find B P .

(a) Set the problem up on the Cartesian plane. Let A be the origin. What coordinates can we assign to each of B, C, and D l
(b) What are the coordinates of P i

(c) Find B P .

Problem 17.13

In A A B C , we have A B — B C — 8 and Z A B C = 45°. In this problem, we find A C .

(a) Why should 45° make us think of the graph of y = x l

(b) What point on the line y = # is 8 units from the origin?

(c) Let B be the origin. Find coordinates for A and C such that A A B C satisfies the problem. Find A C .

Problem 17.9
t V
The area of the shaded region between line k and the axes in the diagram at right is 36. If line k
passes through (12, 0), then what is the slope of k?

Solution for Problem 17.9: The shaded region is a right triangle. The vertex of the right angle of this triangle is the origin, (0,0),
which is labeled A in the diagram. Vertex B is (12,0), so we know that leg A B has length 12. Because the area of the triangle is
36 we know that half the product of the lengths of the legs of the triangle is 36. Therefore, we have (A B M A C ) = 72 so
A C — 72/ A B = 6. So, vertex C‫ ׳‬is (0,6), and the slope of A: is (6 - 0 ) /( 0 - 12) = - 1 / 2 . □

Problem 17.10
t V
Let A be ( —5 ,6 ), B be ( - 7 , 9), and C be (1,10).

(a) Find the length of the median of A A B C from B to ‫־‬A C .

(b) Find the length of the altitude of A A B C from A to ‫־‬B C .

Solution for Problem 17.10:

(a) The midpoint of A C is ^ - . — - — ^ = ( - 2 , 8). The distance from B \o this point is

/ [ - 2 25^ = 2(9 - 8) + *[(7 -) ‫ ־‬T T = >/26.

(b) Solution 1: Find the coordinates of the foot o f the altitude. To find the length of the altitude from A to side B C , we try to find
the point P on B C such that A P ± B C . First, we find the equation whose graph passes through B and C. The slope of
this line is (10 - 9 ) / [ l - ( - 7 ) ] = 1/8. The line passes through (1,10), so an equation of the line in point-slope form is

V~ = g ( # — 1). Rearranging this equation gives# — Sy = —79.

Let point P be (#, y). Since (#, y) is on S 3 , it satisfies the equation x - 8 y = -7 9 . Because A P ± ‫־‬B C , the product of
the slopes of these two segments must be - 1 . Since the slope of ‫־‬B C is 1/8, we know the slope of A P is - 8 So Z f i
passes through (#, y) and ( - 5 , 6), and it has slope - 8 . This gives us

y - 6
= - 8.
# - (-5 )

Rearranging this equation gives 8# ± y = —34.


t h ô c w c t ü m /Ч-Р ^-1/‫־‬tI !

X - Sy = -7 9 ,
8x 4- y = -3 4 .

Multiplying the second equation by 8 then adding the result to the first equation gives us the equation 65# = -3 5 1 , which
yields # — - 2 7 /5 . Substituting this into either equation above then gives us y = 46/5. Therefore P = ( - 2 7 / 5 4 6 /5 )
so we have ‫״‬ v ' ' h

/1 256
V 25 + 25
/260
‫ ־‬V 25
_ 2>/65
5 ‘

Solution 2: Use some geometric insight. The algebra in our first solution is a little messy. The system of equations we have
to solve isnt very nice, and all the fractions give us plenty of opportunities to make mistakes. Once we see all that algebra
coming our way, we might stop and think if there are any geometric insights we can use to simplify the problem.

Concept: When the algebra in an analytic geometry problem starts to get ugly, try using
| geometric facts to simplify the problem.

Here, we seek the length of the altitude from A to B C . The altitude makes us think about area.
We can easily find the length of B C :

B C = y j ( - 7 - 1)2 + (9 - 10)2 = ^ 5 B

Therefore, if we can find [A B C \ we can easily find the length of AT5 since
( A P ) ( B C ) / 2 = [A B C ]. ‫׳‬ -

Is [A B C ] easy to find? To answer this, we think about what kind of triangle A B C is. If it is
isosceles or right, we can probably find the area easily. We can use slope to quickly check if it is
right. We've alread^seen that the slope of B C is 1/8. The slope of A B is - 3 / 2 and the slope of A C is 2 /3 The product
of the slopes of A B and A C is - 1 , so A B J_ A C .

Concept: When given the coordinates of the vertices of a triangle, check if the triangle is
! special in any way. Most notably, check if the triangle is a right triangle. If it is,
this fact will probably simplify the problem, because we know so much about
right triangles.

Now the area of A A B C is easy to find. Because A B A C = 90°, we have [A B C ] = ( A B ) ( A C ) / 2. Using the distance
formula, we find that A B = V 1 3 and A C = 2 \ / l3 , so

_ (n/ Ï 3 ) ( 2 x/ Î 3 )
[A B C ] = = 13.

Finally, we have

2 [A B C ] 26 26 v/65 2>/65
BC ~ ^ 65 \/6 5 ’ v/65 “ 5 ‫־‬

A little geometry sometimes goes a long way in analytic geometry problems!


Problem 17.11
Source: MATHCOUNTS t V

Find the number of points (a, b) such that a 2 4‫ ־‬b2 = 25 and the area of the triangle with vertices ( - 5 , 0), (5 ,0 ) and (a, b) is

Solution for Problem 17.11: Let our triangle be P Q R , with P = (a, 6), Q = ( - 5 , 0), and R = (5, 0). Side Q R is a horizontal
segment with length 10. Because the area of A P Q R is 10, we know that the length of the altitude from P to Q R is 2. Since
Q R is along the #-axis, the fact that P is 2 units from Q R means that P is either 2 units above or 2 units below the #-axis
Therefore, point P is on the line y = - 2 or on the line y = 2. From here we present two solutions.

Solution^ 7: Algebra. Because P is on either y = 2 or y = - 2 , we must have either b = 2 or b = - 2 . When b = 2, the equation
a 4 2 5 = 6 ‫ ־‬gives us a 2 = 25 - b2 = 21, so a = ± > /2 1 . So, two possible points P are (v/2 1 ,2 )a n d (-> /2 1 , 2). Similarly,
letting 6 = - 2 gives us two more points that satisfy the conditions in the problem, namely, (>/21, - 2 ) and (-> /2 1 - 2 )
Therefore, there are 4 different points that fit the description of the problem.

Solution 2: Geometry. Because (a, b) satisfies the equation # 2 + y 2 = 25. we know that (a, b) is on the
circle centered at the origin with radius 5. We also know that (a, b) is either on the line y = 2 or the line У
y = - 2 . Each of these lines is 2 units from the center of the circle, so both lines go inside the circle.
Therefore, both lines intersect the circle twice, as shown in the diagram at right. Each of the 4 points of
intersection between a line and the circle gives us a point that satisfies the restrictions of the problem. □

In our second solution, we interpreted the equation a 2 + b2 = 25 as telling us that the point (a, b) is on the circle with center
(U,U) and radius 5. Any time we see an equation that is in the standard form of a circle, we might consider using analytic
geometry. This gives us a new tool for tackling algebraic problems:

Concept: Interpreting an equation as a geometric figure on the Cartesian plane can often help
I us solve problems.

This street goes both ways! Sometimes it is useful to put a geometry problem on the Cartesian plane and use algebraic tools to
solve the problem.

Problem 17.12
------------------------------------------------------------------------------------------------------------------------------------------------t v
A B C D is a rectangle with A B = Sand A D = 12. Points A‫ ־‬and 1‫ ׳‬are on A B and C D , respectively, such that A X = 7 /3
and D Y = 1. X V meets diagonal A C at point P. Find B P .

Solution for Problem 17.12: We seek a length, so we might try building right triangles and using the Pythagorean Theorem (Try
doing so on your own.) Analytic geometry offers us another way to find lengths: if we can find the coordinates of two points, we
can use the distance formula to find the distance between them. So, we try turning this problem into an analytic geometry
problem. One reason we do so is that rectangles are particularly easy to describe with coordinates. We start by letting A be the
origin, and letting two sides of the rectangle be along the coordinate axes. This makes a lot of the coordinates in the problem
equal to 0, and 0‫׳‬s are easy to deal with.

Concept: When setting a geometry problem up on the Cartesian plane, choose the origin and
| the axes in a convenient matter. Often this means letting the origin be the vertex of a
right angle in the problem, so that the sides of the angle are along the axes.

In the diagram at right, we let A B be along the y-axis and A D be along the # ‫־‬axis.
Because £ £ = 8 and A D = 12, point B is (0 .8 ) and point D is (12,0). Because У
* is on A B such that .4 X = 7/3, we have X = (0, 7/3). Similarly, Y is 1 unit
above D, so Y = (12,1). Now, if we can find the coordinates of point P, we can use
the distance formula to find B P . We know that P is on ‫־‬A C . Because has slope 53
(8 - 0 )/(1 2 - 0) = 2 /3 and it passes through the origin, the equation whose graph
is 5 3 is y = 2 # /3 . (This is why we like having so many Os in our coordinates - the
resulting equations for lines connecting points in the diagram are simpler.) Point P is
also on X Y . The slope of £ 7 is (1 - 7 /3 )/(1 2 - 0) = - 1 / 9 and its ‫־‬/-intercept
is (0, 7/3), so the slope-intercept form of the equation of this line is

1 7
‫ ־ = ״‬5‫ ״‬+ 3 •
Rearranging this equation gives # + 9y = 21. So, P is the intersection of the graphs of y = 2 # /3 and # + 9y = 21.
Subst'tuting y = 2 # /3 into # + 9y = 21 gives # + 6# = 21, from which we find # = 3. Therefore, we have y = 2 # /3 = 2
so P is (3, 2). Finally, because B is (0,8), we have

B P = \ j i 3 - 0 ) 2 + ( 2 - 8 ) 2 = V 9 + 3 6 = V 4 5 = 3\/5.

A little algebra sometimes goes a long way in geometry problems! □

WARNING!! Don't get carried away with using analytic geometry to solve geometry pn
While our solution to Problem 17.12 was pretty straightforward with
О
geometry, you'll find that most geometry problems are easier to solve with ge
methods than with analytic geometry.

For example, suppose we wished to find the length of the median to the hypotenuse of a right triangle with sides of length 14, 48,
and 50. It is much faster to use the simple geometric fact that the median to the hypotenuse of a right triangle is half the length
of the hypotenuse than it is to assign coordinates to the vertices of the triangle, find the coordinates of the midpoint of the
hypotenuse, then use the distance formula.

After fimshmg this chapter, try flipping back through this book and finding problems that you can solve with analytic geometry.
ou find some common trends among the ones you can solve, such as rectangles, midpoints, and right triangles. You'll also
find some trends among problems that are hard to solve with analytic geometry, such as circles, angles that are not 45° or 90°
and complicated diagrams.

Our last sentence suggests that problems with 45° angles might be candidates for analytic geometry. Let's see why.

In A A B C , we have A B — B C — 8 and / . A B C = 45°. In this problem, we find A C .

Solution for Problem 17.13: We present both an analytic geometry solution and a ‫׳‬pure geometry' solution.

Solution 1: Analytic geometry. The 45° angle might make us think about analytic geometry
because the graph of the line y = x makes a 45° angle with the coordinate axes. We also notice
that A B = B C = 8 means that A and C are both on the circle with center B and radius 8. In
other words, if we let B be the origin, then the coordinates of A and C are solutions to the
equation

x 2 + y 2 = 64,

because the graph of this equation is the circle with center (0 ,0 ) and radius 8. We know that the
graph of y = x makes a 45° angle with the axes, so we let C be along the the at-axis, at (8,0),
and A be on the graph of the line y = x.

We can find the coordinates of A either by drawing an altitude from A to the x-axis, or by using our equation for the circle.
Because A is on both the circle and the graph of y = x, we can find the coordinates of A by substituting y = x into our
equation for the circle. This gives us s 2 + z 2 = 64, from which we find x = 4 ^ 2 . (In our diagram, A is on the right side of the y
-axis, so x must be positive.) Therefore, point A is ( 4 ^ 2 , 4 ^ 2 ) , and we can use the distance formula to find

AC = /( 4 v^ - 8 ) 2 + (4 n/ 2 - 0 ) 2

= / 3 2 — 64x/2 + 64 + 3 2

= \ J 1 2 8 - 6 4 v /2

= 8\/2 - V 2.

Solution 2. Geometry. Our analytic geometry solution might inspire a purely geometric solution. At the point
where we were finding the coordinates for A, we suggested drawing an altitude from A to B C . Suppose
we do that right from the beginning. (We might also be inspired to do this by the 45° angle; drawing this
altitude builds a 45-45-90 triangle). From 45-45-90 triangle ABX, we have
A X = X B = A B /y /2 = 4>/2. Therefore, wehave C X = C B - B X = 8 - 4>/2. We can then

apply the Pythagorean Theorem to A A X C to find A C = 8 \ / 2 - \p 2 , as before. □

Concept: Solving a problem in two ways is a good way to check your answer

Our two solutions in the last problem also show that analytic geometry solutions and purely geometric solutions to the same
problem are often closely related to each other.

Exercises

a 1 7 ‫־‬1 0 ‫ ׳‬We USGd S,° PeS t 0 determine that A A I3 C is a ri9ht triangle. How else could we have determined that
A A B C is a right triangle?

Type your solution, notes and/or work here. Show Solution

Find the area of the triangle bounded by the graphs of the equations y = #, y = —x, and y = 6.

Type your solution, notes and/or work here. Show Solution

What is the shortest possible distance between a point on the graph of x 2 + r/2 + 6 x - 8V = 0 and a point on the graph of
x - 14a• + y 2 + 10,v + 65 = 0?

Type your solution, notes and/or work here. Show Solution

In A A B C , we have A B = 8, B C = 12, and A A B C = 30°. Find A C

Type your solution, notes and/or work here. Show Solution

Find the radius of each circle that passes through (9 ,2 ) and is tangent to both the #-axis and the ;(/‫־‬axis.
Hint

Hint

Type your solution, notes and/or work here. Show Solution


17.4 Proofs with Analytic Geometry
In this section, we learn how to prove geometric facts using analytic geometry.

Problems
Problem 17.14 4 Jump to Solution

In this problem, we prove that the midpoint of the hypotenuse of a right triangle is the circumcenter of the triangle.

(a) Let the right triangle be A A B C , with right angle at C. Why is it useful to choose C to be the origin of the Cartesian
plane?

(b) Suppose C is the origin. Why should we choose the axes of the Cartesian plane such that the legs of A A B C are along
the axes?

(c) Let A be (a, 0) and B be (0. 6 ). What is the midpoint of the hypotenuse?

(d) Show that the midpoint of the hypotenuse is equidistant from the vertices of the triangle.

Problem 17.15 4 Jump to Solution

In this problem, we prove that the median of a trapezoid is parallel to the bases of the
trapezoid, and that the length of the median is the average of the lengths of the bases. Let
the trapezoid be W X Y Z , where W is the point (0.0), X is the point (2a, 0), Y is the
point (26, 2d), Z is the point (2c, 2d), and we have a > 0, 6 > c, and d > 0.

(a) Why is W X Y Z a trapezoid? Which two sides are parallel?

(b) Find the coordinates of the midpoints of the legs of W X Y Z. Why did we use 2a, 26,
2 c, and 2 d to describe the coordinates of the vertices of W X Y Z , instead of using
just a, 6 , c, and d?

(c) Show that the median of W X Y Z is parallel to the bases of W X Y Z , and equal in length to the average of the lengths of
the bases.

(d) Explain why our proof is valid by describing why for every trapezoid there is some choice of origin and axes such that the
vertices of the trapezoid can be described with the coordinates ( 0 , 0 ), ( 2 a, 0 ), (26, 2 d), and ( 2 c, 2 d).

Problem 17.16 4 Jump to Solution

In this problem we prove that the medians of a triangle are concurrent. Let our triangle be A B C , where A = (0.0),
B = (2a, 0), and C = (2 6 ,2c).

(a) Find the coordinates of the midpoint of A B . Find an equation whose graph is the line containing the median from C to
‫־‬A B .

(b) Find an equation of the line whose graph is the line containing the median from B to AC .

(c) Find the intersection of the lines you found in parts (a) and (b).

(d) Prove that the intersection point you found in part (c) is on the median from A to B C .

Problem 17.17 4 Jump to Solution

Suppose line k divides rectangle A B C D into two pieces of equal area. Prove that line k passes through the intersection of the
diagonals of A B C D .

Problem 17.14 t V

Use analytic geometry to prove that the midpoint of the hypotenuse of any right triangle is the circumcenter of the triangle.

Solution for Problem 17.14: Our first step is to set up the problem on the Cartesian plane. We must be very careful when we do
so. See if you can figure out why the set-up in the Bogus Solution below is not sufficient:

Bogus Solution: We let our triangle be A B C , and we let C = (0 ,0 ), A = (4 .0 ), and B = (0 ,6 ).


.. Legs A C and B C are along the axes, so they are perpendicular. Therefore, A A B C
y is a right triangle with hypotenuse A B . The midpoint of A B is (2,3). Let this
midpoint be M . We have

A M = \ J ( 2 - 4 ) 2 + (3 - 0 ) 2 = V l3 ,

D M = yj{2 - 0 )2 + (3 - 6 )2 = ^ 1 3 ,

C M = yj{2 - 0 )2 + (3 - 0 )2 = V / 13.

Since M is equidistant from the vertices of A A B C , point M is the circumcenter of


A ABC.

This does indeed show why the midpoint of the hypotenuse of this particular right triangle is the circumcenter of this right
j triangle. However, what about other right triangles with different side lengths? Our 'proof doesn't cover every possible right
triangle.

WARNING!! When setting up a proof on the Cartesian plane, we must be very careful. Our proof
must address all possible configurations of the problem, so our analytic geometry
O representation of the problem must cover all possible configurations.

We might start by letting .4 = (a, 6 ), B = (c, d), and C = (e, / ) . But that's six variables! Hopefully we can find a simpler
representation of A A B C . When setting up a geometry problem on the Cartesian plane, we can start with a diagram, then add
the coordinate axes. We try to do so in a way that simplifies our problem. We can choose any point to be the origin, so we choose
one of the vertices of the right triangle to be our origin.

We choose the vertex of the right angle, point C, to be the origin because this allows us to choose our axes so that the legs of
the right triangle are along the axes. So, we let .4 = (a, 0) and B = ( 0 , 6 ). Notice that a and/or 6 could be positive or negative.
For example, in the case below, both a and 6 are negative.

Since C is the vertex of the right angle, side A B is the hypotenuse. Let M be the midpoint of A B , so that

To show that M is the circumcenter of A A B C , we must show that M is equidistant from all three vertices. This is a job for the
distance formula:

‫־״״‬/(I-‫■’)״‬ +G -*)'-'F¥•
“1-■(0-!)/■‫־׳‬-G -‫־')״‬
We have A M = B M = C M , so M is the circumcenter of A A B C . Notice that every step in our proof is valid even if a or 6 or
both are negative. □

Important: When setting up a geometry problem on the Cartesian plane, choose your origin and
your coordinate axes wisely. Typically, we do so in a way that makes the coordinates
Z of important points in the problem as simple as possible.

Notice that our proof in Problem 17.14 only applies to right triangles, not to all triangles. The points we chose to represent the
vertices, (0,0), (a, 0), and ( 0 , 6 ), are always the vertices of a right triangle (when a and 6 are nonzero). We cannot use these
three points as our vertices to prove a fact about all triangles, since any triangle with vertices ( 0 , 0 ), (a, 0 ), and ( 0 , 6 ) must be a
right triangle.

Problem 17.15 t V

Use analytic geometry to prove that the median of a trapezoid is parallel to the bases of the trapezoid, and that the length of the
median is the average of the lengths of the bases.

Solution for Problem 17.15: In Section 8.2 here, we saw geometric proofs of these facts. Here, we try to find an analytic geometry
proof. We might start by letting the vertices of our trapezoid be (a, 6 ), (c, d), (e, / ) , and (g, h). Yikes. Eight variables! Maybe we
can do better. As noted in Problem 17.14, we can start with a diagram, then add the coordinate axes to set up a proof on the
Cartesian plane. Here, we choose the x-axis to contain one of the bases such that the other base is above the x-axis. We then
choose the leftmost of the two vertices on the :r-axis to be the origin. So, one vertex is the origin and the other is (a, 0) with
a > 0. Well call the origin W and the second vertex on the ;r-axis X , as shown below.

У
Z Y

We placed the ;r-axis such that the endpoints of one base are on the :r-axis because this forces the endpoints of the other base
to have the same ;*/-coordinate, since the bases are parallel. By choosing the ‘lower’ base to be :r-axis, we force this common y-
coordinate to be positive. Therefore, the other two vertices of the trapezoid can be represented by ( 6 , d) and (c, d), where d > 0
and 6 > c. Finally, we let Y be ( 6 , d) and Z be (c, d), so X Y and W Z are the legs of the trapezoid. Now, we have represented
our trapezoid with only 4 variables, instead of 8 .

Notice that every trapezoid can be represented in this way. Because 6 and c can take on negative values, letting the vertices be
W = (0.0), X = (a, 0), Y = ( 6 , d), and Z = (c, d) with a > 0, d > 0, and 6 > c will also represent trapezoids in which Z
and/or Y end up to the left of the y-axis. Two examples are shown below. In each case, we have W at the origin, X to the right
of W on the :r-axis, and Y X and Z W as the legs (because 6 > c).

У У

But what about trapezoids in which the bases are not horizontal?

There's no reason we have to make our :r-axis and y-axis horizontal and vertical! They only have to be perpendicular. For
example, consider the trapezoid with 'slanted‘ bases below. We similarly make our axes 'slanted‘ so that the :r-axis includes one
base.

Now, we’re ready for our proof. We'll use the diagram in which Y Z is horizontal and both Y У
and Z are to the right of the */-axis, as shown at right. (However, our proof will be valid for all
------------- c ------------
set-ups we have discussed.) The midpoint of leg W Z is ( - , - ) and the midpoint of leg X Y
A A
a -F 6 d d
is ( —- — , - ) . The median connects these two points, so the median is on the line y = - .
A A A

Therefore, the median is horizontal, so it is parallel to the bases. We can also use the
coordinates of the midpoints of the legs to determine that the length of the median is

a 4- 6 c a + 6 —c
~2 2 = 2 ’

The lengths of the bases are W X = a and Y Z = 6 — c, so the average of the lengths of the bases is also (a + 6 — c ) / 2. So,
the length of the median equals the average of the lengths of the bases. Notice that every step of this proof is valid for all the
other arrangements we showed. We also could have avoided the fractions in our solution by being a little clever about assigning
variables to coordinates. Because we are going to have to work with midpoints to get information about the median, we know
well have to divide expressions by 2 . Therefore, we might make our coordinates W = (0.0), X = (2a, 0), Y = (26, 2d), and
Z = (2c, 2d). Then, the endpoints of the median are (a 4- 6 , d) and (c, d). No fractions! See if you can finish the problem from
here. □

Concept: When we set up a geometry problem involving midpoints on the Cartesian plane, we
I often use 2a, 26, 2c, etc. for coordinates, rather than just a, 6 , c, etc. This helps us
' avoid expressions involving fractions when we find midpoints of segments in the
problem.

Speaking of midpoints, these are often a sign that an analytic geometry approach might work. This is because coordinates of
midpoints are particularly easy to find.

Problem 17.16 t V

Prove that the medians of any triangle are concurrent.

Solution for Problem 17.16: We must prove a statement about medians, so we're
working with midpoints. This might make us consider analytic geometry. First, we
have to set up the problem on the Cartesian plane. Let A A B C be our triangle, and
let the medians be A D , B E , and C F . We let A be the origin and choose our axes
so that B is on the :r-axis, as shown at right. Since we will have to find the
coordinates of the midpoints of the sides of the triangle, we let A be (0 ,0 ) and B be
(2a, 0). We have no restrictions on C except that it cannot be on the :r-axis, so we
let C be (26, 2c), where c ^ 0. Again, we use 26 and 2c because we will be finding
the midpoints of the sides of the triangle. Specifically, the midpoints of the sides of
A A B C are D = (a 4- 6 , c), E = ( 6 , c), and F = (a, 0).

Now that we have the coordinates of two points on each median, we can find the
linear equations whose graphs include the medians. We then must show that there
is a point (.‫׳‬r, y) that satisfies all three equations. First, well find the equation for .4/3
. The slope of .4/3 is c /(a 4- 6 ) and *4/3 passes through (0,0), so an equation
whose graph is .*4/3 is

y - 0 = ( x - 0 ).
a 4- 6

Notice that the right side of this equation is undefined if a 4- 6 = 0. Since the slope
of *4/3 is c /(a 4- 6), we only have a 4- 6 = 0 if *4/3 is a vertical line. Specifically, if
a 4- 6 = 0, then A and D are both on the y-axis.

WARNING!! When using analytic geometry for a proof, be extra careful not to overlook special
cases.
О

To take care of this special case, we multiply both sides of the equation by a 4- 6 to get

(a+ b)y = c (x — 0 ).
If *4/3 is vertical, then a 4- 6 = 0, and .*4/3 is the graph of x = 0. Our equation above becomes x = 0 when a 4- 6 = 0, so this
equation describes S 3 even if S 3 is a vertical line.

Notice that we use (0,0)


when writing a point-slope form of the equation of instead of using 4- 6 This makes our *4/3 (a ,c).
equation considerably simpler, and is one of the reasons we like to make as many of our coordinates 0 as possible.

Concept: Don’t make analytic geometry proofs harder than they need to be!

Similarly, the slope of b b is c /(b — 2a) and h l i passes through (2a, 0), so an equation whose graph is b i t is

У - 0 = ( x - 2 a).
6 — 2a
Multiplying both sides by 6 — 2a gives

(6 - 2 a )y = c (x — 2 a).

If 6 = 2a, then B E is the graph of x 2a, since B E must pass through (2a, 0) and ( 6 , c). If 6 = 2a, then
(6 — 2 a )y = c (x — 2 a) becomes 0 — c(x 2a), which simplifies to x = 2a, So, the equation ( 6 — 2a )y = c (x — 2a)
describes B E even if B E is a vertical line.

And finally, the slope of fab is 2c/(26 — a) and fab passes through (a, 0), so for this line we have

2 c
У- 0 =
26 — a
(x- a).

Again, we multiply both sides by the denominator of the slope to get

(26 — a )y = 2 c (x - a),

which describes fab even if fab is a vertical line.

We now have the three equations

*4/ 3: ! / ( * + &) = * c‫׳‬


2 /(6 - 2 a) = x c - 2ac,

fcfi■ у(2b - a) = 2xc - 2ac,

We can find the coordinates of the intersection of the graphs of the first two equations by substituting x c = y(a 4- 6 ) into
y(b — 2a) = x c — 2ac. This gives us

2 /( 6 - 2 a) = y (a + 6) - 2 ac.

Subtracting y (a 4■ 6 ) from both sides gives us y(b — 2a) — y (a 4■ 6 ) = —2ac. Simplifying the left side then gives
—3ay = —2ac. We know that a ^ 0, so we can divide by —3a to find y = 2c/3. Since we also have x c = y(a 4- 6 ), we find
x = y(a 4- 6 ) / c = 2(a. 4- 6)/3.

/ 2(a 4- 6 ) 2c.\
So, we know that A D and B E meet at All we have left is to see if this point is on C E . We do so by
V 3 ’ 3 )
substituting x = 2(a + 6 )/3 and y = 2 c/3 into our equation for fab from above. Substituting these expressions into
2 / ( 2 6 — a) = 2 x c — 2 ac gives us a left side of

, _! 4 2c 46c 2ac
2 /( 2 6 - a) = y (26 - a) = —-------- — ,

and a right side of

2(a 4- 6 )
2 x c — 2 ac = 2 c — 2ac

4ac 46c
i + r k
46c 2ac
3 ‘

, . x / 2 ( a + 6) 2c \
We see that the two sides are the same for the ordered pair (x , y) = I ----- ------. — J. so this point is on our third median, as

well! Therefore, we can conclude that the medians of a triangle are concurrent. □

Problem 17.17 t V

Suppose line k divides rectangle A B C D into two pieces of equal area. Prove that line k passes through the intersection of the
diagonals of A B C D .

Solution for Problem 17.17: It's quite easy to describe a general rectangle with coordinates. We let A
be the origin and choose the axes so that two sides are along the axes. Because A B C D is a
rectangle, it is also a parallelogram, so its diagonals bisect each other. This means we want to prove
that к passes through the midpoint of A C . Therefore, we let В = (26,0) and D = (0, 2d) (instead
of using В = (6 ,0 ) and D = (0, d)). Point C is on the vertical line through В and on the horizontal
line through D, so C = (2 6 ,2d). This means the midpoint of A C is (6, d).

Having dealt with A B C D , it's time to take care of k. First, we have to think about ways in which k
could intersect the rectangle. If k intersects two consecutive sides (excluding the vertices of the
rectangle), then the area of the triangle it forms is smaller than the area of the triangle formed by
drawing a diagonal. For example, in the diagram at right, we have [.4 X K ] < [A B D ] = [A B C D ]/2
. So, it is impossible for such a line to bisect the area of A B C D .

Following similar logic, if k passes through a vertex of A B C D and bisects the area of A B C D , then line k must contain a
diagonal of A B C D . So, line k clearly passes through the intersection point of the diagonals in this case.

The other possibility is that k intersects opposite sides of the rectangle (excluding the
У
vertices). Shown at right is our set-up on the Cartesian plane. Line k intersects A B and
C D at P = (/>, 0) and Q = (ry, 2d) as shown. If k bisects the area of A B C D , then it
splits it into two trapezoids with equal area. The bases of trapezoid A P Q D have lengths
p and q, and A P Q D has height 2d, so we have

\A P Q D \ = (2d) ( E J i ) =«*(P + «)•

The bases of P B C Q have lengths 26 — p and 26 — q, so we have

26 — p 4 26 — q
[P B C Q ] = (2d) (■ d(4b - p - q).
)-
Setting these equal gives us d(p 4■ q) = </(46 — p — q). Dividing by d gives us p 4 q = 46 — p — q, so 2(p 4■ q) = 46.

Finally, we find that p 4- q = 26. (We also could have used the fact the [A P Q D ] = ~ [A B C D \x o show that p 4- q = 26.)
A

But how does this help? We want to show that P(} passes through the midpoint of AC. The midpoint of AC is (6, d). Now that
we know that p 4- q = 26, we see that the midpoint of P Q is

( Р+Ч 0 + 2 d\ _
( 2 ’ 2 J ‫( ־‬M )•

Therefore, line k passes through the intersection of the diagonals of A B C D . z

Exercises

1 7 .4 .1 : t V

In part (a) of Problem 17.4.2, we will use analytic geometry to prove a fact about parallelograms. Which of the following
groups of vertices can we use to represent the vertices of the parallelogram, to prove a fact for all parallelograms? If a group
cannot be used, describe a parallelogram whose vertices cannot be represented by the given points. You can assume all
variables represent nonzero numbers. For example, if we wished to prove a result about all triangles, we cannot use (0,0),
(a, 0), and (0,6) to represent the vertices of the triangle. This is because any triangle with these points as vertices is a right
triangle. So, these points cannot represent the vertices of any triangle that is not a right triangle.

(a) ( 0 ,0 ) , (o , 0), (6 ,6 ), (6 , c )

Type your solution, notes and/or work here. Show Solution

(b) (a , 0), (0 ,6 ), ( - a , 0), ( 0 , - 6 )

Type your solution, notes and/or work here. Show Solution

(c) ( 0 ,0 ) , (6 ,6 ), (6 + a , 6), (a , 0 )

Type your solution, notes and/or work here. Show Solution

(d) ( 0 , 0),(6 - a , c), (6 , c ) (a , 0 )

Type your solution, notes and/or work here. Show Solution

1 7 .4 .2 : t V

(a) Use analytic geometry to prove that the diagonals of a parallelogram bisect each other.

Type your solution, notes and/or work here. Show Solution

(b) ★ Use analytic geometry to prove that if the diagonals of a quadrilateral bisect each other, then the quadrilateral is a
parallelogram.

Type your solution, notes and/or work here. Show Solution

1 7 .4 .3 : t V

Use analytic geometry to show that if the diagonals of a quadrilateral are perpendicular and bisect each other, then the
quadrilateral is a rhombus.

Type your solution, notes and/or work here. Show Solution

1 7 .4 .4 ★ : t V

Notice that in Problem 17.16, the sum of the equations we found for *4/3 and is the equation for C
ЬРF . Why could we have
used this observation to deduce that *4/3, b b , and fab are concurrent?

Type your solution, notes and/or work here. Show Solution


17.5 Distance Between a Point and a Line
The distance between a point and a line is the shortest distance from the point to a point on the line.

Problems
Problem 17.18 4 Jump to Solution

Find the distance between the point (2. 7) and the graph of the line x — 3y = 1.

Problem 17.19 4 Jump to Solution

In this problem, we find a formula for the distance between the point (x’o,t/o) and the line
y
A x 4 B y 4 C = 0, where A, B, C, x ‫״‬, and y0 are all constants. We'll find a formula assuming
that A B ^ 0; you'll be asked to show that this formula also works when A B = 0 as an
Exercise.

Let k be the line, P be the point (xo, yo), and Q be the foot of the altitude from P to k. Let R
be the point on k that has ^-coordinate x 0.

For the following parts, assume P is above line k on the Cartesian plane, and that k has a
positive slope. (The proofs for all other possibilities are essentially the same.)

(a) Explain why Q R /P Q = —A /B .

(b) Explain why P R = ( A x o 4‫ ־‬Byo 4‫ ־‬C ) /B . (Hint: Remember that R is on k, and has the same ^-coordinate as P. By
how much do we have to increase the ;(/-coordinate of R to get the ;«/-coordinate of P?)

IA xq 4‫ ־‬B y 0 4‫ ־‬C\
(c) Show that the distance from point P to line k is ------- . -----.
s/A * 4 B 1

Problem 17.20 4 Jump to Solution

Use your formula from Problem 17.19 to check your answer to Problem 17.18.

Problem 17.21 4 Jump to Solution

How many ordered pairs (x , y) satisfy both 14x — 4 % 4 0 = 49 ‫־‬and x 2 4‫ ־‬y 2 = 1?

We start by finding the distance between a specific point and a specific line.

Solution for Problem 17.18: Let point P be (2, 7) and let line k be the graph of the line x — 3y = 1.
Let Q be the point on k that is closest to P, so we have h (§ J_ k. Therefore, the product of the
slopes of these lines is —1. Writing x — 3y = 1 in slope-intercept form gives y = x / 3 — 1/3, so
the slope of k is 1/3. Therefore, the slope of P Q is —3. Since Q is on the line through (2. 7) with
slope —3, it is on the graph of y — 7 = —3 (x — 2). Rearranging this equation gives 3 x 4‫ ־‬y = 13.

We now have two equations whose graphs pass through point Q, so we find the coordinates of Q by
solving the system of equations

x - 3y = 1 ,
3x + y = 13.

Solving this system of equations gives (x , y) = (4,1), so point Q is (4,1). As a quick check, we note that the slope of P Q is
(1 — 7 ) /( 4 — 2) = —3, so P Q is indeed perpendicular to k. (If this slope had not come out to —3, then we would have known
we made a mistake.)

Concept: Checking intermediate steps while working on problems will help you catch a lot of
i errors.

Now, we use the distance formula to find

PQ = (2 - 4 ) 2 + (7 - l )2 = v 3 + 4‫ ׳‬C = 2VTÖ.

We could follow the procedure we used in Problem 17.18 to find the distance between any point and any line. However, the
procedure requires a lot of steps, and often the numbers will get pretty ugly. Instead, let’s see if we can find a formula to use to
find the distance between a point (xo, yo) and the line that is the graph A x 4 ‫ ־‬B y + C = 0.

Problem 17.19 t V

Find a formula for the distance between the point (xo, yo) and the line Ax 4 B y + C = 0, where A, B, C, x‫״־‬, and yo are all
constants.

Solution for Problem 17.19: Let P be the point (x’o, yo) and k be the graph of A x 4 ‫ ־‬B y 4 C = 0. We could proceed as we did
in our solution to Problem 17.18, by finding the slope of k, then finding the equation of the line through P that is perpendicular to
k. Then, we find the intersection of k and this new line, and then...

That looks like a lot of work. Before diving into pages of algebra, let’s see if we can use some geometric insights to simplify the
problem.

Concept: Algebra is not the only tool we have to solve problems about the Cartesian plane.
I Combining geometric insights with algebra can lead to very nice solutions.

We start by drawing a diagram with P, k, and the perpendicular segment from P to k. Let point Q
be the foot of the perpendicular from P to k. We seek a length, and we have a right angle, so we
y
think about building right triangles. We build a right triangle by drawing a vertical segment from P
to line k, meeting k at point R, as shown.

Since P and R are on the same vertical line, the x-coordinate of R is x ‫״‬. Since R is on k, its
coordinates satisfy the equation A x 4 ‫ ־‬B y 4 C = 0. We can now use this equation to find the y
-coordinate of R. Let y R be the ;«/-coordinate of R, so R is (xo, y R). Because R is on k, we must
have

A xq 4 Bypi 4 C = 0.

Solving for y n gives us

—A x 0 — C
Vr
‫־‬I T

It's still not clear how we can find the coordinates of Q. Right triangles make us think of similar
y
triangles. Drawing the altitude Q S to the hypotenuse of right triangle A P Q R gives us plenty of
similar triangles.

But what good are they? We haven't used any information about the equation of our line yet, so
we focus on that. When we do so, we see that we can relate lengths of segments in our diagram
to the slope of k. Specifically, we see that the slope of line A: equals S R /Q S . (In our diagram, we
assume the slope of k is positive. Essentially the same approach will work if the slope of k is
negative.) We can also write the slope of k in terms of A and B. Putting the equation
A x 4 B y 4 C = 0 in slope-intercept form gives

A C
y = - B x - n ’

so the slope of k is —A /B . Therefore, we have

SR _ A
QS ‫־‬ B'

Our similar triangles give us a way to relate A Q S R to A P Q R . We have A Q S R ~ A P Q R , so

SR QR
QS ‫־‬ PQ‫׳‬

Combining this with S R /Q S = —A / B gives us Q R /P Q = —A / B . Rearranging this gives P Q = ( —B /A )( Q R ) .

Unfortunately, it’s not so clear how to find Q R . But we can find P R . Points P and R have the same x-coordinate, so P R equals
the difference in the ;«/-coordinates of P and R:

—A x 0 — C A x 0 4 Byo 4 C
PR = yo~yR= yo-------------h --------= n
--------------- ------------ •

Now, we're close. We have P R , and we can relate P Q to Q R . Finally, we use the Pythagorean Theorem to finish. We have
P R 2 = P Q 2 4 Q R 2 and Q R = - ( A / B ) P Q , so we have

/ ,4x 0 4 Byo 4 C

Multiplying both sides by B 2 gives

( A x 0 4 B y 0 4 C )2 = B 2 • P Q 2 4 A 2 • P Q 2 = ( A 2 4 B 2 ) ( P Q 2).

Dividing both sides by A 2 4 B 2 then taking the square root of both sides, gives

p n _ \A x Q4 B y 0 4 C\
\J A 2 4 B 2

We need the absolute value on the right side because length must be positive.

Our proof does not address the cases in which k is horizontal or vertical - you'll be asked to tackle these cases as an Exercise.
Moreover, our proof assumes that the slope of k is positive, and that P is above the line. However, the proofs are essentially the
same for other possible configurations in which k is neither horizontal nor vertical.

Also, our proof doesn't address the possibility that P is on k. We can quickly show that our formula works in this case. If
P = (xo, yo) is on the graph of A x 4 B y 4 C = 0, then we must have A x q 4 B yo 4 C = 0. When we substitute
A xo 4 B yo 4 C = 0 in our formula, then our formula gives us a distance of 0 , which is indeed the correct distance between
point P and line k when P is on k. □

Important: The distance between the point (xo, yo) and the graph of the equation
A x 4 B y 4 C = 0 is
Z
|i4 x 0 4 B y Q4 C\
V -4 2 + f i 2

Let's check our formula by using it to solve Problem 17.18.

Important: Whenever you derive a formula, you should test it on a specific case.

Z
Problem 17.20 t V

Use your formula from Problem 17.19 to check your answer to Problem 17.18 by using the formula to find the distance between
(2, 7) and the graph of x — 3y = 1.

Solution for Problem 17.20: See if you can figure out where we go wrong here:

Bogus Solution: The distance between the point (2. 7) and the graph of the equation x — 3y = 1 is

y |2 — 3 ■7 + 1| 18 18 yTO 9%/iO
n/ i 2 + 32 _ _ 5 ‫־־‬

Our answer in the Bogus Solution doesn’t match our answer to Problem 17.18. Is our formula wrong? No! We applied it
incorrectly.

WARNING!! In our formula for the distance between a point and a line, the linear equation is in the
q form A x 4 B y 4 C = 0, not A x 4 B y = C.

So, we write our linear equation as x — 3y — 1 = 0. The distance between (2. 7) and the graph of this equation is

1 2 - 3 - 7 - 1| 20_____ 20_ V lO f -
v /F + 3 5 ‫־‬ v ffi “ > /I 6 ■ 7 1 6 ’
which matches our original answer. □

Our formula can also be used to solve less straightforward problems.

Solution for Problem 17.21: We could solve the equation 14x — 4 % 4 49 = 0 for x in terms of ;</ and substitute the result into
x 2 4 y 2 = 1, but that will lead to some pretty ugly algebra. Before we do so, we think about analytic geometry, because the
equation x 2 4 y 2 = 1 is in the standard form of an equation whose graph is a circle. We like circles.

Concept: If an algebra problem has an equation whose graph is a circle, think about using
I analytic geometry.

The graph of the equation x 2 4 y 2 = 1 is a circle with radius 1. The graph of the equation
14x — 4 % 4 49 = 0 is a line, but does it intersect the circle? The graphs of both equations are shown
at right. Unfortunately, we can’t quite tell if the line hits the circle once or twice. And our graph might be
off by just a little bit, so we can't even be sure if the line hits the circle at all! Each point on the circle is 1
unit from the origin. So, if the line passes within 1 unit of the origin, it must intersect the circle twice,
since it must go inside the circle. The distance between (0 .0 ) and the graph of 14x — 4Sy 4 49 = 0
is

114-0-48• 0 4 49[ _ 49
_ 7 14 2 + ( —48)2 ‫ ־־‬V 1 4 2 + ( - I F ‫׳‬

Rather than multiplying out the squares in the denominator, we remember the Pythagorean triple {1 4 .4 8 ,5 0 }, so we know
142 4 ( —48)2 = 502. Therefore, the distance between the line and the origin is 49/50. So, the line goes inside the circle, which
means it intersects the circle twice. This tells us that there are two ordered pairs that satisfy both of the given equations. □

Exercises

1 7 .5 .1 : t V

Find the distance between (3 ,4 ) and the graph of the equation 4 x — 3y 4 7 = 0.

Type your solution, notes and/or work here. Show Solution

1 7 .5 .2 : t V

Find the coordinates of the points on the graph of y = 10 that are a distance of 5 from the graph of x 4 3y = 9

Type your solution, notes and/or work here. Show Solution

1 7 .5 .3 : t V

Our proof of the formula for the distance between a point and a line does not address the cases in which the line is either
horizontal or vertical. Complete the proof of the formula by showing that it holds if the line is horizontal or vertical.

Type your solution, notes and/or work here. Show Solution

1 7 .5 .4 : t V

Find the distance between the graphs of 3 x = 4y 4 8 and 3x = Ay 4 17.

Type your solution, notes and/or work here. Show Solution

1 7 .5 .5 : t V

At how many points does the graph of 2x — 3y = 48 intersect the graph of (x — 3 ) 2 4 (y 4 2 ) 2 = 100?

Type your solution, notes and/or work here. Show Solution


17.6 Advanced Analytic Geometry Problems
Problems
Problem 17.22
4 J u m p t o S o lu t io n

In t h is p r o b le m , w e f in d t h e a r e a o f t h e r e g io n b o u n d e d b y t h e lin e s 2 ,t: + 3y = 21 and 5 * + 2 2 5 = /‫־‬


axes. a n d t h e c o o r d in a te

(a ) G r a p h t h e t w o e q u a t io n s . D o w e h a v e a s im p le f o r m u la t o f in d t h e a r e a o f t h e r e g io n ?

(b ) D is s e c t t h e r e g io n in t o p ie c e s y o u k n o w h o w t o h a n d le , a n d u s e t h e s e p ie c e s t o s o lv e t h e p r o b le m .

Problem 17.23
4 J u m p t o S o lu t io n

(a ) W h a t p o in t is t h e im a g e o f r o t a t in g t h e p o in t ( - 1 , 3 ) a n a n g le o f 1 8 0 ° a b o u t ( - 6 , - 7 ) ?

(b ) W h a t p o in t is t h e im a g e o f r o t a t in g t h e p o in t ( 4 , 5 ) a n a n g le o f 9 0 ° c lo c k w is e a b o u t t h e p o in t ( - 4 , 2 ) ?

(c ) W h a t p o in t is t h e im a g e o f r e f le c t in g t h e p o in t ( 5 . - 3 ) o v e r t h e lin e y = 8?

(d ) W h a t p o in t is t h e im a g e o f r e f le c t in g t h e p o in t ( 3 , - 1 ) o v e r t h e g r a p h o f t h e lin e 2 * - y + 5 = 0?

Problem 17.24
S ource: AMC 7 2 4 J u m p t o S o lu t io n

^ D E ^ e aT 2 7 ( > ‫ ״‬a n d m ) Z r e s p e c ,iv e l* w i, h 8 = ( ° ‫ ׳‬° ) • C = (2 2 3 .0 ), D = (6 8 0 .3 8 0 ), a n d


k ( 6 8 9 , 3 8 9 ) . In t h is p r o b le m , w e f in d t h e s u m o f a ll p o s s ib le * - c o o r d in a t e s o f A .

(a ) W h a t d o e s th e in fo r m a tio n a b o u t th e a re a o f A A B C te ll u s a b o u t th e p o s s ib le lo c a tio n s o f p o in t .4 ?

(b ) W h a t d o e s t h e in f o r m a t io n a b o u t t h e a re a o f A A D E t e ll u s a b o u t t h e p o s s ib le lo c a tio n s o f p o in t .4 ?

(c ) C o m b in e y o u r o b s e r v a t io n s f r o m t h e f ir s t t w o p a r t s . H o w m a n y p o s s ib le p o in t s . 4 a r e t h e r e ?

(d ) W h a t f ig u r e is f o r m e d w h e n y o u c o n n e c t a ll t h e p o s s ib le p o i n t s . 4 y o u f o u n d in p a r t ( c ) ?

^ th a t A c o T b T ^ ‫״‬ P a r t W ) t 0 f ' n d t h e S U m ° f a " P ° S S i b l e •Z:‫ ־‬C ° ° r d i n a , e S o f * “ actually furling any o f t h e points

Problem 17.25
4 J u m p t o S o lu t io n

w e f!n d V Z A V B ° f A X Y Z 'd 'e P e r p e n d ' c u l a r l n A X V Z ■w e h a v e b o t h / W Z = 90° and X K = 8 In t h is p r o b le m

(3 ) 7 P r° b le m U p o n t h e C a r t e s ia n p la n e . C h o o s e y o u r o r ig in w is e ly , t h e n f in d t h e c o o r d in a t e s o f X Y Z A and B If
y o u c h o o s e y o u r o r ig in w is e ly , y o u s h o u ld o n ly n e e d o n e v a r ia b le . '

(b ) U s e t h e in f o r m a t io n in t h e p r o b le m t o f in d t h e v a lu e o f t h e v a r ia b le .

(c ) F in d Y Z .

Problem 17.26
Source: AMC 7 2 4 j u m p t o S o lu t io n

r s P ' a ? 8 8 an 6 ‫ ! ״‬l S , T a r e 0 f S l d e 1 ■ L e t t h e v e r t i c e s ° f t h e s q u a r e • , a k e n c o u n t e r c l o c k w i s e , * A B C
a n d ‫ ״‬. A l s o , l e t t h e d i s t a n c e s f r o m P t o A . B . a n d C , r e s p e c t i v e l y , b e u . v, a n d ‫ ״‬, W h a t i s t h e g r e a t e s t d i s t a n c e t h a t P c a n b e
f r o m ‫ ״‬i f ‫ ־‬r + v 2 = w 2?

Problem 17.22

F in d t h e a r e a o f t h e r e g io n b o u n d e d b y t h e lin e s 2 x + 3 y = 21 and 5 * + 2 y = 2 5 a n d t h e c o o r d in a te

Solution for Problem 17.22: W e s t a r t b y g r a p h i n g t h e t in


a n d s h a d in g t h e b o u n d e d r e g io n a s s h o w n b e lo w .

Im p o rta n t: J u s t a s w it h m o s t g e o m e tr y p r o b le m s , o u r f ir s t s te p w it h m a n y a n a ly tic g e o m e tr y
^ p r o b le m s is d r a w in g a d ia g r a m .

f 3 q U a ,d r ‫ ״‬a * e r a l ' b u t f 5 n o t o n e o f o u r 5 P e d a l t y p e s o f q u a d r i l a t e r a l . S o , i t ' s n o t i m m e d i a t e l y


o b v io u s h o w t o f in d t h e a r e a o f t h e s h a d e d r e g io n . T h e r e fo r e , w e u s e a t a c t ic f r o m o u r s t u d y o f f u n k y
a r e a s : w e d iv id e t h e s h a d e d r e g io n in t o p ie c e s w e k n o w h o w t o h a n d le . W e d r a w d ia g o n a l P R w h ic h
c u t s t h e s h a d e d r e g io n in t o t w o t r ia n g le s . W e c o u ld a ls o h a v e c u t t h e r e g io n in t o t w o t r ia n g le s b y
d r a w in g Q S . b u t w e c a n e a s ily f in d t h e a r e a s o f A P Q R a n d A P S R . T h e g ra p h o f 5 * + 2y = 25
( lin e k in t h e d ia g r a m ) in t e r s e c t s t h e * - a x i s a t ( 5 , 0 ) a n d i t in t e r s e c t s t h e g r a p h o f 2 x + 3y = 2 1 ( lin e

!\ / QwR ll- r ln / ? (39r!oSo P? r h and the a'1i,Ude fr°m R 10


] _ ( 5 ) ( o ) / 2 = 2 o / 2 . S im ila r ly , t h e g r a p h o f 2 x + 3 y = 2 1 m e e t s
iS5' This qive s us
t h e ‫־‬/ - a x i s a t ( 0
7 ) so
/ 5 = 7 . T h e a ltitu d e fr o m R t o P S h a s le n g th 3 , s o [ P S / ? ] = (3 )(7 )/2 = 2 1 / 2 . F in a lly , w e h a v e

[P Q R S ] = [P Q R \ + [ P S R ] = f H = 23.

‫״ ״ “ ״ ״ ־ ״ ״‬ b> * “ ‫ «" ״ ־‬P « S S " * ‫״ ״ * ״ ״ * ־‬ ‫־ ״‬ • » * S ‫ ״ • ־ ״ ־ » « ־ ־‬s o lv e , ‫« ״‬

L e t 's t r y s o m e t r a n s f o r m a t i o n s o n t h e C a r t e s ia n p la n e .

Problem 17.23

(a ) W h a t p o in t is t h e im a g e o f r o t a t in g t h e p o in t ( - 1 , 3 ) a n a n g le o f 1 8 0 ° a b o u t ( - 6 , - 7 ) ?

(b ) W h a t p o in t is t h e im a g e o f r o t a t in g t h e p o in t ( 4 . 5 ) a n a n g le o f 9 0 ° c lo c k w is e a b o u t t h e p o in t ( - 4 , 2 ) ?

(c ) W h a t p o in t is t h e im a g e o f r e f le c t in g t h e p o in t ( 5 . - 3 ) o v e r t h e lin e y = 8?

(d ) W h a t p o in t is t h e im a g e o f r e f le c t in g t h e p o in t ( 3 . - ! ) o v e r t h e lin e 2 x - y + 5 = 0?

Solution for Problem 7 7.23:

(a ) Let Z b e ( - 1 3 ) . le t Y b e ( - 6 - 7 ) , a n d le t Z b e t h e im a g e o f X u p o n a 1 8 0 “ r o ta tio n a b o u t Y B e c a u s e t h e a n g le o f
o t a t io n is 1 8 0 . w e h a v e Z X Y Z = 1 8 0 » S o , p o i m K is o n M F u rth e rm o re , b e c a u s e Z is t h e im a g e o f X upon m t ln
t ) , w e have X Y — Y Z . S in c e Y is a ls o o n Z Z , w e k n o w t h a t Y is t h e m id p o in t o f Z Z . S o , i f Z is ( a . b) w e h a v e

( ^ ^ ) = ‫״‬ - 7 1

F r o m t h is , w e f in d a = -1 1 and b = - 1 7 , s o t h e im a g e o f X u p o n 1 8 0 ° r o t a t io n a b o u t Y is ( - 1 1 , - 1 7 ) .

W e a ls o c o u ld h a v e s o lv e d t h is p r o b le m b y n o t in g t h a t b e c a u s e Y is t h e m id p o in t o f X Z
w e k n o w t h a t th e tr a n s la tio n t h a t m a p s Z t o Y a ls o m a p s V ' t o Z . T o g e t f r o m X t o K w e Y
g o le f t 5 u n it s a n d d o w n 1 0 u n it s . S o . t o g e t f r o m V ‫ ־‬t o Z , w e a ls o g o le f t 5 u n it s a n d d o w n
1 0 u n its ,to ( - 6 - 5 , - 7 - 1 0 ) = (-1 1 ,-1 7 ).

(b ) L e t ( 4 , 5 ) b e p o in t . 4 a n d ( - 4 , 2 ) b e p o in t B a s s h o w n in t h e d ia g r a m a t r ig h t . I f p o in t C is
th e im a g e o f ,4 u p o n a 9 0 c lo c k w is e r o t a t io n a b o u t B . t h e n w e m u s t h a v e A B = C B a n d
AB Y CB T h e s l o p e o f A B i s 3 / 8 , s o w e k n o w t h a t t h e s l o p e o f ‫־‬C B is - 8 / 3 . T h is
m e a n s t h a t C is o n t h e lin e t h r o u g h B w i t h s lo p e - 8 / 3 . B u t w h e r e o n t h i s lin e i s it ?

W e c a n f in d p o in t C b y n o tin g t h a t t o g e t f r o m .4 t o B , w e g o d o w n 3 u n its a n d le f t 8 u n its


S o . to S e t fro m p o in t B t o C , w e g o r ig h t 3 u n it s a n d d o w n 8 u n it s , w h ic h m e a n s C is

v e r tic a l i J , " 1 ’ ‫ * ? ־‬T " 8 iS ' " U S t r a t e d i n t h e f , g u r e a t r i g h t W e f i r s t b u i l d r i g h t t r i a n g l e A A B X s o t h a t A X i s


v e r t 'c a ! a n d B Z is h o r iz o n t a l. W h e n w e r o t a t e t h i s t r i a n g l e 9 0 » c lo c k w is e , w e g e t A B Y C . in w h ic h B Y = B X = 8
and
o r — AA — 3 . T h e r e fo r e , p o in t C is 8 u n it s b e lo w a n d 3 u n it s t o t h e r ig h t o f B .

(C ) im a T lT f? ’ " 2 iS 1 1 T b e l0 W t h e h 0 r iZ 0 n , a l lin e y = 8 ' 5 0 it s 'm a g e * 1 1 u n its d ir e c tly a b o v e y = 8 . T h e re fo re th e


h a t th e a a p h o f v ~ l i f , h e ' ° " T " ° f *8 = ‫ ׳‬is < 5 ■ 8 + 11) = ( 5 . 1 9 ) . W e c a n q u ic k ly c h e c k t h is b y n o t in g
( 5 1 9 ) is h e i m L e o f f 5 37 " 7 b ls e c t o r ^ w it h e n d p o in t s ( 5 , - 3 ) a n d ( 5 . 1 9 ) . T h is t e lls u s t h a t
( o , J. J ; i s t h e i m a g e o f ( 5 , — 3 ) u p o n r e f l e c t i o n o v e r t h e g r a p h o f y = 8.

(d ) L e t lin e k b e t h e g r a p h o f 2 a : - y + 5 = 0 . L e t P b e ( 3 , - 1 ) , a n d le t / ? b e it s im a g e
u p o n r e f l e c t i o n o v e r k. B e c a u s e / ? i s t h e i m a g e o f P u p o n r e f l e c t i o n o v e r k, P R a n d k У к

a r e p e r p e n d ic u la r . T h e s lo p e o f k is 2 , s o t h e s lo p e o f P R is - 1 / 2 . T h e r e f o r e , R is o n
t h e lin e t h r o u g h P w i t h s lo p e - 1 / 2 . A p o in t - s lo p e f o r m o f t h i s lin e 's e q u a t io n is

2‫׳‬ ‫ ־ = )!־(־‬2 (* ‫ ־‬3)•

B u t w h i c h p o i n t o n t h e g r a p h o f t h i s l i n e i s / ? ? I f w e c a n f i n d t h e p o i n t w h e r e ~PR m e e t s
k. w e c a n u s e t h i s p o i n t t o f i n d R , b e c a u s e t h i s i n t e r s e c t i o n p o i n t i s t h e m i d p o i n t o f ‫־‬P R

L e t t h e i n t e r s e c t i o n p o i n t o f P R a n d k b e Q . W e h a v e t h e e q u a t i o n s o f b o t h * p A a n d k.
s o w e c a n f in d Q b y s o lv in g t h e s y s te m o f e q u a tio n s

2a: - y + 5 = 0,

y + l = — i ( * - 3 ) .

A h d in g t h e s e t w o e q u a t io n s g iv e s 2 x + 6 = 4 - '1 S o l v i n g t h i s e q u a t i o n g i v e s * = - 9 / 5 . S u b s titu tin g t h is in to e ith e r

o f t h e a b o v e e q u a t io n s g iv e s y = 7 / 5 . S o , p o in t Q is ( - 9 / 5 , 7 / 5 ) .

N o w w e ' r e r e a d y t o f i n d R . Let t h e c o o r d i n a t e s o f R b e ( x R , y R). B e c a u s e Q i s , h e m i d p o i n t o f P R . w e m u s t h a v e

+ - 1 + y R\ _ / 9 7 \

V 2 2 ‫־‬ ) ~ V 5 ’ 5 /

f h e ^ o r t \ X R )1 r 2 7 _ 9 /5 ^ ( _ 1 + y R ) / 2 = 7 / 5 9 'v e s u s 2 3 . 3 / 5 , 1 9 / 5 ‫) ־ ( = ) * ״‬, W e c o u l d a l s o h a v e f o u n d
t h e c o o r d i n a t e s o f / 7 b y n o t i n g t h a t t o g e t f r o m P t o Q , w e g o 2 4 / 5 u n i t s t o t h e l e f t a n d 1 2 / 5 u n i t s u p S o t o g e t f r o m Q to
R. w e g o u n its 2 4 / 5 t o t h e le f t a n d 1 2 / 5 u n its u p f r o m ( - 9 / 5 , 7 / 5 ) t o ( - 3 3 / 5 , 1 9 / 5 ) .

Problem 17.24
Source: AMC 12 t V

T n a ^ g 'e S g T f i C a n c l . 4 ‫ ״‬£ h a v e a r e a s 2 0 0 7 a n d 7 0 0 2 . r e s p e c tiv e ly , w it h Z ? = (0 ,0 * C = (2 2 3 .0 ), D = (6 8 0 .3 8 0 ). a n d


t, ( b o J , o o J ) . F in d t h e s u m o f a ll p o s s ib le r - c o o r d in a t e s o f . 4 .

Solution for Problem 7 7.24 T h e h u g e n u m b e r s i n t h e p r o b l e m m a k e i t h a r d


t o d r a w o u r g r a p h t o s c a l e , s o w e s t a r t w i t h a r o u g h s k e t c h . W e 'l l h a v e t o u s e
t h e in f o r m a t io n a b o u t t r ia n g le s A B C a n d A D E t o d e te r m in e th e p o s s ib le
p o in t s A . W e s ta r t w it h t h e f a c t t h a t [A B C ] = 2 0 0 7 . B a s e ‫־‬B C o f th is
t r ia n g le h a s le n g t h 2 2 3 , s o i f w e le t /» ! b e t h e le n g t h o f t h e a l t it u d e t o s id e
B C o f A A B C , w e have

h = 2 [A B C ] = 2 (2 0 0 7 ) _
18.
BC 223 -

T h is t e lls u s t h a t A is 1 8 u n it s f r o m S in c e B C is a lo n g t h e r r - a x is , w e
now k n o w t h a t A is o n t h e g r a p h o f e ith e r у = 18 or у = - 1 8 . T r ia n g le
A B C g a v e u s s o m e i n f o r m a t i o n a b o u t . 4 ; le t 's t a k e a lo o k a t A A D E T h e

d is ta n c e f o r m u la t e lls u s t h a t D E = 9 v ^ , s o i f w e le t h 2 b e t h e le n g t h o f t h e a lt it u d e f r o m . 4 t o B E , w e h a v e

= 2 [A D E ] _ 2 (7 0 0 2 ) _ 2 (7 7 8 ) _

D E 9 ^ 2 ~ ~ W ~ W 2 •

X iS . 7 7 8 / 2 ™ *У ‫ ׳ ״‬e c o u ld f in d t h e e q u a t io n s o f t h e t w o lin e s t h a t a r e 7 7 8 s f r a w a y f r o m Ш b u t th a t
it m ig h t b e p r e t t y d if f ic u lt . I n s t e a d , w e g o b a c k t o o u r d ia g r a m a n d s e e i f w e c a n f in d a n y t h in g t o s im p lif y t h e p r o b le m

C o n c e p t:
O n e re a s o n w e d r a w d ia g r a m s f o r a n a ly t ic g e o m e t r y p r o b le m s is t h a t t h e y m ig h t
in s p ir e g e o m e tr ic s h o r tc u ts .
I
W e v e f o u n d t h a t A m u s t b e o n o n e o f t w o lin e s p a r a lle l t o a n d th a t

t h e s e t w o lin e s a r e e q u id is t a n t f r o m S ? . S im ila r ly , w e f o u n d t h a t . 4 m u s t

b e o n o n e o f t w o lin e s p a r a lle l t o a n d e q u id is t a n t f r o m W e a d d a ll f o u r
o f t h e s e lin e s t o o u r d ia g r a m . T h e f o u r p o s s ib le p o in t s A a r e w h e r e p a ir s o f
t h e s e f o u r l i n e s i n t e r s e c t . W e l a b e l t h e s e A h A * A :, a n d A , , a s s h o w n .
A , A 2A :S/L, is a p a r a l l e l o g r a m b e c a u s e t h e o p p o s i t e s i d e s o f A , A 2A 3 A 4
a r e a lo n g p a r a lle l lin e s . W e c o u ld f in d t h e c o o r d in a t e s o f t h e v e r t ic e s o f t h e
p a r a lle lo g r a m A , A 2A ;, A t . b u t t h a t l o o k s l i k e a l o t o f w o r k . I n s t e a d , w e
t h i n k a l i t t l e b i t t o s e e i f w e c a n a v o i d a l l t h a t w o r k . B e c a u s e A , A 2A ;,A ., i s
a p a r a l l e l o g r a m , w e t h i n k a b o u t w h a t ’s s p e c i a l a b o u t p a r a l l e l o g r a m s and
h o w t o u s e t h e s e s p e c ia l p r o p e r t ie s t o f in d t h e s u m o f t h e * - c o o r d in a t e s o f
t h e v e r tic e s .

S p e c ific a lly , w e f o c u s o n p a r a lle lo g r a m f a c t s t h a t m ig h t b e u s e d t o d is c o v e r s o m e th in g a b o u t t h is s u m o f * - c o o r d in a t e s T h is

th e f a r t t h a t t h 9 h ^ m ' d p ° ' n t S ' S I ‫ ״‬C .e f l n d i n g t h e ™ d p o i n t o f a s e g m e n t r e q u i r e s a d d i n g c o o r d i n a t e s . T h i s i n t u r n l e a d s u s t o


e f a c t t h a t t h e d ia g o n a ls o f a p a r a lle lo g r a m b is e c t e a c h o th e r , w h ic h m e a n s t h e y h a v e t h e s a m e m id p o in t In o t h e r w o r d s if *
^ a n d ^ a r e t h e * - c o o r d in a t e s o f A , A , , 1 , a n d A „ r e s p e c tiv e ly , t h e n t h e * - c o o r d in a t e o f t h ^ i n t e r l l n o ^ g o a

A t -A3 a n d •4 2 ' 4 '‫ ־‬e q U a ' S b ° t h + X 3^ 2 a n d + ^ ) / 2 W e w an, * , + * 2 + * 3 + * , s o w e k n o w w e , e c lo s e !

W e le t P b e t h e in t e r s e c t io n o f d ia g o n a ls A ^ a n d A ^ U , a n d le t th e
c o o r d in a te s of P be ( x P , y P ). W e ju s t saw above th a t
* p = ( * ! + * 3) /2 = ( * 2 + x 4)/ 2 . S o , w e have
* i 4 2 = 4* + 2* = 3* ‫־‬x P. w h ic h m eans th a t our d e s ir e d sum
Xi + *2 + * 3 + x 4 e q u a l s 4 * p . A l l w e h a v e t o d o n o w i s f i n d P . W e k n o w
P is t h e in t e r s e c t io n o f d ia g o n a ls , 4 i - 4 3 a n d A 2A 4, b u t w e w o u ld lik e t o
f in d a f a s t e r w a y t o f in d ‫״‬ t h a n f i n d i n g a l l t h e v e r t i c e s o f A ! A 2A 3 A 4 O u r
d ia g r a m g iv e s u s o u r s lic k a p p r o a c h . B e c a u s e P is t h e m id p o in t o f b o t h
d ia g o n a ls , it is e q u id is t a n t f r o m o p p o s i t e s i d e s A ! A 2 a n d A ^ A t. a n d i t i s .

equidistant f r o m o p p o s ite s id e s A 2A 3 a n d .4 ! A , Aha! is a ls o I

e q u id is ta n t fr o m - 4!.42 a n d . 43A 4, a n d *B (5 i s e q u i d i s t a n t f r o m . 4 2. 43 a n d »

A , A 4 . So, P i s t h e i n t e r s e c t i o n p o i n t o f t l C a n d S / ? . T h i s p o i n t i s e a s y t o
f in d !

B e c a u s e B a n d C a r e o n ,h e * - a x is , £ 3 is t h e * - a x is . S o , a ll p o in t s o n s a tis fy y = 0 . T h i s m e a n s , h e ;‫־‬/ - c o o r d i n a t e o f P i s

e q l l T y l w - H (,3 8 9 6 8 0 ) 8 R0 ) / ( 6 8 9 6 ‫ ־‬H 0 ) = 1 th e ‫ ״‬n e P aS 8eS ^ ( 6 8 0 ‫ ־‬3 8 ° )■ - ‫^ ״‬ <s t h e g r a p h o f t h e


J , 6 8 0 ~ * )1 7 ‫ ) ״‬R e a r r a n 9 'n 9 t h i s e q u a t io n g iv e s * - y = 3 0 0 . P o in t P is o n t h e g r a p h o f t h i s lin e , a n d t h e *
e o is , s o t e * - c o o r d i n a t e o f P is 3 0 0 . T h e r e f o r e , o u r d e s ir e d s u m o f * - c o o r d i n a t e s is 4 ( 3 0 0 ) = 1 2 0 0 . □

w e Z n ^ h ° W ,9 a . ° m e t r i C i n S i g h t s c a n h e l p u s w ' t h a n a l y t i c g e o m e t r y p r o b l e m s . L e t ' s t a k e a l o o k a t a c o u p l e e x a m p l e s o f h o w
w e c a n u s e a n a ly tic g e o m e tr y t o t a c k le c h a lle n g in g g e o m e t r ic p r o b le m s . J u s t a s w e s a w w h e n u s in g a n a ly tic g e o m e tr y f o r
g e o m e t r ic p r o o f s , o n e o f t h e m a in k e y s in e a c h p r o b le m is c o n v e n ie n t ly s e t t in g u p t h e p r o b le m o n t h e C a r te s ia n p la n e

Problem 17.25

M e d ia n s X .4 a n d ) H o f A X Y Z a r e p e r p e n d ic u la r . I f Z .Y X Z = 9 0 ° and X Y = 8 , f in d Y Z .

Solution for Problem 17.25: W e h a v e t w o b ig c lu e s t o t r y a n a ly t ic g e o m e t r y : m id p o in t s a n d


p e r p e n d ic u la r lin e s W e le t Z , t h e v e r te x o f t h e r ig h t a n g le , b e t h e o r ig in . T h is a llo w s u s t o
p la c e 1 o n th e * - a x is a n d Z o n t h e ‫־‬/ - a x is . B e c a u s e X Y = 8 , w e c a n le t Y b e ( 8 , 0 ) . T h e n
w e c a n le t Z b e ( 0 , 2 z ) f o r s o m e v a lu e o f z . W e u s e 2 z in s te a d o f ju s t z b e c a u s e w e k n o w
w e ‫ ״‬b e w o r k in g w it h m id p o in t s in t h is p r o b le m . I f w e f in d z . w e c a n u s e t h e d is t a n c e f o r m u la
t o f in d Y Z , s in c e

YZ = t/ (2 z ~ ( ) ) 2 + (0 - 8 )2 = v 4 ‫׳‬z 2 + 64 = 2 V z2 + 16.

W e n o w c a n fin d t h e c o o r d in a te s o f A a n d B in t e r m s o f z . B e c a u s e A is t h e m id p o i n t o f Y Z .
it s c o o r d in a t e s a r e ( 4 , z ) . B e c a u s e B is t h e m id p o in t o f X Z . it s c o o r d in a t e s a r e ( 0 , z ) B u t
n o w w h a t? ' ' ‫׳׳‬

W e h a v e n 't u s e d , h e f a c t t h a t Z . 4 - L Y B . B e c a u s e Z ^ 1 Y B . t h e p r o d u c t o f t h e s lo p e s o f H I a n d H is - 1 . T h e s lo p e o f

Z A is z / 4 a n d t h e s lo p e o f Y B is - z / 8 . T h e r e fo r e , w e h a v e

(!) (?) ‫>־ ־‬-


T h i s g i v e s u s - z 2/ 3 2 = -1 , so z 2 = 3 2 . N o w w e c a n f in d Y Z u s in g o u r fo r m u la f r o m a b o v e . W e h a v e

V'Z = 2v/P T T 6 = 2v/32TT 6 = 2v/48 = 873.

Problem 17.26
Source: AMC 12 t V

and“ ‫״‬ u ‫״‬e l ? ! 7 ™ P la ? 3 8 8 9 iV e n S Q U a re ° f S id e 1 ' L e t t h e v e r t ic e s o f t h e s q u a r e , t a k e n c o u n t e r c lo c k w is e b e 4 . B C


fro m W + ( >e ‫״‬% n C e S 10 ‫ ־‬B ‘ a n d C ‫ ־‬r e S P e C tiV e ly ' b e ‫ ־ ״‬a n d w ■W h a t i s g r e a t e s t d is ta n c e , h a t P c a n b e

Solution for Problem 17.26: S q u a r e s a r e e a s y t o r e p r e s e n t o n t h e C a r t e s i a n p l a n e . W e e v e n k n o w e a c h


s id e h a s le n g t h 1 , s o w e c a n le t t h e v e r t ic e s o f o u r s q u a r e b e A = (0 ,0 ), B = (1 0 ) C = (1 1)

a n d 0 ■( 1 ’ 0 ) ‫ ־ ״‬u r g i v e n n a t i o n u 2 + = U ,2 in v o lv e s s q u a r e s o f d is t a n c e s 'T h i s is a n o t h e r
c l u e t h a t a n a l y t i c g e o m e t r y m i g h t b e h e l p f u l : s q u a r i n g t h e d i s t a n c e s g e t s r i d o f t h e s q u a r e r o o t s i g n in
D C
t h e d i s t a n c e f o r m u l a . W e l e t t h e c o o r d i n a t e s o f P b e ( * , y ). s o w e c a n u s e , h e d i s t a n c e f o r m u l a t o

A В

ul = 8£ = (* - o)2+ (?/- o)2,


‫! ״‬ = P B 2 = ( * - l ) 2 - F ( y - 0 ) 2,
w2 = P C 2 = ( * — l) 2 + ( ‫־‬/ — l ) 2 .

S u b s titu tin g th e s e in to u2 + v 2 = w 2 g iv e s u s

x 2 + y 2 + ( x - l ) 2 + y 2 = (* - l ) 2 + (y - i ) 2.

T h e ( * — l ) 2 t e r m s c a n c e l, a n d a lit t le r e a r r a n g in g g iv e s

x 2 + y 2 + 2y - 1 = 0.

T h e g r a p h o f t h i s e q u a t io n is a c ir c le . T o le a r n m o r e a b o u t t h i s c ir c le , w e c o m p l e t e t h e s q u a r e in y o n t h e le f t . T h is g iv e s u s

x 2 + (y + l ) 2 = 2.

T h is m e a n s t h a t ( x y ) i s o n a c i r c l e w i t h c e n t e r ( 0 , - 1 ) a n d r a d i u s v 2 ‫׳׳‬. M o r e o v e r , w e c a n r e v e r s e o u r s t e p s a b o v e t o s e e t h a t

s : s r s res,rictionsplacedonpoimp intheproblemSo•nowour prob,emistobnd ppi2 ‫״<״‬


W e a d d t h is c ir c le t o o u r d ia g r a m a t r ig h t . B e c a u s e b o t h t h e c e n t e r o f , h e c ir c le a n d p o in t D a r e o n t h e
.‫־‬/ - a x i s , t h e p o i n t o n , h e c i r c l e t h a t i s t h e f a r t h e s t f r o m D is o n , h e o p p o s it e s id e o f t h e c ir c le f r o m D
In o t h e r w o r d s , it is t h e b o t t o m m o s t ' p o i n t o f t h e c i r c l e , w h i c h w e l a b e l a s Q in t h e d i a g r a m B ecause

° iS 2 u n i , s Uop t h e c e n ,e r o f t h e c ir c l®. a n d t h e r a d iu s o f t h e c ir c le is V 2. t h e d is t a n c e b e t w e e n D D C
a n d Q is 2 + v 2 ‫ ׳‬, w h ic h is t h e r e f o r e o u r d e s ir e d g r e a t e s t p o s s ib le d is t a n c e , o
В

7
A
\
V )
IQ

W h a t is t h e im a g e o f ( 5 , 6 ) u n d e r e a c h o f t h e f o llo w in g t r a n s f o r m a t io n s :

(a ) A 1 8 0 ° r o t a t io n a b o u t t h e o r ig in .

T y p e y o u r s o lu tio n , n o te s a n d / o r w o r k h e re .
Show Solution

(b ) T h e tr a n s la tio n t h a t m a p s ( 2 . - 3 ) t o ( 0 . 6 ) .

T y p e y o u r s o lu tio n , n o te s a n d / o r w o r k h e re .
Show Solution

(c ) R e fle c tio n o v e r t h e g r a p h o f * = -3 .

T y p e y o u r s o lu tio n , n o te s a n d / o r w o r k h e re .
Show Solution

(d ) A 9 0 ° c o u n te r c lo c k w is e r o ta tio n a b o u t ( - 3 , 2 ) .

T y p e y o u r s o lu tio n , n o te s a n d / o r w o r k h e re .
Show Solution

(e ) R e fle c tio n o v e r t h e g r a p h o f y = *.

T y p e y o u r s o lu tio n , n o te s a n d / o r w o r k h e re .
Show Solution

(f) R e fle c tio n o v e r t h e g r a p h o f 2 * + 3y = - 5

T y p e y o u r s o lu tio n , n o te s a n d / o r w o r k h e re .
Show Solution

r n
z as r s s i s s s f 31 “ (‫ ־‬4‫ ־ ׳‬3l *‫“ ״״ ■״‬ ‫■״״״״׳ «״ ״ ״ ״ “ ״ ״ ״״״־ ״‬
T y p e y o u r s o lu tio n , n o te s a n d / o r w o r k h e re .
Show Solution

II Z V Z is a s q u a r e w it h W X = 6 . Q u a r t e r c ir c le s c e n te r e d a t e a c h v e r te x o f W X Y Z a re

W C rT sbovT S t,U a re ^ Sh° W a The in ,e r s e c tio n s o f th e s e d a r t e r c ir c le s fo rm s q u a re

(a ) F in d t h e a r e a o f A B C D w it h a n a ly t ic g e o m e tr y .

T y p e y o u r s o lu tio n , n o te s a n d / o r w o r k h e re .
Show Solution

(b ) F in d t h e a r e a o f A B C D w it h o u t a n a ly t ic g e o m e tr y .

T y p e y o u r s o lu tio n , n o te s a n d / o r w o r k h e re .
Show Solution

Ï Ï Î Ü ‫״‬ "‫“ ״‬ 5 *"■‫״‬ ‫~ ״■״״׳ “״' ״־‬ - ‫ »״־״‬- . ‫ ״‬12


T y p e y o u r s o lu tio n , n o te s a n d / o r w o r k h e re .
Show Solution

L e t p o in t C b e ( 3 , 2 ) . P o in ts ,4 a n d B a re o n t h e g r a p h o f 2 * - y = - 7 s u c h t h a t A A B C is e q u ila t e r a l. F in d A B .

T y p e y o u r s o lu tio n , n o te s a n d / o r w o r k h e re .
Show Solution

S o lv e P r o b le m 1 7 . 2 5 w it h o u t u s in g a n a ly t ic g e o m e t r y

H in t

T y p e y o u r s o lu tio n , n o te s a n d / o r w o r k h e re .
Show Solution

G iv e n t h a t x 2 + y2 - 1 4 * + 6 6 + / ‫־‬, w h a t i s , h e l a r g e s t p o s s i b l e v a l u e t h a t 3 * + 4 j ! c a n h a v e ? ( S ource: AHSME)

H in t

T y p e y o u r s o lu tio n , n o te s a n d / o r w o r k h e re .
Show Solution

L e t A C a n d P I ) be t w o p e r p e n d ic u la r c h o r d s o f a c ir c le w it h r a d iu s 8 , a n d le t t h e
t w o c h o r d s in t e r s e c t a t P . F in d a ll p o s s ib le
v a lu e s o f P A 2 + P B 2+ P C 2+ P D 2.

T y p e y o u r s o lu tio n , n o te s a n d / o r w o r k h e re .
Show Solution
Challenge Problems

17.49: t V

Find an equation whose graph is a circle that passes through the origin and both the ^-intercept and the ;(/-intercept of the
graph of x — 3y = 18.

Type your solution, notes and/or work here. Show Solution

1 7 .5 0 : t V

(a) The graph of the equation 3 x — y = 0 is rotated 90° clockwise about the origin to produce line k. Find an equation
whose graph is line k.

Type your solution, notes and/or work here. Show Solution

(b) The graph of the equation 3 x — y = 6 is rotated 90° clockwise about the origin to produce line k. Find an equation
whose graph is line k.

Type your solution, notes and/or work here. Show Solution

17.51 Source: AMC 12 t V

The graphs of x 2 -\-y 2 = 4 412 ‫־‬x 4 6 ‫(;־‬/ and x 2 -F y 2 = k -F 4x 4 1 2 ‫־‬y intersect when k satisfies a < k < b, and for no
other values of k. Find b — a.

Type your solution, notes and/or work here. Show Solution

1 7 .5 2 : Source: AMC 12 t V

Flow many points are equidistant from the :r-axis, the ;(/‫־‬axis, and the graph of the equation x 4 y = 2?

Type your solution, notes and/or work here. Show Solution

1 7 .5 3 : t V

Point X is on diameter P Q of a circle. Prove that if A B is a chord of the circle parallel to P Q , then
X A 2 + X B 2 = X P 2 + X Q 2.

Type your solution, notes and/or work here. Show Solution

1 7 .5 4 : t V

Let U be the point ( —4, 3) and V be the point (2, —1).

(a) Let A be ( —1, 3). Let A ‫ ׳‬be the image when A is rotated 90° clockwise about U . Let A " be the image when A ‫ ׳‬is rotated
90° clockwise about V. Find A ".

Type your solution, notes and/or work here. Show Solution

(b) Let B be (7, —2). Let B ‫ ׳‬be the image when B is rotated 90° clockwise about U. Let B " be the image when B ‫ ׳‬is
rotated 90° clockwise about V. Find B ".

Type your solution, notes and/or work here. Show Solution

(c) Find the midpoint of .4,4‫ ״‬and the midpoint of B B " . Notice anything interesting?

Type your solution, notes and/or work here. Show Solution

(d)^Let C be (a, b). Let C ‫ ׳‬be the image when C is rotated 90° clockwise about U . Let C " be the image when C ‫ ׳‬is rotated
90° clockwise about V. Show that your observation in part (c) can be used to quickly find C " for any a and b.

Type your solution, notes and/or work here. Show Solution

1 7 .5 5 : t V

Let A B C D be a square, and let P be a point on side C I) . Construct squares B P W X b


and A P Y Z externally on B P and A P , respectively, as shown. Let O, O h and 0 2 be the
centers of squares A B C D , B P W X , and A P Y Z , respectively. Show that quadrilateral
O O 1P O 2 is a parallelogram.

Type your solution, notes and/or work here. Show Solution

1 7 .5 6 ★ : Source: AHSME t V

Find the largest value of y / x for pairs of real numbers (.r, y) that satisfy (x — 3 ) 2 4 6 = 2(3 — /);) ‫־‬.

Type your solution, notes and/or work here. Show Solution

1 7 .5 7 ★ : t V

Let A 1A 2 and B 1B 2 be two perpendicular chords of a circle with radius r, each passing through a fixed point P. Show that
( A \ A 2 )2 -F ( B ] B 2)2 is independent of the position of the chords. (In other words, show that for all pairs of perpendicular
chords of this circle that meet at P, the sum of the squares of the lengths of the chords is the same.)

Type your solution, notes and/or work here. Show Solution


17.7 Summary

Important: The graph of the equation y — t/! = m (x — x ! ) is a line through ( x ! , t/ !) with slope
m. This is called a point-slope form of the equation.
Z
The standard form of a linear equation is A x + B y = C, where, if possible, A, B,
and C are integers, A is positive, and A, B, and C have no common factors besides
1 .

The slope-intercept form of a linear equation is y = m x -I- b, where m is the slope


of the line and b is the ;(/‫־‬coordinate of the ;(/-intercept.

Important: The distance in the plane between the points ( x !, y ! ) and (x’ 2 , t/ 2 ) is

^ \]{X2 - X l Y + (V2 -VlY■

This is often referred to as the distance formula.

Important: The midpoint of the segment with endpoints ( x !, t/!) and (x-2 , t/2 ) is

Z x! + x 2 j / i + y 2

Important: If two non-vertical lines have the same slope, then they are parallel. Conversely, if two
^ non-vertical lines are parallel, then they have the same slope.

If two non-vertical lines are perpendicular, then the product of their slopes is —1.
Conversely, if the product of the slopes of two lines is —1, then the two lines are
perpendicular.

Important: The standard form of an equation whose graph is a circle is

Z (x - h)2 + ( y - k)2 = r2,

where h, k, and r are constants with r > 0. The center of the circle is (h, A:) and the
radius of the circle is r.

P roblem Solving S trateg ies

Concepts:
■ When the algebra in an analytic geometry problem starts to get ugly, try using
geometric facts to simplify the problem.

■ When given the coordinates of the vertices of a triangle, check if the triangle is
special in any way. Most notably, check if the triangle is a right triangle. If it is, this
fact will probably simplify the problem, because we know so much about right
triangles.

■ Interpreting an equation as a geometric figure on the Cartesian plane can often


help us solve problems.

■ When setting a geometry problem up on the Cartesian plane, choose the origin and
the axes in a convenient matter. Often this means letting the origin be the vertex of
a right angle in the problem, so that the sides of the angle are along the axes.

■ Solving a problem in two ways is a good way to check your answer.

■ Checking intermediate steps while working on problems will also help you catch a
lot of errors.

■ When we set up a geometry problem involving midpoints on the Cartesian plane,


we often use 2a, 2b, 2c, etc., for coordinates rather than just a, b, c, etc. This helps
us avoid expressions involving fractions when we find midpoints of segments in
the problem.

■ Algebra is not the only tool we have to solve problems about the Cartesian plane.
Combining geometric insights with algebra can lead to very nice solutions.

■ When stuck on a problem, focus on information that you haven’t used yet.

■ If an algebra problem has an equation whose graph is a circle, think about using
analytic geometry.

■ One reason we draw diagrams for analytic geometry problems is that they might
inspire geometric shortcuts.
R eview P roblem s

17.27: t V

Find an equation, in standard form, whose graph is the line through ( —5 ,2 ) that is perpendicular to the graph of
5.T — 2y 4- 7 = 0.

Type your solution, notes and/or work here. Show Solution

1 7 .2 8 : t V

Find the area of the triangle with vertices (3,4), ( 8 ,4), and ( —6 , —6 ).

Type your solution, notes and/or work here. Show Solution

1 7 .2 9 : t V

Let A be ( —2,4), B b e ( —7,1), and C be ( —1, —5).

(a) Find an equation whose graph is a line that contains the median from A to the midpoint of B C .

Type your solution, notes and/or work here. Show Solution

(b) Find the length of the median from A to B C .

Type your solution, notes and/or work here. Show Solution

(c) Find the length of the altitude from A to B C .

Type your solution, notes and/or work here. Show Solution

(d) Find the area of A A B C .

Type your solution, notes and/or work here. Show Solution

1 7 .3 0 : t V

Find the constant a such that the graph of (2a — 3 )# 43) ‫־‬a — l ) y = 3 is parallel to the segment with endpoints (2, —4)
and ( —1 , 2 ).

Type your solution, notes and/or work here. Show Solution

17.31 t V

Graph the equation x 2 — 6x 4‫ ־‬y 2 4 -4 y = —3. Find the area enclosed by your graph.

Type your solution, notes and/or work here. Show Solution

1 7 .3 2 : t V

Let P be ( —2, 5) and Q be ( 8 , —3). Find the standard form of the equation of the circle with P Q as a diameter.

Type your solution, notes and/or work here. Show Solution

1 7 .3 3 : t V

Let point X be (10,1), and let C be the graph of (x — l )2 4‫( ־‬y 4 1 0 = 2(2 ‫־‬.

(a) Let Y be the point on C that is closest to X . Find X Y.

Type your solution, notes and/or work here. Show Solution

(b) ★ Find the coordinates of Y.

Type your solution, notes and/or work here. Show Solution

1 7 .3 4 : t V

Find all possible values of a such that the distance between the point (a, 3) and the graph of 5a; — 12y = 8 is 7.

Type your solution, notes and/or work here. Show Solution

1 7 .3 5 : t V

Point A is ( —3, 5). The midpoint of A B is (4, 2) and the midpoint of A C is (3,1). What is BC?

Type your solution, notes and/or work here. Show Solution

1 7 .3 6 : t V

Point O is the origin and point B is (0,13). Point A has positive coordinates such that A O = 5 and A B = 12. Find the
coordinates of A

Type your solution, notes and/or work here. Show Solution

1 7 .3 7 : t V

What is the image of (5, —2) when reflected over each of the following:

(a) The x ‫־‬axis.

Type your solution, notes and/or work here. Show Solution

(b) The graph of x = —3.

Type your solution, notes and/or work here. Show Solution

(c) ★ The graph of 2x — y = —1.

Type your solution, notes and/or work here. Show Solution

1 7 .3 8 : t V

Find the length of the segment that is a chord of both the graph of x 2 4- y 2 = 36 and the graph of x 2 4- 2x 4‫ ־‬y 2 = 30.

Type your solution, notes and/or work here. Show Solution

1 7 .3 9 : t V

Use analytic geometry to show that if A B = A C in A A B C , then the altitude from A to B C is also the median from A to
‫־‬B C .

Type your solution, notes and/or work here. Show Solution

Two adjacent vertices of a square are ( —4, 5) and ( —3, 7). What are all possible points besides these two that could be
vertices of this square?

Type your solution, notes and/or work here. Show Solution

1 7 .4 1 t V

Show that the image of the point (a, b) upon reflection over the line y = x is ( 6 , a).

Type your solution, notes and/or work here. Show Solution

1 7 .4 2 : t V

Point X is (2 ,3 ) and point Y is ( 6 ,3). Find all possible points P such that A X Y P is equilateral.

Type your solution, notes and/or work here. Show Solution

1 7 .4 3 : t V

Use analytic geometry to show that the sum of the squares of the lengths of the diagonals of a parallelogram equals the sum
of the squares of the lengths of the sides of the parallelogram.

Type your solution, notes and/or work here. Show Solution

Find the equation whose graph is the circle that passes through the points ( —1, 5), (4,4), and (5.9).

Type your solution, notes and/or work here. Show Solution

1 7 .4 5 : t V

In triangle A B C , let I), E, E be the midpoints of B C , A C , A B , respectively,

(a) Show that triangles A B C and D E E have the same centroid.

Type your solution, notes and/or work here. Show Solution

(b) Let G i, G?, G 3 be the centroids of triangles A F E , B D F , C E I) , respectively. Show that triangles A B C and G 1G 2G 3
have the same centroid.

Type your solution, notes and/or work here. Show Solution

Points X and Y are on .4 B such that A X = X Y = B Y . If A is (0 .9 ) and B is (4,0), what is the slope of the line through
X and the origin?

Type your solution, notes and/or work here. Show Solution

1 7 .4 7 : t V

Point P is ( 6 . 2). Points Q and R are on the æ-axis such that Z .P Q R = 30° and Z .P R Q = 60°. What is the area of A P Q R
?

Type your solution, notes and/or work here. Show Solution

1 7 .4 8 : t V

Point X is (4, —3), point Y is ( —2, 5), and point Z is (c, 3).

(a) For what values of c is A X Y Z isosceles?

Type your solution, notes and/or work here. Show Solution

(b) For what values of c is A X Y Z a right triangle?

Type your solution, notes and/or work here. Show Solution


La w o f Cosines

New facts often trigger леи‫ ׳‬ideas - Alex F. Osborn

CHAPTER

I
Introduction to Trigonometry

of ‫ ״ ״ *""״״ ־‬- ‫— ־״״״״״ ׳ ״‬

18.1 Trigonometry and Right Triangles


Problems
Problem 18.1
4 J u m p t o S o lu tio n
Y o u m a y u s e a c a lc u la t o r o n t h is p r o b le m .

1t
В ‫׳‬

C — — --------------- D
c ! X;

(a ) I n t h e d i a g r a m a b o v e , B C ‫־‬a 3 . 5 . W h a t i s E F t o t h e n e a r e s t t e n t h ?

(b ) U s e t h e g iv e n in f o r m a t io n a b o u t A A B C t o f in d Y Z / X Y t o t h e n e a r e s t 0 . 0 1 .

You may use a calculator on this problem.

(a) In the diagram above, I1C « 3.5. What is E F to the nearest tenth?

(b) Find Y Z / X Y to the nearest 0.01.

Solution for Problem 18.1:

(a ) W e have A A B C ~ A D E F b y A A S im ila r ity . T h e r e fo r e , w e h a v e B C / A B = E F / D E . s o

£ F ‫> ״ ־‬£> ( i f ) ‫( ־‬i2)( ¥ ) 47‫•־־‬


(b ) W e have A A B C ~ A X Y Z b y A A S im ila r ity . T h e r e fo r e , w e h a v e Y Z / X Y = B C / A B 3 9 0 ‫ ־־־‬N o tic e t h a t E F / D F

“ ” p‫״ “׳‬s M‫ ־ ״ ״ “• ״“*־‬1* '• “ '‫ «״״‬/ / /



T h e r a tio

length of a leg opposite an acute angle of a right triangle


length of the hypotenuse of the right triangle

is s o c o m m o n a n d im p o r t a n t t h a t w e g iv e it a n a m e . T h is r a t io is c a lle d t h e sine o f t h e a c u t e a n g l e . W e d e n o t e t h e s i n e o f Z . 4 a s

iiv t 'u f C ^ r i9 h t W e h a V e = 90 ' B° = a - C A = b■ AB = c . S o , o u r d e f in it io n o f s in e

sjnyj _ length of leg opposite A a


length of hypotenuse ~ c.'

2 r rt0hbe e r T t i 8 ‫ ] ״‬,r e ,U S e d , * T " ‫ ״‬I " 3 1 9 1 6 5 ‫ ׳‬t 0 S h ° W t h a t a " r i9 h t t r ia n 9 le s w it h a n a c u te a n g le o f


2 3 , t h e r a t io o f t h e le n g t h o f t h e le g o p p o s i t e t h e 2 3 ° a n g le t o t h e le n g t h o f t h e h y p o t e n u s e is t h e
s a m e S p e c ific a lly , w e f o u n d t h a t t h is r a t io is a p p r o x im a t e ly 0 . 3 9 So w e w r ite s in 2 3 ° ~ 0 3 9 to in d > th ■
t r ia n g le w it h a n a c u te a n g le o f 2 3 ° , t h e r a t io ' ^ 0 , i J 1 ln d lc a t e t h a t ln a n y r ig h t

length of the leg opposite the 23° angle


length of the hypotenuse
is a p p r o x im a te ly 0 .3 9 .

U S S U lT “ no' ,h*on‫״‬ ‫״‬ ‫ * ״ ■ ״ ״״׳ * ״ “ ״‬- — «


length of the leg adjacent to the angle
length of the hypotenuse

W e d e n o t e t h e c o s in e o f Z . 4 a s c o s A , s o in o u r d ia g r a m a t r ig h t , w e h a v e ^

cos 4 _ length of the leg adjacent to the angle _ b

length of the hypotenuse —c

s in A
ta n A =
cos A

» S S S S S °' “ ‫• ״ ״ ״‬ w.‫" * * ־‬ ’‫—־״'»’ * * ״ " * * • " ״ ״‬ ‫״*״״< ־׳‬


‫״>׳״■'״״ “ “ ״‬-

Problems ------------------------------
Problem 18,2
4 J u m p t o S o lu t io n
S h o w t h a t t h e t a n g e n t o f a n a c u te a n g le in a r ig h t t r ia n g le e q u a ls t h e r a t io

length of the leg opposite the angle


length of the leg adjacent to the angle'

Problem 18.3 --------------------------------------------------------------------------------


4 J u m p t o S o lu t io n

S u p p o s e A A B C is a 4 5 - 4 5 - 9 0 r ig h t t r ia n g le w it h Z C = 9 0 ° . U s e A A B C t o f in d s i n 4 5 ° , c o s 4 5 ° . a n d t a n 4 5 ° .

Problem 18.4
4 J u m p t o S o lu tio n
(a ) F in d s i n 3 0 ° , c o s 3 0 ° , a n d t a n 3 0 °

(b ) F in d s in 6 0 ° , c o s 6 0 ° . a n d t a n 6 0 °

Problem 18.5
4 J u m p t o S o lu tio n
In A P Q R . w e h a v e Z P = 9 0 °, P Q = 3, a n d Q R = 7

(a ) F in d s i n Q a n d c o s R

(b ) F in d t a n Q .

‫״‬ M“ №‫ ״״״‬- <‫ “>״ ״״‬- ‫ « “ ״‬- * ‫ ׳‬- ‫» —«< ־‬ p».‫׳‬. ‫־‬.

Problem 18.6 ‫־־‬


4 J u m p t o S o lu t io n
S u p p o s e t h a t Z . 4 is a n a c u t e a n g le .

(a ) E x p la in w h y s i n A = c o s (9 0 ° - A ).

(b ) E x p la in w h y s i n 2 A + cos2 A = 1.

Problem 18.7
4 J u m p t o S o lu t io n
L e t A A B C b e a r ig h t tr ia n g le w it h Z C = 9 0 °.

(a ) S h o w th a t s in A a n d c o s .4 m u s t b e b e tw e e n 0 a n d 1

‫״‬ I Z Z S A T J Z 'Z Z Z .4 ‫־‬ ,r " "‫— ־ ״ ־ ״ ״ * ־‬ ' ‫״‬ ‫״‬ ‫* ־ ־ ־‬ - » ■ « ‫״‬ < ‫ ״‬.‫ ״‬y —

P r o b le m 1 8 .2

Show that the tangent of an acute angle in a right triangle equals the ratio

length of the leg opposite the angle


length of the leg adjacent to the angle

S o / u f / o n for Problem 18.2: L e t A A B C b e r i g h t t r i a n g l e w i t h Z C = 9 0 ° , B C = a. A C = b a n d


AB - c , a s s h o w n a t r ig h t . S in c e t a n g e n t is t h e r a t io o f s in e t o c o s in e , w e h a v e

tan A = h in A _ 7 _ “ _ length of the leg opposite the angle


COS6 ! 4‫־‬ length of the leg adjacent to the angle‘

P u ttin g ^ th e Q r e s u lt o f P r o b le m 1 8 . 2 t o g e t h e r w it h o u r d e f in it io n s o f s in e a n d
c o s in e , w e h a v e t h e f o llo w in g f o r r ig h t t r ia n g le

Important:

z sin 4 _ length of leg opposite A


length of hypotenuse ”
a
c’
_ length of leg adjacent to A b
cos A —
length of hypotenuse — c’
tan A = length of leg opposite A a
length of leg adjacent to A ~ Ji'

S O H C A H T O A ‫׳‬r e d e f ' n i t i 0 n S ' ‫ ״‬e h a V e n ° c h 0 i c e b u t t 0 s i m P ‫ ׳‬y m e m o r i z e t h e m O n e c o m m o n m n e m o n ic f o r m e m o r iz in g t h e m is

Sine = Opposite/Hypotenuse;
Cosine = Adjacent/Hypotenuse;
Tangent = Opposite/Adjacent.

W e c a n 't q u it e d o S O H C A H T O A j u s t i c e in p r i n t - it 's a m u c h m o r e e f f e c t i v e m e m o r i z a t i o n t o o l w h e n s p o k e n . " S o - c a h - t o e - a h . "

W e c a n f in d t h e v a lu e s o f t r ig o n o m e t r ic f u n c t io n s o f s o m e a n g le s w it h o u r u n d e r s ta n d in g o f s p e c ia l r ig h t t r ia n g le s .

Problem 18.3

Find sin 45°, cos 45°, and tan 45°.

S olu b b n for Problem 18.3: A t r i g h t i s 4 5 - 4 5 - 9 0 t r i a n g l e A B C w i t h r i g h t a n g l e a t C . T h e r a t i o o f a l e g o f


4 5 - 4 5 - 9 0 t r i a n g l e t o i t s h y p o t e n u s e i s 1 /s/2, s o w e h a v e

s in 4 5 ° = s in A = — = -L
AB y/2‫׳‬

W e u s u a lly t r y t o a v o id w r it in g f r a c t io n s w it h a s q u a r e r o o t in t h e d e n o m in a to r . W e c a n w r it e l/ s/ 2
w i t h o u t a s q u a r e r o o t in t h e d e n o m i n a t o r b y m u l t i p l y i n g t h e n u m e r a t o r a n d d e n o m i n a t o r b y v 2 ‫׳׳‬:

• s i‫ ״‬4 5 ‫= ״‬ * =
s/2 s/2 s/2 2 '
S im ila r ly , w e h a v e

co s4 5 ° = c o s A = — = - L = ^ ?
AB s/2 2 '

F in a lly , s in c e t h e le g s o f a 4 5 - 4 5 - 9 0 t r ia n g le h a v e t h e s a m e le n g t h , w e h a v e

ta n 4 5 ° = t a n .4 = ^ = 1,
AC

Problem 18.4

Find sin 30°, cos 30°, and tan 30°. Find sin 60°, cos 60°, and tan G0C

Solution for Problem 18.4: W e s t a r t w i t h A A B C a t r i g h t , i n w h i c h Z A = 3 0 °, Z C = 9 0 °, a n d Z B = 60°


B e c a u s e A A B C is a 3 0 - 6 0 - 9 0 r ig h t t r ia n g le , w e h a v e B C / A B = 1 /2 , so

sin3° ° - B_C = _1 and


cos 60° = ! § = !
AB 2

W e a l s o h a v e A C /A B = \/3 /2 , so

» ‫ ״‬6 0 • . AC \/3
— and cos 30° - ----- .
AB AB

S in c e A C / В С = \ / 3 / 1, w e h a v e

tan 30° = — = — = — Z ? = Z?
AC s/ 3 v/3 x/3 3 '
D

Angle sin cos tan


30° 1 À •h
3
45° h h 1
■2 2
60° sa 1 vS
2 2

» Ü T l Ï r “ T‫” • ״‬ “ ‫״ ״‬ " > ‫» ״ ' ״ ״ ׳ « ׳ • ״' ׳ “ ״‬ ‫־‬ »>‫— ־‬ » f • 5 -9 0 « » « ‫ ־ ־ ״‬t r ia n g le s a n d in

X t e, r gh “ “ ‫“ ״ * ״‬ ‫“ ״‬ * ‫“ ״‬ “ ‫ ״ ״ * ׳‬-* ” ‫ ־ ״ ־ ״ ״ ־ ״ ' “ ־ * ״‬I‫ ־‬. ‫ ״ • ־ ־ ״‬e v a l u a t e , й, т т ш ol

Problem 18.5

In A PQR, we have Z P = 90°, PQ = 3, and QR = 7.

(a) Find sin Q and cos R.

(b) Find tan Q.

(c) Find (sin R )2 -F (cos R )2.

Solution for Problem 18.5:

(a ) W e f ir s t u s e t h e P y th a g o r e a n T h e o r e m t o d e te r m in e t h a t P R = 2 s / l0 . T h e n , w e d r a w o u r
d ia g r a m w it h th e s id e s la b e le d as a t r ig h t . W e h a v e s in Q = P R /Q R = 2 v /l0 /7 and
oosR = P R /Q R = 2 v /1 0 /7

(b ) W e have 2 V10

tanQ‫™ ־‬ ^ .
PQ 3

(c ) W e f ir s t fin d s in R = PQ /Q R = 3 /7 . Then, w e have

( s in R )2 + (c o s R )2 =
i.+ « . l
49 49

W e t y p ic a lly w r it e ( s i n R f a n d ( c o s R ) 2 a s s i ‫ ־ ״‬R a n d c o s ‫ ־‬R . r e s p e c t iv e ly , a n d s im ila r ly f o r o t h e r p o s it iv e in t e g e r p o w e r s o f


t r i g o n o m e t r i c f u n c t i o n s . S o , w e d w r i t e t h e r e s u l t w e f o u n d in p a r t ( c ) a s s i n 2 R + cos2 R = 1

P a r t s ( a ) a n d ( c ) a b o v e s u g g e s t a c o u p le in t e r e s t in g r e la t io n s h ip s b e t w e e n t r i g o n o m e t r i c f u n c t io n s . L e t 's in v e s t ig a t e .

P r o b le m 1 8 .6

Suppose that Z -4 is an acute angle.

(a) Explain why sin .4 = cos(90° - A).

(b) Explain why sin2 A - f c o s 2 A = 1.

Solution for Problem 18.6:

h a V e 9Z f 3 S = 99 0 ° r - nz 4 f o 4 / ? C ^ a n 9 'e W <‫ ־‬h a v e s i n A = B C /A B W e a ls o

c o s (9 0 ° - A) = cos B = —
AB

T h e re fo re , w e h a v e s in A = c o s ( 9 0 ° - . 4 ) f o r a n y a c u te a n g le Z . 4 ,

(b ) W e h a v e s in A = B C / A B a n d c o s .4 = AC/AB. so

s in 2 A + cos2 4 = g g + ^
A B 2 ' A B 2 A B 2 ’

T h e P y th a g o r e a n T h e o r e m g iv e s u s B C 2 + A C 2 = A B 2 so w e have

B C 2 + A C 2 A B 2
sin2 A + cos2 A = = 1.
A B 2 A B 2

ZZZr i Ä r S T S r c r Ä “ ‫ ״׳ ■* ״‬- ‫* ־״ ׳״ ״“'•־‬ ‫״‬- — «•

sin2 A + cos2 A = 1 ,

sin A = cos(90° - A).

O n c e a g a ia n o te th a t s in 2 A = ( s i n A ) 2 ( a n d lik e w is e f o r a ll t r ig o n o m e t r ic f u n c t io n s r a is e d t o p o s it iv e in t e g e r p o w e r s , .

Problem 18.7

Let A A R C be a right triangle with Z C = 90°.

(a) Show that sin A and cos A must be between 0 and 1.

(b) Show that tan A must be greater than 0, but that tan .4 has no upper bound.

Solution for Problem 18.7:

(a ) r '9 h t T h 3 V e S in A = B C / A B B e c a u s e A B is t h e h y p o t e n u s e o f A A B C it
is o n g e r t h a n b o t h le g s . S p e c ific a lly , w e h a v e A B > B C . s o B C / A B < 1. F u r t h e r m o r e

0 < sin A < I6' ‫׳‬ P0SmVe' 50 BC/AB > ° C° mbinin9 these ine^ a'™es gives us

S im ila r ly , w e h a v e c o s A = AC/AB and A B > A C > 0, so 0 < cos A < 1.

(b ) B e c a u s e tan A = B C / A C a n d B C a n d A C a r e p o s i t i v e w e h a v e t a n A ~> (1 r ‫» ״‬ . x
p o s i t i v e v a l u e , l e t A C = 1 T h e n w e havp t a n 4 — h x 1 ‫ ׳‬A, ’ . 0 see * 'a n A c a n t a k e o n a n y
c a n b u ild a r ig h t t r ia n g le w it h le q s 4 C - 1 a n d / ? Z W e have r e s tr ic t io n s o n B C . F o r a n y p o s itiv e n u m b e r x . w e

Important: If Z A i s a c u t e , t h e n 0 < s i n A < 1, 0 < cos A < 1, a n d ta n A > 0 T h e re


no
^ upper bound on the value o f tan A.

 Π= ! r : S Г Л Г Г - - — •4 - • ‫׳״• ׳ ״‬

WARNING!! D e g re e s a re n o t th e o n ly u n it s f o r a n g le s . I f y o u u s e y o u r c a lc u la t o r t o c o m p u te

o I' f a n d y 0 U d 0 n 1 9 e t ( ) • 5 ( o r 1 / 2 ) a s t h e r e s u lt • t h e n e it h e r y o u a r e n 't u s in g y o u r
c a lc u la t o r c o r r e c tly , o r y o u r c a lc u la t o r is s e t t o u s e u n it s b e s id e s d e g r e e s f o r a n g le s I f
y o u r c a lc u la t o r is s e t t o u s e s o m e o t h e r u n it s o f m e a s u r e b e s id e s d e g r e e s , r e s e t y o u r
c a lc u la t o r t o d e g r e e s b e fo r e c o n tin u in g .

Solution for Problem 18.8: W e h a v e s i n С = AB/AC


have
so we have A C — A B / (sin C). Since sin C = sin59° xs 0.857 we

A В 12
AC - ---------- Я 5 ------------ s s 1 4 0
s in C 0.857 U•

l â UTlc ,Л?Дс‫־‬,:Г\Т «?""КГсТ T "’Sin*•


В С « (1 4 .0 X .5 1 5 ) « 7 .2 . о 3 ’П С е C = 59 ' w e have cos 59° «
‫«״■״‬
0 .5 1 5 , so

Problem 18.9

Solution for Problem 18.9: O h n o ! T h e r e ' s n o d i a g r a m ! W h a t w i l l w e d o ?

W e s t a r t b y d r a v .in g o u r o w n d ia g r a m . D o n t le t w o r d y , s e e m in g ly c o m p lic a t e d g e o m e tr y p r o b le m s

th e " ‫ ״‬n e t ° t ' t T t , m e 1 0 d r a W 3 d ia 9 r a m f ° r t h e P r° b le m ' la b e l i l w it h in f o r m a t io n y o u k n o w


t h e n g e t t o w o r k . A s y o u d is c o v e r m o r e a b o u t t h e d ia g r a m , a d d w h a t y o u le a r n t o t h e d ia g r a m

S t h r ‫ ״ ״‬I h rt, h W i t h t h e p e r p e n d ' c u l a r b i s e c , o r o f A B T h e p e r p e n d i c u l a r b i s e c t o r s o f a t r i a n g l e


p a s s t h r o u g h t h e c r c u m c e n t e r o f t h e t r ia n g le . T h e _ c ir c u m c e n te r o f a r ig h t t r ia n g le is t h e m id p o in t

c L easXTnd ACS0' P° im ° '' th6 midP°im °f Theref°re' if We f'nd A B D C ■ or B D ■ we

Z I T d b J ^ Z Z T " ,1 3 ^ ^ M D = 1 and s in ^ M D B = 1 /5 . W e 'd lik e to f in d B D . F ro m

M B 1

B D ~ 5 '

U n f o r t u n a t e ly , w e d o n 't k n o w M B . H o w e v e r , w e d o k n o w D M . a n d w e k n o w t h a t

D M
= cos Z MDB.
BD

a n d w e k n o w th a tOOS ^ ^ ^ ^ th iS e q U a ,i° n W e 9 ‫ ׳‬v e n C O fi Z M D B . but w e d o k n o w « ‫ ״‬Z M D B .

c o s 2 Z M D B + s in 2 Z M D B = 1.
T h e re fo re , w e h a v e

cos2 Z M D B = 1 - s in 2 Z M D B = 2 4 /2 5 .

Ü / S S W Ä ‫ ״‬.? “ “ ' “ 2 ‫״‬ O S ' ^ A F‫“ ״‬ D M /B D = coeZ M D B . «

B P — 5______ 5^ s/6 5s/6


cos Z M D B 2v/6 2v ‫ ׳‬6 ‫ ׳‬v/6 12'
F in a lly , w e h a v e

A C = 2A jD = 2 B D =
5у6‫׳‬
6

O u r p r o c e s s o f f in d in g B D u s in g M D = 1 a n d s in Z M D B = 1 / 5 in r ig h t t r ia n g le M D B is a n e x a m p le o f t h e f a c t t h a t :

Important:
| [ , nW a b a V V hl l e n 9 t b 0 f a S i d e o f 3 r i 9 h t t r i a n 9 ‫ ׳‬e a n d <h a v a l u e o f a t r i g o n o m e t r i c
z le n g t h s ^ o f ! h e W a n g le 3 C U te ^ ° f t r i a n 9 'e ' t h 6 n W 6 C 3 n f 'n d a " t h e s id e

T r i g o n o m e t r i c f u n c t i o n s a r e u s e d f o r s u r v e y i n g l a n d a n d m e a s u r i n g l a r g e d i s t a n c e s . H e r e 's a n e x a m p l e h o w

Problem 18.10

am 500 feet from a building. When I look at the top of the building, I look upward at an
angle of 7 from horizontal. When I look at the point where the building touches the
nearest foot? d° WnWard at a‫ ״‬angle of 2 ° from h0r‫׳‬z°ntal. How tall is the building to the

S o / u f i o n for Problem 18.10: W e l a b e l t h e d i a g r a m a s s h o w n a t r i g h t . W e s e e k t h e l e n g t h


o . D . W h ile w e c a n 't f in d C D d ir e c t ly , w e c a n u s e t r ig o n o m e t r y t o f in d B C a n d B D
S p e c ific a lly , w e have tan Z C A B = BC/BA. so BC = BA tan 7 ° 6 1 .4 ‫־ ־‬ W e
a ls o h a v e tan Z B AD =
B D / B A . s o B D = B A t a n 2 ° 1 7 . 5 ‫־־־‬. T h e r e f o r e t o t h e
n e a r e s t f o o t , t h e h e ig h t o f t h e b u ild in g is C D = C B + B D « 7 9 f e e t , c

S in e c o s in e a n d ta n g e n t a r e n o t t h e o n ly t r ig o n o m e t r ic f u n c t io n s . T h r e e m o r e t r ig o n o m e t r ic f u n c t io n s
a re cosecant ( d e n o t e d
e s c ), secant ( d e n o t e d s e c ) , a n d cotangent ( d e n o t e d c o t ) . T h e s e a r e d e f i n e d a s f o l l o w s :

1
esex -
S in t
1
s e c x -
COS X
1
cot x =
tanx

n e e d t 0 m e m 0 r iz e th e s e r i9 h t n o w - У ™ 11‫ ׳‬b a v e e " ° 9 ‫ ״‬h e x p e r i e n c e w i t h t h e m in la t e r t e x t s t h a t y o u 'll c o m e t o k n o w

Let AA B C b e a r ig h t tr ia n g le w it h Z A B C = 9 0 °, A B = 12, and A C = 1 6 . F in d s i n A , c o s A , a n d t a n A

T y p e y o u r s o lu tio n , n o te s a n d / o r w o r k h e re .
Show Solution

F in d t h e m is s in g s id e s in t h e t r ia n g le s b e lo w t o t h e n e a r e s t 0 . 1 .

T y p e y o u r s o lu tio n , n o te s a n d / o r w o r k h e re .
Show Solution

H in t

T y p e y o u r s o lu tio n , n o te s a n d / o r w o r k h e re .
Show Solution

S h o w t h a t i f 0 is t h e m e a s u r e o f a n a c u t e a n g le , t h e n c o s 0 = s i n ( 9 0 ° - 0).

T y p e y o u r s o lu tio n , n o te s a n d / o r w o r k h e re .
Show Solution

In t r ia n g le A B C . w e h a v e Z B = 9 0 ° a n d s in A = 5 / 7 . F in d t a n C .

T y p e y o u r s o lu tio n , n o te s a n d / o r w o r k h e re .
Show Solution

T y p e y o u r s o lu tio n , n o te s a n d / o r w o r k h e re .
Show Solution

W it h o u t u s in g a c a lc u la t o r , e v a lu a te s in 1 5 °

H in t

H in t

T y p e y o u r s o lu tio n , n o te s a n d / o r w o r k h e re .
Show Solution
18.2 Not Just For Right Triangles
So far, we have only defined trigonometric functions for angles in right triangles. However, these functions are useful for far more
than just right triangles. But to see why, we'll start with a special right triangle.

P roblem s
Problem 18.11

In A A B C at right, we have Z C = 90°, A B = 1. and Z.A = 9. Find expressions for A C and


B C in terms of 9.

Problem 18.12

We commonly refer to the circle centered at the origin with radius 1 as the unit circle,
which is shown at right. (0 J )

Let O be the origin, A be the point (1 ,0 ) and 13 be a point on the unit circle such that both
coordinates of 13 are positive and A B O A = 8. Find the coordinates of 13 in terms of 8 .
é a
' Ö

(0 M )

Problem 1 8 . 1 1 ______________________________ _________________ _______________________

In A A B C , let A C = 90°, A B = 1, and A A = 8. Find expressions for A C and B C in terms of 8.

Solution for Problem 78.77: Because A B = 1, we have s in /1 = ‫ ־‬B C Therefore,


B C = sin A = sin 8. Similarly, we have cos A = A C / A B - A C , so
A C = cos A = cos 9. These side lengths are shown in the diagram at right. □

We can extend our observation in Problem 78.11 to build a definition of sine and cosine for angles that are not acute. First, we II
take a slightly different look at depicting the cosine and sine of an acute angle.

t V
Problem 1 8 . 1 2 ______________________________________________________________ _________________ __________________

We commonly refer to the circle centered at the origin with radius 7 as the unit circle. Let O be the origin, .4 be the point (1 0)
I d /T be int on ,he uni, circle such that both coordinates of /7 are positive and Z B O .4 = 8 . Find the coordinates of B m
terms of 9.

Solution for Problem 78.12: To find the coordinates of B in terms of 9, we draw altitude B X y
from B to the x-axis. Doing so builds a right triangle with an acute angle with measure V.
Furthermore, the hypotenuse, O B , of this triangle is a radius of the unit circle, so O B - 1
Now, we have the same problem as in Problem 18.11, and we have O X = cos 9 and
X B = sin 6. Therefore, the coordinates of B are (cos 0, sin 0). □

Problem 18.12 to use the unit circle


Rather than using right triangles to define cosine and sine, we can extend our observation in

For example, point B at right is on the unit circle. 60° counterclockwise from (1,0).
Drawing altitude ‫־‬B P from B to the x-axis forms 30-60-90 triangle A B P O , from which we
haVe P O = 1 /2 and B P = \ / 3 / 2 . Therefore, the coordinates of B are ( l/ 2 , v 3 / 2 ) , so
we have cos 60° = 1 /2 and sin 60 = \/3 /2 .
Let’s see how this definition allows us to define sine and cosine of angles that are not
acute.

P roblem s
4 Jump to Solution
Problem 18.13

= 150° and such that C is 150° counterclockwise


Let point A be (1 ,0 ) and let point C be on the unit circle such that A C
from A along the circle. Find the coordinates of C.

4 Jump to Solution
Problem 18.14

= 225° and such that I ) is 225° counterclockwise


Let point A be (1 ,0 ) and let point D be on the unit circle such that A I )
from A along the circle. Find the coordinates of D.

Find cos 150° and sin 150c

Solution for Problem 18.73: Following our definition of cosine and sine, we let A be (1 ,0 ) and we
let point C be the point on the unit circle that is 150° counterclockwise from A. The coordinates
of C then are (cos 150°, sin 150°). To find these coordinates, we first determine where point C
is along the unit circle. The point (0 ,1 ) is 90° counterclockwise from A and ( - 1 , 0 ) is 1817
counterclockwise from A, so C is on the minor arc from (0, l) t o ( - 1 , 0 ) . At right, we have our
unit circle, the origin, O, and points .4 and C.
Once again, we build a right triangle by drawing the altitude from C to the x-axis. Because
A C = 150°, we have A A O C = 150° Therefore, we have A C O P = 180° - 150° = 3 0 ° .
So, A C O P is a 30-60-90 triangle. Since O C = 1 and A C O P = 30°, we have O P = t/ 3 / 2
and C P = 1/2. Because C is to the left of the y-axis and above the x-axis, its :;;-coordinate is
= —y /3 /2 and
negative and its ;!/-coordinate is positive. So, the coordinates of C are ( - \ / 3 / 2 , 1/2), which means cos 150
sin 150° = 1/2. □
The coordinate axes divide the Cartesian plane into four quadrants. We usually label these
quadrants with Roman numerals, as shown at right. We refer to them as the first, second third an
fourth quadrant, respectively. So, point C in Problem 78.73 is in the second quadrant. AH points in
the second quadrant are to the left of the y-axis and above the x-axis, so all points in the second
quadrant have a negative x-coordinate and a positive y-coordinate. We sometimes refer to angle
measures with quadrants, as well. When we say that a 150° angle is a 'second quadrant angle, this
means that the point that is 150° counterclockwise from ( 1 , 0 ) on the unit circle is in the second
quadrant. Similarly, because 60° is between 0° and 90°, a 60° angle is a 'first quadrant' angle.

Evaluate cos 225° and sin 225°.

Solution for Problem 78.74: Again, we let A be (1,0), and we let point D be the point on the unit y
circle that is 225° counterclockwise from A. Since ( - 1 , 0 ) is 180° counterclockwise from A an
(0. - 1 ) is 270° counterclockwise from A, we know that D is in quadrant III, as shown at right.

We draw altitude ‫־‬O P from D to the x-axis, forming right triangle O O P . Because A E F is 180°

and A E D is 225°, we have D F = 45°, so A O O P is a 45-45-90 triangle. Since O D = 1, we


have O P = D P = V 2 /2 . Point D is \ F l j 2 to the left of the y-axis and V 2 /2 below the x-axis,
so its coordinates are ( - n/ 2 / 2 , - n/ 2 / 2 ). Therefore, we have
cos 225° = sin 225° = —v 2 / 2 ‫ ׳‬. □
Now that we have a handle on sine and cosine, we can revisit tangent. Once we have defined sine and cosine in terms of the unit
circle, we define the tangent of any angle 8 for which cos 8 ^ 0 as
sin 6
ta n 0 = ----- p.
cos 9

If cos 6 = 0. then ta n 6 is undefined.

P roblem s
4 Jump to Solution
Problem 18.15

(a) Use the unit circle to find sin 0° and cos 0°, then evaluate t an 0

(b) Find sin 90°, cos 90°, and ta n 90°.

4 Jump to Solution
Problem 18.16

Evaluate cos 300°, sin 300°, and ta n 300° without a calculator.

4 Jump to Solution
Problem 18.17

Explain w h y s in (1 8 0 ° - 8 ) = s in 8 for any acute angle 8.

4 Jump to Solution
Problem 18.18

Show that for any triangle A A B C , we have [A B C ] = ~ ( A C ) ( B C ) s in C

t V
Problem 18.15

(a) Find sin 0°, cos 0°, then evaluate ta n 0

(b) Find sin 90°. cos 90°, and ta n 90°.

Solution for Problem 18.7 5:


= 1 and sin 0 ° = 0, so
(a) The point that is 0° counterclockwise from (1 ,0 ) is simply (1,0). Therefore, we have cosO‘
ta n 0 ° = (s in 0 ° ) /( c o s 0 °) = 0 .
= 1. Therefore, we have
(b) The point that is 90° counterclockwise from (1 ,0 ) is (0,1). So, we have cos 90° = 0 a n d s in 9 0 ‘

ta n 90° = (s in 90° ) /( c o s 9 0 °) = 1 /0 = • ■•

Uh-oh! We can’t divide by 0, so ta n 90° is undefined.

When we put the values of sine and cosine of 0° and 90° together with those of 30 ,
45 °, and 60°, a curious pattern emerges:

0 ° 30° 45° 60° 90°


A Vl A A
sin
cos à2 h à à â
2 2 2 2

Problem 1 8 . 1 6 _______________________ ___________________

Evaluate cos 300°, sin 300°, and ta n 300° without a calculator.

Solution for Problem 18.16: We let A be (1,0), and we let point B be the point on the unit circle y
that is 300° counterclockwise from A The point (0, - 1 ) is 270° counterclockwise from A and
going 360° counterclockwise from A takes us back to A so we know that B is in quadrant M as
shown at right. (We could also have determined that B is in quadrant IV by noting that 30U
counterclockwise is the same as 60 clockwise.)
We draw the altitude from 13 to point P on the x-axis. Because 13 is 300° counterclockwise from
A, we have A B = 60°, so A B O A = 60. From 30-60-90 triangle B P O , we have B P = \ / 3 / 2
and O P = 1 /2 so the coordinates of 13 are (1 /2 , - t / 3 / 2 ) . What’s wrong with this conclusion:

Bogus Solution: Because 13 is (1 /2 , - V 3 / 2 ‫)׳‬, we have sin 300° = 1 /2 and cos 300° = - t/3 /2 .

We have sine and cosine backwards here! Cosine is the x-coordinate and sine is the y-coordinate.________________

WARNING!! Be careful not to get sine and cosine backwards. Once you have found the
appropriate point on the unit circle, an easy way to remember which coordinate is
O cosine and which is sine is to use alphabetical order: cosine is before sine and x is
before y.

So, because 13 is (1 /2 , - t / 3 / 2 ) , we have cos 300° = 1 /2 and sin 300° = - t / 3 / 2 . Therefore, we have
sin 300° _ - \ / 3 / 2 ___ ^
ta n 300c
cos 300° 1/2


We can use the unit circle to ,see’ trigonometric relationships.

Problem 18.17

Explain why sin(180° - 8 ) = sin 8 for any acute angle 8.

Solution for Problem 18.17: Let .4 be (1 ,0 ) and B be ( - 1 , 0 ) Let C be (^os 6», sin 9) so y
that C is 9 counterclockwise from A Let D be the point (co s(l8 0 - 9), sin(180 V)),
so D is 180° - 0 counterclockwise from A Because ( - 1 , 0 ) is 180° counterclockwise
from A and D is 180° - 0 counterclockwise from A , point D is 9 clockwise from ( - 1 ,0 ) ,
as shown in the diagram. We draw altitudes C X and D Y to the x-axis, since these lengths
equal s in 0 and s in ( 180° - 0), respectively. Because O D = O C = 1, we have
A O Y D = A O X C by SA Congruence for right triangles. This gives us C X - U Y . so
sin 0 = sin(180° — 9). □

With a little bit of casework, we can extend this proof to show that sin(180° - 8) = sin 8 for all angles 0, not just acute angles.
This relationship is not worth memorizing. After working with trigonometric functions and the unit circle more, this relationship,
and others like it, will become automatic to you. We'll be exploring how in later books in the Art of Problem Solvmg series.

t V
Problem 18.18

Show that for any triangle A A B C , we have [A B C ] — (A C ) ( B C ) sin C.

Solution for Problem 18.18: We consider separately the cases in which C is acute, right, or obtuse.

Case 7‫ ־‬Z C is acute. To find the area, we seek an expression for the altitude from A to B C .So, we draw A
altitude A X , forming right triangle A v 4 X C . From this triangle, we have s i n C = A X / A C . so
A X = A C s in C. Therefore, we have

[A B C ] = 1~{ A X ) { B C ) = i ( ^ C ) ( B C ) s i n C .

[A B C ] = \ { A C ) { B C ) . So, we have
Case 2: Z C is right. If Z C is right, then we have both s i n C - 1 and A!
.

J
[A B C ] C - { A C ) { B C ) sin C.

Case 3: A C is obtuse. If A C is obtuse, then altitude A X is outside A A B C , as shown at nghlW e A


have sin A A C X = A X / A C , so A X = A C sin A A C X . Because A A C X = 180 - A A C B ,
we have

[A B C ] = \ { A X ) ( B C )

= l ( A C ) ( B C ) sin A A C X
2
= ~ ( A C ) ( B C ) s in (l8 0 ° - A A C B ).
2

Because
sin( 180° - Z A C B ) = sin Z .A C B

for any angle Z .A C B , we have

[A B C ] = \ ( A C ) ( B C ) sin A A C B ,
A

as desired.
We have covered all three possible cases for Z C , so we have shown that

Using our unit circle, we can also evaluate trigonometric functions for angle meas‫ ״‬es y
greater than 360°, as well as for negative angle measures. For example, since going dbU
counterclockwise from ( 1 , 0 ) brings us back to ( 1 , 0 ), the point that is
360° 4- 60° = 420° counterclockwise from (1 ,0 ) at right is point B shown. From the
coordinates of 13, we see that cos 420° = 1 /2 and sin 420° = V 3 /2 ‫ ״‬To evaluate
trigonometric functions of negative angles, we go clockwise from ( 1 , 0 ) rather than
counterclockwise. So, because B is 300° clockwise from (1,0), we have
cos(—300°) = 1 /2 and s in (—300°) = > /3 /2 .

Exercises

Evaluate each of the following:

j | (a) sin 12 0 °
Show Solution
Type your solution, notes and/or work here.

(b) sin 330°


Show Solution
Type your solution, notes and/or work here.

(c) ta n 135°
Show Solution
Type your solution, notes and/or work here.

(d) cos ( - 4 5 ° )
Show Solution
Type your solution, notes and/or work here.

(e) cos 2 1 0 °
Show Solution
Type your solution, notes and/or work here.

(f) ta n 720°
Show Solution
Type your solution, notes and/or work here.

Explain why sin 2 0 -4- cos2 0 = 1 for any angle measure 9.


Show Solution
Type your solution, notes and/or work here.

lax 9 < 360° satisfy the equation sin 0 = 0.31?

Show Solution
Type your solution, notes and/or work here.

Explain why cos(360° + 8 ) = cos 8 for any angle 8.


Show Solution
Type your solution, notes and/or work here.

Explain why cos(180° - 8 ) = - cos 8 for any acute angle 8.

Show Solution
Type your solution, notes and/or work here.
18.3 Law of Sines and Law of Cosines
SAS Congruence tells us that if two sides of one triangle and the angle between them are equal to the corresponding sides and
angle of another triangle, then the two triangles are congruent. Therefore, the third sides of these two triangles have the same
length. In other words, if we know two side lengths of a triangle and the measure of the angle between those sides, then there is
only one possible length of the third side of the triangle. Hmmm... Can we find that length if we know the two side lengths and
the angle measure?
Similarly, ASA and AAS Congruence tell us that if two angles and a side of a triangle equal the corresponding angles and side of
another triangle, then the two triangles are congruent. In other words, if we know two angles and a side in one triangle, then there
is only one possible value for each of the remaining two side lengths. Can we find these two side lengths if we know the angles
of the triangle and the other side length?

In this section, we use trigonometry to answer both of these questions.

Note: you can use your calculator for the problems in this section.

P roblem s
4 Jump to Solution
Problem 18.19

In A A B C , let A C = 15, B C = 12, and Z C = 34°. In this problem, we find A B to the nearest hundredth.

(a) Draw altitude ‫־‬B X from B to A C . Find C X and B X to the nearest hundredth.

(b) Find X A to the nearest hundredth.

(c) Use parts (a) and (b) to find A B to the nearest hundredth.

4 Jump to Solution
Problem 18.20

Let A A B C be an acute triangle with a = B C , b = A C , and с = A B . In this problem, we prove that

c2 = a2 4 b2 — 2 ab cos C.

(a) Draw altitude ‫־‬B X from В to ~AC. Express B X and C X in terms of a, b, and/or trigonometric functions of Z C .

(b) Express A X in terms of a, b, and/or trigonometric functions of Z C .

(c) Use Л A B X to show that c 2 = a 2 4‫ ־‬b2 - 2abcosC .

(d) What happens in the equation c2 = a2 4 b2 - 2 ab cos C if Z C is a right angle?

4 Jump to Solution
Problem 18.21

In A P Q R , suppose P R = 12, Z Q P R = 66 °, and Z P R Q = 63°. In this problem, we find P Q and QR.

(a) Let T be the foot of the altitude from P to Q R . Find P T to the nearest hundredth.

(b) Use P T to find P Q to the nearest hundredth.

(c) Find Q R to the nearest hundredth.

4 Jump to Solution
Problem 18.22

Suppose that A A B C is an acute triangle with a = B C , b = A C , and c = A B . Prove that

a
sin A sin В sin C

4 Jump to Solution
Problem 18.23

Find B C , C l) , and B D to the nearest tenth in the diagram at right.

t V
Problem 1 8 . 1 9 _______________________________________________ ‫־‬

In А Л /iÇ J e t A C = J 5 , B C = 12, and Z C = 34°. Find .4 D to th e nearest hundredth.

Solution for Problem 78.79: It’s not immediately obvious how to find A B . We know how to use b
trigonometry to find side lengths in right triangles, so we start by drawing an altitude from B to A C
as shown. This creates right triangle A B C X with the 34° angle as one of its acute angles. From

right triangle A C B X , we have = sin C ~ 0.559, so B X ~ 0.559(Z ?C ) ~ 6.71.

Similarly, we have C X / B C = cos C ~ 6.829, so C X % 9.95.

This doesn’t tell us A B yet, but we now have the length of one leg of A B X A. If we can find the
other, we can use the Pythagorean Theorem to find A B . Fortunately, X ; 4 is easy to find:

X A = A C - C X « 5.05.

Now, we can use the Pythagorean Theorem to find

A B = y / B X * + X A * » 8.40.

UJ

In Problem 18.19, we were given two side lengths of a triangle, and the measure of the angle between these two sides. We then
found the third side. There was nothing particularly special about the side lengths or the angle. We might be able to follow
essentially the same process for any triangle. Let’s give it a try.

,Problem 1 8 . 2 0 ___________________________________________________________________________________________ * V 1

Let A A B C be an acute triangle with a = B C , b = A C , and c = A B . Suppose we know a, b, and Z C . Find a formula that |
we can use to find c. __ __ __ __

Solution for Problem 78.20: We use Problem 18.19 as a guide. In fact, this problem is essentially the
same as Problem 18.19, but with variables a, b, and Z C in place of the numbers that were given in
that problem. We can use the same diagram, and we’ll use the same steps. We draw altitude B X
from B to .4C . Then, we have s in C = B X / B C , so B X = B C sin C = a sin C . We also have
cosC = C X /B C , so C X = B C cos C — a cos C . Therefore, we have
A X = A C — C X = b — a cos C . Next we apply the Pythagorean Theorem to A A B X to find
A B 2 = B X 2 4 A X 2. Substituting our expressions for these three sides gives us

c? = a 2 s in 2 C 4 (b - a cos C ) 2
= a 2 s in 2 C 4 b2 - 2ab cos C 4 a2 cos2 C
= a 2 (s in 2 C 4 cos2 C ) 4 b2 — 2ab cos C .

Since sin 2 C 4 cos2 C = 1, we have


c2 = a2 4 b2 - 2ab cos C .

We can test our new formula with the data from Problem 18.19.

Check formulas you prove by trying them on specific examples you have solved
Concept:
without the formula.

In Problem 18.19, we have« = 12, 6 = 15, and Z C = 34°, so we have

c2 = a 2 4 b2 — 2abcos C « 70.546.

Taking the square root of both sides gives c ~ 8.40, which agrees with our answer from Problem 18.19.

With a little more casework (which you’ll supply as an Exercise), we can show that this equation holds for any triangle A B C , not
just for acute triangles.

Important: Let a = B C , b = A C , and c = A B in A A B C . The Law of Cosines states that

Z c2 = a 2 4 b2 — 2 ab cos C.

i-----— ---------------------------------------------------------------------------------------------------------------------------------------
Notice that when Z C = 90° in the Law of Cosines, we have c o s C = 0, so the law becomes c2 = a 2 4 b2, which is just the
Pythagorean Theorem.
Problem 18.19 is just a specific example of Problem 18.20, so we call Problem 18.20 a generalization of Problem 18.19.

Concept: Whenever we have to prove a general statement like the Law of Cosines, it's often
useful to first consider a specific example of the general statement. If we can solve a
specific example, we can sometimes use that solution as a guide to prove the
generalization.

Now that you’ve learned the Law of Cosines, you’re probably thinking there must be a Law of Sines, too. Of course, you’ve read
the section title, so you know you’re right. We’ll discover the Law of Sines the same way we discovered the Law of Cosines -
we’ll start with a specific example.

Problem 1 8 . 2 1 ____________________________________________________________________________________________ * v

Suppose P R = 12,1'.Q P R = 66 °, and A P R Q = 63° in A P Q R . Find P Q and R Q to the nearest hundredth.

Solution for Problem 18.21: We start by building a right triangle, since we know how to use
trigonometry to find lengths of sides in a right triangle. We draw altitude P I to side Q R . From right
triangle A P T R , we have P T / P R = s in R, so P T = P R s in R « 10.69. We also have
T R / P R = cos R, so T R = P R cos R ~ 5.45. Now, we can use right triangle A P Q T to find
lengths Q T and PQ. First, we find that Z Q = 180° — 66° — 63° = 51°. We have
P T / P Q = s in Q, so we have P Q = Р Т / (s in Q ) % 13.76. We also have P T / Q T = ta n Q.
so Q T = P T /( t a n Q ) ~ 8 .6 6 . Finally, we have Q R = Q T 4 T R zz 14.11. □

Let’s try using our approach in Problem 18.21 to produce another law.‫׳‬

t V
Problem 1 8.2 2 ~ ~ ~ ~ ~ — — — ‫־‬

Suppose that A A B C is an acute triangle with a = B C , b = A C , and c = A B . Prove that |


a _ b _ c |
- — — — — — __ __ _ _ _ _ s \n A __s i n B __ sin _ I

Solution for Problem 18.22: We use Problem 18.21 as a guide. We draw altitude i4 T to B C . From right
triangle A A T C , we have s in C = A T /A C , so A T = A C s m C = b s in C . From right triangle
A A B T , we have s in B = A T /A B , so A T = A B sin B = c s in B . These two expressions for
A T must be equal, so we have b sin C = c s in B. Dividing this equation by s in B and by s in C, we
have

s in B s in C

We can follow essentially the same steps starting with the altitude from C to A B to show that b / (sin B ) = a /(s in ^4). So, we
have
a
sin A sin В sin C

Important: Let a = B C , b = A C , and c = A B in A A B C . The Law of Sines states that

a b _ c
Z sin A sin В sin C

I--------------------------------------------------------------------------------
As with the Law of Cosines, you’ll tackle the cases in which A A B C is right or obtuse in the Exercises.

The Law of Sines gives us another look at why 'SSA Congruence‫ ׳‬is not a valid
theorem to prove triangle congruence. Suppose we have Z .4 = 30°, B C = 3, and
A C = 4 in A A B C . The Law of Sines then gives us

BC _ AC
sin A sin В ’

from which we find s in В = ( A C / B C ) s in A = 2 /3 .

Next, suppose we have Z D = 30°, E F = 3, and D F = 4 in A D E F . Again, the


Law of Sines gives us
EF _ DF
s in D s in D '

from which we find sin E = ( D F / E F ) sin D = 2 /3 . So, we have


sin В = sin E — 2 /3 . Do we have A B = Z D ?

Not necessarily! There are two angles 9 for which


0° < 0 < 180° and s in 9 = 2 /3 . The two intersections of (0 , 1 )
the graph of у = 2 / 3 with the unit circle correspond to -
these two angles; these are shown at right. So, we cannot
deduce that Z B = Z E , and therefore we cannot conclude
that A A B C = A D E F .

Let's give the Law of Cosines and the Law of Sines a try:

t V
P roblem 1 8 . 2 3 __________________________ ,

Find B C J J D, and B D tothejiearest tenth in the diagram at right

65'> D

Solution for Problem 18.23: In A A B C , we know two side lengths and the angle between these sides, so we can use the Law of
Cosines to find B C . This gives us
B C 2 = 82 4 122 - 2 (8 )(1 2 ) cos 66° « 129.9,

so B C « 11.4. Turning to A B C D , we have


Z B C D = 180° - 65° - 85° = 30°.

Now, we know one of the side lengths of A B C D and all of the angle measures of A B C D , so we can use the Law of Sines to
find the missing side lengths. Specifically, we have
BC BD CD
s in D s in Z B C D s in Z C B D

which means we have


11.4 BD CD
s in 65° s in 30° s in 85°

Solving for B D , we have

B D = Z Z L . s in 30° « 6.3,
s in 65°

and solving for C I ) gives

C D = - î ilL • s in 85° ^ 12.5.


sin 65°

Exercises

Find the missing sides below to the nearest tenth,

(a)

Show Solution
Type your solution, notes and/or work here.

Show Solution
Type your solution, notes and/or work here.

Find the missing sides below to the nearest tenth,

(a)

Show Solution
Type your solution, notes and/or work here.

Show Solution
Type your solution, notes and/or work here.

In this problem, we prove that the Law of Cosines works when the given angle is obtuse. Let A A B C be an obtuse triangle
with Z C > 90°. Let A B = c, C A = b, and B C = a, as usual.

(a) Let D be the foot of the altitude from A to f ô . Find A D , B D , and C D in terms of a. b, c, and trigonometric functions
of Z C .
Show Solution
Type your solution, notes and/or work here.

(b) Show that c 2 = o 2 4 b2 — 2ab cos C.


Show Solution
Type your solution, notes and/or work here.

IC aw of Sines hold! U l I V VI «V4.

Show Solution
Type your solution, notes and/or work here.

A surveyor is 3 kilometers from the base of a mountain. The mountain's face slopes up at an angle of 30° from horizontal. If
..---------
the surveyor- rv,ooe,‫״‬-oc
measures in Hpnrpps from horizontal to the top of the mountain, approximately how tall is the mountain to the
1 0 degrees from horizontal to the top

nearest hundredth of a kilometer?


Show Solution
Type your solution, notes and/or work here.

Let A A B C be a triangle with B C = a, A C = b, A B = c, and with circumradius R.

(a) Draw A A B C and its circumcircle. Let point D be on the circumcircle such that A D is a diameter. Find sin Z A D B in
terms of the side lengths of A A B D .
Show Solution
Type your solution, notes and/or work here.

(b) What angle of A A B C equals Z A D B , and why?


Show Solution
Type your solution, notes and/or work here.

(c) Show that


a
= 2 R.
sin A sin В sin C

This is the Extended Law of Sines.


Show Solution
Type your solution, notes and/or work here.

Suppose A C = 5, B C = 6, and A B = \/3 T . Find Z B C A .


Show Solution
Type your solution, notes and/or work here.
18.4 Summary
We write the cosine of an angle 9 as cos 9 and the sine of 9 as sin 9. We define sine and cosine using the unit circle, which is the
circle on the Cartesian plane with radius 1 centered at the origin.

Important: Let point ;4 be (1 ,0 ) and point B be on the unit circle such that B is 9 degrees
■y counterclockwise from A. Then, we define cos 9 and sin 9 to be the ^ ‫־‬coordinate and
;*/-coordinate, respectively, of point B.

sin 9
We write the tangent of an angle 9 as ta n 9, and define it as ta n 9 = ----- -. Sine, cosine, and tangent are examples of
cos 9
trigonometric functions.

For an acute angle Z A of a right triangle A A B C , we have:

length o f leg opposite A a


s in A
length o f hypotenuse c
length o f leg adjacent to ;4 b
cos A
length o f hypotenuse c
length o f leg opposite A a
ta n .4
length o f leg adjacent to ,4

We proved several trigonometric identities, which are statements involving trigonometric functions of angles that are true for all
angles. The most important is:

Important:
s in z A + cos2 .4 = 1

While we proved several other identities, it is more important to understand how these identities are derived than to memorize all
the identities.

We then used trigonometry to find three important triangle relationships:

Important: Let B C = a, A C = b, and A B = c in A A B C . Then, we have

. [A B C \ = l o f t sin C‫׳‬.

■ c2 = a 2 + b2 — 2ab cos C. This is called the Law of Cosines.

a c
. This is called the Law of Sines.
sin A sin B sin C

P roblem Solving S trateg ies

Concepts:
If we have the length of a side of a right triangle and the value of a trigonometric
function of either of the acute angles of the triangle, then we can find all the side
lengths of the triangle.

Check formulas you prove by trying them on specific examples you have solved
without the formula.

Whenever we have to prove a general statement like the Law of Cosines, it's often
useful to first consider a specific example of the general statement. If we can
solve a specific example, we can sometimes use that solution as a guide to prove
the generalization.
R eview P roblem s

1 8 .2 4 : t V

ln A A B C at right, find s in A, cos A and ta n Æ

Type your solution, notes and/or work here. Show Solution

1 8 .2 5 : t V

In A P Q R , let Z Q = 90°, Z P = 71°, and P R = 16. Find P Q and Q R to the nearest tenth.

Type your solution, notes and/or work here. Show Solution

1 8 .2 6 : t V

Show that if A X Y Z is a right triangle with Z X = 90°, then ( ta n Y ) ( ta n Z ) = 1.

Type your solution, notes and/or work here. Show Solution

1 8 .2 7 : t V

In A P Q R , we have Z Q = 90° and s in P = 1 /4 . Find sin R.

Type your solution, notes and/or work here. Show Solution

1 8 .2 8 : t V

Evaluate each of the following:

(a) s in 135°

Type your solution, notes and/or work here. Show Solution

(b) s in 630°

Type your solution, notes and/or work here. Show Solution

(c) c o s ( - 12 0 °)

Type your solution, notes and/or work here. Show Solution

(d) cos 315°

Type your solution, notes and/or work here. Show Solution

(e) ta n 300°

Type your solution, notes and/or work here. Show Solution

(f) ta n 150°

Type your solution, notes and/or work here. Show Solution

1 8 .2 9 : t V

The angle of elevation is the angle above the horizontal at which a viewer must look to see an object that is higher than the
viewer. Similarly, the angle of depression is the angle below the horizontal at which a viewer must look to see an object that is
below the viewer.

Object Horizontal
Viewer•
Angle of Depression

Angle of Elevation
Viewer•
Horizontal O bject

Answer each of the following problems to the nearest foot.

(a) A surveyor measures the angle of elevation from her feet to the top of a building as 5°. The surveyor knows that the
building is 500 feet tall. Assuming the ground is flat and level between the surveyor and the building, how far away is the
surveyor from the building?

Type your solution, notes and/or work here. Show Solution

(b) l‫׳‬m standing at the peak of a mountain that is 14,000 feet above sea level. The angle of depression from this peak to a
nearby smaller peak is 4°. On my map, these two peaks are represented by points that are 1 inch apart. If each inch on
my map represents 1.2 miles, and there are 5280 feet in a mile, then how many feet above sea level is the second peak?

Type your solution, notes and/or work here. Show Solution

(c) ★ A bee is on a hill looking at a building. The building is 400 feet tall. The angle of elevation from the bee to the top of the
building is 4° and the angle of depression from the bee to the bottom of the building is 2°. What is the shortest distance
the bee will have to fly to reach the building?

Type your solution, notes and/or work here. Show Solution

1 8 .3 0 : t V

Let 0 ! and 0-2 be the two values of 9 such that 0 ° < 9 < 180° and sin 9 = 0.48. What is 9\ 49 ‫־‬p.

Type your solution, notes and/or work here. Show Solution

1 8 .3 1 t V

Suppose that A B = 7, B C = 8 , and / . A B C = 45°. Find [A B C ].

Type your solution, notes and/or work here. Show Solution

1 8 .3 2 : t V

If ta n 9 is negative, then which quadrant(s) could 9 possibly be in?

Type your solution, notes and/or work here. Show Solution

1 8 .3 3 : t V

Recall that sec x = -------- . Show that for any angle x for which cos x ^ 0, we have t a n 2 x 4 1 ‫ = ־‬see2 x.
cos x

Type your solution, notes and/or work here. Show Solution

1 8 .3 4 : t V

Two strips of width 1 overlap at an angle of a as shown. Show that the area of the overlap (shown
shaded) is l / ( s i n a ) .

Type your solution, notes and/or work here. Show Solution

1 8 .3 5 : t V

Point A is on the unit circle in the first quadrant such that A is 9 degrees counterclockwise from (1,0). Point B is on the pr­
axis such that A B is tangent to the unit circle. Show that ta n 9 = A B . (Do you now see why we use the name 'tangent' for
this trigonometric function?)

Type your solution, notes and/or work here. Show Solution

1 8 .3 6 : t V

Find the missing side lengths in the triangles below to the nearest tenth.

(a)

Type your solution, notes and/or work here. Show Solution

(b)

Type your solution, notes and/or work here. Show Solution

1 8 .3 7 : t V

Find the missing side lengths in the triangles below to the nearest tenth.

(a)

Type your solution, notes and/or work here. Show Solution

(b)

Type your solution, notes and/or work here. Show Solution

1 8 .3 8 : t V

If I walk north 3 miles, turn 32°, then walk another 2 miles, how far will I be from my initial starting point (to the nearest 0.1
mile)?

Type your solution, notes and/or work here. Show Solution

1 8 .3 9 : t V

What's wrong with the following proof of the Pythagorean Theorem:

In the text, we proved that if a = B C , b = A C , and c = A B , then c2 = a 2 4‫ ־‬b2 — 2ab cos C. If Z C = 90°, then
we have cos C = 0. Therefore, if Z C = 90°, we have c 2 = a 2 46 2 ‫־‬.

Type your solution, notes and/or work here. Show Solution

In A P Q R , we have P Q = 3, Q R = 5, and P R = 6 .

(a) Find cos P.

Type your solution, notes and/or work here. Show Solution

(b) ★ Find sin Q.

Type your solution, notes and/or work here. Show Solution


C hallen g e P roblem s

18.41 t V

Show that if the sides of A A B C have lengths a, b, and c, and A A B C has circumradius R, then

abc
[A B C ] =
4R

Type your solution, notes and/or work here. Show Solution

1 8 .4 2 : Source: AHSME t V

Two rays with common endpoint O form a 30° angle. Point A lies on one ray, point B on the other ray, and A B = 1. What is
the maximum possible length of O B ?

Type your solution, notes and/or work here. Show Solution

1 8 .4 3 : Source: AHSME t V

How many triangles have area TO and vertices ( —5, 0), (5,0), and (5 cos#, 5 s in # ) for some angle #?

Type your solution, notes and/or work here. Show Solution

Four congruent 3 0 9 0 ‫־‬60‫ ־‬triangles are constructed on the sides of a square as shown at right. The
hypotenuse of each of these triangles has length 2. The outer vertices of these triangles are
connected as shown to form quadrilateral A B C D . What fraction of A B C D is shaded?

Type your solution, notes and/or work here. Show Solution

1 8 .4 5 : Source: AMC 12 t V

A circle centered at O has radius 1 and contains the point A Segment A B is tangent to the circle at A and Z A O B = #.
Suppose point C is on O A such that B C bisects /.A B O . Show that O C = 1 /(1 4‫ ־‬sin #).

Type your solution, notes and/or work here. Show Solution

1 8 .4 6 : Source: AHSME t V

If sin x = 3 cos x, then what is (sin a*)(cos a*)?


Type your solution, notes and/or work here. Show Solution

1 8 .4 7 : t V

Line £ intersects the ;r-axis at an angle of 50° Line k makes an angle of 140° with the :r-axis. The intersection of £ and k has a
/;‫־‬coordinate of 10. Find all possible values of the distance between the ^ ‫־‬intercepts of £ and k.

Type your solution, notes and/or work here. Show Solution

1 8 .4 8 : t V

Right triangle A B C has inradius 1 and sin A = 12/13. Find the length of the hypotenuse of A B C .

Hint

Type your solution, notes and/or work here. Show Solution

1 8 .4 9 : t V

What's wrong with this proof of AA Similarity:

Suppose we have A A B C a n d A D E F w ith / A = Z D and Z B = Z E . The Law of Sines gives us

BC AC . EF DF
and
sin A sin В sin D sin E '

Therefore, we have B C / A C = (sin A ) / (sin B ) and E F / D F = (sin D ) / (sin E ). Since Z A = Z D and


Z B = Z E , we have

(sini4)/(sin B ) = (sin B)/(sin E ).

So, we have

B C _ sin A _ sin D _ E F
AC sin B sin E DF'

Because Z A = Z D and Z B = Z E , we have Z C = Z F . We can follow essentially the same steps as above to
deduce

BC : AC : AB = E F : D F : DE.

Type your solution, notes and/or work here. Show Solution

1 8 .5 0 ★ : Source: AHSME t V

Points A, B, C, and D are on a circle of diameter 1, and X is on diameter A D . Suppose


B X = C X and % /B A C = Z B X C = 36°. Show that

A X = (cos 6 ° )(s in 12° ) /( s in 18°).

Hint

Hint

Type your solution, notes and/or work here. Show Solution

1 8 .5 1 ★ : t V

An object moves 8 cm in a straight line from A to B, turns at an angle of a, where 0° < a < 180°, and then moves 5 cm in a
straight line to C. What is the probability that A C < 7?

Hint

Type your solution, notes and/or work here. Show Solution

1 8 .5 2 ★ : Source: AMC 12 t V

Equilateral triangle A B C has been creased and folded so that vertex A now rests at A ' on B C as /1
A
shown. If B A ' = 1 and A 'C = 2, then what is the length of crease P Q ?

Hint

Hint В

Type your solution, notes and/or work here. Show Solution


Archimedes' Circles

Let no one ignorant of geometry enter here. - Inscription above Plato's Academy

CHAPTER
I
I Problem Solving Strategies in Geometry
In this chapter we review many of the most powerful geometry problem-solving strategies we have learned in the text by tackling
some challenging problems with them. We also apply many of our geometric tools to proofs that require multiple insights.

This chapter is meant to extend the lessons of this book, so many of the Exercises and Challenge Problems in this chapter are
significantly more difficult than most of the problems elsewhere in this text. One excellent strategy for mastering the problems
you cannot solve on your first try is to review the solutions, then try the problems again on your own a few days later.

19.1 The Extra Line


We cleverly added extra lines to diagrams to solve various problems throughout this book. In this section we reinforce the most
common indications that extra lines might be helpful.

P roblem s

One very common type of ‫׳‬extra line we draw is a perpendicular from a point to a line. Usually, our goal in doing so is to build a
useful right triangle. Here's a classic example.

Solution for Problem 79.7: Circles + tangent lines = draw a radius to a point of
b
tangency. We draw radius O E because O E _L A B . Since the diagonals of a rhombus
are perpendicular and bisect each other, А Л О В is a right triangle with legs of length
O B = 8 and О A = 15. Therefore, A B = 17. From here, we can solve the problem
in several ways. Here are a couple:

Solution 7: Use similar triangles. Lots of right angles usually means there are similar
triangles. Here, we have

А АО В ~ A A E O ~ A O E B .

Therefore, O E / O A = O B /A B , so

O E = ( O A ) ( O B ) / A B = 120/17.

Solution 2: Use area. Since

[A O B ] = ( A O ) ( O B ) / 2 = ( O E ) ( A B ) / 2,

we have

O E = ( A O ) ( O B ) / A B = 120/17.

(Yeah, I like the area approach better, too.) z

Concept: When in doubt, build right triangles. One very common way to do so is to draw radii to
I points of tangency. Nearly always try this when you have circles and tangents in a
problem.

Solution for Problem 19.2: We have a problem about area, and we unfortunately can't find anything about [B D E \ easily. We do,
however, know how to relate [A M C \ and \B M C \ to [A B C ]. Therefore, we connect M to C. (We might a so have thought to do
this because C M is a median, and we know something about medians.)

Since C M is a median of A A B C , we have [B M C ] = [A M C] = 12. Now we look for ways


to relate either of these to what we want, [B E D ]. Triangles A B E D and A B M C overlap.
When we remove A B D M from both, we are left with two triangles that share side D M :
A E D M and A C D M . Since D M | E C , the altitudes to side D M of these triangles are the
same. Therefore, [E D M ] = [ C D M , which means [B E D ] = [B M C ] = 12.□

Concept: Connecting points that are originally not connected in a diagram can be extremely
I useful! This doesn't mean you should connect everything in your diagram
' immediately, however. Look for segments to draw that will be helpful, particularly
those connecting important points, or those that form segments, triangles, or angles
you know something about immediately.

Solution for Problem 19.3: We start by noting that

Z D = 360° - 120° - 90° - 90° = 60°.

Next, we might think to draw B D , since that will give us a couple right triangles. However, we only have one side of those two
right triangles, and we don't know anything about the acute angles of them. We'd like to make the 120° and 60° angles useful.
This gets us thinking about 3 0 9 0 ‫־‬60‫ ־‬triangles.

With a right angle at / . B A D and a 60° angle at Z D , we see that extending D C past C and A B past B
t
to meet at point E will give us a 3 0 9 0 ‫־‬60‫ ־‬triangle. Moreover, since / C B E = 180° — 120° = 60° and
/ B C E = 90°, A C B E is also a 3 0 9 0 ‫־‬60‫ ־‬triangle. Now we can solve the problem.

From 3 0 9 0 ‫־‬60‫ ־‬A C B E we have B E = 2 B C = 8 and E C = B C \/3 = 4 \/3 . From A D A E we have

D E = A E (2 /y /3 ) = ( A B + B E ) ( 2 / V 3 / 3 / \ 2 2 = (3‫׳‬.

Therefore, C D = E D — E C = 1 0 \/3 /3 . □

Concept: 60°, 30°, and even 120° angles are often good clues to build 3 0 9 0 ‫־‬60‫ ־‬triangles by
! dropping altitudes or extending segments.

Problem 19.4 Source: AMC12 t V

| A B is tangent at A to the circle with center O, point D is interior to the circle, and D B intersects the
circle at C. If B C = C D = 3, O P = 2, and A B = 6 , then find the radius of the circle. ____ I

Solution for Problem 19.4: We have a problem with segment lengths and a circle, so we think of Power
of a Point. We have a tangent, but only part of a secant. We continue B D past D until it hits the circle
at E . We do this not only because it lets us use Power of a Point, but because B D seems to end
rather abruptly in the middle of the circle. Segments that seem to end suddenly in a diagram (i.e. ones
that, if continued, will hit an important circle or segment) are often candidates to be extended. The
power of point B gives us

( B C ) ( B E ) = B A 2,

and substitution gives

3(3 4‫ ־‬C E ) = 36.

Therefore, C E = 9. Here are a couple ways we can finish:

Solution 1: Build right triangles. We know the length of chord C E and we'd like to find the length of a
radius. We draw radius O C and drop a perpendicular from O to C E . This gives us a couple right
triangles in A O D E and A O C F . Since O F is part of a radius that is perpendicular to chord C E , it
bisects C E . Therefore, C F = 9 /2 and D F = C F — C D = 3/2. So,

OF = VOD2 - D F2 =
z

Finally, we have

OC = V O F 2 + F C 2

Solution 2: Keep doing what worked. Extending a line that suddenly stopped worked once,
so try it again! O D stops rather suddenly. So, we extend it in both directions to hit the
circle a t ./ and K , as shown. Now, we have two intersecting chords, so we can use Power
of a Point! The power of point D gives

(J D )(D K ) = (C D )(D E ).

If we let the radius of the circle be r, we have

( r + 2 ) ( r — 2) = (3 )(9 — 3).

Solving for r, we find r = V22. □

Concept: Segments that stop suddenly inside figures (particularly triangles, quadrilaterals, or
circles) are great candidates to be extended.
1

Concept: If you successfully use a certain tactic in a problem to get some information, but you
still haven’t solved the problem, try using that same tactic again in a different way.
1 Maybe it still has more information to give!

Notice that our path to the solution is easy to see in the final diagram of the last solution. This is because the side lengths are
labeled.

Concept: Label lengths in your diagram as you find them, even if you have to label them in
terms of an important variable.

Exercises

1 9 .1 .1 : Source: ARML t V

A square is inscribed in a circle of radius 1 as shown at right. Circles © P and © Q are the largest
circles that can be inscribed in the indicated segments of the circle. The segment joining the centers
of circles P and Q intersects the square at A and B. Find A B .

Hint

Type your solution, notes and/or work here. Show Solution

1 9 .1 .2 : t V

In the diagram at right, a quarter-circle centered at one vertex of the square connects two other
vertices of the square. A small circle is tangent to the large circle and to two sides of the square as
shown. Each side of the square is 4 units long. What is the radius of the small circle?

Hint

Type your solution, notes and/or work here. Show Solution

1 9 .1 .3 : Source: MATHCOUNTS t V

A round table is pushed into a corner as shown in the diagram. Point A is on the outer edge of the
table and is 2 inches from one of the walls. Given that the radius of the table is 37 inches, how many
inches is point A from the other wall?

Hint

Type your solution, notes and/or work here. Show Solution

1 9 .1 .4 ★ : t V

W X Y Z is a parallelogram, B is the midpoint of X Y , and A is on W & such that Z A _L B A . %


Prove A Y = Y Z .

Hint

Type your solution, notes and/or work here. Show Solution


19.2 Assigning Variables
In many problems, finding the answer is not simply a matter of performing routine calculations. Often we need to discover
relationships between angles or between lengths. Usually, these relationships will be equations of some sort. One very helpful
way to keep track of this information is to assign variables to lengths and/or angles. We can then write equations that are easy to
read, and we can use the variables to keep track of the information in our diagram.

As you'll see, we often use this strategy together with our ,extra line' tactic from the last section, particularly when we build right
triangles to use the Pythagorean Theorem.

P roblem s
Problem 19.5 4 Jump to Solution

In the figure at right, A D = A E , and F is the intersection of E l!) with the bisector of Z C . If Z B = 36° /1
, how many degrees are there in Z C F E ?

BL

Problem 19.6 Source: AMC 10 4 Jump to Solution

Points A, B, C, and D lie on a line, in that order, with A B = C D and B C = 12. Point E is not on the line, and
B E = C E = 10. The perimeter of A A E D is twice the perimeter of A B E C . Find A E .

Problem 19.7 4 Jump to Solution

W X Y Z is a rectangular sheet of paper with W X = 10 and X Y = 12. The paper is folded so that W coincides with the
midpoint of X Y . What is the length of the fold?

Variables are great help in angle-chasing problems. As you might guess, the fact that the angles of a triangle add to 180° is our
most commonly used equation-forming tool. Try assigning variables and chasing angles in this problem.

Problem 19.5 t V

In the figure at right, A D = A E , and F is the intersection of E ] 3 with the bisector of Z C . If Z B = 36°
, how many degrees are there in Z C F E ? yr

Solution for Problem 19.5: We can't deduce the measures of any more angles directly from
Z B = 36°, so well have to use relationships among the angles to learn more. We could
start off by writing a bunch of equations like:

Z B 4- Z A C B + Z A = 180°
ZADE = ZAED
ZED B + Z D B C + ZBCE +Z D E C = 360°,

but there are so many different angles that it's hard to keep track of them all. Instead, we
assign variables. We assign a variable to what we want, Z C F E = x, and we also pick
variables for angles that are easy to relate to other angles in the diagram. We are given that
A A D E is isosceles, so we let Z A D E = ZuAED = z. Similarly, our angle bisector gives
us Z A C F = Z F C B , so we call these both y.

Now we look for other angles we can label in terms of x, y, or 2 . Triangle A A D E gives us Z A = 180° — 2z. Then, A A B C
gives us Z A 4‫ ־‬Z B A Z A C B = 180°, so

180° - 2 z A 36° A 2y = 180°.

Therefore, y = z — 18°.

Now we can reduce the number of variables in our diagram. If we don’t see our path to solution
A
now, we can redraw the diagram using only x and z. Since Z F E A is an exterior angle of
A F E C , we have

ZC FE A ZFC E = ZDEA.

Therefore,x A z — 18° = 2, so x = 18°.

We could also have written Z D E C = 180° — 2 in our diagram, then used the sum of the
angles of A F E C to solve the problem. □

Concept: Assign variables to important angles and use your mastery of basic geometry to
I express other angles in terms of those variables. Typically, you should assign
' variables to angles you seek and to angles you know a lot about first. Once you start
chasing angles, reach for all the angle tools you know, most notably the facts you
know about triangles, straight lines, parallel lines, and angles that intersect circles.

We can also chase lengths with equations. Our main tools here are the Pythagorean Theorem, similar triangles, and Power of a
Point. As we’ve seen throughout this book, the Pythagorean Theorem is the most heavily used of these three for finding lengths.

Problem 1 9.6 Source: AMC 70 t V

Points A, B, C, and D lie on a line, in that order, with A B = C D and B C = 12. Point E is not on the line, and
B E = C E = 10. The perimeter of A A E D is twice the perimeter of A B E C . Find A E .

Solution for Problem 19.6: We start with a diagram. We draw everything we are
t
given in the problem. Then we look for more lengths we can find in the diagram
and were almost immediately stuck. So, we start assigning variables. Since we
want A E , we let A E = x and look for other sides we can express in terms of x.

We suspect all these equal side lengths (A B = C D and B E = C E ) will


mean A E = D E , so we try to prove it. Since B E = C E , Z E B C = Z E C B .
Therefore, Z E B A = Z E C D , so A A B E = A D C E by SAS Congruence. So, we have D E = A E = x, too.

This isn't enough, so we look for more lengths we can express in terms of x. A B and C D are all we have left. We look back at
the problem for information we haven't used yet and see that bit about the perimeter of A A E D being twice that of A B E C .
From this, we have

A E -P E D A A B A B C A C D = 2(10 + 10 4- 12).

Substitution gives 2x 42 ‫ ־‬12 4 ‫־‬A B = 64, so A B = 26 — x.

Adding all this to our diagram, were still stuck. The isosceles triangles are a clue
to build right triangles and use the Pythagorean Theorem. We draw altitude E F ,
thereby building right triangles A E F C and A E F A . From A E F C we have
E F = 8 . Now we can build an equation for x by applying the Pythagorean
Theorem to A E F A . E F 2 A A F 2 = A E 2, so

64 A (32 — x ) 2 = x 2.

Therefore, 64 = x 2 — (32 — x ) 2. We factor the right side of this equation as the difference of squares to find:

64 = [x - (32 - x ))[x 4- (32 - x )] = (2 x - 32)(32).

Solving for x gives x = 17.

Make sure you follow that nifty use of the difference of squares factorization. This manipulation can often save you a few extra
steps of nasty algebra (and will reduce careless mistakes). z

Concept: If you need a length in a problem, try building right triangles. Some of the best tools
! for building right triangles are drawing altitudes of triangles and trapezoids, and
drawing radii to points of tangency.

Problem 1 9.7 t V

W X Y Z is a rectangular sheet of paper with W X = 10 and X Y = 12. The paper is folded so that W coincides with the
midpoint of X Y . What is the length of the fold?

Solution for Problem 19.7: We start with a diagram. Instead of just drawing the folded
paper, we also include the paper as it originally was, since we know so much about
rectangles. Our diagram shows W X Y Z , and the fold A B that leads to W
coinciding with W , the midpoint of X Y . Since A W A B is just the 'folded over' (i.e.
reflected) version of A W A B , we have A W A B = A W A B . We only initially know
X W = W 'Y = 6 and X W = Y Z = 10. Well have to assign some variables.
We let A X = x and B Z = y, so we then have A W = 10 — x and
W B = 12-t/.

We then use our congruent triangles to note that A W = A W = 10 — x and


W ' B = W B = 1 2 — y. Right triangle A X A W gives us an equation for x:

x2 4 1 0 ) = 62 ‫ — ־‬x ) 2.

Solving this equation for x gives x = 16/5.

Unfortunately, we don’t have a right triangle that quickly gives us y. So, we borrow a
tactic from last section and build one. We draw B C such that B C A X Y as shown.
Since B C Y Z is a rectangle, we have C Y = y, B C = 10, and C W = 6 — y. We
could use the Pythagorean Theorem as before on A W C B . However, we can find
the answer a little faster by noting that

Z C W B = 180° - Z A W B - Z A W 'X
= 90° - Z A W 'X
= Z X A W ‫׳‬.

Therefore, right triangles A W 'X A and A B C W are similar. So, we have

.X W * 4
W C ‫־־‬ BC ‫־‬

Substitution gives

16/5 6

t- 6 / ‫־‬ 10 ’

so y = 2/3.

Therefore, W B = B W '= 12 — y = 34/3. In right triangle A A W B, we now have B W = 3 4 /3 and


A W = 10 — x = 34/5, so we can find A B :

A D = y /A W ’2 + W B 2

34 \ / 34
‫־־‬ 15

Notice once again that we use a little algebraic manipulation, in this case factoring out the 34, to simplify our work, z

Exercises

1 9 .2 .1 : Source: AMC 12 t V

Triangles A B C and A B D are isosceles with A B = A C = B D , and B D intersects A C at E .


Given that B D J_ A C , find all possible values of Z C 4‫ ־‬Z D .

Type your solution, notes and/or work here. Show Solution

1 9 .2 .2 : Source: H M M T t V

A point on a circle inscribed in a square is 1 and 2 units from the two closest sides of the square. Find the area of the square.

Type your solution, notes and/or work here. Show Solution

1 9 .2 .3 : t V

A B C D is a rectangular piece of paper with A B = 8 and A D = 12. We fold the paper so that B coincides with D. What is
the length of the fold?

Hint

Type your solution, notes and/or work here. Show Solution

1 9 .2 .4 ★ : Source: Mandelbrot t V

3 B
Square A B C D has side length 1 . Point E is chosen on side B C so that A E 4- E B = —, and point F is
_____ —
chosen on side C D so that A F bisects Z D A E . Find D F .

Hint

Hint

Hint

Type your solution, notes and/or work here. Show Solution


19.3 Proofs
Geometry proofs present a challenge by offering us so many different avenues of exploration. Our task is to narrow down the
possible approaches to the ones that are most likely to bear fruit. Before trying the problems in this section, read through
Problem 13.10 again, noting particularly how we keep track throughout the problem of What We Know and What We Want. Then,
try applying this approach to these problems. We will also use some of our techniques from the first two sections of this chapter,
especially adding extra lines.

P roblem s
Problem 19.8 4 Jump to Solution

M and N are the midpoints of sides B C and C D , respectively, of square A B C D . A M and A N A b


meet B D at X and Y , as shown. Show that B X = X Y = Y D.

D
A - c

Problem 19.9 4 Jump to Solution

0^4 and © / i are externally tangent. D is on ©^4 and E on 0 / i such that is a


common external tangent of the two circles. © C is tangent to © A © ii, and D E
as shown. Given that a is the radius of © A b is the radius of Q B, and c is the
radius of © C , prove that

J_ _ J _ J_
y /c y /ü y /b

Problem 19.10 Source: Mandelbrot 4 Jump to Solution

In A A B C let L be the foot of the angle bisector from A to B C , and let M be the
midpoint of B C . We also draw the circumcircle of A A B C . Finally, we construct the
circle through points A, L, and M . This circle intersects the rest of the diagram in
several remarkable ways, so we label points Z, Y, and X where the circle meets A B ,
A C and the circumcircle of A A B C , respectively.

(a) Prove that B Z = C Y .

(b) Prove that A X B Z = A X C Y .

(c) Prove that X L is the diameter of the circumcircle of A A L M .

In some of the following problems, we will explore in search of the solution. After some of these problems, we will present brief
solutions based on our discoveries. The purpose of the exploration is to give you some insight into how an experienced geometer
thinks about problems. The purpose of the solutions following the exploration is to give you a feel for how to write up clear
solutions once you find them.

Solution for Problem 79.8: We make our list of what we know and what we want. We want to prove a statement about lengths, so
we start with information we know about lengths.

What We Know What We Want


AB = BC = CD = DA BX = X Y = YD
BM = M C = CN = DN

It sure seems reasonable that B X = Y D , but we have to prove it. Our main tool for proving segments equal is congruent
triangles, so we look for congruent triangles in our diagram. A A B X and A A D Y look congruent, so we go after these two
triangles.

Important: Draw precise diagrams. Triangle congruences, parallel lines, collinear points, and
much, much more stand out in precise diagrams.

Since A B C D is a square, we have A D = A B and Z A D B = Z A B D = 45°. We just need another angle to prove
congruence. The angles at X and Y don’t look easy to work with, but angles Z X A B and Z D A Y are also parts of right
triangles A B A M and A D A N . Since A D = A B and B M = D N (both are half the side length of the square), we have
j A B A M = A D A N by LL Congruence (or by SAS Congruence). So, we have

ZXAB = ZM AB = ZNAD = ZY AD,

which means A A B X = A A D Y by ASA Congruence.

We have more information for our table now. Most importantly, we've reduced our problem to just proving B X = X Y . We
quickly brainstorm for a few other statements that are equivalent to B X = X Y and we see that B Y = 2 B X or B Y = 2 D Y
will also give us what we want. Also, if we show B X = B D / 3 or X Y = B D / 3, we’ll be finished.

What We Know What We Want


AB = BC = CD = DA BX = XY
BM = M C = CN = DN B Y = 2B X
BX = YD B Y = 2D Y
AADY 2 AABX X Y = B D /Z
B X = B D /Z

This gives us a lot to shoot for. We don’t have any more congruent triangles that are interesting to investigate, but the ratios in
our ‫׳‬What We Want’ list suggest looking for similar triangles. Parallel lines mean similar triangles. Specifically, we have
A D Y N ~ A B Y A because D N || A B . Seeing B Y = 2 D Y in the ,What We Want’ list, we focus on what
A D Y N ~ A B Y A tells us about B Y and D Y :

DY DN C D /2 1
B Y ‫ ־־‬A B ‫־‬ AB 2 ‫־‬

We've found something we want! Now we retrace our steps to write a nice solution:

Since A B = A D , Z A B M = Z A D N and D N = B M (each is half the length of a side of the square), we have
A A D N = A A B M by SAS Congruence. Therefore,

ZD AY = ZD AN = ZM A B = ZXAB.

Together with A B = A D and Z A B X = Z Y D A (since each is 45°), we have A A B X = A A D Y by ASA Congruence.


Therefore, B X = D Y .

Because D N || A B , we have Z N D Y = Z Y B A and Z Y A B = Z Y N D, so A D Y N ~ A B Y A. Therefore,

D Y / B Y = D N / A B = ( C D / 2 ) / A B = 1/2.

Therefore, D Y = B Y j 2, so B X = B Y / 2 and X Y = B Y — B X = B Y / 2 also. Thus, B X = X Y = Y D. (See if you


can find other solutions!) □

Problem 19.9 t V

©^4 and © / i are externally tangent. D is on ©^4 and E on © / i such that D Ê is a


common external tangent of the two circles. © C ‫ ׳‬is tangent to © A Q B , and D E
as shown. Given that a is the radius of © A b is the radius of ©7i, and c is the
radius of © C ‫׳‬, prove that

J_ _ J _ J_
y /c y /a y /b ‘

Solution for Problem 19.9: There’s not much working backwards we can do here, but we can at least get rid of the fractions by
mutliplying our desired equation by yfabc. This gives

y fa b = y f b c ,+ y /a c .

Going forwards, we can use all the information we know about tangent circles and lines. We connect centers and draw radii. The
perpendicular lines thus formed and the square roots in our target equation both point in the same direction: the Pythagorean
Theorem. In looking for a good right triangle to build, we remember Problem 12.22, in which we built a right triangle to find the
length of a common tangent to two circles. Inspired by this, we draw a line through C perpendicular to A D and B E . Rather than
listing everything we know in a table, we add it to our diagram:

Concept: A large diagram in which we keep close track of what we discover is a very effective
I way to keep track of What We Know.

Now we have our right triangles. Since X D E C and C Y E F are rectangles, we have X D = C F = Y E = c. Therefore,
A X = a — c and B Y = b — c. Since A C = a + c, we can apply the Pythagorean Theorem to A A X C to find

XC = y /A C 2 - A X 2

= y j { * + c ) 2 ‫( ־‬a - c ) 2

= y/Aac
= 2 y/ac.

Our result is part of the equation we want to prove! Since X C F D is a rectangle, we have D F = 2 y/ac as well. Therefore, the
common external tangent of tangent circles with radii a and c has length 2 y/ac. We can apply this result to our other two pairs of
tangent circles to find F E = 2 \/b c and D E = 2 y/âb. Since D E = D F -h F E , we have

2 y/ab = 2 y/ac 4 2 ‫־‬y/bc.

Dividing this equation by 2 y/abc gives the desired

J_ _ J _ J_
y /c y /a . y /b *

Notice that we used both of our tactics from earlier in this chapter. We drew many extra lines to build right triangles and we
labeled lengths we could find in terms of our variables. As we did so, we found lengths that are in our expression we sought. This
brought us right to the solution. □

Concept: Always compare new problems to problems you have already solved. Problem 19.9 is
I very similar to Problem 12.22, in which we found the length of a common tangent.
' Thinking of this common external tangent problem gave us a quick path to the
solution of our new problem.

Problem 19.10 Source: Mandelbrot t V

In A A B C let L be the foot of the angle bisector from A to B C , and let M be the
midpoint of B C . We also draw the circumcircle of A A B C . Finally, we construct the
circle through points A, L, and M . This circle intersects the rest of the diagram in
several remarkable ways, so we label points Z, Y, and X where the circle meets A B ,
A C and the circumcircle of A A B C , respectively.

(a) Prove that B Z = C Y .

(b) Prove that A X B Z = A X C Y .

(c) Prove that X L is the diameter of the circumcircle of A A L M .

Solution for Problem 79.7 0:

(a) We don’t have an obvious pair of congruent triangles to go after to show that B Z = C Y . (We suspect that well need
B Z = C Y to tackle the triangles in the second part, so we don’t go after those triangles immediately.) We pull out our
other length tools that this diagram invites us to use. The circles suggest Power of a Point. Point B gives us
( B Z ) ( B A ) = ( B M ) ( B L ) and C gives ( C Y ) ( C A ) = ( C L ) ( C M ) . These equations include our target lengths! The
Angle Bisector Theorem gives us A B / B L = A C /C L , which has one of our target lengths, and the midpoint gives us
B M = C M . We put all this information together:

What We Know What We Want


(B Z )(B A ) = (B M )(B L ) BZ = CY
(C Y )(C A ) = (C L ) (C M )
A B /B L = A C /C L
BM = CM

Solving our first two equations in the What We Know column for B Z and C Y gives us B Z = ( B M ) ( B L ) / ( B A ) and
C Y = ( C L ) ( C M ) / (C A ). We’d like the expressions on the right sides of these equations to be equal:

What We Know What We Want


(B Z )(B A ) = (B M )(B L ) BZ = CY
(B M )(B L )/(B A ) =
(C Y )(C A ) = (C L ) (C M )
(C L ) ( C M ) / (C A )
A B /B L = A C /C L
BM = CM

Since C M = B M , our second line of what we want reduces to B L / B A = C L / C A . This is just the reciprocal of the
third line in What We Know! Now we have a path from What We Know to What We Want, so we can write our proof:

From the power of point B, we have ( B Z ) ( B A ) = ( B M ) ( B L ), or B Z = ( B M ) ( B L ) / ( B A ). The Angle


Bisector Theorem applied to bisector A L of A A B C gives B L / B A = C L / C A , so

B Z = ( B M ) ( B L / B A ) = (.B M ) ( C L / C A ).

B M = C M because M is the midpoint of B C , so B Z = ( C M ) ( C L / C A ) . Finally, the power of point C


gives us ( C Y ) ( C A ) = ( C L ) ( C M ) , from which we have C Y = ( C M ) ( C L / C A ) = B Z .

(b) The previous part gave us B Z = C Y , so if we find two pairs of equal y


corresponding angles in A B Z X and A C Y X , we will prove the triangles are
congruent.

We have circles, so we look for angles that are inscribed in the same arc. Angles
Z X B Z and Z X C Y are the same as Z X B A and Z X C A , respectively. These

two angles are both inscribed in X A of the larger circle, so we have


Z X B Z = Z X C Y . Unfortunately, we can’t do the same for any other pair of
angles of our triangles. We therefore start chasing angles, looking for a pair of equal
angles that we might relate to the angles in our triangles. We continue looking for
equal inscribed angles, and find Z A Z X = Z A Y X since both are inscribed in

A X of the smaller circle. We can quickly relate each to angles in our triangles:

Z B Z X = 180° - Z A Z X = 180° - Z A Y X = Z X Y C .

Therefore, by ASA Congruence, we have A X B Z = A X C Y . Notice how marking equal angles and sides as we find them
makes the congruent triangles stand out.

(c) We don’t have a lot of tools for proving a segment is a diameter of a circle. One v

possible approach is to show that one of the angles inscribed in either X Y L or

X Z L is a right angle, thus making the arc 180°. We consider the labeled points on
the small circle, looking for a point to be the vertex of our sought-after right angle.

Marking the equal sides given by A X B Z = A X C Y helps guide us. Our


congruent triangles tell us B X = C X , so A X B C is isosceles. Therefore,
midpoint M of B C is also the foot of the altitude from X to B C . Since Z X M C is

a right angle, Z X M L = Z X M C , and Z X M L is inscribed in X Y L , we have

X Y L = 180°. Therefore, X L is a diameter of the small circle.

As an Exercise, you’ll find another approach to this part.

Exercises

1 9 .3 .1 : t V

In triangle A B C , let the bisector of angle B A C intersect B C at D and the circumcircle of triangle A B C at E. Prove that
A E • D E = B E 2.

Hint

Type your solution, notes and/or work here. Show Solution

1 9 .3 .2 : t V

In the diagram, ZA bisects ZBZC, and YA bisects ZBYX. Prove that


ZYCZ + ZYBZ 2ZYAZ.

Hint

Type your solution, notes and/or work here. Show Solution

1 9 .3 .3 : t V

Prove Z L Z X = Z L Y X in Problem 19.10 without using the fact that X L is a diameter of the small circle. Use this to find
an alternate proof that X L is a diameter of the circumcircle of A A L M .

Type your solution, notes and/or work here. Show Solution

1 9 .3 .4 : t V

Quadrilateral W X Y Z has right angles at Z W and Z Y and an acute angle at Z X . Altitudes are
dropped from X and Z to diagonal W Y , meeting W Y at O and P as shown. Prove that
W O = BY.

Type your solution, notes and/or work here. Show Solution

1 9 .3 .5 ★ : t V

Let A B, and C be points on a line, in that order. We draw semicircles on segments B C , A C , and A B. Then the figure they
enclose is called an arbelos, which is shaded in the diagram below at left. The arbelos has many interesting properties, two of
which we prove in this problem.

(a) Take point D on arc A C such that D B J_ A C . Prove that the arbelos has the same area as the circle with diameter
‫־‬B D .

Hint

Type your solution, notes and/or work here. Show Solution

(b)*Line B D from the previous part divides the arbelos into two parts. Inscribe a circle in each part. Prove that the two
circles have equal radius.

C A

Hint

Hint

Hint

Hint

Type your solution, notes and/or work here. Show Solution

Extra! A ‫׳‬proof without words’ of the Pythagorean Theorem is shown


at right. Alexander Bogomolny describes it as an ‫׳‬unfolded
V variant' of a proof by abu I’Hasan Thabit ibn Qurra Marwan
al’Harrani popularized by Monty Phister.

Extra! Josey and Beth are standing 100 feet apart with a 100 foot rope connecting their
ankles such that the rope is pulled taut 1 inch above the ground between them. Josey
V steps one foot towards Beth, so the rope is relaxed along the ground. Is it now
possible to lift the center of the rope high enough that a person can walk under it
without ducking or moving either of the girls? Is it possible for Josey to lift the rope
above her head without her and Beth getting closer (and without her lifting her foot)?
19.4 Summary
This entire chapter is about problem solving strategies. Here are several of the most useful approaches we used to solve
problems throughout this book.

■ When in doubt, build right triangles. One very common way to do so is to draw radii to points of tangency. Nearly always try
this when you have circles and tangents in a problem. Altitudes of triangles and trapezoids also often produce useful right
triangles.

■ Connecting points that are originally not connected in a diagram can be extremely useful! This doesn’t mean you should
connect everything in your diagram immediately, however. Look for segments to draw that will be helpful, particularly those
connecting important points, or those that form segments, triangles, or angles you know something about immediately.

■ 60°, 30°, and even 120° angles are good signs to try to build 3 0 9 0 ‫־‬60‫ ־‬triangles by dropping altitudes or extending
segments.

■ Consider extending segments that stop suddenly inside figures such as triangles, quadrilaterals, or circles.

■ If you successfully use a certain tactic in a problem to get some information, but you still haven’t solved the problem, try
using that same tactic again in a different way. Maybe it still has more information to give!

■ Label lengths as you find them, even if you have to label them in terms of an important variable.

■ Assign variables to important angles and use your mastery of basic geometry to express other angles in terms of those
variables. Typically, you should first assign variables to angles you seek and to angles you know a lot about.

■ Once you start chasing angles, reach for all the angle tools you know, most notably the facts you know about triangles,
straight lines, parallel lines, and angles that intersect circles.

■ When trying to find a geometric proof, keep careful track of what you know and what you want.

■ Draw large, precise diagrams - triangle congruences, parallel lines, collinear points, and much, much more stand out more
clearly in large, precise diagrams than in small, scribbled ones.

Compare new problems to problems you have already solved.


C hallen g e P roblem s

t V
19.11

A X Y Z is a right triangle with right angle Z X . P is on X Z . The triangle is folded over V P so that
point X lands on side Y Z . Given that X Y = (') and P Y = P Z , find, with proof, Z X Z Y .

Show Solution
Type your solution, notes and/or work here.

Source: ARML t V
1 9 .1 2 :

Square L M N P is inscribed in right triangle A B C as shown. Given P N = G, compute B


(A P )(N C ).

Show Solution
Type your solution, notes and/or work here.

t V
19.1 3 :

Four circles of radius 2 are arranged so that each is tangent to two others, and their centers are the vertices of a square of
side length 4. A small circle is inside the square such that it is tangent to the four circles. What is the radius of the small
circle?

Show Solution
Type your solution, notes and/or work here.

Source: MATHCOUNTS t V
1 9 .1 4 :

A A B C and A P Q R are equilateral in the diagram at right. A Q , R C , and B P are congruent


and bisect their respective angles. The four interior regions A Q P B , A Q R C , B P R .C , and
Q P R all have the same area. Given that A B = 20, what is P B ?

Show Solution
Type your solution, notes and/or work here.

t V
1 9 .1 5 :

Point P is inside rectangle A B C D . Prove that A P 2 4 ‫ ־‬C P 2 = B P 2 4‫ ־‬D P

Hint

Hint

Show Solution
Type your solution, notes and/or work here.

t V
1 9 .1 6 :

W X Y Z is a parallelogram. M is the midpoint of W Z and O is the midpoint of Y Z . M O and X Z meet at N . Find


Z N /N X .

Show Solution
Type your solution, notes and/or work here.

t V
1 9 .1 7 :

In A A B C , Z B A C = 60°, Z A C B = 45°. and D is on B C such that A D = 18 and Z B A D = Z C A D . Find the area of


A ABC.

Hint

Hint

Hint

Show Solution
Type your solution, notes and/or work here.

Source: ARML t V
1 9 .1 8 :

In the diagram at right, the circle and the square have the same center. If the area of shaded region B
T a
A B C equals the area bounded by C D and minor arc C D , compute the ratio of the side of the square
1
to the radius of the circle.
\
> \

Show Solution
Type your solution, notes and/or work here.

t V
19.1 9 :

In quadrilateral A B C D , A B || C D . A C and B D meet at E. Points M and N are the midpoints of A E and D E ,


respectively. B M and B E trisect Z A B C , and C E and C N trisect Z B C D . Prove that A B C D is a rectangle.

Hint

Hint

Show Solution
Type your solution, notes and/or work here.

Source: ARML t V
19.2 0 :

Four congruent circles are tangent to each other and tangent to the edges of a sector as shown. If the
straight edges are joined to form a right circular cone with vertex at P, the radius of the base would be
2 /3 the slant height of the cone. Compute the ratio of the radius of the sector to the radius of each
circle.

Hint

Hint

Show Solution
Type your solution, notes and/or work here.

Source: MATHCOUNTS t V
1 9 .2 1 :

A B C D is a square. Parallel lines m, n, and p pass through vertices A, B, and C, respectively. The distance between m and
n is 7 units, and the distance between n and p is 9 units. Find the number of square units in the area of square A B C D .

Hint

Hint

Show Solution
Type your solution, notes and/or work here.

t V
1 9 .2 2 :

Chords X Y and T U of © 0 bisect each other. Furthermore, X Y = T U . Prove that the two chords meet at the center of the
circle.

Show Solution
Type your solution, notes and/or work here.

Source: AHSME t V
19.2 3 :

In the figure, points B and C lie on line segment A D , and A B , B C , and C D


are diameters of circles O, N , and P, respectively. Circles O, N , and P all
have radius 15, and the line A G is tangent to circle P at G. If A G intersects
circle N at points E and F, then find E E .

Hint

Hint

Show Solution
Type your solution, notes and/or work here.

t V
1 9 .2 4 :

Find the volume of a sphere that is inscribed in a regular octahedron with side length 12.

Hint

Hint

Hint

Show Solution
Type your solution, notes and/or work here.

t V
1 9 .2 5 :

In the diagram at right, O is the center of the circle. Find the area of the shaded region given that
Z A O C = 120° and O M = M C = 6 .

Show Solution
Type your solution, notes and/or work here.

t V
1 9 .2 6 :

Squares are erected externally on the sides of A A B C as shown at right. The altitudes from
vertices A, B, and C are drawn and extended to divide these squares into rectangles. Let R lt
for 1 < i < 6 , be the areas of the rectangles as shown. Prove that Ro = R z R a = Rs, and
Ra = R h what does this result say if Z A C B = 90°?

Hint

Hint

Show Solution
Type your solution, notes and/or work here.

t V
1 9 .2 7 :

Shown at right is right rectangular prism A B C D E F G H . Prove that A A C F is acute.

Hint

Hint

Show Solution
Type your solution, notes and/or work here.

t V
19.28:

M and N are the midpoints of diagonals B D and A C , respectively, of trapezoid A B C D B


at right. A B || C D , A B = 8 , and M N = (>.

(a) Prove that M N || C D . (Find a rigorous proof - this is one of those ‫׳‬obvious’ facts
that requires a careful proof.) D

Hint

Show Solution
Type your solution, notes and/or work here.

(b) Find C D.

Hint

Hint

Show Solution
Type your solution, notes and/or work here.

t V
1 9 .2 9 ★ :

A circle in the plane has center O. Two chords, with midpoints M and N , intersect at P. Prove that
M N < O P . When does equality occur?

Hint

Hint

Show Solution
Type your solution, notes and/or work here.

Source: USAMTS t V
1 9 .3 0 ★ :

Let R and S be points on the sides B C and A C , respectively, of triangle A B C , and let P be the intersection of A R and
U S . Determine the area of triangle A B C if the areas of triangles A P S , A P B , and B P R are 5, 6 , and 7, respectively.

Hint

Hint

Show Solution
Type your solution, notes and/or work here.

Source: Mandelbrot t V
1 9 .3 1 :

In the diagram to the right, points A and B are on © 0 . Point C is on major arc A B . Line i is the
angle bisector of Z A C B and line m is the perpendicular bisector of A C . Lines t and m meet at
point P, point M is the midpoint of minor arc A B , and minor arc A B has measure 9. Prove that
Z A P M = 9/2.

Hint

Show Solution
Type your solution, notes and/or work here.

t V
1 9 .3 2 :

Sphere Z is tangent to all 6 edges of regular tetrahedron A B C D . Given that each edge of the tetrahedron has length 12 , find
the radius of sphere Z.

Hint

Hint

Hint

Show Solution
Type your solution, notes and/or work here.

t V
1 9 .3 3 ★ :

A A B C is an equilateral triangle with side length 6 . Prove that for any point P we choose inside A A B C , the sum of the
distances from P to the sides of A A B C is the same.

Hint

Show Solution
Type your solution, notes and/or work here.

Source: H M M T t V
1 9 .3 4 ★ :

U and C are points on the sides of A M N H such that M U = s, U N = 6 , N


N C = 20, C H = s, H M = 25. If A U N C and M U C H have equal
areas, what is s?

Hint

Show Solution
Type your solution, notes and/or work here.

t V
1 9 .3 5 ★ :

Let C ! and C 2 be two circles that are externally tangent, and let their centers be O ! and 0 -2, respectively. Let P be a point on
C i and let Q be a point on C 2 such that P Q is a common external tangent to circles C ! and C 2 , and let M be the midpoint of
P Q . Prove that Z 0 !A / 0 2 = 90°.

Hint

Hint

Show Solution
Type your solution, notes and/or work here.

t V
1 9 .3 6 ★ :

Four balls of radius 1 are all tangent to each other. What is the radius of the smallest sphere that encloses them?

Hint

Show Solution
Type your solution, notes and/or work here.

t V
1 9 .3 7 ★ :

Diagonals A C and B D of regular heptagon A B C D E F G meet at X . Prove that A B 4‫ ־‬A X = A D .

Hint

Show Solution
Type your solution, notes and/or work here.

You might also like